Sunteți pe pagina 1din 220

 

1. ¿QUÉ AFIRMACIÓN ES FALSA RESPECTO A LOS NIÑOS CON PRIMOINFECCIÓN


TUBERCULOSA?

LA ÚNICA EVIDENCIA DE ENFERMEDAD PUEDE SER LA SEROCONVERSIÓN


1. 1.
DEL MANTOUX.
2. 2. RARA VEZ SE DETECTA RADIOLÓGICAMENTE EL COMPLEJO PRIMARIO.
3. 3. LOS NIÑOS SON MUY CONTAGIOSOS Y PRECISAN AISLAMIENTO.
4. 4. PRECISAN TRATAMIENTO TRAS EL DIAGNÓSTICO DURANTE 9-12 MESES.
Gráfico de respuestas
Comentario

Los niños con tuberculosis son muy poco contagiosos, ya que tienen escasa capacidad de
propagar el bacilo. Son muy poco bacilíferos debido a que la fuerza muscular es mucho menor que
la de un adulto, con lo que la tos de un niño es muy poco efectiva para esta propagación
(respuesta 4 falsa). El resto de las respuestas son ciertas, y ten especial cuidado con la 1, porque
efectivamente la tuberculosis produce una sintomatología más bien larvada.(R3)

2. ¿Cuál de las siguientes características define a las máculas?:

1. 1. Tamaño menor de 1 cm.


2. 2. Afectar la epidermis.
3. 3. Desaparecer a la vitropresión.
4. 4. Ser cambios de coloración cutánea.
Gráfico de respuestas
Comentario
¡No se debe fallar! Las lesiones elementales son una pregunta sencilla, que no debe suponerte
ningún problema, por lo que debes estudiarlas fijándote en lo fundamental. La Mácula es, por
definición, un cambio de coloración de la piel (respuesta 5 correcta). El tamaño no es una
característica definitoria, por lo que la respuesta 1 es incorrecta. Existen máculas que desaparecen
a la vitropresión y otras que no (respuesta 3 incorrecta), como son las producidas por
extravasación sanguínea, denominadas Máculas purpúricas, que poseen un color violáceo. Una
mácula puede afectar sólo a la epidermis o extenderse también a la dermis (respuesta 2 falsa). La
mácula es una lesión primaria (respuesta 4 falsa), dentro de la clasificación de las Lesiones
Dermatológicas (apréndela bien, te será muy útil).(R4)

3. Lactante de 3 meses de edad con episodios de púrpura frecuentes desde su


nacimiento, lesiones eccematosas en torso y cara interna de extremidades, e infecciones
respiratorias recidivantes. En su analítica destaca una trombocitopenia, linfopenia y
descenso de IgM con elevación de IgA e IgE. El diagnóstico más probable es:

1. 1. Síndrome de Wiskott-Aldrich.
2. 2. Síndrome de Fanconi.
3. 3. Síndrome de Bloom.
4. 4. Síndrome de Duncan.
Gráfico de respuestas
Comentario

Pregunta de dificultad media sobre un tema poco importante en el ENARM, las inmunodeficiencias.
Recuerda repasar bien este tema para el ENARM. Este ID no ha sido muy preguntada en el
ENARM, pero es un caso muy típico, por lo que deberías intentar recordarla ya que las cosas

 
 
 
 
típicas son las que aparecen en el ENARM. El síndrome de Wiskott- Aldrich es una
inmunodeficiencia primaria que involucra tanto a linfocitos B como T y además, otro tipo de células
sanguíneas como las plaquetas. En su forma clásica, el SWA tiene un patrón característico de
descubrimientos que incluye: un aumento en la tendencia a sangrar causada por una menor
cantidad de plaquetas, infecciones recurrentes por bacterias, virus y hongos, y eccema de la piel.
Además existe un aumento en la incidencia de malignidad, incluyendo linfoma y leucemia, y un
aumento en la incidencia de enfermedades autoinmunes en algunos pacientes. Se hereda como un
trastorno recesivo ligado al X, por lo tanto, sólo los varones son afectados por esta enfermedad.
Cuando aparecen síntomas de infección es necesaria una búsqueda completa de infecciones y
determinar el tratamiento antimicrobiano más efectivo. La única cura permanente para el SWA es
un transplante de médula ósea o un transplante de células madre del cordón umbilical.(R1)

4. A 2-year-old boy is brought to the doctor's office for a routine visit. His mother reports
that the child is able to kick a ball and he can walk up and down stairs holding your hand.
He follows two-step instructions such as “Pick up your shoes and put them in the closet"
and builds towers of 4 or more blocks. He can repeat words but is unable to construct 4-
word sentences. Which of the previous aspects is more concerning?

1. 1. Language delayed development


2. 2. Motor skills
3. 3. Cognitive impairment
4. 4. Social interaction
Gráfico de respuestas
Comentario
Language delayed development. A 24-month-old child is able to recognize body parts and objects
by their names, point to objects when they are named, say several single words, repeat words of a
conversation and use simple phrases. The boy in this case does not use phrases yet, so there is a
delay in language development. The rest of milestones are adequate for the child's age.(R1)

5. ¿A partir de qué semana del embarazo pueden verse por ultrasonido abdominal los
movimientos cardiacos fetales?:

1. 1. 7ª semana.
2. 2. 8ª semana.
3. 3. 10ª semana.
4. 4. 14ª semana.
Gráfico de respuestas
Comentario

Pregunta sobre el ultrasonido del primer trimestre, es un dato puramente memorístico.

El saco gestacional puede visualizarse a partir de la 5ª semana a través de un USG transvaginal.


En cambio, si el USG es abdominal habría que esperar un poco más (6ª semana). Con el latido
cardíaco, ocurre algo similar, pero se demora una semana: se ve a la 6ª por vía vaginal y a la 7ª
transabdominal. Respuesta 1 correcta.(R1)

6. Un RN comienza con conjuntivitis a las 12 horas de vida ¿Cúal es la etiología más


probable?

1. 1. Gonococo.
2. 2. Pseudomonas.

 
 
 
 
3. 3. Chlamydia.
4. 4. Estreptococo B.
Gráfico de respuestas
Comentario

Durante los primeros días de vida, habría que tener en cuenta al gonococo como primera opción
(respuesta 1 correcta).

La conjuntivitis por Chlamydia aparece más tarde, entre los días 5 y 14.

Actualmente, se emplean colirios de tetraciclinas, puesto que el nitrato de plata es muy irritante y,
por sí mismo, era capaz de producir conjuntivitis químicas.(R1)

7. Con respecto a los miomas uterinos, ¿Cuál de los siguientes enunciados es correcto?

1. 1. Se encuentran en por lo menos las ¾ partes de las mujeres en edad fértil.


2. 2. Se encuentran en menos de 1/6 parte de las mujeres mayores de 35 años.
3. 3. Se estima que menos de la mitad de los miomas uterinos producen síntomas.
4. 4. Desde el punto de vista anatomopatológico, son tumores tipo vascular.
Gráfico de respuestas
Comentario

Los miomas son los tumores más frecuentes del tracto genital femenino. Estan compuestos de
fibras musculares lisas. La edad de máxima incidencia es en los 35-45 años. Alrededor de un 50-
80% de los miomas son asintomáticos, siendo un hallazgo casual en una ecografia. Tienen una
prevalencia del 25% en la raza blanca y del 50% en la raza negra.(R3)

8. Carlos es un niño que sube escaleras alternando los pies, salta, imita trazo circular,
puede decir YO, MIO, TUYO, ayuda a recoger objetos. ¿Cuál es la edad en meses de
Carlos, desde el punto de vista del desarrollo madurativo?:

1. 1. 24
2. 2. 30
3. 3. 42
4. 4. Ninguna de las anteriores
Gráfico de respuestas
Comentario

Pregunta de elevada dificultad, no te preocupes si la has fallado.

El conocimiento de la pauta normal del desarrollo psicomotor es fundamental para detectar


precozmente las desviaciones que puedan indicar la presencia de una disfunción del SNC. El
desarrolo neurológico sigue una dirección cefalocaudal. Existen numerosos hitos que pueden
evaluarse para valorar el desarrollo psicomotor. La prueba más empleada es el test de Denver, que
valora los siguientes aspectos: personal-social, lenguaje, motricidad fina y grosera en niños desde
el nacimiento a los 6 años.

En este caso en particular, se cuenta que el niño ya sube escales, salta, realiza combinaciones de
palabras, que corresponderían a hitos de 24 meses. Sin embargo, también se dice que para subir y

 
 
 
 
bajar las escaleras alterna los pies, imita trazos circulares, ayuda a recoger objetos...todo ello hitos
un tanto más complejos que corresponderían más a un desarrollo madurativo de 30 meses.(R2)

9. Un niño de 2 años tiene un cuadro de fiebre alta sin foco claro, con adenopatías
laterocervicales como único hallazgo en la exploración. El estado general es bueno.
Recibe ibuprofeno y amoxicilina. A las 72 horas, ya sin fiebre, presenta exantema de
tronco y raíz de miembros, sin afectar a la cara. Dos días más tarde toda la sintomatología
ha desaparecido. ¿Qué diagnóstico establecería?:

1. 1. Escarlatina.
2. 2. Enfermedad de Kawasaki.
3. 3. Eritema infeccioso.
4. 4. Exantema súbito.
Gráfico de respuestas
Comentario

Pregunta de dificultad media pero que presenta datos típicos que ya nos deberían ir sonando en la
primera vuelta.

Un niño con fiebre alta que al desaparecerle ésta, presenta un exantema que respeta cara y
extremidades inferiores debe orientarnos hacia el diagnóstico de un exantema súbito, enfermedad
producida por el HHV- 6.

También es característico de esta enfermedad la presencia de leucocitosis con neutrofilia en las


primeras 24 horas y su asociación con las convulsiones febriles. Debemos estudiar esta
enfermedad puesto que ya ha caído varias veces en el examen.(R4)

10. El síndrome de Turner es una anomalía:

1. 1. Cromosómica estructural, tipo inversión.


2. 2. Monogénica, ligada cromosoma X.
3. 3. Poligénica, multifactorial.
4. 4. Cromosómica, tipo monosomía sexual.
Gráfico de respuestas
Comentario

Concepto fundamental. El síndrome de Turner forma parte del grupo de anomalías que afecta a los
gonosomas. Se caracteriza por talla baja y disgenesia gonadal en mujeres que tienen un sólo
cromosoma X y la ausencia de todo o parte del segundo cromosoma, ya sea X o Y.(R4)

11. A 5-year-old Caucasian child with a delayed development and recurrent respiratory
infections that have frequently required hospital admission and intravenous
antibiotherapy presents to the emergency department for an intense nosebleed.
Coagulation tests show an increased prothrombin time. Which of the following is the most
likely deficit in coagulation factors in this case?

1. 1. Factor VII
2. 2. Fibrinogen
3. 3. Antithrombin
4. 4. Factor V
Gráfico de respuestas

 
 
 
 
Comentario
Factor VII. Repetitive respiratory infections are common in cystic fibrosis, a potentially lethal
autosomal recessive disease. Other typical symptoms are malabsorption and diarrhea due to
pancreatic involvement. Vitamin K has a lipidic structure and it requires emulsion by pancreatic
juices for absorption. Its deficiency affects vitamin K dependent factors II, VII, IX and X.(R1)

12. ¿Cuál de las siguientes correlaciones clínico-terapéuticas es correcta?:

1. 1. Erosio interdigitalis blastomicetica-griseofulvina.


2. 2. Botón de Oriente-antimoniales.
3. 3. Eczema marginado-nistatina.
4. 4. Pitiriasis versicolor-crotamiton.
Gráfico de respuestas
Comentario
Pregunta básica, que no se debe fallar. Las Enfermedades Infecciosas de la piel es uno de los
temas más importantes en la Dermatología de cara al MIR. La Pediculosis pubis (ladillas) se trata
con Lindane tópico o Permetrina al 1%, como todas las Pediculosis. La Erosio interdigitalis
blastomicetica está producida por Candida, en la cual nunca se debe usar Griseofulvina, sino que
su tratamiento de primera elección son los Imidazoles. El Botón de Oriente es una Leishmaniasis
cutánea y su tratamiento, como en la visceral, son los Antimoniales pentavalentes (respuesta 3
correcta). El Eccema marginado de Hebra es una Tiña inguinal y éstas se tratan habitualmente con
Imidazoles de una forma eficaz. La Pitiriasis Versicolor es la micosis más frecuente y está
producida por Malassezia furfur. Su tratamiento son los Imidazoles.(R2)

13. Señale la correcta entre las siguientes afirmaciones respecto a la gripe:

Una de las cepas de la vacuna antigripal estacional administrada a la población española


1. 1.
en los últimos años ha sido A (H1N1).
El virus A (H1N1) de la actual pandemia está formado por la recombinación de genes de
2. 2.
la gripe A humana y porcina y no incluye genes de la gripe A aviar.
Los virus de la gripe son virus con ADN monocatenario y diversas proteínas, de las cuales
3. 3.
la hemaglutinina y la neuraminidasa no son estructurales y permiten su clasificación.
La pandemia de gripe A (H1NI) de 1918 se llamó “gripe española” porque se inició en
4. 4.
España y afectó a la población española de manera particularmente intensa.
Gráfico de respuestas
Comentario

En realidad, la pregunta podía resolverse sin saber demasiado sobre la epidemia que motivó esta
pregunta.

Existen tres tipos de gripe estacional: A, B y C. Los virus de tipo A se clasifican en atención a las
distintas combinaciones de dos proteínas presentes en la superficie del virus, la hemaglutinina (H)
y la neuraminidasa (N). Existen numerosos subtipos de virus gripales de tipo A. Durante los últimos
años, los más frecuentemente responsables de enfermedad en humanos han sido H3N2 y H1N1.

En la situación actual, con brotes epidémicos anuales durante los meses fríos, la composición de la
vacuna de la gripe estacional se compone de estos dos subtipos de virus (H1N1 y/o H3N2) y/o
virus B. La vacunación se realiza mediante una vacuna trivalente inactivada, que contiene las
cepas que circularon en la temporada gripal anterior. Por lo tanto, la respuesta correcta es la 1. Lo
que no se incluye en la vacuna son virus gripales del tipo C, puesto que justifican muy pocos casos
de enfermedad en humanos.

 
 
 
 
No te dejes engañar por la respuesta 4. La “gripe española” se inició en EEUU. Recibió esta
denominación porque las primeras noticias se difundieron en periódicos de Boston, publicados en
español.(R1)

14. Un paciente de 48 años, etílico crónico y diagnosticado de cirrosis hepática ingresa


en Urgencias con un cuadro de confusión moderada y asterixis de varias horas de
evolución. ¿Cuál de estas circunstancias cree que NO influyó en el desencadenamiento
de esta complicación?

1. 1. Ingesta de gran cantidad de carne en las horas previas.


2. 2. Tratamiento con un ansiolítico los días previos.
3. 3. Tratamiento diurético por su hepatopatía.
4. 4. Tratamiento con betabloqueantes para profilaxis de hemorragias por varices.
Gráfico de respuestas
Comentario

Un paciente cirrótico con confusión y asterixis padece, casi con total seguridad, un cuadro de
encefalopatía hepática. Lo que nos preguntan, por tanto, es que señalemos el factor que no es
capaz de desencadenarla. Algunos tratamientos relativamente frecuentes en los pacientes
cirróticos, como los diuréticos del asa (que a veces se usan para el control de la ascitis) pueden
producir encefalopatía hepática por las alteraciones del medio interno que producen (alcalosis
metabólica). Sin embargo, el uso de betabloqueantes para disminuir la presión portal no se
relaciona con este problema (respuesta 4).(R4)

15.   Neonato de 30 días que


presenta cuadro de vómitos de 12 horas de evolución asociado a evacuación con hilos
de sangre, según los padres. En la exploración se observa regular estado general y
cierta distensión abdominal. Se decide realizar una radiografía de abdomen. ¿Qué es lo
que se observa en la imagen y cuál es la asociación más compatible?:

1. 1. Imagen en donuts- invaginación intestinal.


2. 2. Neumatosis intestinal- enterocolitis necrotizante.
3. 3. Imagen normal- alergia a las proteínas de la leche de vaca.
4. 4. Neumoperitoneo- enterocolitis necrotizante.
Gráfico de respuestas
Comentario

 
 
 
 
En la imagen se puede ver gas dentro de las paredes del intestino (neumatosis intestinal), un
hallazgo muy característico en la enterocolitis necrotizante.

El cuadro clínico de un neonato con vómitos y distensión abdominal que presenta una deposición
con sangre, es también muy característico de esta afección. Si este cuadro no se diagnostica a
tiempo, puede evolucionar en choque y muerte,

La principal duda se podría establecer con la respuesta 3 y la 4. Respecto a la alergia a las


proteínas de leche de vaca, puede presentar un cuadro similar con evacuaciones sanguinolientas,
pero no se apreciarían las alteraciones radiológicas que vemos en esta imagen. Respecto a la
respuesta 4, en el caso que aparezca un neumoperitoneo secundario a perforación de alguna de
las asas, el aire se vería entre las asas y la pared abdominal, no dentro de la pared de las asas
como en este caso.(R2)

16. ¿Cuál de los siguientes enunciados es verdadero respecto al cuadro de la pregunta


anterior?:

Se debe dejar al niño en dieta absoluta con infusión de cristaloides y, si es preciso, colocar
1. 1.
una sonda nasogástrica.
Se deben retirar de la dieta las proteínas de la leche de vaca, y substituirla por un
2. 2.
hidrolizado.
3. 3. Deberá realizarse siempre una resección del segmento afectado.
Lo más importante es que se continúe con la lactancia materna, pues previene las posibles
4. 4.
complicaciones.
Gráfico de respuestas
Comentario

Ante un cuadro de enterocolitis necrotizante es importante dejar en reposo el intestino por lo que
se deja al niño en dieta absoluta, aportando los líquidos necesarios por vía intravenosa. En algunos
casos se coloca una sonda nasogástrica para descomprimir el tubo digestivo.

Suele emplearse antibioticoterapia de amplio espectro, con cobertura contra anerobios y gram
negativos. Se suele utilizar cefotaxima y vancomicina con clindamicina en algunas ocasiones. La
cirugía solo está indicado en algunos casos (perforación).

La respuesta 4 podría resultar confusa, ya que se sabe que la lactancia materna es un factor
protector para esta patología, sin embargo, una vez producida la enterocolitis, es imprescindible
que se mantenga al paciente a dieta absoluta.(R1)

17. Recién nacido, de 37 semanas de edad gestacional, que presenta hipoplasia de la


extremidad superior derecha con cicatrices cutáneas, microcefalia, coriorretinitis,
cataratas y crisis convulsivas. En la TC se observa atrofia de parénquima cerebral y
cerebeloso, sin calcificaciones. El diagnóstico más probable es:

1. 1. Infección congénita por parvovirus.


2. 2. Infección congénita por varicela-zoster.
3. 3. Toxoplasmosis congénita.
4. 4. Infección congénita por CMV.
Gráfico de respuestas
Comentario

 
 
 
 
Cuando una embarazada que no ha pasado la varicela la contrae durante el primer trimestre de
gestación, hay riesgo de que el feto presente la llamada fetopatía por VVZ, y cuyo rasgo más
específico consiste en la presencia de cicatrices deformantes de morfología lineal siguiendo una
distribución metamérica. Además, puede presentar microcefalia, coriorretinitis, crisis, entre
otras.(R2)

18. ¿Cuál de las siguientes opciones acerca del tratamiento de la bronquiolitis es


INCORRECTA?:

Actualmente el tratamiento de primera elección en las bronquiolitis moderado-severas es


1. 1.
la adrenalina en aerosol.
Se han descartado los corticoides orales porque no son eficaces y además pueden tener
2. 2.
efectos secundarios importantes como alteraciones del crecimiento.
El único antiviral que se ha demostrado efectivo contra el VRS es la ribavirina, que estaría
3. 3.
indicada en los casos más severos.
La teofilina se podría utilizar en el caso de que el niño presente apneas que puedan resultar
4. 4.
peligrosas, aunque su eficacia no está totalmente demostrada.
Gráfico de respuestas
Comentario

Esta pregunta acerca del tratamiento farmacológico de la bronquiolitis tiene cierta dificultad y es
importante que domines este tema. Recuerda que el tratamiento de primera elección en las
bronquiolitis moderado- severas es la adrenalina en aerosol que es el fármaco más eficaz al
disminuir el edema y la secreción de moco. Ya que en la bronquiolitis no hay broncoconstricción el
salbutamol no es muy eficaz. Los corticoides son ineficaces y además pueden tener efectos
secundarios importantes. La ribavirina es un antiviral que se administra en aerosol. Estudios con
alta evidencia científica demuestran que no es efectiva en el tratamiento de bronquiolitis por VRS,
es teratógena. Su uso sólo debe ser considerado en casos graves en pacientes de alto riesgo. La
teofilina se podría utilizar en caso de apneas graves.(R3)

19. Un lactante de 6 meses, sin antecedentes de interés y con desarrollo psicomotor


normal, presenta salvas de contracciones bruscas en flexión y extensión de la cabeza,
tronco y extremidades que le aparecen al despertarse. En el EEG encontramos una
alteración bilateral con ondas lentas de alto voltaje caóticas y asíncronas (hipsarritmia).
¿Cuál es el fármaco con el que se podría iniciar el tratamiento en este tipo de epilepsia?

1. 1. ACTH.
2. 2. Etosuximida.
3. 3. Lamotrigina.
4. 4. Fenobarbital.
Gráfico de respuestas
Comentario

El síndrome de West apenas tiene importancia para el MIR, pero debes conocer los rasgos más
característicos:

- La hipsarritmia es un fenómeno que no aparece en las crisis, sino en períodos intercríticos.


Consiste en un ritmo de fondo de ondas lentas, entre las que se intercalan otras más rápidas.

- El síndrome de West aparece en niños con solamente unos meses de vida, casi siempre
secundario a otras enfermedades neurológicas.

 
 
 
 
- Son característicos los “espasmos salutatorios”, que son movimientos flexoextensores del tronco.

- En esta entidad en concreto, existe cierta respuesta terapéutica a la ACTH, aunque el ácido
valproico podría ser también una opción.

Esta pregunta podría ser anulada, ya que las opciones 1 y 2 pueden ser empleadas con
eficacia.(R1)

20. Ante un paciente diabético de 60 años que acude a Urgencias con disminución del
nivel de conciencia, oliguria, presentando en la analítica glucemia de 800 mg/dl, sodio de
150 mEq/l, bicarbonato de 20 mEq/l y creatinina de 3 mg/dl, la actitud prioritaria será:

1. 1. Intubación orotraqueal.
2. 2. Realizar una TC.
3. 3. Reposición de líquidos.
4. 4. Tratamiento intensivo con insulina i.v.
Gráfico de respuestas
Comentario

Este paciente presenta una descompensación hiperglucémica hiperosmolar. Los diabéticos que
presentan esta complicación son los de tipo 2. Este cuadro suele producir niveles muy altos de
glucosa, (generalmente > 600 mg/dl), deshidratación importante con aumento de los productos
nitrogenados y sodio sérico normal o bajo, por el efecto de la hiperglucemia. No suele aparecer
acidosis. La cetonuria también es rara y, si aparece, es leve. Para el tratamiento, lo fundamental es
la hidratación del paciente, ya que lo que predomina es el déficit de líquidos, que a veces llega a
ser de hasta 10 litros. A pesar que que la hiperglucemia suele ser severa, la insulina no es el
tratamiento fundamental, aunque ayuda a corregir el cuadro.(R3)

21. Uno de los siguientes procedimientos diagnóstico NO es posible efectuarlo en


consultorio:

1. 1. Prueba con KOH al 10%.


2. 2. Biopsia de cervix.
3. 3. Biopsia de endometrio.
4. 4. Prueba de Micro Track.
Gráfico de respuestas
Comentario

El test de Micro Track es una prueba de inmunofluorescencia directa para detectar inclusiones de
Chlamydia en el frotis endocervical.

Es una prueba que debe hacerse en el laboratorio. Respuesta 4.(R4)

22. Señale la respuesta CORRECTA respecto a la estenosis hipertrófica de píloro:

1. 1. Es más frecuente en niñas.


2. 2. Presenta acidosis metabólica hiperclorémica con hipopotasemia.
3. 3. La prueba diagnóstica de elección es la Rx de abdomen.
4. 4. El tratamiento consiste en la pilorotomía extramucosa de Ramstedt.
Gráfico de respuestas

 
 
 
 
Comentario

La EHP es más frecuente en NIÑOS. Da alcalosis metabólica porque pierden ácido en los vómitos.
La prueba diagnóstica es el ultrasonido y el tratamiento es la pilorotomía de Ramsted.(R4)

23. ¿Cuál de estos agentes etiológicos es el que más veces origina la oftalmía del recién
nacido?:

1. 1. Chlamydia trachomatis.
2. 2. Especies de Hemophilus.
3. 3. Streptococcus pneumoniae.
4. 4. Staphylococcus aureus.
Gráfico de respuestas
Comentario

Pregunta muy importante. La oftalmía del RN es causada más frecuentemente por Chlamydia
trachomatis serotipo D-K (respuesta correcta 1). Suele aparecer tras un periodo de incubación de
7-14 días. Habitualmente se presenta en el contexto de una afectación sistémica. Para el
diagnóstico se emplea la detección de cuerpos intracelulares en el gram o el ELISA. El tratamiento
se realiza con tetraciclina tópica y eritromicina sistémica.(R2)

24. Neonato de 2 días de vida con perímetro craneal de 30 cm, hombre, procedente de un
embarazo mal controlado, con edad gestacional de 38 semanas y peso de 2,400 g,
adenopatías laterocervicales e inguinales, con fontanela de 4x4 cm,
hepatoesplenomegalia y una opacidad en la pupila de ambos ojos, soplo sistólico en 2º
EII. La madre recuerda que tuvo un proceso febril en los primeros meses. ¿Qué patología
sospecha en este niño?:

1. 1. Sífilis congénita.
2. 2. Rubéola congénita.
3. 3. Infección intraparto por virus herpes simple.
4. 4. Toxoplasmosis congénita.
Gráfico de respuestas
Comentario

Recuerde la tríada de Gregg para diagnosticar una rubéola congénita: microcefalia, catarata
(opacidad de pupila) [puede presentar también glaucoma o retinitis en sal y pimienta], conducto
arterial persistente (soplo sistólico) y sordera. La rubéola es más grave y frecuente si la madre se
infecta durante el primer trimestre de gestación. Respuesta 2 correcta.(R2)

25. Indique cuál de las siguientes afirmaciones respecto a los cálculos biliares
pigmentarios es FALSA:

Los cálculos terrosos de bilirrubinato cálcico se encuentran fundamentalmente en la


1. 1.
vesícula biliar.
2. 2. Su incidencia es mayor en pacientes afectos de enfermedades hemolíticas.
3. 3. Suelen acompañarse de un exceso de bilirrubina indirecta en el líquido biliar.
4. 4. Su frecuencia es mucho mayor en países orientales.
Gráfico de respuestas

 
 
 
 
Comentario

Los cálculos biliares más frecuentes son los de colesterol, cuando se componen de sales cálcicas
su consistencia suele ser sólida y blanda, no terrosa.(R1)

26. Una mujer de 49 años acude a consulta por presentar rectorragia de heces de aspecto
mucoide. En el examen endoscópico se encuentra una tumoración plana de superficie
vellosa, de 4 x 3 cm, que se reseca endoscópicamente al parecer de forma completa.
Histológicamente, se objetiva un adenoma velloso, descartándose infiltración neoplásica
del tallo vascular. ¿Cuál es la actitud que debe adoptar respecto a esta paciente?:

Resección segmentaria de la zona donde implantaba el pólipo, con anastomosis término-


1. 1.
terminal posteriormente.
Realizar los test genéticos oportunos para descartar una poliposis colónica familiar entre
2. 2.
sus familiares.
Realizar una TC cerebral, para descartar tumores endocraneales, ya que los pólipos
3. 3.
adenomatosos pueden verse en el síndrome de Turcot.
4. 4. Observación periódica.
Gráfico de respuestas
Comentario

Se trata de un adenoma velloso resecado completamente. El siguiente paso es el seguimiento


endoscópico periódico.(R4)

27. ¿Cuál es el mecanismo fisiopatológico predominante en la bronquiolitis aguda?:

1. 1. Disminución del diámetro de las pequeñas vías aéreas por edema, moco y restos celulares.
2. 2. Disminución del diámetro de las grandes vías aéreas por edema y broncoconstricción.
Disminución de la superficie alveolar ventilada por ocupación por moco, restos celulares y
3. 3.
células inflamatorias.
4. 4. Disminución de la superficie alveolar por colapso por falta de surfactante pulmonar.
Gráfico de respuestas
Comentario

La fisiopatología de la bronquiolitis nunca ha sido preguntada. Durante una infección por VRS se
produce una obstrucción bronquiolar por edema, acúmulo de moco y detritus celulares que
conlleva una disminución del radio de la vía aérea y aumento de la resistencia al paso de aire, pero
disminuye el número de bronquíolos (respuesta 4). La bronquiolitis es una enfermedad de la via
aérea de pequeño calibre y no de las grandes vías aéreas (respuesta 2) ni de los alvéolos
(respuesta 3).(R1)

28. Gestante de 37 semanas a la que se le practica monitorización fetal no estresante con


el siguiente resultado: FCF 135 lpm ondulatoria normal, movimientos fetales escasos y
no ascensos de la FCF, ¿qué actitud tomaría?

1. 1. Cesárea urgente.
2. 2. Amniocentesis tardía.
3. 3. Prueba de POSE.
4. 4. Funiculocentesis.
Gráfico de respuestas
Comentario
 
 
 
 
Se trata de una pregunta sencilla sobre el registro cardiotocográfico en un control de una gestante.
Los parámetros de normalidad son:

- Frecuencia cardiaca fetal entre 120- 160 latidos por minuto.

- Variabilidad de 10-25 lat/min.

- Ascensos o aceleraciones transitorias por encima de 15-20 lat/min.

- Ausencia de desaceleraciones.

En conclusión, el registro de la paciente lo podemos catalogar de RAF negativo, por la falta de


ascensos en la frecuencia cardíaca. La causa más frecuente de esta situación es el sueño fetal. No
obstante, ante esta circunstancia estaría indicada la prueba de Pose (monitorización fetal
estresante), como siguiente paso.(R3)

29. A 5-year-old African child had contact with chickenpox virus during her holidays ten
days ago. Her parents want to know what to do in order to avoid the development of the
illness. What would be the best option in this case?

1. 1. Administer polivalent immunoglobulin


2. 2. Varicela vaccination
Reassure the parents. Explain to them that the child might develop the infection within 10
3. 3.
days
4. 4. Initiate acyclovir
Gráfico de respuestas
Comentario
Reassure the parents. Explain to them that the child might develop the infection within 10 days.
Once infection occurs, there is no effective treatment beyond symptomatic relief. In this case it is
appropriate to warn the parents that the child will suffer the disease. It should be advised not to
expose other unprotected children.(R3)

30. Which of the following is not an indication for dialysis in a patient with chronic renal
failure?

1. 1. Severe renal osteodystrophy.


2. 2. Severe vasculopathy and bleeding diathesis.
3. 3. Uremic pericarditis.
4. 4. Severe peripheral neuropathy.
Gráfico de respuestas
Comentario

Pregunta muy importante no debe olvidarla.

Las indicaciones absolutas de diálisis son:

1. Síntomas urémicos: náuseas, vómitos.

2. Pericarditis urémica.

 
 
 
 
3. Hiperkalemia no por exceso dietético o tratamiento con IECA.

4. Acidosis metabólica severa no controlable con tratamiento médico.

5. Sobrecarga de volumen que no sea por transgresión dietética (sal).

6. Edema pulmonar no debido a insuficiencia cardíaca descompensada y que no responda a


medidas terapéuticas simples.

7. Desnutrición: albúmina inferior a 4 g que no sea por pérdidas urinarias.

8. Encefalopatía urémica: desorientación, confusión, asterixis, mioclonías o crisis convulsivas.

9. Tendencia hemorrágica con sangrado atribuible a uremia.

10. HTA que no responda bien al tratamiento.

11. Aclaramiento de creatinina 5 ml/min o concentración de creatinina superior a 12 mg% con peso
> 70 kg (10 ml/min para pacientes diabéticos).

Pero también hay indicaciones relativas:

A. Sintomatología digestiva: anorexia, náuseas, vómitos.

B. Neurológica: tendencia al sueño, neuropatía periférica.

C. Dermatológicas: prurito.

D. Osteodistrofia renal severa.


(R2)

31. A 18-day-old newborn is brought to the doctor's for a routine visit. The girl is healthy,
and has a good appetite. Physical development is adequate. Her mother followed all
scheduled obstetric visits and instructions during pregnancy. Physical examination is
unremarkable, but a click in the hip is perceived in Barlow maneuver. What should be
done next in order to make an adequate diagnosis?

1. 1. Ultrasound of the hips


2. 2. MRI of the pelvis
3. 3. Surgical examination
4. 4. X ray of the hips
Gráfico de respuestas
Comentario
Ultrasound of the hips. The child described in this clinical case has a high suspicion of congenital
hip dysplasia. Most countries have standard newborn exams that include a hip joint exam screening
for early detection of hip dysplasia. Sometimes during an exam a "click" or more precisely "clunk" in
the hip may be detected (although not all clicks indicate hip dysplasia). The condition can be
confirmed by ultrasound, which is the best test at this age since the cartilage is not yet ossified.(R1)

 
 
 
 
32. Respecto al neumotórax, señale la opción FALSA:

1. 1. Es la introducción de aire en el espacio pleural.


La repercusión clínica depende de la reserva funcional ventilatoria del paciente y del
2. 2.
grado de colapso pulmonar.
La causa más frecuente de neumotórax espontáneo secundario es la ruptura de bullas
3. 3.
subpleurales apicales.
4. 4. El diagnóstico se confirma con radiografías de tórax en inspiración y espiración forzada.
Gráfico de respuestas
Comentario

El neumotórax es muy importante de cara al MIR. El neumotórax se define como la entrada de aire
en el espacio pleural, que despega ambas superficies serosas y provoca un colapso pulmonar. Se
distinguen varios tipos, como el traumático, que a su vez puede ser abierto (es típico el bamboleo
mediastínico) o cerrado; el espontáneo, que puede ser primario, típico del varón joven, alto,
delgado y fumador, y que se debe a la ruptura de bullas apicales subpleurales (por eso la opción 3
es incorrecta, no se relaciona con la forma secundaria) y que recidiva en la mitad de los casos y el
espontáneo secundario a enfermedad pulmonar previa, sobre todo EPOC y silicosis; el neumotórax
a tensión que se debe a un mecanismo valvular que permite la entrada pero no la salida de aire y
que requiere la rápida descompresión de la cavidad pleural; y el neumotórax catamenial, típico de
mujeres mayores de 25 años, en relación con la menstruación y que recidiva con frecuencia. Se
debe sospechar ante la presencia de dolor agudo y disnea, y se confirma mediante una radiografía
de tórax en inspiración y espiración máxima.(R3)

33. Hombre de 50 años


hipertenso y diabético no insulindependiente que acude a urgencias por nauseas y
vómitos alimentario-biliosos, distensión abdominal y dolor abdominal cólico
generalizado. Laboratorios en urgencias: Hb 16,3 g/dl, VCM 86 fl, 16.500 leucocitos,
plaquetas 325.000/mm3, INR 0,9, bilirrubina 1,2, AST 52, ALT 32, GGT 73, amilasa 46 UI/ml,
glucemia 86, urea 88, creatinina 1,6, sodio 149, potasio 4,3. Se realiza una Rx de abdomen

 
 
 
 
que muestra la imagen adjunta. ¿Cuál de entre las siguientes sería la actitud MENOS
adecuada?

1. 1. Solicitar un ultrasonido abdominal.


2. 2. Dieta absoluta.
3. 3. Reposición hidroelectrolítica.
4. 4. Pautaría analgésicos.
Gráfico de respuestas
Comentario

El cuadro clínico es sugestivo de obstrucción intestinal, lo mismo que la radiografía adjunta, donde
se aprecian claros niveles hidroaéreos. La causa más frecuente de obstrucción intestinal en la
población general son las adherencias postquirúrgicas. En este caso, no nos hablan de
antecedentes de cirugía abdominal, por lo que posiblemente la causa sea una hernia, que es lo
habitual en personas no sometidas a laparotomía. Teniendo esto en cuenta, al menos de inicio no
sería necesario realizar un ultrasonido abdominal, sino una exploración física más completa (ingles,
escroto, etc.) buscando una posible hernia.

El 90% de las obstrucciones de intestino delgado se resuelven con sonda nasogástrica, dieta
absoluta y reposición hidroelectrolítica. La cirugía será necesaria si sospechamos estrangulación,
el dolor o la fiebre van en aumento, o no se resuelve en 3- 4 días. Podemos, además, administrar
analgesia. Ten cuidado con la antigua creencia de que no se debe dar analgesia ante un cuadro de
abdomen agudo, porque esta idea se considera hoy día completamente falsa.(R1)

34. Tras ligera mejoría con el objeto de filiar la etiología, realiza una colonoscopía donde
se observa una mucosa de recto normal y al llegar a sigma, abundantes divertículos
rodeados por una mucosa intensamente hiperémica, edematosa y con algunos restos de
secreción purulenta. ¿Cual sería su actitud a continuación?

1. 1. Procedería a retirar el colonoscopio inmediatamente.


Continuaría hasta ciego para descartar enfermedad de Crohn que justifique ese patrón
2. 2.
mucoso.
3. 3. Continuaría por lo menos hasta ángulo esplénico para ver la extensión de la enfermedad.
Trataría de llegar hasta ángulo hepático dada la alta probabilidad de que se trate de una
4. 4.
colitis isquémica cuyas lesiones más graves se observarían en colon transverso.
Gráfico de respuestas
Comentario

La descripción que nos hacen de la colonoscopía se corresponde con la de una diverticulitis aguda.
En esta entidad, está contraindicada la colonoscopía porque implica riesgo de perforación, por lo
que la respuesta correcta sería la 1.

En la diverticulitis aguda, el cuadro típico consiste en un dolor abdominal en fosa ilíaca derecha en
una persona mayor, normalmente con fiebre y leucocitosis. En este caso, queda claro que la
imagen que hemos encontrado en la radiografía de abdomen no se corresponde, en realidad, con
una auténtica obstrucción intestinal. En el contexto de una diverticulitis, se justifica por el íleo
adinámico que puede acompañar a esta patología. En un íleo, cuando se realiza la Rx en
bipedestación, podemos encontrar dilatación de las asas y niveles hidroaéreos, sin que
necesariamente exista una verdadera obstrucción mecánica, como es el caso.(R1)

 
 
 
 
35. Femenino posmenopáusica fue tratada de un carcinoma mamario infiltrante de 1 cm
mediante tumorectomía y linfadenectomía. Esta última fue negativa. ¿Qué tratamiento
añadiría?:

1. 1. Ovariectomía bilateral.
2. 2. Monoquimioterapia con taxol.
3. 3. Poliquimioterapia.
4. 4. Radioterapia y tamoxifeno.
Gráfico de respuestas
Comentario

Sobre esta pregunta, debemos hacer una serie de observaciones. Durante los últimos años, han
cambiado algunos conceptos sobre el cáncer de mama.

Ante un tumor tan pequeño, puede plantearse una cirugía conservadora en vez de una
mastectomía radical, con iguales resultados en términos de supervivencia. Pero debe quedar clara
una cosa:para que los resultados sean los mismos, es preciso complementar la tumorectomía con
radioterapia.

No está indicada la quimioterapia, puesto que los ganglios axilares son negativos y no existe
ningún tipo de factor de mal pronóstico. Recuerde queel factor pronóstico más importante en el
cáncer de mama es el número de ganglios afectados, y por ello la indicación más típica de
quimioterapia.

La hormonoterapia, actualmente, se reserva para las pacientes con receptores estrogénicos


positivos. Como en esta paciente no nos dicen nada de esto, la pregunta no está bien construida.

Hasta hace poco, se trataba con hormonoterapia a todas las pacientes postmenopáusicas,
independientemente de la positividad de los receptores estrogénicos. Hoy día, para indicar
hormonoterapia (tamoxifeno u otros) habría que conocer la presencia o ausencia de receptores
estrogénicos en el tumor, dato que no nos ofrecen.(R4)

36. Entre otras manifestaciones, la talla baja está presente en múltiples síndromes
genéticos y polimalformativos, como los que se exponen a continuación, EXCEPTO en
uno. ¿Cuál de ellos NO suele cursar con talla baja?

1. 1. Síndrome de Turner.
2. 2. Síndrome de Down.
3. 3. Síndrome de Silver-Russell.
4. 4. Síndrome de Klinefelter.
Gráfico de respuestas
Comentario

Una pregunta relativamente sencilla sobre la talla baja, ya que el síndrome de Klinefelter produce
precisamente lo contrario: talla alta, sobre todo a expensas de la mitad inferior del cuerpo.

Para acertar esta pregunta, en realidad no era necesario conocer rarezas como las que enuncian
las respuesta 3.

 
 
 
 
El síndrome de Silver-Russell, que es causa de talla baja, también se asocia con malformaciones
esqueléticas (asimetría) y alteraciones morfológicas del cráneo.

Los síndromes de Down y Turner son más conocidos como causas de talla baja. Recuerde que,
por el contrario, el síndrome de Klinefelter (XXY), igual que el síndrome XYY, produce talla
alta.(R4)

37. A 4-year-old girl is brought to consultation by her anxious mother who has noted that
she is getting swollen. The pediatrician reassures her and starts the examination of the
child. Physical examination confirms the presence of generalized edema. Blood
laboratory test results are: Sodium 142 mEq/L, Potassium 4 mEq/L, Albumin 2 g/dL,
Globulin 5 g/dL, Creatinine 0.96 mg/dL. Urinalysis reveals proteinuria 4+. What should be
done next?

1. 1. Perform a renal biopsy


2. 2. Autoimmune serological tests
3. 3. Renal ultrasound
4. 4. Start Oral Prednisone
Gráfico de respuestas
Comentario
Start oral prednisone. This child edema is due to hypoalbuminemia caused by renal losses, as
verified in the urine sediment. The most common cause of nephrotic syndrome in children is
minimal changes nephropathy. The biopsy would show absence of glomerular lesions, as opposed
to other glomerulonephritis. The biopsy is rarely done. The best attitude is to start treatment with
steroids, since the response is good in most cases. Biopsy is only performed if evolution is torpid or
worsening despite therapy or if there are increased creatinine levels.(R4)

38. ¿Cuál de las siguientes respuestas NO es una indicación de inducción de parto?:

1. 1. Gestación a termino con rotura prematura de membranas de más de 12 horas.


2. 2. Feto pequeño para edad gestacional hemodinámicamente estable.
3. 3. Muerte fetal intraútero.
4. 4. Gestación cronológicamente prolongada.
Gráfico de respuestas
Comentario
La corioamnionitis es criterio de inducción por el elevado riesgo de sepsis neonatal, así como el
riesgo materno. En una gestación a término con 12 o 18 horas de bolsa rota y que no se a puesto
de parto espontáneamente, el riesgo de infección supera los beneficios de esperar a que se ponga
de parto espontánemente, así que se debe proceder a la inducción/estimulación según condiciones
del cervix. La muerte fetal intraútero también cursa con elevado riesgo materno como puede ser
una coagulación intravascular diseminada, entre otras, por lo que se ha de inducir. A partir de la
semana 42 se debe proceder a la inducción, aunque algunos autores lo recomiendan desde la
semana 41. Por el contrario un feto pequeño hemodinámicamente estable (es decir, sano) no
cumple ningún criterio de inducción, beneficiándose de continuar madurando y deberá ser inducido
si se produce un estancamiento en el crecimiento fetal, signos de sospecha de pérdida de
bienestar fetal o alteraciones hemodinámicas (estudio doppler).(R2)

39. La amenorrea con infantilismo sexual, obesidad, déficit mental, retinitis pigmentaria,
polidactilia e hipertricosis aparece en:

 
 
 
 
1. 1. Síndrome de Turner.
2. 2. Síndrome de Laurence-Moon-Bield.
3. 3. Síndrome de Kallman.
4. 4. Síndrome de Savage.
Gráfico de respuestas
Comentario

Es una pregunta muy difícil de un síndrome que prácticamente no debe conocer, porque es muy
improbable que una pregunta como esta pueda salir en el ENARM.

Turner (45X0 , 46 XX y mosaicos con talla corta, coartación de aorta, cubitus valgus...), el sindrome
de Kallman (causa de amenorrea central con atrofia del bulbo olfatorio e infantilismo sexual y
gonadotropinas descendidas con cariotipo femenino o masculino) que le permitan sacar la
pregunta por descarte a que sepa que el Laurence-Moon-Bield es una causa de amenorrea central
que cursa con diabetes, oligofrenia e hipogonadismo.(R2)

40. Which of the following hormones binds to a 7-transmembrane or serpentine receptor?

1. 1. Insulin.
2. 2. TSH.
3. 3. Prolactin.
4. 4. Glucocorticoids.
Gráfico de respuestas
Comentario
Las hormonas ejercen sus acciones mediante la unión a distintos receptores de membrana,
citosólicos o intranucleares. Entre los receptores de membrana, existen hormonas que presentan
receptores de 7 dominios, unidos a proteínas G que conllevan la generación de AMP cíclico, como
PTH, ACTH, TRH, TSH, somatostatina y glucagón. La insulina actúa sobre receptores unido a una
tirosin quinasa, la GH y prolactina a través de receptores de citocinas, y los glucocorticoides
mediante receptores citosólicos.(R2)

41. El aumento del estridor en el recién nacido cuando esta en decúbito supino sugiere
la existencia de:

1. 1. Síndrome obstructivo bronquial.


2. 2. Laringotraqueitis.
3. 3. Laringomalacia.
4. 4. Fistula traqueoesofágica.
Gráfico de respuestas
Comentario

La laringomalacia es más frecuente en menores de 2 años (70% promedio de los casos de


estridor), es más frecuente en el sexo masculino con una relación de 2:1.

Se debe a un defecto intrínseco o a la maduración retardada de las estructuras que soportan la


laringe. La vía aérea se obstruye parcialmente durante la inspiración por colapso de pliegues
aritenopiglóticos flácidos, aritenoides y epiglotis (grados I-III según Olney o tipo I-V según Holinger
), lo cual empeora en posición supina, con el llanto o agitación y en relación a infecciones de la vía
aérea superior. Su evolución es benigna y autolimitada.(R3)

 
 
 
 
42. Which of the following patients should not take oral contraceptives?

1. 1. 36-year-old woman. Healthy. Non-smoker.


2. 2. 15-year-old woman with polymenorrhea.
30-year-old woman with a past history significant for adnexectomy due to ovarian
3. 3.
endometrioma.
4. 4. 22-year-old with familial hypertriglyceridemia.
Gráfico de respuestas
Comentario

Es muy importante que conozcan el mecanismo de acción, los efectos secundarios y las
contraindicaciones de los anticonceptivos orales de cara al ENARM pues han sido preguntados en
diversas ocasiones.

Las contraindicaciones absolutas son pacientes con riesgo cardiovascular (fumadoras mayores de
35 o no fumadoras mayores de 40), antecedentes de enfermedad tromboembólica, HTA mal
controlada, diabetes con afectación vascular, vasculopatía inflamatoria, cardiopatías graves,
pacientes con afectación hepática importante (adenoma hepático o hepatopatías activas), pofiria
aguda intermitente, antecedentes de ictericia durante el embarazo (colestasis intrahepática),
cáncer de mama u otros tumores hormonodependientes, discrasia sanguínea y sangrado genital
anormal no filiado.(R4)

43. Niña de 10 años que acude a revisión de salud. La exploración física es normal, con
un peso y una talla en el percentil 50 y TA 109/65. Un análisis de orina de rutina muestra
una densidad específica de 1.035, pH 6, sangre 2+, sin proteínas. El sedimento urinario
muestra de 5-10 hematíes por campo. ¿Cuál sería la conducta más apropiada a seguir?

1. 1. Determinar creatinina y nitrógeno en sangre.


2. 2. Determinar anticuerpos antinucleares y complemento.
3. 3. Volver a repetir el sedimento de orina en 15 días.
4. 4. Realizar una tomografía axial computarizada abdominal.
Gráfico de respuestas
Comentario

Caso clínico en el que les presentan una niña de 10 años con exploración física, talla y TA
normales, el análisis de orina tiene una densidad y un pH dentro de los valores normales, el único
dato alterado es el número de hematíes por campo (normal 0-4).

Puesto que no tiene ningún síntoma (análisis de orina de rutina), crece correctamente y no indican
ninguna otra alteración en el sedimento, carecen de sentido las opciones 1, y 2 , por tanto, la
conducta inicial más adecuada sería repetir el sedimento en 15 días, ya que el aumento de
hematíes podría ser debido, por ejemplo, a la realización de ejercicio físico intenso, algo muy
común en esta edad.(R3)

44. ¿Cuál de los siguientes NO es criterio de carcinoma colorrectal hereditario no


polipósico?

1. 1. Cáncer de colon demostrado histológicamente en al menos 3 familiares.


2. 2. Cáncer de colon que afecta a dos generaciones de la familia.
3. 3. Mutación en hMSH2.
4. 4. Uno de los miembros afectos de la familia ha de ser menor de 50 años.

 
 
 
 
Gráfico de respuestas
Comentario

Los criterios diagnósticos del síndrome de Lynch (carcinoma colorrectal no asociado a poliposis)
son clínicos, no genéticos. Aunque ya conocemos algunas de las mutaciones que pueden estar
detrás de esta entidad, todavía existen otras que no conocemos, por lo que la definición, hoy por
hoy, sigue basándose en datos como los que enuncian las opciones 1, 2 y 4.

Aprovechamos para recordar que, en el síndrome de Lynch tipo II, el cáncer de colon se asociaría
a otras neoplasias (ovario, endometrio...) como se ha preguntado en alguna ocasión.(R3)

45. RN de bajo peso para su edad gestacional que a las 96 h de vida comienza con llanto
agudo persistente, rechazo del alimento, irritabilidad, hipertonía y convulsiones.
Presenta rinorrea y obstrucción nasal, así como una frecuencia respiratoria de 55
respiraciones/min y una sudoración excesiva. Entre las actitudes terapéutico-
diagnósticas a tomar ante este caso, NO se encuentra:

1. 1. Administración de fenobarbital.
2. 2. Administración de clorpromacina.
3. 3. Solicitar opiáceos en orina materna.
4. 4. Administración de naloxona.
Gráfico de respuestas
Comentario

Ante la presencia de hiperexcitabilidad neurológica extrema en un recién nacido (llanto agudo,


polifagia, polipnea, taquicardia, convulsiones) junto con síntomas difusos (rinorrea, diarrea), ha de
pensar en un síndrome de abstinencia a opiáceos. Lógicamente, no se podría administrar
naloxona, porque es el antagonista de los opiáceos y produciría un síndrome de abstinencia mucho
mayor. La naloxona puede ser útil en el tratamiento de la intoxicación, pero no en la
abstinencia.(R4)

46. Los lactantes postérminos posmaduros se caracterizan por todo lo siguiente,


EXCEPTO:

1. 1. Lanugo.
2. 2. Pelo abundante en la cabeza.
3. 3. Mayor capacidad de alerta.
4. 4. Pérdida de tejido subcutáneo.
Gráfico de respuestas
Comentario

Pregunta de dificultad intermedia que habíamos repasado previamente. Los lactantes postérmino
tienen la piel seca y apergaminada, por esta pérdida de agua en la piel se produce una disminución
del espesor del tejido celular subcutáneo( opción 4 correcta). Presentan también signos de
hipermadurez , como las uñas largas y abundante pelo en la cabeza, y mayor capacida de alerta.
La piel de los recién nacidos pretérmino la piel es delgada y suave y puede estar recubierta de un
vello escaso y fino denominado lanugo, que desaparece en unas semanas ( opción 1 incorrecta).
La piel del recién nacido a término tiene un mayor espesor y esta cubierta por vérnix caseosa, una
especie de crema blanquecina con misión protectora).(R1)

47. NO es correcto en el sarcoma de Kaposi asociado al SIDA:

 
 
 
 
1. 1. Puede producirse con un número normal de CD4.
2. 2. La afección pulmonar se suele asociar a derrame pleural.
3. 3. La afección de ganglios linfáticos implica mal pronóstico.
4. 4. Puede manifestarse como ictericia obstructiva.
Gráfico de respuestas
Comentario
El sarcoma de Kaposi se saba que está producido por el virus herpes humano tipo 8. Aparece en
los pacientes VIH y en general puede aparecer con cualquier cifra de CD4. Las lesiones más
habituales son las mucocutáneas, aunque puede existir afectación visceral, dentro de ellas es el
tubo digestivo uno de los más habituales. Actualmente para su tratamiento lo más útil es conseguir
revertir la situación de inmunodeficiencia y por ello el tratamiento antirretroviral es fundamental.(R3)

48. Which of the following is not a contraindication for intrauterine device (IUD) insertion?

1. 1. Congenital uterine anomalies.


2. 2. Submucous myomas.
3. 3. Breast cancer.
4. 4. Copper allergy.
Gráfico de respuestas
Comentario
Toda distorsión de la cavidad uterina provocada tanto por malformaciones congénitas como por
miomas submucosos, dificultan la inserción del dispositivo y aumentan la posibilidad de perforación
uterina o expulsión del mismo. El dispositivo más ampliamente utilizado con fin anticonceptivo es el
denominado T de Cobre por lo que aquellas pacientes con alergia a este metal o con enfermedad
de Wilson tienen contraindicado su uso. Los efectos adversos más frecuentes en las usuarias de
DIU son las hemorragias genitales y el dolor pélvico, por lo que pacientes con dismenorrea no
deben utilizarlo. Debido a que el DIU no produce ninguna alteración hormonal no tiene ninguna
influencia sobre la evolución de un cáncer de mama.(R3)

49. Respecto a las nuevas vacunas conjugadas frente a neumococo, señale la opción
FALSA:

La vacuna 13-valente puede administrarse en primovacunación si ésta ya se inició con la


1. 1.
vacuna 7-valente.
La vacuna 10-valente puede administrarse como refuerzo a los 12-18 meses si la
2. 2.
primovacunación se realizó con vacuna 7-valente.
La vacuna 13-valente puede administrarse como refuerzo a los 12-18 meses si la
3. 3.
primovacunación se realizó con vacuna 7-valente.
4. 4. Las vacunas 10-valente y 13-valente están aprobadas para niños menores de 5 años.
Gráfico de respuestas
Comentario

Pregunta difícil.

La vacuna antineumocócica 10-valente sólo está aprobada para menores de 2 años, mientras que
la 13-valente puede emplearse en menores de 5 años.(R4)

50. Señale la respuesta FALSA en relación a las deceleraciones del registro


cardiotocográfico:

 
 
 
 
Para que puedan ser consideradas como tales deben tener una duración mayor de 15
1. 1.
segundos y ser de más de 15-20 lpm de amplitud.
Las deceleraciones tardías son como imágenes especulares de las contracciones uterinas,
2. 2.
que suceden 40-50 segundos después de la contracción.
3. 3. Las deceleraciones tardías suelen ir precedidas de aceleraciones.
4. 4. Las deceleraciones variables suelen tener un principio y un fin bruscos.
Gráfico de respuestas
Comentario

Conceptualmente, una deceleración es un descenso de la línea de base de más de 15 lpm,


durante más de 15-20 segundos. Las deceleraciones NO deben estar presentes en condiciones
normales. No obstante, el significado de cada tipo de deceleración es absolutamente distinto. Debe
conocer tres:

-Deceleraciones precoces (DIPS tipo I): Aparecen a la vez que la contracción uterina. Son las más
frecuentes. Suelen deberse a la estimulación vagal, por compresión de la cabeza fetal.

-Deceleraciones tardías (DIPS tipo II): Existe un decalaje de unos 20 segundos entre la contracción
uterina y la deceleración. Orientan a acidosis fetal (peor pronóstico). Si ve DIPS tipo II, debe
realizar una microtoma fetal para medir el pH. No van precedidas de aceleraciones (respuesta 3
falsa).

-Deceleraciones variables (DIPS umbilicales): Son inconstantes en morfología y sincronía.


Sugieren patología de cordón, y el pronóstico es intermedio entre las I y II. Tienen un comienzo y
fin muy brusco, con una morfología típica en W.(R3)

51. Una de las siguientes expresiones es INCORRECTA para Chlamydia:

1. 1. Son microorganismos bacterianos.


2. 2. Tienen ácido ribonucleico y desoxirribonucleico.
3. 3. Son parásitos intracelulares.
4. 4. Producen ATP.
Gráfico de respuestas
Comentario
Pregunta díficil sobre la microbiología de las clamidias, tema infrecuente en el examen MIR. Las
clamidias son microorganismos bacterianos, de crecimiento intracelular, que contienen ADR y
ARN. Se reproducen por fisión binaria, y a diferencia de las rickettsias, no producen energía (
opción 4 falsa) ni poseen pared rígida. Además, como dice la opción 5, producen cuerpos de
inclusión en células eucarióticas huéspedes.(R4)

52. Paciente de 42 años de edad con dolor abdominal localizado en fosa ilíaca derecha
(FID) de 5 días de evolución. Refiere anorexia, vómitos biliosos ocasionales, escalofríos
y sudoración intensa. En FID existe defensa y se palpa una masa dolorosa, resto del
abdomen es normal. El tacto rectal es muy doloroso en zona derecha de pared rectal. La
temperatura es de 38,7ºC. BH: 23.500 leucocitos (88% segmentados, 15 cayados). La RX
de abdomen y tórax es normal. La siguiente exploración indicada para el diagnóstico del
cuadro sería:

1. 1. Enema opaco.
2. 2. USG abdominal.
3. 3. TC abdominal.

 
 
 
 
4. 4. Colonoscopía.
Gráfico de respuestas
Comentario

Pregunta importante, puesto que de cara al examen lo que más se ha preguntado es el diagnóstico
de la apendicitis aguda. El diagnóstico de este cuadro se hace fundamentalmente por la clínica y la
BH y sólo en casos dudosos se recurren a técnicas complementarias, siendo el USG de primera
elección por su inocuidad y por ser una técnica no invasiva.(R2)

53. ¿Cuál de los siguientes signos NO suele aparecer en la clínica del cáncer de mama?:

1. 1. Mastodinia.
2. 2. Telorragia.
3. 3. Tumoración.
4. 4. Retracción del pezón.
Gráfico de respuestas
Comentario

El cáncer de mama tiene distintas presentaciones como retraccion del pezón, piel de naranja,
telorragia, palpación de tumoraciones, adenomegalias, pero no mastodinia, respuesta 1, ya que
este signo es dolor mamario ciclico, que se ve en la enfermedad fibroquística mamaria.(R1)

54. La localización más frecuente de los tumores del SNC en la infancia es:

1. 1. Infratentorial.
2. 2. Supratentorial.
3. 3. Intramedular.
4. 4. No existe una localización más frecuente.
Gráfico de respuestas
Comentario

La localización más frecuente de los tumores del SNC en la infancia es infratentorial (respuesta 1
correcta) siendo más frecuente en los adultos la localización supratentorial.(R1)

55. Un enfermo padece una GN donde la IF descubre C3 en pared capilar y nódulos


mesangiales. En el ME aparecen depósitos intramembranosos y asimismo tiene
hematuria, C3 persistentemente disminuido en suero y factor nefrítico C3 +. Indica la
afirmación más probable en relación a esta nefropatía:

1. 1. Se suelen detectar grandes cantidades de IgG con IF.


2. 2. La lipodistrofia parcial es una enfermedad asociada a veces.
3. 3. Si los depósitos fueran subendoteliales, tendría peor pronóstico.
4. 4. Su recurrencia en el riñón trasplantado es excepcional.
Gráfico de respuestas
Comentario

La glomerulonefritis mesangiocapilar (GNMC) se caracteriza por asociar una proliferación de las


células del mesangio y un engrosamiento de las paredes capilares del glomérulo. Esta GN recibe a

 
 
 
 
menudo otros nombres, como glomerulonefritis membranoproliferativa, glomerulonefritis lobular y
glomerulonefritis crónica hipocomplementémica.

Los principales rasgos distintivos de esta afección son el descubrimiento habitualmente antes de
los 30 años, la frecuencia e intensidad de la hipocomplementemia, la posible asociación con
lipodistrofia parcial, la frecuencia con que el síndrome nefrótico se acompaña de hematuria,
hipertensión y déficit de la función renal, el carácter excepcional de las remisiones y la progresión
del curso clínico, con una evolución a la insuficiencia renal terminal.

Aunque las manifestaciones clínicas son muy parecidas, la GNMC se clasifica en tipo I, cuando los
depósitos que engrosan las paredes capilares son subendoteliales y el complemento se activa por
la vía clásica; y tipo II, cuando se localizan en la propia lámina densa de la MBG (“enfermedad por
depósitos densos”). En este segundo tipo, el complemento se activa por la vía alterna, mediante el
factor nefrítico o C3NeF, mucho más frecuente en la tipo II (la opción 1 es la falsa).

Algunos autores han descrito un tipo III, que asocia depósitos subepiteliales y espículas (como las
de la nefropatía membranosa) a las lesiones propias de una GNMC tipo I.(R2)

56. Mujer de 29 años, con endometriosis ovárica diagnosticada hace 1 año y en


tratamiento con ACO. En esta revisión presenta un marcador tumoral de Ca 125=
35mUI/ml (0.0-35.0 mUI/ml) y por ultrasonido transvaginal visualizamos un quiste
homogéneo ecomixto de punteado fino, de 28 mm, en ovario derecho y otro de 35 mm en
ovario izquierdo. Actualmente no desea gestación y la paciente refiere tener una
dismenorrea leve tolerable. ¿Qué actitud terapéutica consideraría la más correcta?:

1. 1. Quistectomia ovárica bilateral por laparotomía.


PAAF de ambos quistes ováricos para tener un resultado anatomopatológico por marcador
2. 2.
tumoral límite.
3. 3. Seguiría con el mismo tratamiento ACO.
4. 4. Cambiaría el tratamiento a análogos de la GnRH.
Gráfico de respuestas
Comentario

Dada la estabilidad clínica de la paciente y que los quistes endometriósicos son de pequeño
tamaño (<4 cm) el tratamiento quirúrgico no parece la mejor alternativa.

El marcador Ca125 en el límite alto de la normalidad no justifica la necesidad de obtener un


resultado anatomopatológico.

Dentro del tratamiento médico tanto los análogos de la GnRH como los ACO son opciones válidas.
Ya que la paciente tolera bien los anticonceptivos y no existe clínica, en principio no parece
justificado cambiar de fármaco.(R3)

 
 
 
 

57. Una mujer de 84 años acude a nuestra


consulta por dolor crónico, a veces intenso en la zona media de la columna vertebral
torácica, que aumenta al sentarse y levantarse. A la exploración se aprecia marcada
cifosis y contractura de la musculatura paravertebral. También se palpa una masa pulsátil
en el epigastrio. La radiografía se muestra en la imagen. ¿Cuál es la causa más probable
del dolor?

1. 1. Espondiloartrosis.
2. 2. Hiperostosis anquilosante esquelética difusa.
3. 3. Osteomalacia.
4. 4. Osteoporosis con aplastamiento vertebral.
Gráfico de respuestas
Comentario

Aunque nos hablen de una masa pulsátil en epigastrio, que tal vez te haya hecho pensar en un
posible aneurisma aórtico, difícilmente podemos justificar un dolor vertebral crónico a partir de este
hallazgo. Afortunadamente, el autor de la pregunta no ha tenido la maldad de incluir la respuesta
“aneurisma aórtico” entre las opciones. Posiblemente lo tenga, eso habría que confirmarlo, pero lo
que tienes que tener claro es que, a los 84 años, la respuesta más probable un aplastamiento
vertebral asociado a osteoporosis. De hecho, es lo que se puede ver en esta radiografía de tórax
lateral (respuesta 4 correcta).(R4)

58. Los pólipos benignos más frecuentes en el intestino son:

1. 1. Pólipos inflamatorios.
2. 2. Lipomas.
3. 3. Pólipos hiperplásicos.

 
 
 
 
4. 4. Adenomas.
Gráfico de respuestas
Comentario

Una pregunta que se presta poco al posible razonamiento, ya que se trata de un dato puntual. Tal
como dice la respuesta 3, los pólipos benignos más frecuentes son los hiperplásicos. Recuerde
que, desde el punto de vista del potencial maligno, los que deberían preocuparnos son los
adenomatosos.(R3)

59. En el recién nacido señale que reflejos deben estar presentes al nacimiento:

1. 1. Reflejo tónico cervical y succión.


2. 2. Reflejo de marcha y moro.
3. 3. Reflejo de moro y succión.
4. 4. Reflejo de paracaídas y de moro.
Gráfico de respuestas
Comentario

Los reflejos en general tienen un valor adaptativo; así, las funciones más habituales de los reflejos
son:

• Supervivencia: respirar, tragar, rotación y natación.

• Protección frente a estimulación no deseada: parpadeo, de retirada.

• Reducir tensión: succión.

• Supervivencia en un pasado evolutivo (sin utilidad actual): reflejo de Moro, abrazo.

• Establecer interacción con padres/cuidadores: rotación, succión, prensión.

• Sin función clara: Babinski.

La mayor parte de estos desaparecen a partir de los 6 meses, ya que el control voluntario asociado
a la maduración cerebral progresa.

El reflejo del moro y el de succión deben estar presentes en la exploración del recién nacido.(R3)

60. Con respecto a la ictericia neonatal, es FALSO:

1. 1. Toda ictericia dentro del primer día de vida debe ser considerado patológico.
2. 2. Al realizarse una exanguineotransfusión existe un rebote a los 30 minutos.
El diagnóstico de ictericia secundaria a leche materna se realiza midiendo la bilirrubina en
3. 3.
la leche de la madre.
4. 4. El manejo primario de la ictericia neonatal es la fototerapia contínua.
Gráfico de respuestas
Comentario

Pregunta fácil, que al leerla con detenimiento puede llegar a la respuesta corecta.

 
 
 
 
Respuesta 1: ya sabemos que toda ictericia en el primer día de vida es patológica.

Respuesta 2: supongamos que no tenemos ni idea de las complicaciones de


exanguineotransfusión por lo que la dejamos pendiente.

Respuesta 3: ¿el diagnóstico de ictericia por leche materna se realiza midiendo la bilirrubina en la
leche materna? ¿No suena ilógico medir bilirrubina en la leche materna?

Respuesta 4: no todos los pacientes requieren de fototerapia, existen indicaciones muy precisas de
su uso, pero en caso de requerir tratamiento el primer paso sería este.

Con todo el análisis de las respuestas, claramente la opción que debe elegir es la número 3.(R3)

61. Niña de 7 años que, desde hace varios días, presenta bruxismo y se despierta por la
noche en numerosas ocasiones refiriendo prurito anal. En la exploración se aprecia
irritación perianal y vaginal, con discreto flujo vaginal. ¿Qué exploración complementaria
realizaría en primer lugar?

1. 1. Hemograma con recuento de eosinófilos.


2. 2. Test de Graham o del celofán.
3. 3. Frotis vaginal.
4. 4. Coprocultivo.
Gráfico de respuestas
Comentario

Pregunta relativamente fácil sobre el diagnóstico del Enterobius vermicularis (oxiuros), la


parasitosis de mayor incidencia en España, y por tanto tema muy preguntable.

Si bien diagnosticamos muchos parásitos intestinales, en este caso helmintos, por medio del
estudio de las heces, el examen de la cinta adhesiva perianal (test de Graham) es de uso casi
exclusivo para los oxiuros. Recuerda que estos gusanos residen en el colon ascendente, y que la
hembra migra hasta la región perianal durante las noches para depositar allí sus huevos. Es
característico el gran prurito que genera en esta zona (también puede asociar bruxismo).(R2)

62. A 17-year-old boy who was diagnosed with Crohn disease 2 years ago presents to his
physician with a 5-day history of tender red lumps on both shins. He also says that he
has worsening diarrhea and abdominal pain. Current medications include azathioprine
and over-the-counter analgesics. Which of the following is the most appropriate
management?

1. 1. Bedrest, wet dressings and antidepressant drugs.


2. 2. Biopsy of healthy skin and directly prescribe opioid analgesics.
3. 3. Suspect the presence of intestinal cancer as the cause of the skin lesions.
4. 4. Optimize the treatment of the inflammatory bowel disease.
Gráfico de respuestas
Comentario
El eritema nodoso, es un tipo característico de lesión de la piel que consiste en la presencia de
nódulos dolorosos que presentan signos inflamatorios (enrojecimiento y aumento de temperatura) y
afectan predominantemente a la zona pretibial (delante de la tibia), en las extremidades inferiores,
aunque puede tener otras localizaciones. Las lesiones suelen desaparecer en un plazo de entre 1 y
3 semanas. La

 
 
 
 
existencia de lesiones cutáneas, diarrea y dolor es sugestivo de brote de su enfermedad de Crohn,
por lo que es necesario ajustar el tratamiento de la misma e iniciar tratamiento con esteroides.(R4)

63. Un RN ha sido intubado en la sala de partos por presentar apnea desde el nacimiento.
La madre había recibido un derivado opiáceo. ¿Cuál de los siguientes fármacos y vía de
administración utilizaría de 1ª elección?:

1. 1. Naloxona por vía intratraqueal.


2. 2. Flumacenil por vía intravenosa.
3. 3. Adrenalina por vía intratraqueal.
4. 4. Bicarbonato sódico por vía intravenosa.
Gráfico de respuestas
Comentario

Una causa de depresión respiratoria neonatal es la administración de opiáceos como anestésico a


la madre. La depresión respiratoria provocada se reversibilizará a través de la administración al
recién nacido de naloxona. La naloxona, antagonista opiáceo, se puede administrar por vía
intratraqueal aprovechando la intubación del niño (respuesta correcta 1).(R1)

64. Señale cuál de las siguientes características o acciones fisiológicas NO es atribuible


a la progesterona:

1. 1. Es sintetizada en el cuerpo lúteo.


2. 2. Su aumento súbito condiciona la aparición de la menstruación.
3. 3. Es segregada por las células trofoblásticas de la placenta.
4. 4. Posee acción antialdosterónica.
Gráfico de respuestas
Comentario

La progesterona es sintetizada en el cuerpo lúteo materno durante las 7-10 primeras semanas y es
fundamental para la gestación, y a partir de la 10-12 semanas la producción pasa a la placenta.

Los estrógenos inducen la luteólisis, para ello aumenta la producción de PGF, que inhibe la
síntesis de progesterona y la capacidad de unión de la LH a su receptor.(R2)

65. Physical examination of a patient with jaundice reveals a palpable gallbladder. What
is your diagnostic suspicion?

1. 1. Pancreatic cancer.
2. 2. Duodenal stenosis.
3. 3. Gallbladder cancer.
4. 4. Gallbladder cholesterolosis.
Gráfico de respuestas
Comentario

La presencia de ictericia con vesícula palpable debe sugerirnos la presencia de un cáncer de


cabeza de páncreas como enfermedad de base. El aforismo según el cual la presencia de ictericia
indolora es atribuible al cáncer de páncreas no siempre es cierto, ya que la mayoría de los
pacientes presentan dolor como parte de su complejo sintomático; por otra parte, la existencia de

 
 
 
 
una vesícula palpable en el cáncer de páncreas se da sólo en aproximadamente 30% de los casos.
Aún así, con las premisas reseñadas la conjunción de ambos hechos debe hacernos pensar como
primera posibilidad la existencia de un cáncer de páncreas.(R1)

66. ¿Cuál de las siguientes es una CONTRAINDICACIÓN absoluta para el empleo de


anticonceptivos orales?:

1. 1. Epilepsia.
2. 2. Litiasis biliar.
3. 3. Trombofilia.
4. 4. Dislipemias.
Gráfico de respuestas
Comentario

Es muy importante que conozca el mecanismo de acción, los efectos secundarios y las
contraindicaciones de los anticonceptivos orales, pues han sido preguntados en diversas
ocasiones.

Las contraindicaciones absolutas son pacientes con riesgo cardiovascular (fumadoras mayores de
35 o no fumadoras mayores de 40), antecedentes de enfermedad tromboembólica, HTA mal
controlada, diabetes con afectación vascular, vasculopatía inflamatoria, cardiopatías graves,
pacientes con afectación hepática importante (adenoma hepático o hepatopatías activas), pofiria
aguda intermitente, antecedentes de ictericia durante el embarazo (colestasis intrahepática),
cáncer de mama u otros tumores hormonodependientes, discrasia sanguínea y sangrado genital
anormal no filiado.(R3)

67. RN que, al minuto de vida, presenta FC de 110 latidos por minuto, acrocianosis con
esfuerzo respiratorio ausente, así como hipotonía y leve mueca al introducir la sonda de
aspiración. La puntuación de Apgar será:

1. 1. 1.
2. 2. 3.
3. 3. 4.
4. 4. 5.
Gráfico de respuestas
Comentario

0 1 2

Color de la piel todo azul extremidades azules normal

ritmo cardíaco no pulso <100 >100

irritabilidad no respuesta mueca llanto debil tos movimiento

actitud ninguno alguna flexión movimiento activo

respiración ausente débil irregular fuerte

(R3)

 
 
 
 
68. A 59-year-old woman comes to her gynecologist for her annual follow-up visit. She
went through her menopause when she was 53. Physical examination is unremarkable.
However, an ultrasound study is obtained in order to monitor previously diagnosed
uterine myomas. No myomas are seen and an endometrial thickness of 9 mm is found.
Which of the following is the best next step in management?

1. 1. Administration of medroxyprogesterone acetate during the second half of the cycle.


2. 2. Total hysterectomy with bilateral salpingo-oophorectomy.
3. 3. Hysteroscopic directed biopsy.
4. 4. Diagnostic curettage.
Gráfico de respuestas
Comentario

Pregunta de dificultad moderada sobre la hiperplasia endometrial y el cáncer de endometrio.

La hiperplasia endometrial es una proliferación del endometrio, originada por el estímulo de los
estrógenos no compensada por la adecuada secreción de progesterona.

Se clasifica en simple/ compleja, con/ sin atipia.

El diagnóstico es anatomopatológico mediante histeroscopía con biopsia dirigida (RC- 3). Las
indicaciones para realizar esta prueba es la presencia de metrorragia en mujer postmenopáusica
(es obligatorio descartar cáncer de endometrio) y la presencia de un diámetro del endometrio por
ecografía vaginal mayor de 15mm en premenopáusicas y de 5mm en postmenopáusicas (en el
caso mide 9 mm).(R3)

69. Mujer de 53 años de edad diagnosticada de artritis reumatoide hace 15 años, que
consulta por aparición de edema maleolar. La paciente refiere además diarrea y en la
exploración se detecta hepatomegalia. Los exámenes de laboratorio muestra creatinina
2 mg/dl y proteinuria de 3.5 g/24 horas. Respecto a la complicación que presenta esta
paciente señale la FALSA:

1. 1. Se debe al depósito extracelular de proteínas de estructura fibrilar.


2. 2. El órgano más frecuentemente afectado es el riñón.
3. 3. La afectación cardíaca es la causa más frecuente de muerte.
4. 4. Implica un mal pronóstico.
Gráfico de respuestas
Comentario

Nos describen la aparición de una proteinuria en rango nefrótico y deterioro de la función renal en
paciente diagnosticada de artritis reumatoide de larga evolución. Debemos sospechar una
amiloidosis renal secundaria.

El diagnóstico se realiza con biopsia, generalmente de la grasa subcutánea y tinción con Rojo
Congo. Un punto importante es que es la propia enfermedad renal la que es la principal causa de
muerte en los pacientes que desarrollan amiloidosis secundaria.

La afectación cardiaca porduce insuficiencia cardiaca congestiva, y a nivel digestivo produce


diarrea y malabsorción.

 
 
 
 
El origen del trastorno se situa en el depósito de proteína fibrilar de tipo AA extracelular, y su
tratamiento se realiza en base a las complicaciones que van surgiendo (diálisis, diuréticos,
vasodilatadores, transplante hepático o renal…).(R3)

70. Femenino de 81 años, diabética e hipertensa, institucionalizada en una residencia de


mayores, que es trasladada al servicio de urgencias del hospital, tras haber observado
una cuidadora la presencia de una ulceración en región vulvar, que se acompaña de
secreción serohemática y mal olor. Al preguntar a la paciente, que está consciente y
orientada, refiere que "lleva mucho tiempo con esa lesión, que es dolorosa y que
previamente le picaba mucho". En cuanto a la patología que presenta la paciente, es
FALSO:

La edad de aparición más frecuente se encuentra entre los 65-80 años, siendo rara antes de
1. 1.
los 30 años.
La localización más frecuente son los labios mayores, pero puede afectar también a labios
2. 2.
menores, clítoris, horquilla y periné.
3. 3. Son de utilidad en su diagnóstico la vulvoscopia y la tinción con azul de toluidina.
Se extiende principalmente por vía hematógena, siendo más rara la extensión linfática o
4. 4.
por continuidad.
Gráfico de respuestas
Comentario
La vía de extensión del cáncer de vulva es por continuidad o por vía linfática. La extensión por vía
linfática es la forma más importante de propagación. Por ello en el tratamiento del cáncer de vulva
se incluye la linfadenectomía inguinal de manera rutinaria.(R4)

71. Mujer de 35 años, diagnosticada de carcinoma de endometrio en estadio avanzado


por el que recibe quimioterapia, acude a urgencias por presentar desde hace una semana
odinofagia. Una radiografía de tórax fue normal. Se realizó una endoscopia donde desde
el tercio medio del esófago la mucosa presentaba importante afectación con vesículas y
ulceraciones, siendo informada como esofagitis necrotizante. ¿Cuál es su impresión
diagnóstica?

1. 1. Esofagitis por Candida.


2. 2. Esofagitis por virus varicela zoster.
3. 3. Esofagitis por CMV.
4. 4. Esofagitis por VHS.
Gráfico de respuestas
Comentario

Pregunta de considerable dificultad. La causa más frecuente de esofagitis infecciosa es la


candidiásica, pero en este caso no existen hallazgos sugerentes (no hay lesiones blanquecinas en
la mucosa esofágica). La presencia de vesículas y ulceraciones orienta a etiología vírica, puesto
que la quimioterapia no produce lesiones de este tipo.

El CMV no es una buena opción, puesto que suele producir una úlcera única y de gran tamaño, y
aquí son múltiples. Sólo nos queda el VHS y el virus varicela zóster, siendo los dos capaces de
producir vesículas y ulceraciones. La razón para decantarse por el VVZ es el carácter
NECROTIZANTE de la esofagitis, que no sería típica por el virus herpes simple.(R2)
 
 

 
 
 
 
72. Todas las siguientes pacientes pueden tomar anovulatorios, EXCEPTO:

1. 1. 22 años, candidiasis crónica.


2. 2. 36 años, sana.
3. 3. 37 años, fumadora de 10 cig/d durante 10 años hasta los 34 años.
4. 4. 16 años, nuligesta.
Gráfico de respuestas
Comentario

Es muy importante que conozca el mecanismo de acción, los efectos secundarios y las
contraindicaciones de los anticonceptivos orales para el ENARM.

Las contraindicaciones absolutas son pacientes con riesgo cardiovascular (fumadoras mayores de
35 o no fumadoras mayores de 40), antecedentes de enfermedad tromboembólica, HTA mal
controlada, diabetes con afectación vascular, vasculopatía inflamatoria, cardiopatías graves,
pacientes con afectación hepática importante (adenoma hepático o hepatopatías activas), pofiria
aguda intermitente, antecedentes de ictericia durante el embarazo (colestasis intrahepática),
cáncer de mama u otros tumores hormonodependientes, discrasia sanguínea y sangrado genital
anormal no filiado.(R3)

73. Varón de 50 años hipertenso y


diabético no insulindependiente que acude a urgencias por nauseas y vómitos
alimentario-biliosos, distensión abdominal y dolor abdominal cólico generalizado. En la
analítica realizada en urgencias se observa: Hb 16.3 g/dl, VCM 86 fl, 16.500 leucocitos,
plaquetas 325.000/mm3, INR 0.9, bilirrubina 1.2, AST 52, ALT 32, GGT 73, amilasa 46 UI/ml,
glucemia 86, urea 88, creatinina 1,6, sodio 149, potasio 4,3. Se realiza una Rx de abdomen
que muestra la imagen adjunta. ¿Cuál de entre las siguientes sería la actitud MENOS
adecuada?

1. 1. Solicitar una ecografía abdominal.


2. 2. Colocar sonda nasogástrica.
3. 3. Reposición hidroelectrolítica.

 
 
 
 
4. 4. Pautaría analgésicos.
Gráfico de respuestas
Comentario

El cuadro clínico es sugestivo de obstrucción intestinal, lo mismo que la radiografía adjunta, donde
se aprecian claros niveles hidroaéreos. La causa más frecuente de obstrucción intestinal en la
población general son las adherencias postquirúrgicas. En este caso, no hablan de antecedentes
de cirugía abdominal, por lo que posiblemente la causa sea una hernia, que es lo habitual en
personas no sometidas a laparotomía. Teniendo esto en cuenta, al menos de inicio no sería
necesario realizar una ecografía abdominal, sino una exploración física más completa (ingles,
escroto, etc.) buscando una posible hernia.

El 90% de las obstrucciones de intestino delgado se resuelven con sonda nasogástrica, dieta
absoluta y reposición hidroelectrolítica. La cirugía será necesaria si sospechamos estrangulación,
el dolor o la fiebre van en aumento, o no se resuelve en 3- 4 días. Podemos, además, administrar
analgesia. Tenga cuidado con la antigua creencia de que no se debe dar analgesia ante un cuadro
de abdomen agudo, porque esta idea se considera hoy día completamente falsa, se pueden indicar
derivados de morficos.(R1)

74. Tras ligera mejoría con el objeto de filiar la etiología, realiza una colonoscopia donde
se observa una mucosa de recto normal y al llegar a sigma, abundantes divertículos
rodeados por una mucosa intensamente hiperémica, edematosa y con algunos restos de
secreción purulenta. ¿Cual sería su actitud a continuación?

1. 1. Procedería a retirar el colonoscopio inmediatamente.


2. 2. Tomaría biopsias del fondo del divertículo para estudio microbiológico.
3. 3. Continuaría por lo menos hasta ángulo esplénico para ver la extensión de la enfermedad.
Trataría de llegar hasta ángulo hepático dada la alta probabilidad de que se trate de una
4. 4.
colitis isquémica cuyas lesiones más graves se observarían en colon transverso.
Gráfico de respuestas
Comentario

La descripción que hacen de la colonoscopia se corresponde con la de una diverticulitis aguda. En


esta entidad, está contraindicada la colonoscopia porque implica riesgo de perforación.

En la diverticulitis aguda, el cuadro típico consiste en un dolor abdominal en fosa ilíaca derecha en
una persona mayor, normalmente con fiebre y leucocitosis. En este caso, queda claro que la
imagen que han encontrado en la radiografía de abdomen no se corresponde, en realidad, con una
auténtica obstrucción intestinal. En el contexto de una diverticulitis, se justifica por el íleo adinámico
que puede acompañar a esta patología. En un íleo, cuando se realiza la Rx en bipedestación, se
puede encontrar dilatación de las asas y niveles hidroaéreos, sin que necesariamente exista una
verdadera obstrucción mecánica, como es el caso.(R1)

75. En el proceso de diagnóstico diferencial de una amenorrea los resultados obtenidos


son los siguientes: En la 2º fase de estudio (administración de estrógenos) no se ha
producido hemorragia genital, ¿cuál de las siguientes medidas diagnósticas NO nos va
a ayudar a dilucidar el origen de la amenorrea a partir de los resultados obtenidos?:

1. 1. Exploración genital.
2. 2. Determinación de gonadotropinas.
3. 3. Histerografía.

 
 
 
 
4. 4. Administración de una tanda de estrógenos y progestágenos.
Gráfico de respuestas
Comentario

Ante una amenorrea en primer lugar se realiza un test de embarazo, si es negativo seguimos el
estudio con TSH y PRL, en caso de que sean normales damos una pequeña cantidad de
progesterona (5 días) si la paciente tiene la regla es porque existía una anovulación que impedía el
aumento de la progesterona. Si no aparece la regla administramos estrógenos y progestágenos
durante 3 meses, si no tiene la regla a pesar de una adecuada secuencia hormonal debemos
pensar en alteraciones genitales estructurales (como es este caso y hacia ello van dirigidas las
pruebas que podemos hacer). Si tiene la regla tras al administración hormonal combinada
debemos pensar en un problema ovárico o hipotálamo-hipofisário y para ello ya determinaremos
gonadotropinas.(R2)

76. Con relación a la sífilis tardía, uno de los siguientes enunciados es FALSO:

Alrededor de un 30% de enfermos con sífilis latente no tratados presentan clínica


1. 1.
manifiesta de sífilis tardía.
2. 2. Las manifestaciones cardiovasculares se limitan a los grandes vasos.
Los aneurismas sifilíticos de aorta abdominal muy a menudo se localizan por debajo de las
3. 3.
arterias renales.
4. 4. Los gomas esqueléticos con mayor frecuencia afectan a los huesos largos de las piernas.
Gráfico de respuestas
Comentario

Cuando hablamos de sífilis tardía nos referimos a aquella que acontece a partir de un año post-
infección, y dentro de ella englobamos la sífilis terciaria y cuaternaria. La lesión característica de la
sífilis terciaria es el goma, lesión granulomatosa que con mucha frecuencia afecta a huesos largos
de piernas ( opción 4 correcta). También pertenecen a esta fase otros cuadros:
cardiovasculares- recordar que típicamente se afecta la aorta ascendente y neurológicos (
neurosífilis asintomática, meningitis crónica, ACVA). Hay dos cuadros de neurosífilis que se
incluyen dentro de la sífilis cuaternaria: tabes dorsal ( desmielinización de cordones posteriores,
que con el tiempo puede provocar ulceraciones plantares y deformidades articulares, y
frecuentemente acompañado de atrofia óptica y parálisis general progresiva ( con las
características pupilas de Argyll- Robertson).(R3)

77. Mujer de 40 años que acude a consulta porque desea esterilización definitiva.
Antecedentes ginecoobstétricos: 5 gestaciones, 2 partos eutócicos, 2 cesáreas y un
aborto. Antecedentes personales: obesidad (IMC=32), HTA en tratamiento con diuréticos
y fumadora de 15 cigarrillos/día. Apendicectomizada y reintervenida por peritonitis
fecaloidea. ¿Cuál de los siguientes métodos consideraría el más adecuado?

1. 1. Ligadura de trompas por laparoscopía.


2. 2. Obstrucción tubárica bilateral por histeroscopía.
3. 3. Implante subdérmico.
4. 4. Dispositivo intrauterino.
Gráfico de respuestas
Comentario

Dado que la mujer desea una anticoncepción definitiva deberemos realizar un bloqueo tubárico.
Éste se puede realizar mediante laparoscopía, laparotomía o por histeroscopía, siendo ésta última

 
 
 
 
la mejor opción para nuestra paciente dados los antecedentes médicos y quirúrgicos que presenta.
Se trata además de un procedimiento ambulatorio y que no precisa de anestesia general.(R2)

78. ¿Cuál es la causa más frecuente de obesidad en niños?:

1. 1. Síndromes dismórficos.
2. 2. Obesidad nutricional.
3. 3. Causas endocrinas.
4. 4. Causas genéticas.
Gráfico de respuestas
Comentario

Pregunta que no debe fallar. Recientemente se ha publicado que México ocupa el primer lugar en
obesidad infantil nutricional y la causa son malos hábitos higienico dietéticos.(R2)

79. Un estadio T1 b del cáncer de próstata corresponde a:

1. 1. Nódulo único no palpable visible por ultrasonido transrectal.


2. 2. Afectación de un solo lóbulo.
3. 3. Afectación exclusiva de la zona transicional.
4. 4. Descubrimiento incidental en la pieza quirúrgica.
Gráfico de respuestas
Comentario

Pregunta que si no conoce el TNM puede resultar muy difícil. Pero si sabe que en el estadio T1 el
tumor no es palpable ni visible puede descartar las opciones 1 y 2. El estadio T1b, corresponde al
hallazgo de >5% de tejido neoplásico en la muestra de tejido prostático obtenido porque el paciente
ha sido intervenido de HBP (RTU ó Cirugía abierta).(R4)

80. ¿Cuál de las siguientes afirmaciones relativas a la patología anorrectal en el VIH es


cierta?:

1. 1. Hay un riesgo aumentado de cáncer anal.


La ulceración anal es frecuentemente secundaria a una infección y responde precozmente a
2. 2.
los antibióticos tópicos.
Los pacientes HIV asintomáticos pueden tener serios problemas de cicatrización tras
3. 3.
cirugía anorrectal.
La amputación abdominoperineal es el tratamiento de elección para el carcinoma
4. 4.
escamoso del ano en los pacientes HIV.
Gráfico de respuestas
Comentario
Los pacientes VIH + pueden presentar la patología anorrectal común a la población en general
como son fístula anal (15%), absceso (30%), hemorroides (17%); sin embargo presentan otras
manifestaciones características, debido a la práctica de relaciones sexuales anales que los
predispone a enfermedades de transmisión por esa vía: el 95% de los homosexuales presentan
anticuerpos contra el virus del herpes tipo 2, del 90 al 100% son seropositivos para el
citomegalovirus (CMV), la ileocolitis sintomática por CMV es la infección intestinal más común en
pacientes con SIDA, ocurriendo en aproximadamente el 10%; el 55% de los homosexuales son
portadores del gonococo, la mayoría en forma asintomática en tanto que, los condilomas
acuminados se han reportado en un 40 a 60% de los homosexuales y pueden mantenerse
asintomáticos, la CDC informó un incremento en la incidencia de los condilomas acuminados de

 
 
 
 
500% de 1966 a 1981. Se ha comprobado como hay una incidencia de carcinoma anorrectal en los
VIH positivos con condilomas.(R1)

81. Femenino primigesta de 32 semanas


de gestación. Acude a urgencias por sensación de dinámica. Exploración: cérvix cerrado
y formado. RCTG: 2 contracciones en 10 minutos, FCF basal 140 latidos, RAF positivo.
Se realiza USG transvaginal para la medición de la longitud cervical obteniéndose la
imagen que se muestra. ¿Cuál de las siguientes afirmaciones es INCORRECTA?

1. 1. Se aconseja reposo en esta paciente.


2. 2. Si el test de la fibronectina es positivo, es un factor de riesgo para parto pretermino.
3. 3. Ante 2 contracciones en 10 minutos es necesario instaurar tocólisis.
4. 4. Un RAF positivo no requiere realizar una prueba de Pose.
Gráfico de respuestas
Comentario

La imagen adjunta nos muestra una longitud cervical considerable (línea punteada). Es muy poco
probable que esta situación termine en un parto pretérmino. Más bien parece una amenaza de
parto pretérmino, pero de bajo riesgo (cérvix no modificado, ausencia de antecedentes de parto
pretérmino, no se trata de una gestación múltiple). En estos casos, no es obligado instaurar
tocolisis, aunque tengamos dinámica uterina (respuesta 3 falsa). Puede ser suficiente con
recomendar reposo y alta domiciliaria. En el caso de que no ceda la dinámica uterina, puede
valorarse un período de observación de 12-24 horas inicialmente.(R3)

82. Acude a urgencias un pre escolar de 5 años por cuadro de fiebre y diarrea de 2 días
de evolución. La exploración es normal y el paciente presenta buen estado general,
constantes vitales normales y buena tolerancia oral, por lo que es dado de alta
recomendándole una alimentación normal e ingesta frecuente de líquidos. 48 horas
después vuelve a consultar por persistencia del cuadro, con unas 10-12 deposiciones
diarias, abundantes, de consistencia líquida y sin productos patológicos, con
temperatura máxima de 38.7º C. En la exploración, el niño se encuentra febril y decaído,
con ojos hundidos y algo ojerosos. No presenta signo del pliegue pero la mucosa bucal
está algo seca. La auscultación cardiopulmonar y la palpación abdominal no presentan
alteraciones. La frecuencia cardíaca es de 105 lpm y la tensión arterial de 95/50. El peso
en el momento actual es de 17.700 kg y el peso previo era de 18.400 kg. Se extrae una

 
 
 
 
analítica sanguínea en la que se observa pH de 7.37; sodio de 133 mEq/l; potasio de 4
mEq/l. Con los datos aportados, ¿cuál de las siguientes afirmaciones le parece
incorrecta?

1. 1. El paciente presenta una deshidratación leve.


2. 2. El paciente presenta acidosis metabólica.
3. 3. La rehidratación oral es el tratamiento de elección.
4. 4. La mejor forma de evaluar la respuesta al tratamiento es la medición de la diuresis.
Gráfico de respuestas
Comentario
Este caso clínico ilustra una deshidratación isonatrémica o isotónica (se considera hipotónica a
partir de niveles de sodio menores o iguales a 130 mEq/l, aunque los niveles normales de natremia
son de 135- 145 mEq/L), sin acidosis (ya que el pH normal es de 7,35 a 7,45). En las
deshidrataciones, el peso nos sirve para estimar el grado de deshidratación. En este caso el
paciente ha perdido 700 gr, que es aproximadamente un 4% de su peso, por lo que consideramos
que la deshidratación que presenta es leve ya que ha perdido menos de un 5% del peso (si
hubiese perdido 5- 10% sería moderada, y grave si la pérdida hubiese sido mayor). La tendencia
actual es rehidratar al paciente por vía oral ya que es la más adecuada y fisiológica, de manera
que reservamos la vía intravenosa cuando la oral está contraindicada (intolerancia oral, íleo
paralítico, deshidratación grave…). La monitorización de la diuresis es la mejor manera de evaluar
la respuesta a la rehidratación.(R2)

83. Acude a consulta una mujer de 19 años que presenta lesiones eritematoescamosas,
edema y alguna vesícula en la cara, escote, dorso de las manos y antebrazos. Las
lesiones tienen 12 horas de evolución y han aparecido tras una excursión al campo. Entre
los antecedentes personales destaca acné vulgar en tratamiento con retinoides tópicos
y doxiclicina oral. El diagnóstico más probable es:

1. 1. Erupción lumínica poliforma.


2. 2. Reacción fototóxica.
3. 3. Urticaria solar.
4. 4. Fotodermatosis por hipersensibilidad.
Gráfico de respuestas
Comentario

Paciente que muestra lesiones en áreas fotoexpuestas y, por tanto, es obvio pensar en que la
exposición solar tiene mucho que ver con el cuadro. Para incidir más en esta pista, nos dicen que
las lesiones aparecen tras una excursión al campo. El paciente está en tratamiento para su acné
con retinoides tópicos y doxiciclina oral. Unos de los efectos secundarios característicos de las
tetraciclinas es la fotosensibilidad. En el capítulo de farmacología se habla de él en el apartado de
reacciones adversas de las tetraciclinas.

En cuanto a los retinoides tópicos, acentúan la reacción fototóxica que producen las tetraciclinas.
De hecho, se suele evitar pautar retinoides en pacientes que se exponen al sol de manera habitual
para evitar este efecto. Acuérdate que la erupción fototóxica se da en la primera exposición y la
clínica es monomorfa, como cualquier quemadura en áreas fotoexpuestas. Entre las causas
destacan las tetraciclinas, alquitrán, psoralenos y retinoides. Es importante que no la confundas
con la reacción fotoalérgica, que se da en la 2ª exposición, aparece incluso en áreas cubiertas y
está relacionada con el consumo de sulfamidas, PABA y fenotiacinas.(R2)

 
 
 
 
84. Respecto a las glomerulonefritis, señale la asociación FALSA:

1. 1. Enfermedad de Goodpasture - anticuerpos antimembrana basal.


2. 2. Enfermedad de Berger - depósitos de IgA.
3. 3. Glomerulonefritis focal y segmentaria - "spikes".
4. 4. Glomerulonefritis rápidamente progresiva - semilunas.
Gráfico de respuestas
Comentario
Pregunta de dificultad moderada- baja acerca de asociaciones típicas en GN. Es importante
elaborar listas con "datos típicos" ya que solamente con ellos podemos sacar directamente varias
preguntas en el MIR. La OPCION FALSA ES LA 3, debido a que las "espículas" o "spikes" son
típicas de la nefropatía membranosa y no de la HSF. Recordamos a continuación la histología
típica de la GNM: engrosamiento uniforme y difuso de la pared de los capilares glomerulares, sin
proliferación celular asociada. El hallazgo más característico, cuando la biopsia se impregna con
sales de plata, es la presencia de múltiples prolongaciones argirófilas de la membrana basal hacia
el espacio urinario de Bowman (espículas o Spikes). Mediante la IF se observan depósitos de IgG y
C3 a lo largo de toda la pared capilar, en forma de pequeños gránulos distribuidos de modo muy
uniforme. Al ME la membrana basal presenta en su vertiente externa (subepitelial) depósitos
redondeados, de densidad elevada y dispuestos a intervalos más o menos regulares. Entre estos
depósitos subepiteliales se observan, de modo casi constante, proyecciones de la membrana
basal, que se corresponden con las espículas argirófilas observadas con el microscopio óptico. Los
podocitos están total o parcialmente fusionados.(R3)

85. Una niña de 12 meses es hospitalizada con un cuadro de deshidratación y diarrea de


3 días de duración. Una semana antes pesaba 10 kg en una consulta de vigilancia. Había
estado con fiebre de 39ºC y con 10 y 12 evacuaciones diarias durante los últimos días.
No había orinado en las últimas 18 horas. La exploración física puso de manifiesto ojos
hundidos y secos, y piel sin turgencia (signo del "pliegue"). ¿Qué es lo primero que haría
usted?:

1. 1. Pedir un recuento sanguíneo completo y hemocultivos.


2. 2. Recoger una muestra de orina para cultivo, electrólitos y densidad.
Empezar a administrar solución de Ringer lactato, 20 mL/kg intravenosos después de
3. 3.
tomar sangre para determinar electrolitos y nitrógeno ureico.
4. 4. Transfundir plasma fresco compatible.
Gráfico de respuestas
Comentario

Los casos clínicos de deshidrataciones son habituales en el ENARM.

En esta pregunta nos presentan un caso de deshidratación probablemente isotónica por pérdidas
digestivas (gastroenteritis) y de grado severo ya que lleva 18h en anuria.

La prioridad en su manejo es la rehidratación, en este caso la vía de elección es la iv empezando


con 20ml/kg de cristaloide como expansor vascular para luego continuar con glucosalino.

Recuerde que la causa más frecuente de choque en los pacientes de edad pediátrica es la
hipovolémica y que en un niño deshidratado la mejor forma de valorar la respuesta al tratamiento
es el control de la diuresis.(R3)

 
 
 
 
86. Señale cuál de las siguientes es la víscera que con MENOR frecuencia se lesiona en
los traumatismos abdominales:

1. 1. Bazo.
2. 2. Intestino.
3. 3. Páncreas.
4. 4. Riñón.
Gráfico de respuestas
Comentario

Es una pregunta contestable con la información que viene en el manual. La víscera más
frecuentemente dañada en el trauma abdominal es el bazo seguida del hígado. Sólo por extensión
la siguiente estructura en dañarse sería las vísceras huecas y el riñón dada su localización. Por
último, el páncreas es el órgano que más raramente se daña (opción 3 correcta). Es muy frecuente
que se acompañe de lesiones de órganos y vasos de la vecindad. La causa principal de la
mortalidad se debe a lesiones vasculares asociadas. El tratamiento vendrá dado por la magnitud
de la lesión. En las lesiones pequeñas bastará con un drenaje adecuado. Cuando hay laceración
considerable será necesaria la cirugía resectiva.(R3)

87. Paciente de 45 años de edad, ejecutivo, es ingresado en el hospital a causa de vómitos


de sangre roja. No refiere sintomatología previa ni historia significativa. Necesitó 5
unidades de sangre antes de la endoscopia. ¿Cuál es el diagnóstico más probable?:

1. 1. Gastritis.
2. 2. Esofagitis.
3. 3. Síndrome de Mallory Weiss.
4. 4. Ulcera duodenal.
Gráfico de respuestas
Comentario
Se trata de una pregunta directa, sobre cuál es la causa más frecuente en la población general de
hemorragia digestiva alta. Por su prevalencia, la causa más frecuente es la úlcera duodenal,
aunque en valores relativos sangran más las úlceras gástricas, porque suelen ser más grandes
(son la segunda causa más frecuente de hemorragia digestiva alta). Hay que recordar que si se
tratara de un paciente cirrótico, la causa más frecuente de este tipo de sangrado sería la
hemorragia digestiva alta por rotura de varices esofágicas.(R4)

88. Paciente de 65 años visto por el cardiólogo con anterioridad por el seguimiento de
una angina estable de moderados esfuerzos, que se controlaba adecuadamente hasta
hace dos meses con propanolol y nitroglicerina sublingual durante las crisis.
Actualmente viene a urgencias por nuevo cuadro anginoso, refiere que las crisis se están
haciendo últimamente más frecuentes y que aparecen con menos esfuerzo. En la
exploración física se observa TA de 140/85 mmHg, 90 lpm, con el resto dentro de lo
normal. En el ECG basal tiene una onda T negativa en V5-V6 y descenso del ST en el
periodo sintomático. ¿Qué tratamiento le propondría?

Ingreso hospitalario y coronariografía urgente tras control del dolor con nitroglicerina
1. 1.
sublingual.
Ingreso hospitalario, reposo y tratamiento con AAS, clopidogrel y heparina de bajo peso
2. 2. molecular, tratamiento antianginoso betabloqueantes y nitroglicerina y realización de
prueba de esfuerzo pronóstica 48 horas después de controlado el dolor.
3. 3. Ingreso en la unidad coronaria instaurando trombolíticos.

 
 
 
 
Ingreso hospitalario, tratamiento con AAS, heparina no fraccionada y control del dolor y
4. 4. tras ello, alta hospitalaria con tratamiento y realización de prueba de esfuerzo para
estadificación del riesgo de forma ambulatoria.
Gráfico de respuestas
Comentario

Cuando nos encontramos ante una angina inestable, el protocolo a seguir es el siguiente:

1. Ingreso hospitalario.

2. Monitorización y reposo.

3. Inicio de la antiagregación con AAS asociada o no a clopidogrel (este último debe omitirse si el
paciente puede ser sometido a revascularización coronaria quirúrgica en los próximos 5 días). En
los pacientes de alto riesgo se pueden utilizar inhibidores de la glucoproteína IIb- IIIa.

4. Anticoagulación con HNF o HBPM.

5. Tratamiento con fármacos antianginosos, preferiblemente betabloqueantes, calcioantagonistas


y/ o nitratos.

Posteriormente es necesario valorar el pronóstico del paciente. Para ello, hay que realizar una
prueba de detección de isquemia y, en función de los hallazgos encontrados, se hace o no una
coronariografía.(R2)

89. Mujer de 80 años de edad ingresada en el hospital por haber presentado un infarto de
miocardio, presenta dolores cólicos en la parte inferior del abdomen y un estreñimiento
agudo. A la exploración se aprecia un abdomen distendido y timpánico, con disminución
o ausencia de ruidos intestinales. Existe dolor a la palpación pero no dolor al rebote. La
radiografía de abdomen revela una importante dilatación del colon, en especial del ciego.
¿Qué cuadro clínico sospecharía?:

1. 1. Apendicitis aguda.
2. 2. Pseudoobstrucción intestinal aguda o síndrome de Ogilvie.
3. 3. Síndrome de intestino irritable.
4. 4. Pancreatitis postanginosa.
Gráfico de respuestas
Comentario

Pregunta tipo ENARM en la que se le pide reconocer el cuadro clínico para poder contestar a cerca
de su tratamiento. Utiliza esta pregunta para dejar algunos puntos claros sobre la
pseudoobstrucción intestinal.

Se trata de un trastorno crónico que cursa con signos y síntomas de obstrucción pero NO existe
lesión obstructiva. Cuando el episodio es agudo se denomina síndrome de Ogilvie y es típico de
pacientes ancianos.

El paciente refiere sintomatología intermitente, de tipo cólica, con importante distensión abdominal
que afecta sobre todo al colon derecho y transverso, existiendo un elevado riesgo de perforación
cuando el diámetro cecal es mayor de 12 cm. En ausencia de dilatación severa el tratamiento es

 
 
 
 
conservador con enemas, neostigmina o colonoscopias descompresivas. En caso de perforación o
si existe alto riego de que esta ocurra esta indicada la resección quirúrgica (hemicolectomia
derecha o transversa).(R2)

90. Una paciente embarazada de 8 semanas acude por detección en el urocultivo de


control de 100.000 UFC de E. coli. Refiere encontrarse asintomática y hasta ahora el curso
del embarazo ha sido estrictamente normal. En este caso:

1. 1. Se trata de una bacteriuria asintomática, por lo que no precisará tratamiento alguno.


Se deberá hacer una exploración completa del tracto genitourinario, especialmente alguna
2. 2.
prueba de imagen, para filiar el trastorno que ha provocado el cuadro clínico.
3. 3. Se deberá iniciar un tratamiento antibiótico con quinolonas.
4. 4. Se podría tratar con cotrimoxazol.
Gráfico de respuestas
Comentario
Se trata de una paciente embarazada con una bacteriuria asintomática y, dado que pertenece al
grupo de las que se deben tratar se deberá iniciar un tratamiento con antibióticos. Las quinolonas
deben evitarse durante todo el embarazo pero las sulfamidas pueden usarse en el primer y
segundo trimestre.(R4)

91. Respecto al RCTG (registro cardiotocográfico), indique la respuesta INCORRECTA:

1. 1. El patrón de variabilidad sinusoidal indica recuperación fetal tras una hipoxia prolongada.
Si la pérdida de variabilidad no va acompañada de deceleraciones tardías probablemente
2. 2.
se trata de una lesión del cerebro fetal producida anteriormente.
3. 3. El primer tipo de variabilidad que se pierde con la hipoxia es la de corto plazo.
4. 4. La causa de disminución de variabilidad durante la hipoxia es la depresión del SNC.
Gráfico de respuestas
Comentario

Pregunta sobre el RCTG que no presenta excesiva dificultad y es imortante para el


nacional. Tenemos que recordar que el patrón sinusoidal implica gravedad ya que aparece en los
estados premortem y en las isoinmunizaciones Rh, por lo que la opción 1 es incorrecta. La hipoxia
moderada produce acidosis fetal y dips de tipo II. La disminución de la variabilidad se debe a una
pérdida del control del sistema nervioso autónomo, bien por hipoxia o por lesión cerebral.(R1)

92. Señale la FALSA en relación a las lesiones traumáticas del bazo:

1. 1. Es el órgano más frecuentemente lesionado en traumatismos no penetrantes.


Clínicamente cursa con signos de irritación peritoneal en el área esplénica y signos de
2. 2.
hemorragia.
3. 3. En el diagnóstico son útiles la ecografía y la TAC.
El tratamiento requiere la esplenectomía en todos los casos de lesión esplénica en los
4. 4.
niños.
Gráfico de respuestas
Comentario

Es una pregunta contestable con el manual. El órgano más frecuentemente lesionado en


traumatismos no penetrantes y en general es el bazo. Clínicamente se observan signos generales
de hemorragia y locales de irritación peritoneal en el área esplénica. En raros casos (<5%) puede
haber una rotura esplénica diferida por un hematoma eventualmente contenido por la cápsula,

 
 
 
 
manifestándose generalmente dentro de la primera semana tras el trauma. El diagnóstico se
establece por ecografía o TC. Si el paciente está hemodinámicamente inestable, la punción-lavado
peritoneal es indicación de cirugía sin dilación. El tratamiento en ausencia de lesiones significativas
y de hemorragia persistente puede ser consevador (esplenorrafia), reservando la esplenectomía
para lesiones extensas del parénquima. En niños es preferible el tratamiento conservador siempre
que sea posible ya que la esplenectomía no está exenta de morbilidad: aumenta la infección por
gérmenes capsulados...(R4)

93. Uno de los siguientes psicofármacos ha sido relacionado con la aparición de


priapismo. Señale cuál:

1. 1. Trazodona.
2. 2. Clozapina.
3. 3. Tioridacina.
4. 4. Metilfenidato.
Gráfico de respuestas
Comentario
Se han descrito bastantes casos de priapismo con trazodona. Los efectos secundarios de los
fármacos psicotrópos son muchos y complicados de recordar. Lo más sencillo es aprenderlos por
grupos y recordar los efectos secundarios más llamativos, y por tanto más preguntables. Los
cuadros resumen del manual te pueden ayudar.(R1)

94. Mujer de 29 años de edad, presenta secreción serosanguinolenta por el pezón de la


mama izquierda. Al examen físico: mamas sin alteraciones. ¿Cuál es la causa más
frecuente de este tipo de secreción?:

1. 1. Fibroadenoma.
2. 2. Mastopatía fibroquística.
3. 3. Absceso.
4. 4. Papiloma intraductal.
Gráfico de respuestas
Comentario

El papiloma intraductal es la causa más frecuente de secreción patológica a través del pezón ya
sea unilateral o localizada a un único conducto. La secreción asociada al papiloma puede ser clara
o hemorrágica. Generalmente afecta a los conductos principales, localizándose a 1-2 cm del
pezón. Suele aparecer en mujeres premenopáusicas. El tratamiento quirúrgico consiste en la
extirpación de la zona previa localización del conducto que produce la telorragia.

El fibroadenoma es el tumor benigno más frecuente de la mama, formado por proliferación de


elementos epiteliales y mesenquimatosos. Es pseudoencapsulado, bien delimitado del tejido
mamario adyacente, móvil, esférico, aunque puede ser multilobulado. Es estrogenodependiente, es
posible que aumente de tamaño con al toma de anticonceptivos orales, en la gestación y en la
lactancia. Su mayor incidencia es entre los 15 y 35 años. El diagnóstico se basa en la clínica. Se
palpa un nódulo de características benignas, confirmándose por pruebas de imagen

La mastopatía fibroquística es una enfermedad benigan y crónica con una alteración en la


proliferación del estroma y del parénquima mamario, desarrollando tumores o quistes palpables. Es
la patología más frecuente en la mujer premenopáusica y es muy rara después de la menopausia.
El síntoma más frecuente es la mastodinia.

 
 
 
 
En caso de ser un absceso mamario te darían otros datos como síndrome de respuesta
inflamatoria sistémica, datos de inflamación localizada como aumento de volumen, temperatura,
eritema y dolor.(R4)

95. La modificación endocrina que aparece de modo más precoz en la menopausia es:

1. 1. Aumento de FSH.
2. 2. Disminución de FSH.
3. 3. Aumento de LH.
4. 4. Aumento de estrógenos.
Gráfico de respuestas
Comentario

Pregunta básica. En la menopausia la primera alteración hormonal es el aumento de FSH.


Respuesta 1 correcta. y posteriormente el aumento de LH.(R1)

96. El síndrome de Rokitansky-Küster-Hauser se caracteriza por los siguientes hechos,


EXCEPTO:

1. 1. Caracteres sexuales secundarios femeninos normales.


2. 2. Cariotipo 46, XY.
3. 3. Agenesia total o parcial de la vagina.
4. 4. Utero rudimentario no canalizado.
Gráfico de respuestas
Comentario

Deben conocer ciertos datos típicos de alguno de estos síndromes que aunque no son muy
frecuentes como preguntas puede caer algún caso clínico en que le describan uno de ellos. En el
síndrome de Rokitansky lo fundamental es una alteración en la permeabilización de los conductos
de Müller. El fenotipo y el cariotipo es femenino normal. Los ovarios son normales. El útero es
rudimentario y no está canalizado. Hay agenesia de los 2/3 superiores de la vagina, por lo que a la
inspección se observa una vagina corta que termina en un fondo de saco ciego. Presenta
frecuentes malformaciones renales y urinarias.(R2)

 
 
 
 
97. ¿Qué fármaco se puede emplear para tratar por vía oral la infección por Leishmania?

1. 1. Anfotericina B liposomal.
2. 2. Alopurinol.
3. 3. Miltefosina.
4. 4. Irtaconazol.
Gráfico de respuestas
Comentario

El tratamiento de las leishmaniasis se ha realizado tradicionalmente con antimoniales


pentavalentes como glucantime. Estos fármacos deben administrarse por vía intramuscular o
intravenosa, producen con frecuencia pancreatitis y son tóxicos cardiacos (requieren de
monitorización diaria de la actividad electrocardiográfica). La alternativa a este tratamiento es el
empleo de anfotericina B. En su forma clásica, denominada anfotericina B deoxicolato, se trata de
un fármaco muy nefrotóxico y debe administrarse de manera prolongada por vía intravenosa. La
anfotericina B liposomal ha permitido el tratamiento de los pacientes con leishmaniasis con un
riesgo mucho menor de nefrotoxicidad pero con un coste económico muy elevado y, en cualquier
caso, por vía intravenosa. En los últimos años el tratamiento con miltefosina ha supuesto una
revolución en el abordaje de la leishmaniasis, ya que por primera vez se ha comercializado un
fármaco eficaz para el tratamiento de la leishmaniasis con la posibilidad de ser administrado por vía
oral.(R3)

98. Mujer de 37 años que acude a su consulta solicitando método anticonceptivo (no
desea esterilización). Antecedentes personales: enfermedades propias de la infancia. No
refiere intervenciones quirúrgicas. Sin alergias medicamentosas conocidas. Fumadora
de 20 cigarrillos/día. No consumo de alcohol ni de drogas. Antecedentes gineco-
obstétricos: 3 partos eutócicos; lactancia materna. Tiempo medio de menstruación: 4 /
28 días. Sin dismenorrea. Revisiones periódicas anuales. Última hace 3 meses con
citología normal. Exploración: genitales externos y vagina normales; cérvix de multípara
bien epitelizado; útero de tamaño y forma normal; anejos no se palpan patológicos. USG:
útero en anteversión, regular, histerometría 75mm; anexos normales. ¿Qué método
anticonceptivo aconsejaría?:

1. 1. DIU.
2. 2. Anticonceptivos inyectables combinados.
3. 3. Diafragma.
4. 4. Minipíldora de gestágenos.
Gráfico de respuestas
Comentario

En una paciente fumadora de más de 35 años, la anticoncepción hormonal sería una


contraindicación absoluta, debido al riesgo cardiovascular, con lo que las opciones 2 y 4 no serían
planteables. En cuanto al diafragma, recuerde que debe utilizarse junto con una crema espermicida
y su eficacia está por debajo de la del DIU. Por lo tanto, tampoco sería la mejor opción para esta
paciente. En caso de que éste no pueda utilizarse, tal vez sería planteable, pero no como primera
opción.

Con respecto al DIU, sería interesante que recordara las siguientes ideas:

- Mecanismo de acción: reacción inflamatoria endometrial que evita la implantación.

 
 
 
 
- Está contraindicado ante la sospecha de embarazo, infección pélvica aguda o reciente,
sangrado uterino anormal, tratamiento con anticoagulantes o tumores cervicales o uterinos de
cualquier tipo.

- El momento habitual de colocación es durante la menstruación.

- El DIU previene mejor el embarazo normal que el ectópico, por lo que aumenta la frecuencia
relativa de éste último.

- El DIU es un factor de riesgo para enfermedad inflamatoria pélvica, al poner en contacto una
cavidad séptica (la vagina) con el interior del útero.(R1)

99. En el síndrome de Lutembacher existe una de las siguientes valvulopatías:

1. 1. Estenosis mitral.
2. 2. Insuficiencia mitral.
3. 3. Estenosis pulmonar.
4. 4. Estenosis aórtica.
Gráfico de respuestas
Comentario
Se trata de la asociación de comunicación interauricular y estenosis mitral reumática.(R1)

100. Si en un paciente con insuficiencia cardiaca crónica detectamos unas ondas v


prominentes en el pulso venoso yugular y en la auscultación cardiaca se ausculta un
soplo holosistólico en el área del apéndice xifoides que se acentúa con la inspiración
profunda, ¿cuál es la valvulopatía responsable de esta exploración física?:

1. 1. Insuficiencia pulmonar.
2. 2. Insuficiencia tricúspide.
3. 3. Insuficiencia aórtica.
4. 4. Estenosis aórtica.
Gráfico de respuestas
Comentario
La alteración del pulso venoso indica afectación de cavidades derachas (1, 4 y 5 descartadas). El
soplo en el apéndice xifoides indica foco tricúspide y la onda "v" prominente indica paso de sangre
de VD a AD por insuficiencia tricuspídea.(R2)

101. Pre-escolar de 3 años de edad, sexo femenino, que inició hace 5 días con un cuadro
caracterizado por rinorrea, tos, fiebre de 39°C; al que se le agrega el día de su ingreso
dificultad para respirar, aleteo nasal, FR de 56 x’, matidez en tercio superior derecho,
vibraciones vocales aumentadas, en donde se ausculta soplo tubárico y broncofonía el
diagnóstico probable es:

1. 1. Atelectasia.
2. 2. Neumonía.
3. 3. Derrame pleural.
4. 4. Neumotórax.
Gráfico de respuestas
Comentario

 
 
 
 
102. En este caso de tos, fiebre y dificultad respiratoria, asociado a síndrome de consolidación por:
matidez, vibraciones vocales aumentadas, soplo tubárico y broncofonía: todo esto nos habla de
una neumonía, respuesta 2 correcta.(R2)

103. Lactante con hipotensión arterial lábil, incoordinación en la succión-deglución,


frecuentes neumonías aspirativas y síncopes, que presenta además ácido homovanílico
elevado y vanilmandélico disminuido en orina. ¿Qué enfermedad presenta?:

1. 1. Neuroblastoma.
2. 2. Feocromocitoma.
3. 3. Disautonomía familiar.
4. 4. Patología troncoencefálica.
Gráfico de respuestas
Comentario
El neuroblastoma ha sido preguntado en varias ocasiones en el MIR por lo que conviene que
tengas claros algunos conceptos referentes sobretodo a la clínica y a sus factores pronósticos. El
neuroblastoma se clasifica dentro de los PNET, suele diagnosticarse en niños y es muy raro en el
SNC siendo su localización más frecuente en el abdomen. Suele diagnosticarse como una
enfermedad metastásica y la clínica depende de su localización y de la producción de
catecolaminas. La forma más frecuente de presentación es en forma de una masa abdominal.
También se ha descrito el Síndrome de opsoclono- mioclono (Sdme de Kinsbourne), diarrea
intensa debido a la secreción de VIP, la aparición de un hematoma lineal en el párpado e
hipertensión (rara en este tumor, siendo más común en el feocromocitoma) .Para su diagnostico se
precisa la realización de un TC abdominal, una gammagrafia con MIBG y la determinación de
catecolaminas en orina (estarán elevadas por ser un tumor productor), aunque el diagnostico
definitivo se realiza mediante una biopsia de la masa. El tratamiento de elección es la cirugía.(R3)

104. A 13-month-old child is brought to the pediatrician's office for a routine visit. He has
a normal physical development (percentile 60 in growth curves) but his parents are
finding it hard to teach him how to speak and he can't walk properly yet. Physical
examination shows a mild increased cephalic diameter and diffuse moles and freckles on
his skin, especially in the axillary area. What is the most likely diagnosis?

 
 
 
 
1. 1. Neurofibromatosis type 2
2. 2. Von Hippel Lindau Syndrome
3. 3. Neurofibromatosis type 1
4. 4. Turcot's syndrome
Gráfico de respuestas
Comentario
Neurofibromatosis type 1. Neurofibromatosis is an autosomal dominant disorder. Symptoms of NF1,
which may be evident at birth and nearly always by the time the child is 10 years old, include "cafe-
au-lait" spots, iris spots (Lisch's nodules), axillary freckles (Crowe's sign), macrocephaly and bone
deformities. Neurofibromas, which are benign skin tumors, are the hallmark of the disease. NF2 is
less common and is defined by bilateral acoustic neuromas, cataracts at an early age and other
tumors of the nervous system.(R3)

105. Which of the following is false regarding the management of subarachnoid


hemorrhage?

1. 1. Cerebral angiography in the first 48 hours of presentation may facilitate early treatment.
2. 2. Systemic arterial blood pressure must be lowered in order to avoid rebleeding.
Hydrocephalus is a common cause of late neurological impairment and low level of
3. 3.
conscioussness.
4. 4. Nimodipine may prevent vasospasm.
Gráfico de respuestas
Comentario

Pregunta relativamente sencilla sobre el manejo de la hemorragia subaracnoidea.

El tratamiento de la HSA es esencialmente médico (no es susceptible de evacuación quirúrgica) y


va dirigido a reducir la sintomatología dolorosa, soporte vital y evitar el riesgo de resangrado. No se
debe bajar la tensión arterial en fase aguda pues puede existe riesgo de hipoperfusión cerebral
(respuesta 2 incorrecta).

La hemorragia subaracnoidea puede presentar complicaciones de dos tipos: 1) médicas, entre las
que se encuentra el SIADH y 2) neurológicas, como el vasoespasmo, el resangrado y la
hidrocefalia obstructiva (respuesta 3 correcta).

Es frecuente la utlización de calcioantagonistas (nimodipino) para la prevención del vasoespasmo (


respuesta 4 correcta).

Respecto a su diagnóstico, ante la sospecha clínica, el TC craneal sin contraste es el


procedimiento de elección y primera prueba a realizar. La angiografía de cuatro vasos debe ser
realizada lo antes posible, incluyendo los sistemas carotídeos y vertebrobasilares para definir la
localización y morfología del aneurisma, delimitar otros aneurismas no rotos y valorar el grado de
vasoespasmo (respuesta 1 correcta).(R2)

106. Si un niño de 8 meses, regurgitador habitual desde el nacimiento, presenta desde


hace dos días irritabilidad con hematemesis, usted debe pensar en:

1. 1. Enfermedad por reflujo gastroesofágico.


2. 2. Estenosis hipertrófica de píloro.
3. 3. Malrotación intestinal.
4. 4. Fístula traqueo-esofágica.

 
 
 
 
Gráfico de respuestas
Comentario
Al igual que en los adultos, ante una historia de reflujo gastroesofágico de evolución tórpida, hay
que pensar que éste pueda acarrear complicaciones, locales y extradigestivas. Este niño
probablemente presenta esofagitis, pues manifiesta irritablidad (por dolor retroesternal y pirosis) y
hematemesis (por dislaceración de la mucosa).(R1)

107.   A 55-year-old woman presents to


her physician with a 6-week history of fatigue and abdominal discomfort. Physical
examination reveals multiple cervical, axillary and inguinal adenopathies. A CT scan
shows big retroperitoneal adenopathies. A biopsy is obtained and shown in the image.
The pathologist describes it as a lymphoma. Which of the following markers would most
likely be negative?

1. 1. CD20.
2. 2. CD15.
3. 3. CD10.
4. 4. t (14;18).
Gráfico de respuestas
Comentario
Aunque aparentemente difícil, se trata en realidad de una pregunta asequible, incluso sencilla.
Podría adivinarse la respuesta correcta sin ni siquiera mirar la imagen histológica. Bastaba darse
cuenta de un par de detalles. - Las respuesta 4 tiene que estar presente. La traslocación t(14;18)
es propia del linfoma folicular, que forzosamente tiene que ser la enfermedad del paciente, ya que
las dos opciones no pueden ser falsas a la vez. - Como el linfoma folicular es de estirpe B, las
respuestas 1 y 3 deberían también estar presentes, ya que son propias de este tipo celular. La
respuesta que queda es la 2, el CD15, que aparece en otro tipo de neoplasias hematológicas
(típicamente, en la enfermedad de Hodgkin). Lo que se ve en la imagen es el típico aspecto del
linfoma folicular, en el que aparecen estructuras que recuerdan a los folículos linfoides, como su
nombre sugiere.(R2)

108. Femenino de 36 años fumadora de 40 cigarrillos al día que consulta por dolor
epigástrico de 3 meses de evolución que alivia con antiácidos y las comidas. En una Rx
con bario se demuestra en bulbo duodenal una úlcera de 1 cm. A continuación
recomendaría en primer lugar:

1. 1. Duodenoscopia.

 
 
 
 
2. 2. Omeprazol.
3. 3. Test del aliento.
4. 4. Acexamato de Zinc.
Gráfico de respuestas
Comentario

Actualmente es OBLIGATORIO la demostración, después del diagnóstico por imagen de una


úlcera, de infección o no por H. pylori. En los ulcus duodenales no son necesarias las biopsias de
los bordes de la úlcera. Por tanto, en este caso, al haber llegado al diagnóstico por estudio
baritado, sería lo correcto realizar el test del aliento para comprobar si hay colonización por H.
pylori.(R3)

109. Los RN post término y post maduros se caracterizan por lo siguiente, EXCEPTO:

1. 1. Lanugo.
2. 2. Cabello abundante.
3. 3. Uñas largas.
4. 4. Surcos palmares supernumerarios.
Gráfico de respuestas
Comentario

En el recién nacido postérmino presentará piel seca, turgente, elasticidad disminuida, quebradiza,
con descamación y pálida, puede encontrarse teñida de meconio verde o amarillento. Las uñas son
largas, secas y quebradizas. El cabello es largo y seco. Los pliegues palmares y plantares
muestran gran cantidad de surcos supernumerarios.

El lanugo es un dato clínico de prematurez. Respuesta 1 correcta.(R1)

110. Mencione el tipo de degeneración más frecuente que se presenta en los miomas
uterinos.

1. 1. Degeneraron lipomatosa.
2. 2. Degeneración hialina.
3. 3. Degeneración quística.
4. 4. Degeneración roja y carnosa.
Gráfico de respuestas
Comentario

La degeneración hialina es la más fecuente, presente casi en todos los miomas, y consiste en el
desplazamiento de varias fibras musculares por sustancia amorfa y colágena.(R2)

111. Recién nacido niño de 41 semanas + 2 días de edad gestacional, peso 3,700 gramos,
nacido mediante cesárea urgente por riesgo de pérdida de bienestar fetal durante el parto
inducido. Líquido meconial en cantidad normal, teñido de meconio (+/+++). El Apgar al
minuto y cinco minutos es de 8/9. Precisa reanimación con secado, calor radiante y
aspiración orofaríngea de de secreciones levemente teñidas. Posteriormente inicia
lactancia materna con buena succión. A las 4 horas de vida es explorado: presenta buen
estado general, color sonrosado salvo cianosis de manos y pies hasta los tobillos.
Cabeza y cuello normales, no signos de fractura, paladar integro. No signos de
respiratoria. AC: Rítmico, soplo sistólico I-II/VI. AP: Buena ventilación bilateral. Abdomen

 
 
 
 
blando y depresible, no doloroso, se palpa hígado 1 cm bajo el reborde costal y dos
pequeñas masas a nivel de ambas fosas lumbares. Ano normal. El niño está dormido
aunque se despierta y llora durante la exploración. La fontanela es normal, tiene un buen
tono muscular y los reflejos arcaicos están presentes y son normales. Genitales
masculinos, fimosis y el hemiescroto derecho es más oscuro que el contralateral y está
duro. No duele a la palpación y la transiluminación es negativa. Señale la afirmación
CORRECTA:

Habría que hacer un ultrasonido abdominal para descartar anomalías renales y de la vía
1. 1.
urinaria que se asocian a su patología genital.
Si no se hubiera demorado la exploración es probable que la cirugía todavía podría haber
2. 2.
sido curativa.
En este momento está recomendada la actitud conservadora y observar evolución en los
3. 3.
próximos años.
4. 4. La clínica neonatal de esta entidad es diferente a la observada en niños más mayores.
Gráfico de respuestas
Comentario

La torsión testicular neonatal es diferente a la del niño mayor. En el recién nacido no es dolorosa.
La mayoría de ellas ocurren intraútero, por una fijación incompleta de la túnica vaginal a la pared
escrotal (se denomina torsión extravaginal) y en el momento del nacimiento se detecta una masa
escrotal dura, firme, no dolorosa.

El escroto suele estar edematoso y equimótico. La trasiluminación es negativa lo que la distingue


del hidrocele congénito. Está indicada la cirugía ya que el testículo ya es inviable, con fijación del
testículo contralateral, ya que tiene más riesgo de torsión.

El hidrocele es una colección líquida que rodea al testículo y puede ser comunicante (abierta la
comunicación con la cavidad abdominal y con cambios de tamaño a lo largo del día o no
comunicante. En ambos casos está indicado retrasar la cirugía hasta los 2- 2 ½ años ya que la
mayoría se reabsorben solos.(R4)

112. Niña de 3 años de edad, procedente de un asentamiento humano, cuya madre refiere
que presenta dolor urente en región infraescapular derecha, horas antes del ingreso. Al
examen se observa una zona edematosa de 3 x1 cm, bordes regulares, equimotica,
rodeada por un halo eritematoso. El diagnostico presuntivo es mordedura por:

1. 1. Serpiente venenosa
2. 2. Latrodectus mactans
3. 3. Escorpión
4. 4. Loxosceles laeta
Gráfico de respuestas
Comentario

Es una araaña. Dentro del género de las loxosceles, es la que posee mayor distribución en Sud
América, y es sin duda la más tóxica y peligrosa.

Despuésd e 4 o 8 horas, el area de la mordedura se siente dolorosa y con prurito, presentando una
induración central, rodeada de una área pálida de isquemia y una zona de eritema. En general no
existe adenopatia regional.(R4)

 
 
 
 
113. ¿Cuál de las siguientes afirmaciones NO es cierta con respecto al tromboembolismo
pulmonar?:

1. 1. Los émbolos sólo producen infartos pulmonares en una minoría de casos.


2. 2. La congestión pulmonar favorece la formación de infartos.
Aproximadamente la mitad de los émbolos proceden de las venas profundas de muslos y
3. 3.
piernas.
4. 4. Sólo el 50% de los pacientes presentan síntomas o signos de flebitis periféricas.
Gráfico de respuestas
Comentario

El TEP es una enfermedad muy importante para el ENARM. Debes dominar la clínica, el
diagnóstico y el tratamiento, aunque algunas veces también se ha preguntado sobre los factores
predisponentes, que son todos aquellos que determinan una hipercoagulabilidad. La respuesta
claramente falsa es la 3. En el TEP, la inmensa mayoría de los émbolos proceden de las venas
profundas de muslos y piernas, no sólo el 50%. Recuerda que la cirugía es un factor predisponente
muy importante, sobre todo la artroplastia de cadera, seguida de las intervenciones sobre la rodilla.
También son predisponentes, aunque en menor medida, la cirugía prostática y la oncológica
femenina, en cuyo caso los émbolos procederían más bien de las venas pélvicas. Por tanto, la
mayor parte de los émbolos se originan en las venas del territorio iliofemoral.(R3)

114. Un mioma producirá con mayor frecuencia:

1. 1. Sangrado abundante en el momento esperado de la regla (hipermenorrea).


2. 2. Hemorragia acíclica (metrorragia).
3. 3. Reglas muy frecuentes (polimenorrea).
4. 4. Flujo vaginal serosanguinolento.
Gráfico de respuestas
Comentario

La clínica característica del mioma submucoso es la hipermenorrea, o sangrado abundante en el


momento de la regla.(R1)

115. Paciente de 4 años que acude a Urgencias con una historia de secreción nasal
purulenta y de mal olor unilateral desde hace 4 días. El diagnóstico más frecuente es:

1. 1. Atresia de coanas unilateral.


2. 2. Rinitis crónica por rinovirus.
3. 3. Cuerpo extraño intranasal.
4. 4. Granulomatosis crónica nasofaríngea.
Gráfico de respuestas
Comentario

En ocasiones, los niños pequeños se introducen pequeños objetos en la nariz, en un intento


normal de explorar sus propios cuerpos. Lo que se introducen puede ser muy variable: alimentos,
semillas, borradores, juguetes. Es un cuadro relativamente frecuente y, sólo por la edad del niño,
deberías haberte planteado seriamente responder la opción 3. Por otra parte, la clínica que nos
describen encaja bastante bien:

•   Olor fétido o secreción nasal sanguinolenta.


•   Dificultad para respirar por la fosa nasal afectada.

 
 
 
 
•   Sensación de ocupación en la nariz.

(R3)

116. ¿Cuál de los siguientes datos NO es típico de la bronquiolitis?:

1. 1. Bradipnea.
2. 2. Palpación del hígado y bazo.
3. 3. Retracción intercostal.
4. 4. Febrícula.
Gráfico de respuestas
Comentario

Esta pregunta sobre las manifestaciones clínicas de la bronquiolitis es importante los hallazgos
exploratorios son TAQUIPNEA y signos de dificultad respiratoria (tiraje, retracción intercostal y
esternal, aleteo nasal...) Podemos palpar hígado y bazo descendidos por la hiperinsuflación
pulmonar. En la auscultación: espiración alargada, estertores, y sibilancias. El paciente suele estar
afebril o con febrícula.(R1)

117. A un RNT de peso adecuado para la edad gestacional, nacido de parto eutócico (en
el cual el ginecólogo lo elevó sobre el plano materno durante un tiempo generoso sin
antes pinzar el cordón umbilical), que no tiene antecedentes familiares de anemia, le
encontramos pálido nada más nacer. En la biometría hemática se detecta Hb: 12 g/dl, test
de Coombs negativo y los glóbulos rojos (G.R.) son normocrómicos y normocíticos ¿Qué
test NO dejaría de realizar bajo ningún concepto?:

1. 1. Estudio de Hb.
2. 2. Estudio de fragilidad osmótica de GR.
3. 3. Estudio enzimático de GR.
4. 4. Test de Kleihauer-Betke.
Gráfico de respuestas
Comentario

El test de Kelihauer-Betke es un test que se realiza en sangre materna. Este test se considera
positivo si aparece una doble población de hematíes: por un lado, la población normal de una
mujer adulta (rica en hemoglobina adulta), y por otro lado una población rica en hemoglobina fetal.
La presente prueba se realiza en casos de sospecha de transfusión feto-materna (como en el caso
del enunciado de esta pregunta), que puede llegar a ser responsable de anemia, generalmente de
escasa entidad, en el RN.(R4)

118. ¿Cuál sería el diagnóstico más probable en un paciente de 55 años, que comienza
con síntomas de obstrucción nasal y secreción purulenta, proteinuria, elevación muy
importante de la velocidad de sedimentación y positividad de "c-ANCAS" (anticuerpos
anticitoplasma de neutrófilos tipo c)?

1. 1. Sinusitis purulenta por Haemophilus.


2. 2. Síndrome de Goodpasture.
3. 3. Granulomatosis linfomatosa.
4. 4. Granulomatosis de Wegener.
Gráfico de respuestas
Comentario
 
 
 
 
Recuerda el cuadro de granulomas e inflamación necrotizante en los vasos de las vías
aéreassuperiores ( 95% sinusitis),asociado a glomerulonefritis focal y segmentaria ( 75%). Es típica
la secreción hemorrágica o purulenta de la vía aérea.
Otro de los aspectos típicos es la presencia de infiltrados pulmonares bilaterales , cavitados,no
migratorios.Es bastante específico de la enfermedad de Wegener la presencia de c- ANCA
positivo, sobre todo si asocia un cuadro como el descrito.(R4)

119. Hombre de 75 años que presenta


sangre oculta en heces positiva, estreñimiento de reciente aparición y pérdida de 5 kg de
peso. Se realiza una colonoscopia cuyo resultado se muestra en la imagen. ¿Cuál de las
siguientes afirmaciones es FALSA?

1. 1. El uso regular de aspirina reduce el riesgo de padecer esta patología.


2. 2. Los familiares de primer grado deben realizarse una colonoscopia de screening.
Más del 80% de los pacientes asintomáticos con prueba positiva para sangre oculta en
3. 3.
heces padecen esta patología.
4. 4. Los pólipos con mayor riesgo de degeneración son los adenomatosos vellosos.
Gráfico de respuestas
Comentario

Teniendo en cuenta las manifestaciones clínicas y la masa irregular que vemos en la colonoscopia,
evidentemente habría que sospechar un cáncer de colon.

Es cierto que la aspirina es un factor protector para su aparición, y que los pólipos adenomatosos
requieren seguimiento, sobre todo si son vellosos, ya que son precisamente éstos los que
producen mayor riesgo.

También es correcta la 2, ya que este tipo de cáncer tiene un importante carácter hereditario, igual
que sucede en el cáncer de mama.

 
 
 
 
Sin embargo, la opción 3 está claramente equivocada. Un paciente asintomático con test de sangre
oculta en heces positivo pocas veces padece un cáncer de colon. De hecho, la mayoría de los que
tienen un test positivo no lo padecen, con lo que el valor predictivo positivo no es un 80%, sino muy
inferior (20- 30%).(R3)

120. En el tratamiento de la tuberculosis es FALSO que:

1. 1. Con tratamiento correcto la tasa de recidiva oscila en torno al 15%.


2. 2. El esputo es estéril a los dos meses en la gran mayoría de los pacientes.
3. 3. Una vez acabado un tratamiento correcto no hay por qué seguir revisiones de por vida.
4. 4. En general no hay que hacer revisiones periódicas de enzimas hepáticas.
Gráfico de respuestas
Comentario

Si se realiza correctamente el tratamiento antituberculoso, las recidivas son absolutamente


excepcionales, por lo que un 15% muy superior a la cifra real (respuesta 1 incorrecta). Analicemos
el resto de las opciones:

- En más del 80% de los pacientes, ya no pueden cultivarse micobacterias a partir de los esputos
(opción 2 correcta) cuando pasan dos meses de tratamiento.

- La opción 3 es una cuestión de pura lógica. Las enfermedades con tratamiento efectivo no
precisan seguimiento de por vida: se curan y se resuelve el problema. Es el caso de la
tuberculosis, mientras la evolución del paciente sea satisfactoria una vez instaurado el tratamiento.

- Los efectos secundarios de los fármacos son más frecuentes en pacientes infectados por el VIH.
Esto es debido a las alteraciones de su sistema inmune, a la mayor comorbilidad que asocian y a
que muchas veces están polimedicados (respuesta 4 correcta).

- En pacientes en tratamiento con isoniacida, se realiza un primer control donde se evalúan las
transaminasas, retirándose el fármaco si se quintuplica la cifra normal, o si se triplica, en caso de
que el paciente tenga una hepatopatía de base. Esta primera revisión se realiza a todos. En
cambio, las “revisiones periódicas” (es lo que dice la pregunta exactamente) sólo se realizan en
pacientes con factores de riesgo para hepatotoxicidad.(R1)

121. A 3-month-old infant is brought to the pediatrician’s consultation for a routine check-
up. He presents increased peripheral pulses and a continuous (systolic and diastolic)
murmur that radiates to chest. He had a difficult delivery and presented respiratory
distress at birth, but was otherwise healthy. What is the most likely diagnosis?

1. 1. Patent ductus arteriosus


2. 2. Interventricular septal defect
3. 3. Tetralogy of Fallot
4. 4. Congenital lung hypoplasia

(R1)

 
 
 
 

122. Indique cuál de los siguientes


hallazgos NO es sospechoso de malignidad en una mastografía de screening:

1. 1. Retracción de la piel de la mama.


2. 2. Edema alrededor de la areola mamaria.
3. 3. Calcificaciones groseras.
4. 4. Densidad focal asimétrica.
Gráfico de respuestas
Comentario

Las calcificaciones sugestivas de malignidad son finas y con tendencia a la agrupación. Una
calcificación grosera sería más propia de una mastopatía fibroquística o de otras entidades que, en
cualquier caso, no se consideran malignas.(R3)

123. Paciente de 59 años que acude a consulta de ginecología para revisión. Entre sus
antecedentes destaca la existencia de mastodinia cíclica con aumento de densidad
mamaria actualmente asintomática. En la exploración no se detectan tumoraciones a
nivel de las mamas ni adenopatías axilares. Se realiza mastografía de la mama derecha
en la que aparece lo que se muestra en la imagen. ¿Cuál sería la conducta terapéutica
más apropiada?:

La mamografía es normal, por lo que continuaremos con el screening normal con nuevo
1. 1.
control en 2 años.
2. 2. Complementar el estudio con ecografía, y si esta es normal, nuevo control en 1 año.
3. 3. Punción de la lesión en consulta con citología de la misma.
4. 4. Biopsia diferida previo marcaje con arpón.
Gráfico de respuestas
Comentario
 
 
 
 
Aunque en esta paciente no encontramos ningún nódulo en la palpación, la ecografía nos revela un
aumento de densidad de bordes imprecisos, de localización focal. Este hallazgo debe considerarse
sospechoso de malignidad, por lo que es imperativo tomar una muestra para estudio histológico.

Teniendo esto en cuenta, la duda razonable estaría entre las opciones 3 y 4. La respuesta 3 no
puede ser correcta: no puede realizarse una PAAF, porque no existe ninguna zona identificable al
tacto (no hay nódulo). Realizar la punción sería como pinchar a ciegas. Por lo tanto, necesitaremos
marcar la zona con arpón, para obtener la biopsia de la región que nos interesa (respuesta 4
correcta).(R4)

124. Respecto al carcinoma de mama, ¿cuál de las siguientes afirmaciones NO es


correcta?:

1. 1. El tipo anatomopatológico más frecuente es el carcinoma ductal infiltrante.


2. 2. El carcinoma intraductal no evoluciona a lesión palpable.
3. 3. Es muy frecuente la extensión de la enfermedad a los ganglios linfáticos.
4. 4. El principal factor pronóstico es la afectación ganglionar.
Gráfico de respuestas
Comentario

La respuesta incorrecta es la 2. El carcinoma intraductal, en caso de no extirparse a tiempo, crece


y evoluciona hacia una lesión palpable, como cualquier otra neoplasia de carácter maligno. Otra
cuestión es que esto sea más o menos frecuente (cada vez menos, gracias a la detección precoz
mediante mamografía), pero no puede decirse que NO evolucione, porque sí puede hacerlo.(R2)

125. 36 hours after birth, a newborn presents with weak crying, hypotonia and seizures.
He has slight macrocephaly with microphthalmus and bilateral choriorretinitis. Physical
examination at birth showed hepatosplenomegaly. When he was 12 hours old, he was
found to have jaundice and petechiae, especially located in skin folds. Which finding
would you expect to find in a brain CT scan of this patient?

1. 1. Periventricular calcifications.
2. 2. Cerebellar hypoplasia.
3. 3. Diffuse cerebral edema.
4. 4. Diffuse calcifications.

(R4)

126. En el ciclo cardíaco normal, ¿cuál de las siguientes respuestas es la cierta?

1. 1. La onda v del pulso venoso coincide con la sístole auricular.


El cierre de la válvula aórtica precede al de la pulmonar, especialmente durante la
2. 2.
inspiración.
La contracción auricular activa, puesta de manifiesto por la onda P del ECG, es diastólica
3. 3.
precoz.
Cuando la frecuencia cardíaca es superior a 100 lpm, el espacio QT del ECG es 0,46
4. 4.
segundos.
Gráfico de respuestas
Comentario

 
 
 
 
Pregunta de dificultad media que requiere el conocimiento del ciclo cardíaco, en relación con la
onda del pulso venoso yugular, así como varias nociones básicas de la función cardíaca para su
resolución.

La respuesta 1 es falsa, pues la onda v del pulso yugular coincide con el llenado venoso auricular
en la cual la válvula auriculoventricular está cerrada coincidiendo con el fin de la sístole y el
principio de la diástole (relajación isovolumétrica) ventricular. Posteriormente se igualan las
presiones, se abre la válvula y se produce el seno de la presión venosa. La onda que coincide con
la contracción auricular es la onda a (que desaparece en la FA), por lo que la respuesta 1 sería
incorrecta.

La respuesta 2 es la correcta, pues en condiciones normales las válvulas izquierdas siempre se


cierran antes que las derechas (regla nemotécnica: siempre por orden alfabético Aórtica antes que
Pulmonar y Mitral antes que Tricuspídea) especialmente durante la inspiración profunda, en la cual
la presión negativa torácica facilita la mayor entrada de sangre en cavidades derechas, lo que
condiciona un mayor volumen eyectivo en éstas y tardan más en cerrarse las válvulas tricúspide y
pulmonar (fenómeno de Rivero-Carvallo).

La respuesta 3 es falsa, dado que siempre queda un volumen residual en el corazón tras la sístole
(volumen telesistólico final) que es de un 25-40% del total, correspondiendo a un 60-75% de
fracción de eyección, que es bastante menos de lo que indica la respuesta.

La respuesta 4 también es falsa puesto que la contracción auricular corresponde a la fase final
diastólica ventricular, fase de llenado ventricular con ayuda auricular (hasta un 25% del total) muy
importante, sobre todo en pacientes con insuficiencia diastólica en los cuales el paso de ritmo
sinusal a FA descompensa a estos pacientes por no conseguir un volumen telediastólico final
suficiente para conseguir la fracción de eyección necesaria.

La respuesta 5 es falsa, y la explicación es que la frecuencia cardíaca mayor de 100 lpm no


determina el aumento del QT sino el acortamiento, siendo por el contrario causas importantes de
QT largo bradiarritmias (bloqueo AV de tercer grado), hipopotasemia, hipomagnesemia,
hipocalcemia, antidepresivos tricíclicos, etc.(R2)

Ciclo cardíaco

 
 
 
 

 
 
 
 
127. Una paciente de 25 años consulta por adenopatías laterocervicales y axilares. Dice
haber perdido peso durante las últimas semanas, así como ocasional fiebre vespertina y
sudoración nocturna. Asimismo, dice haber notado dolor cervical en algunas ocasiones
cuando sale los fines de semana, aunque no identifica la causa. La biopsia ganglionar
muestra bandas de fibrosis que delimitan nódulos, así como la presencia de células
binucleadas en ojo de búho y células lacunares. Teniendo en cuenta la enfermedad que
padece, señale cuál no sería uno de los llamados síntomas B:

1. 1. Fiebre.
2. 2. Sudoración nocturna.
3. 3. Signo de Hoster.
4. 4. Todas las opciones son síntomas B.
Gráfico de respuestas
Comentario

El signo de Hoster fue preguntado en la convocatoria MIR 2007- 2008 y se define como dolor en
las adenopatías asociado con el consumo de alcohol característico de pacientes con linfoma de
Hodgkin. No se incluye dentro de los síntomas B (fiebre, sudoración nocturna y pérdida de
peso).(R3)

128. ¿Cuál de los siguientes sigue una curva unimodal con máximo en el día 21-22 del
ciclo genital femenino?:

1. 1. GnRH.
2. 2. FSH.
3. 3. Estradiol.
4. 4. Progesterona.
Gráfico de respuestas
Comentario

La liberación de la progesterona es unimodal, tiene un pico en la fase secretora que alcanza el


nivel máximo 8 días tras el pico de LH (es decir 8 días tras la ovulación: 14 y 8 son 21 o 22). Sin
embargo, los estrógenos tiene una liberación bimodal, crecen hasta un pico preovulatorio (24- 36 h
antes de la ovulación)y tiene otro pico menor en la fase lútea. La liberación de GnRH se produce
de manera pulsatil, de modo que los pulsos lentos sobrestimulan FSH, y los rápidos LH. LA
liberación de FSH tiene dos fases un primera meseta pequeña en la primera mitad de la fase
proliferativa y un segundo pico justo antes de la ovulación. La LH tiene un solo pico, el pico
ovulatorio consecuencia del efecto gatillo de los estrógenos. La ovulación es consecuencia directa
de este pico de LH.(R4)

129. Un paciente varón de 52 años acude a su consulta porque, desde hace


aproximadamente un año, presenta debilidad muscular que ha ido en aumento
progresivamente, anorexia, náuseas, vómitos y, esporádicamente, diarrea y dolor
abdominal. Como antecedentes personales sólo refiere episodios recurrentes de disuria,
polaquiuria y tenesmo. Comenta que su médico de atención primaria le ha dicho "que no
sale nada en los cultivos" y le receta antibióticos "que no le hacen nada". En la
exploración, usted detecta una discreta hiperpigmentación de la cara y el cuello, los
pliegues palmares y la mucosa oral. La tensión arterial es de 90/60. Solicita una analítica
en la que destacan: Na 130 meq/l; K 5,8 meq/l y glucosa 65 mg/dl. ¿Qué diagnóstico
sospecha?

 
 
 
 
1. 1. Acantosis nigricans paraneoplásica.
2. 2. Enfermedad de Addison de origen autoinmune.
3. 3. Tuberculosis genitourinaria y de glándulas suprarrenales.
4. 4. Síndrome carcinoide.
Gráfico de respuestas
Comentario

Ante un cuadro de hiponatremia, hiperpotasemia e hipoglucemia, habría que considerar una


posible insuficiencia suprarrenal. La duda, en este caso, estaría entre las opciones 3 y 4. Aunque la
causa más frecuente es autoinmune, esto no justificaría la clínica urinaria que nos mencionan. En
cambio, la tuberculosis genitourinaria encaja perfectamente en esto (de ahí que no responda a
antibióticos, ni crezca ningún germen en el urocultivo, porque no lo estarán sembrando en el medio
adecuado, que es el de Lowestein).(R3)

130. Todos los siguientes agentes se consideran modificadores de la enfermedad en el


tratamiento de la artritis reumatoide EXCEPTO:

1. 1. Prednisona.
2. 2. D-penicilamina.
3. 3. Sulfasalacina.
4. 4. Hidrocloroquina.
Gráfico de respuestas
Comentario
Los fármacos modificadores de la enfermedad son la base del tratamiento de la AR. El metotrexate
es el de elección y ha relegado a los que se mencionan en esta pregunta. El tratamiento con oro
(Crisoterapia) prácticamente se ha abandonado. La administración parenteral (aurotiomalato
sódico) es más eficaz pero tambien más tóxica, mientras que la administración oral (auranofin) es
más segura pero con un efecto más limitado. La D- penicilamina es tan eficaz como las sales de
oro parenterales pero algo más tóxico ya que produce los mismos efectos secundarios y algunos
peculiares como la inducción de enfermedades autoinmunes por lo que también se ha abandonado
su uso. La sulfasalacina y la hidroxicloroquina mantiene un papel en el manejo de la AR, como
único fármaco en pacientes con forma leves y en combinación con metotrexate en lo que
denominamos triple terapia, útil en pacientes en los que la respuesta al metotrexate no haya sido
satisfactoria y como paso previo a la administración de terapia anti TNF. Los corticoides no se
consideran fármacos modificadores de la enfermedad aunque en ocasiones se ha atribuido al
tratamiento con ellos el retraso en la aparición de erosiones.(R1)

131. A 48-year-old patient is seen in the emergency department because of severe pain in
the left eye that started abruptly while he was watching television. Physical examination
reveals redness in his left eye, slight corneal edema and a nonreactive mid-dilated left
pupil. Which of the following options is incorrect?

1. 1. The patient probably has elevated intraocular pressure in his left eye.
2. 2. It is more frequent in patients with aphakia (without lens).
3. 3. This patient probably has a shallow anterior chamber.
4. 4. The patient will most likely report seeing halos around lights.
Gráfico de respuestas
Comentario

 
 
 
 
Pregunta relativamente sencilla sobre el glaucoma agudo. La dificultad de la pregunta radica en
identificarlo, ya que las opciones que nos ofrecen son bastante fáciles. Una persona sin cristalino,
es decir, afáquica, no tendría por qué tener un mayor riesgo de glaucoma.

De hecho, sería al contrario, puesto que al no existir cristalino el humor acuoso circularía con
libertad (respuesta 2 falsa).

Sobre esta enfermedad, han aparecido bastantes preguntas acerca del tratamiento. Vamos a
aprovechar la ocasión para revisar algunos detalles relevantes:

•   El manitol y la acetazolamida son diuréticos osmóticos, y se usan para disminuir lo más


rápido posible la presión intraocular.

•   Los corticoides tópicos servirán para disminuir el componente inflamatorio.

•   NO SE DEBEN DAR MIDRIÁTICOS. Esto sólo aumenta el bloqueo pupilar (recuerde que
la pupila del glaucoma está en midriasis media). Lo que debe dar son MIÓTICOS, como la
pilocarpina u otros fármacos colinérgicos, para así romper el bloqueo pupilar.

(R2)

132. Indique cual de las siguientes situaciones clínicas constituye una contraindicación
para la lactancia materna:

1. 1. Prematurez.
2. 2. Galactosemia.
3. 3. Madre con diagnostico de hepatitis B.
4. 4. Esquizofrenia.
Gráfico de respuestas
Comentario

Pregunta fácil. Entre las contraindicaciones para la lactancia materna se encuentra galactosemia,
fenilcetonuria, infección por VIH (en México, pero no es absoluta en otros paises).(R2)

133. A 50-year-old male is diagnosed with gastroesophagic reflux disease (GERD), and
he is started on ranitidine, with adequate control of his symptoms. Six months later, he
comes to your office for a follow-up visit, where he complains that his symptoms have
reappeared. When asked about any changes on his medications, he explains that he
started a new drug 3 months ago, after he was diagnosed with another disease. Which of
the following is most likely the cause of the worsening of his GERD symptoms?

1. 1. Fluoxetine
2. 2. Hydrochlorothiazide
3. 3. Propranolol
4. 4. Nifedipine
Gráfico de respuestas
Comentario
Nifedipine. There are many factors that affect the lower esophageal sphincter. Among those that
decrease lower esophageal sphincter pressure we can find beta blockers, alpha blockers,
anticholinergics, fats, ethanol or calcium channel blockers such as nifedipine.(R4)

 
 
 
 
134. ¿Cuál es la anomalía funcional que mejor define la EPOC?:

1. 1. Disminución del flujo inspiratorio.


2. 2. Disminución del volumen residual.
3. 3. Disminución del flujo espiratorio.
4. 4. Disminución de la ventilación.
Gráfico de respuestas
Comentario

Pregunta sencilla acerca de la fisiopatología del EPOC y que no debes dudar porque en este tema
hay que tener claro todos los conceptos relacionados con ella. El EPOC se caracteriza
fundamentalmente por un aumento de la resistencia de la vía aérea al paso del aire. Esto provoca
la limitación al flujo aéreo ESPIRATORIO por alteraciones de las vías aéreas pequeñas por fibrosis
e inflamación (y dudosamente por hipersecreción de la mucosa), y/o por la disminución del soporte
elástico de estas vías por destrucción de paredes y septos alveolares (que a modo de resorte
traccionan de ellas para mantenerlas abiertas). Por ello en el EPOC se produce un atrapamiento
aéreo que produce un aumento del VR, típico de las enfermedades obstructivas. Como ves las
opciones 1, 2 y 4 son falsas. La disminución de la ventilación, que se caracterizará sobre todo por
un aumento de la CO2, aunque puede darse en estadios finales y en las reagudizaciones, no es la
anomalía funcional principal del EPOC.(R3)

135. En la histopatología del lupus eritematoso sistémico con afectación renal NO se


encuentra:

1. 1. Depósitos subendoteliales en asa de alambre.


2. 2. Proliferación mesangial.
3. 3. Proliferación extracapilar.
4. 4. Hiperplasia tubular.
Gráfico de respuestas
Comentario
La histología de los estadios de la nefritis lúpica es un tema complicado si no lo estudias con
detenimiento. Los hallazgos descritos en las opciones corresponden (salvo la hiperplasia tubular)
con el estadio 4 o GN proliferativa difusa lúpica que es la forma más grave y más frecuente en
enfermos sintomáticos. Recuerda que la proliferación extracapilar son las denominadas
?semilunas? y que los cuerpos hematoxilinofílicos son patognomónicos. El pronóstico de este
estadio es ominoso y aboca a la insuficiencia renal pese al tratamiento en un 20% de los
casos.(R4)

136. La conducta a seguir en una paciente gestante del primer trimestre con DIU es:

1. 1. Prohibir las relaciones sexuales.


2. 2. Continuar embarazo.
3. 3. Aborto terapéutico.
4. 4. Retirar DIU.
Gráfico de respuestas
Comentario

Si una mujer con DIU se queda embarazada, es recomendable retirarlo cuanto antes, porque
puede dar complicaciones: abortos y parto pretérmino, entre otras. Respuesta 4 correcta.(R4)

 
 
 
 

137. Hombre de 48 años que acude


a consulta por rectorragia de 2 meses de evolución sin dolor anal. En los exámenes de
laboratorio destaca: Hb de 11.6 g/dl, VCM 62, leucocitos 5 300 con fórmula normal,
plaquetas 300 000, glucosa 98, urea 58 mg/dl, creatinina 1.1 mg/dl, AST 32, ALT 22, GGT
14, bilirrubina 0.6 mg/dl, fosfatasa alcalina 68, sodio 136, potasio 4.1. Se realizó una
colonoscopia que mostraba la imagen adjunta a 55 cm de margen anal. ¿Cuál sería la
actitud más adecuada en este momento?:

Biopsia de la lesión ya que lo más probable es que exista displasia grave y remitiría a
1. 1.
cirugía dado su gran tamaño.
2. 2. Polipectomía.
3. 3. Continuaría hasta ciego y sólo realizaría polipectomía si tiene un mínimo de 3 pólipos.
4. 4. Inyección en su base con oleato de etanolamina.
Gráfico de respuestas
Comentario

El paciente consulta por una anemia microcítica de varios meses de evolución, que parece
relacionada con el hallazgo endoscópico que nos muestran. Como puede ver, se trata de una
lesión pediculada (polipiode), aparentemente no ulcerada, pero de cierto tamaño.

No es obligado que se trate de una neoplasia maligna, pero tampoco es descartable, por lo que la
actitud recomendable sería la extirpación (vía endoscópica) y el estudio histológico de la pieza.

No tiene sentido la respuesta 4 (biopsia y extirpación sólo si es un pólipo adenomatoso), porque


eso no resuelve el problema del síndrome anémico por el que consulta este paciente. Aunque no
se trate de una neoplasia maligna, lo más probable es que sea la responsable del sangrado y,
aunque no lo fuese, el tamaño hace muy recomendable la extirpación.(R2)

 
 
 
 
138. Se progresó hasta ciego sin encontrar más lesiones. El paciente posteriormente nos
refiere que su padre tuvo cáncer de colon a los 62 años. ¿Cuál debería ser su actitud
siguiente?:

1. 1. Revisión con colonoscopia cada 5 años.


2. 2. Revisión con colonoscopia cada 10 años.
3. 3. Revisión con colonoscopia a los 6 meses para comprobar que no recidivado.
4. 4. Debe realizarse colectomía ya que se trata de síndrome de cáncer hereditario.
Gráfico de respuestas
Comentario

Se ha demostrado una reducción estadísticamente significativa de la mortalidad por cáncer


colorrectal cuando se realizan colonoscopias en personas con test de sangre oculta en heces
positiva. Dado que, en este caso, tenemos una lesión que muy probablemente ha sido la
responsable del cuadro anémico, y además existen antecedentes familiares de cáncer de colon,
las colonoscopias (cada 3-5 años) estarían claramente justificadas, según las recomendaciones de
la American Cancer Society (respuesta 1 correcta).(R1)

139. Un niño de 5 años llega a Urgencias con convulsiones generalizadas que han
comenzado 45 minutos antes. La anamnesis y el examen físico no revelan ninguna causa
desencadenante específica. El tratamiento inicial incluye la administración de oxígeno y
anticonvulsivantes. Está en condiciones de intubar y ventilar al niño, si fuera necesario.
De los siguientes estudios, ¿cuál NO estaría indicado?:

1. 1. Un ECG.
2. 2. Una TC de cráneo.
3. 3. Glucemia y electrolitos séricos.
4. 4. Niveles séricos de amoníaco y ácido láctico.
Gráfico de respuestas
Comentario

En este caso al tener un paciente de 5 años con crisis convulsivas deberá de realizar TC de cráneo
para descartar alteraciones estructurales, medir glucosa y electrolitos séricos ya que alteraciones
en estos parámetros podría causarlas y algunas enfermedades del metabolismo podrían elevar los
niveles séricos de amoniáco y ácido láctico. La realización de un ECG no es prioritario. Respuesta
1.(R1)

140. Sobre el síndrome de compresión radial indique la respuesta INCORRECTA:

Se comprime fundamentalmente en el arco fibroso proximal del supinador (arcada de


1. 1.
Fröshe).
2. 2. Cursa con dolor en la masa muscular distal al epicóndilo
3. 3. El tratamiento debe ser quirúrgico porque de forma conservadora no puede mejorar.
4. 4. Si la compresión no cede puede acabar con debilidad de la musculatura extensora.
Gráfico de respuestas
Comentario
El síndrome de compresión del nervio radial ocurre en tres zonas fundamentalmente: 1. Tabique
intermuscular lateral. 2. Arco fibroso proximal del músculo supinador. 3. Tercio distal del antebrazo.
El lugar de compresión más frecuente es el segundo (Respuesta 1correcta) y se caracteriza por
dolor en la musculatura epicondílea incluso se puede confundir con una epicondilitis (Respuesta 2
correcta). Refieren dolor nocturno, consecuente a esfuerzos. Tres maniobras pueden

 
 
 
 
desencadenar la clínica: presión sobre el punto de compresión, supinación forzada contra
resistencia e hiperextensión del tercer dedo con la muñeca y el codo extendidos (Respuesta 3
correcta). Cuando la compresión es intensa y de larga duración se puede producir una debilidad de
la musculatura extensora (Respuesta 5 correcta). El tratamiento es inicialmente conservador con
modificación de las actividades que desencadenan el dolor (sobre todo la pronosupinación contra
resistencia) (Respuesta 4 INCORRECTA). En casos refractarios se procede a una liberación
quirúrgica.(R3)

141. Señale la FALSA con respecto a la trombocitemia esencial:

1. 1. La célula más primitiva en la que se identifica la alteración clonal es el megacariocito.


2. 2. Es el más frecuente de los síndromes mieloproliferativos.
3. 3. El diagnóstico es de exclusión.
4. 4. La evolución a leucemia aguda es muy rara.
Gráfico de respuestas
Comentario
Pregunta de baja dificultad. No os confundáis, la alteración primitiva de la trombocitemia esencial
(TE) esta localizada en la célula germinal pluripotencial (opción 1 falsa), y como consecuencia de
ello se ven afectados en mayor medida los megacariocitos. Aunque el diagnostico es de exclusión,
en la actualidad la TE es considerada como el síndrome mieloproliferatico más frecuente (opciones
2 y 4 correctas). La sintomatología más común se debe a trastornos en la microcirculación (opción
3 correcta) como dolores isquémicos en los dedos de los pies o en las plantas y palmas (conocidos
como eritromelalgia), acrocianosis, parestesias, cefalea pulsátil o vértigo. Son menos frecuentes
las trombosis arteriales o venosas de vasos de mayor calibre. Os recuerdo además, que los
fenómenos hemorrágicos son menos frecuentes que los tromboticos. Pero sin duda la evolución a
leucemia aguda es la manifestación menos frecuente de los pacientes con esta enfermedad
(opción 5 correcta) y solo esta aumentada esta complicación en aquellos tratados con 32P o
agentes alquilantes como la hidroxiurea.(R1)

142. Recién nacido, con gestación a término, parto por vía vaginal, que pesa 3,420 Kg y
mide 54 cm, con antecedente de interés de fiebre materna en el postparto inmediato, que
a los dos días de vida presenta fiebre de 38.9ºC, al que se añade dificultad respiratoria,
con disminución de actividad, aletargamiento y rechazo de las tomas de leche materna.
En la exploración física, el RN presenta clara hipoactividad, detectando roncus y
crepitantes bilaterales en la auscultación pulmonar. Las fontanelas no aparecen
deprimidas ni abombadas. Se realiza una Rx de tórax, en la que se observan infiltrados
intersticiales difusos en ambos pulmones. Ante la sospecha de una sepsis neonatal
precoz se solicitan hemocultivos, se realiza una punción para valorar el LCR (leucocitos
0; glucosa 47; proteínas 92; cultivo de LCR y tinción de Gram negativos) y se obtiene una
muestra de sangre (hemoglobina 12.4; leucocitos 21,000 (segmentados del 46%);
plaquetas 42,000; pH 7.32; pO2 136; pCO2: 32; bicarbonato: 16.3. En una sepsis neonatal
precoz, ¿cuál sería la terapia específica que iniciaría?:

1. 1. Infusión de prostaglandinas.
2. 2. Ventilación mecánica y administración de surfactante.
3. 3. Dieta absoluta prolongada y reserva de plaquetas.
4. 4. Ampicilina y aminoglucósido o cefalosporina de 3ª generación.
Gráfico de respuestas
Comentario

Esta pregunta no debería haberle causado ningún problema. El tratamiento empírico de la sepsis
neonatal precoz, al

 
 
 
 
igual que el de las sepsis neonatales tardías, incluye la administración de ampicilina (que cubre la
Listeria y S. agalactiae) y de un aminoglucósido (que cubre los gram negativos, como el E. coli). Si
hay meningitis concomitante, se sustituirá el aminoglucósido por una cefalosporina de tercera
generación de administración parenteral, ya que los aminoglucósidos no alcanzan buenas
concentraciones en el líquido cefalorraquídeo.(R4)

143. A 61-year-old female patient comes to the physician because of a palpable nodule in
her breast, measuring 1,5 cm in diameter. Mammography reveals a nodule measuring 9
mm in diameter, with spiculated margins and clustered microcalcifications located in her
right breast. Thick needle biopsy reveals an invasive ductal carcinoma (grade I). Further
studies show that the lesion stains positive for estrogen receptors and negative for Her-
2. Which of the following is the most appropriate treatment?

Modified radical mastectomy, hormone therapy and chemotherapy, regardless of the


1. 1.
lymph node involvement.
Tumorectomy combined with sentinel node biopsy. Radiotherapy, hormone therapy and
2. 2.
chemotherapy according to the result of the sentinel node biopsy.
Lymphadenectomy. Chemotherapy according to the result of the lymph nodes biopsy.
3. 3.
Mastectomy and tratuzumab.
Tumorectomía, radioterapia, linfadenectomía y danazol. Tumorectomy, radiotherapy,
4. 4.
lymphadenectomy and danazol.
Gráfico de respuestas
Comentario

Es un ejemplo excelente para ir revisando los criterios de aplicación de cada parte de la terapia
combinada del cáncer de mama:

Tumorectomía: SÍ porque se trata de un nódulo menor de 5 cm (mide 9 mm) y no se cumple


ninguna contraindicación de cirugía conservadora.

Exéresis del ganglio centinela: SÍ, siempre que se trate de un ca. infiltrante y hayamos descartado
afectación axilar clínica+ecográfica+citológica.

Radioterapia: SÍ siempre que hagamos cirugía conservadora, y también cuando haya alto riesgo de
recidiva local.

Quimioterapia: DEPENDE del ganglio centinela, estaría indicado si axila positiva o axila negativa
con factores de mal pronóstico.

Hormonoterapia: SÍ puesto que la aplicaremos siempre que haya receptores hormonales positivos.

Trastuzumab: NO porque nuestro tumor no expresa el gen Her2 neu.

Por tanto la respuesta correcta es la 2.(R2)

144. A 45-year-old with no significant past medical history and no current symptoms is
noted to have ventricular extrasystoles on a routine ECG. The extrasystoles are frequent
and homogeneous. Echocardiogram shows no abnormal findings and treadmill stress
testing reveals no signs of ischemia or arrhythmia. Which of the following is the most
appropriate management?

 
 
 
 
No therapy is needed initially. Perform a coronary angiography to rule out coronary
1. 1.
stenosis and prescribe treatment accordingly.
2. 2. Propafenone or Flecainide.
3. 3. Beta-blockers or sotalol and aspirin.
4. 4. No treatment is needed.
Gráfico de respuestas
Comentario

Las extrasístoles ventriculares son muy frecuentes y aparecen, en algún momento, en la mayoría
de las personas sanas, aunque casi siempre son totalmente asintomáticas. No obstante, hay un
reducido número de individuos que tiene una especial sensibilidad y notan las extrasístoles como
“vuelcos” del corazón, lo que les crea ansiedad. Esa sensación de “vuelco” suele ser por la pausa
compensadora que sigue a la extrasístole, que incrementa el llenado cardíaco y la contracción
siguiente es más fuerte.

Las extrasístoles ventriculares carecen de importancia real y no se asocian a arritmias malignas,


siempre y cuando el paciente tenga un corazón estructuralmente normal. Además, pueden ser
expresión de otra enfermedad subyacente, como el hipertiroidismo o la anemia, que conviene
descartar. Una vez que estamos seguros de que el paciente está sano (tal como nos expresan en
esta pregunta), lo más importante es tranquilizarle y explicarle qué factores pueden hacer que sean
más intensas y frecuentes (estrés, estimulantes como la cafeína o el tabaco…). Por ello, la
respuesta correcta es la 4.

Si, a pesar de esto, les producen muchas molestias, se puede emplear un betabloqueante, que
disminuye el número e intensidad de las extrasístoles. No está indicado usar antiarrítmicos por sus
efectos secundarios, que podrían ser incluso arritmias graves, ni tampoco aspirina, como dicen el
resto de las opciones.(R4)

145. Una mujer gestante presenta trombosis venosa profunda y embolismo pulmonar. De
los siguientes tratamientos indique el adecuado ante esta situación clínica:

1. 1. Administrar anticoagulantes orales.


Heparina no fraccionada intravenosa durante 5-7 días y posteriormente heparina
2. 2.
subcutánea hasta el momento del parto.
3. 3. Trombolíticos.
Heparina no fraccionada intravenosa durante 5-7 días y posteriormente anticoagulantes
4. 4.
orales hasta el momento del parto.
Gráfico de respuestas
Comentario
El tratamiento del TEP es un tema frecuente en el examen MIR. Es importante que domines la
heparina y los anticoagulantes orales, porque además se preguntan en muchas otras asignaturas.
Ante un TEP en un paciente estándar, lo habitual es comenzar el tratamiento con heparina,
pasando después a anticoagulantes orales. Sin embargo, estamos ante una mujer embarazada. En
esta situación, está contraindicado el uso de dicumarínicos o warfarina, puesto que son
teratógenos. Por tanto, habría que sustituirlos por heparina subcutánea hasta el momento del
parto. Recuerda que, entre los factores predisponentes a la TVP y al TEP, se encuentran el
embarazo, el puerperio, los anticonceptivos orales y la terapia hormonal sustitutoria, lo que se
explica por el efecto trombofílico de los estrógenos.(R2)

146. Hombre de 25 años, que vuelve de las costas de España, que presenta fiebre elevada
de una semana de evolución, acompañada de cefalea y artromialgias. Tres días antes del

 
 
 
 
ingreso, le aparece un rash maculopapular no pruriginoso, que incluye palmas y plantas.
En la exploración física, el único hallazgo es una lesión pustulosa en la pierna derecha
que posteriormente evoluciona hacia una mancha negra. ¿Cuál es el agente etiológico
más probable?

1. 1. Virus.
2. 2. Rickettsia conorii.
3. 3. Coxiella burnetii.
4. 4. Rickettsia rickettsii.
Gráfico de respuestas
Comentario

Un caso clínico muy representativo de la fiebre botonosa mediterránea. Si apareciese una pregunta
así en el ENARM, deberías acertarla sin problemas.

La rickettsiosis más frecuente en nuestro medio es la fiebre botonosa mediterránea, producida por
Rickettsia connorii. La transmite una garrapata, que produce una lesión cutánea característica,
denominada mancha negra, en el punto de inoculación. El resto del cuadro clínico es similar en el
resto de las rickettsiosis: fiebre, malestar general, mialgias, cefalea intensa y lesiones cutáneas
eritematosas que afectan a palmas y plantas.

El diagnóstico de las rickettsiosis es serológico. Antiguamente, se empleaba la reacción de


Weil- Felix, pero actualmente está en desuso. El tratamiento de elección es la doxiciclina, asociada
a corticoides en las formas más graves.(R2)

147. En relación con el cáncer de mama en estadios localizados, ¿cuál de las siguientes
afirmaciones es FALSA?

1. 1. La cirugía conservadora con radioterapia radical no compromete la supervivencia.


La radioterapia postmastectomía está indicada si existen factores de riesgo locorregional
2. 2.
(metástasis ganglionares, afectación margen de resección, etc.).
3. 3. La quimioterapia está contraindicada en ausencia de metástasis ganglionares.
La consulta multidisciplinaria de distintos especialistas optimiza las decisiones
4. 4.
terapéuticas.
Gráfico de respuestas
Comentario

Pregunta sobre tratamiento del cáncer de mama fácil de contestar con los conocimientos que has
estudiado.

La indicación principal de quimioterapia en el cáncer de mama es la afectación ganglionar. Sin


embargo, hay casos en que, si bien los ganglios sean negativos, hay que administrarla si la
paciente ofrece factores de mal pronóstico.(R3)

Tratamiento del cáncer de mama según grupo de riesgo

 
 
 
 

148. Hombre de 78 años de edad. Antecedentes personales: HTA, cardiopatía isquémica,


amputación de miembro inferior derecho por embolia, diabetes tipo 2 en tratamiento con
hipoglucemiantes orales. Desde hace 16 horas presenta dolor abdominal localizado en
región periumbilical y fosa ilíaca izquierda, tipo cólico, de inicio brusco y que no se
irradia. No vómitos, ni alteración del tránsito intestinal (1 deposición/día.. Exploración
física: tensión arterial 150/95 mm Hg, pulso 80 lpm, temperatura 37,3 ºC, abdomen blando,
depresible, ruidos aumentados, dolor a la palpación en fosa ilíaca izquierda e hipogastrio
con ligera defensa, Blumberg negativo, puñopercusión renal negativa, tacto rectal: dedo
sale manchado de sangre. Exploraciones complementarias: Hb 12 g, Hcto 39%, 16.500
leucocitos (fórmula: 80% segmentados y 20 bandas). Rx abdomen asa de intestino
delgado dilatada con edema de pared. Ante este cuadro clínico, ¿qué diagnóstico
sospecharíamos en primer lugar?

1. 1. Obstrucción intestinal.
2. 2. Pancreatitis aguda necrohemorrágica.
3. 3. Isquemia intestinal aguda.
4. 4. Diverticulitis aguda.
Gráfico de respuestas
Comentario

Normalmente, cuando se trata de una EMBOLIA mesentérica, el antecedente personal que suelen
darnos es la fibrilación auricular. En este caso, no nos dicen que padezca este problema, sino
cardiopatía isquémica, hipertensión y DM tipo 2, que también predisponen. Habría que pensar en
una isquemia mesentérica aguda, pero en este caso no por fibrilación auricular, sino por trombosis
de la mesentérica superior. Los antecedentes personales que nos refieren orientan más a
trombosis que a embolia.(R3)

149. Which of the following is not a risk factor for pneumococcal meningitis?

 
 
 
 
1. 1. Otitis media.
2. 2. Splenectomy.
3. 3. Graft versus host disease (GVHD).
4. 4. Defects in the terminal pathway of complement activation.
Gráfico de respuestas
Comentario

Pregunta difícil, relacionada con las inmunodeficiencias. En la inmunidad contra el neumococo


participan el sistema de inmunoglobulinas, el bazo y las vías iniciales del sistema de complemento.

En la anemia falciforme el bazo, como consecuencia de múltiples infartos de repetición, pierde su


función y se favorecen las infecciones por gérmenes encapsulados como el neumococo. Este
efecto se denomina "autoesplenectomía". Concretamente la sepsis neumocócica es la primera
causa de muerte en estos pacientes.

La enfermedad de injerto contra huésped se asocia a inmunodepresión y susceptibilidad para las


infecciones relacionadas con la enfermedad subyacente.

En la mayoría de los casos la meningitis neumocócica se desarrolla por extensión directa de una
infección de los senos paranasales o del oído medio y la relación otitis media aguda y meningitis
está bien documentada.

La opción falsa es la 4, ya que aunque el déficit en las vías iniciales del complemento hay cierta
susceptibilidad para la infección de bacterias piógenas como el neumococo, el déficit en las vías
finales del complemento se asocia con la infección recurrente por Neisserias (meningococo y
gonococo).(R4)

150. Gestante de 40 semanas de gestación, sin antecedente patológicos de interés, ni


previos ni durante el embarazo, acude para realizar monitores. En el estudio
cardiotocográfico se informa de la presencia de: taquicardia fetal, variabilidad de 15, con
ausencia de aceleraciones y deceleraciones tipo DIP I. Ante estos resultados se decide
realizar una prueba de Pose. En relación a los valores obtenidos en el registro
cardiotocográfico, ¿Cuándo se considera taquicardia fetal?

1. 1. Por encima de 140 latidos/minuto.


2. 2. Por encima de 150 latidos/minuto.
3. 3. Por encima de 160 latidos/minuto.
4. 4. Por encima de 180 latidos/minuto.
Gráfico de respuestas
Comentario

Pregunta teórica sobre la monitorización fetal, que no se presta a razonamiento alguno. Para sacar
esta pregunta debemos saber cuál es el rango normal de la frecuencia cardiaca fetal (FCF). La
normalidad está entre 120 y 160 lpm, por lo que la opción correcta es la 3 (taquicardia >160 lpm).
Bradicardia sería menor de 110- 120 lpm. Recordemos que, tanto en la bradicardia como en la
taquicardia, habría que realizar una microtoma de sangre fetal.(R3)

151. Un paciente de 75 años es traído al servicio de Urgencias por presentar, de forma


brusca, hemiparesia derecha de predominio braquial. A la exploración, muestra
hemianopsia homónima derecha y afasia fluente con nula comprensión. El diagnóstico
de sospecha es:

 
 
 
 
1. 1. ACVA en territorio de la arteria cerebral posterior izquierda.
2. 2. ACVA en territorio de la arteria cerebral media izquierda.
3. 3. ACVA en territorio de la arteria cerebral posterior derecha.
4. 4. Hemorragia subaracnoidea.
Gráfico de respuestas
Comentario

Una pregunta muy importante que no se debe fallar.

El cuadro descrito corresponde a un ACV, probablemente isquémico (aunque esto lo determinará


el TAC). El territorio comprometido es la cerebral media izquierda, que debemos pensar por la
hemiparesia de predominio braquial derecho. Por otro lado, la hemianopsia homónima derecha
también nos ayuda al diagnóstico, así como la afasia, que también se relaciona con lesiones
hemisféricas izquierdas.(R2)

152. Un niño de 5 años con una talla inferior a -2.5 desviaciones estándar (DE) respecto
a su edad y sexo, con edad ósea de 4 años y medio refiere antecedente de retraso del
crecimiento intrauterino con talla al nacimiento inferior a -2 DE. ¿Cuál sería su actitud?

1. 1. Diagnosticar talla baja familiar idiopática y controlar la aparición de una pubertad precoz.
2. 2. Tras descartar otras causas de talla baja, valorar iniciar tratamiento con GH.
3. 3. No precisa tratamiento ni seguimiento.
4. 4. Iniciar tratamiento con levotiroxina a dosis sustitutivas.
Gráfico de respuestas
Comentario

El retraso del crecimiento intrauterino puede ser una causa de talla baja. Esto se debe a que
durante la etapa fetal se produce una alteración del eje GH - IGF-1. En la mayoría de estos niños
CIR este eje se normaliza en los 2 primeros años de vida, de modo que el crecimiento se acelera y
alcanzan percentiles de talla normales para su edad (catch- up). Sin embargo, en un pequeño
porcentaje esto no ocurre, en este grupo se ha visto que el tratamiento con hormona de
crecimiento a partir de los 4 años permite una normalización de la talla y un adecuado desarrollo,
además de prevenir otras complicaciones metabólicas.(R2)

153. Un paciente acude a su consulta y usted hace el diagnóstico de presunción de


diverticulitis aguda, porque relata cuatro signos frecuentes de esta enfermedad. Uno de
los siguientes NO suele acompañar a la diverticulitis aguda de sigma:

1. 1. Dolor en fosa ilíaca izquierda.


2. 2. Escalofríos.
3. 3. Fiebre.
4. 4. Rectorragias.
Gráfico de respuestas
Comentario

Una pregunta relativamente sencilla, donde intentan que confundamos la clínica de la diverticulitis
con la de la hemorragia diverticular. Una diverticulitis se caracteriza por:

•   Fiebre.
•   Dolor en hipogastrio o fosa ilíaca izquierda.
•   Signos de irritación peritoneal.

 
 
 
 
•   Cambios del ritmo intestinal, que pueden constar tanto en diarrea como en estreñimiento.

En un 25% de los casos puede haber hemorragia, pero casi siempre es microscópica, no en forma
de rectorragia (respuesta 4 falsa).

Los divertículos sangrantes acostumbran a ser los que aparecen en colon derecho y constituyen un
cuadro distinto (sangran sin inflamación asociada, es decir, no existe diverticulitis). Son la causa
más frecuente de hemorragia digestiva baja masiva. Lo habitual es que el sangrado cese
espontáneamente, sin recidivas, en cuyo caso no se necesita tratamiento adicional. No obstante,
en un 20% de los casos se precisaría cirugía o embolización.(R4)

154. Señale la FALSA:

1. 1. El ácido fólico se encuentra sobre todo en frutas y verduras.


En condiciones normales, si se suprime la ingesta de ácido fólico, las reservas de esta
2. 2.
vitamina pueden mantener los requerimientos fisiológicos hasta 2 ó 3 años.
El ácido fólico se absorbe en duodeno y primeras porciones de yeyuno y es transformado
3. 3.
en metiltetrahidrofolato por las células intestinales.
4. 4. Los folatos se almacenan en el hígado y hematíes como poliglutamatos.
Gráfico de respuestas
Comentario

El ácido fólico es una sustancia que se encuentra en muchos alimentos, especialmente en


verduras de hoja verde y algunas frutas (opción 2 correcta); pero no se sintetiza en nuestro
organismo, dependiendo exclusivamente de la dieta (opción 1 correcta). Se absorbe, a diferencia
de la Vit.B12, en el duodeno y yeyuno proximal (opción 4 correcta) y luego se almacena en el
hígado y en los hematíes (opción 5 correcta). Pero estas reservas son muy escasas (5- 10 mg) por
lo que en tres a cuatro meses se podrían terminar, si se eliminase el ácido fólico de la dieta (opción
3 falsa).(R2)

155. Gestante de 31 semanas que acude a urgencias hospitalarias por presentar una
tensión arterial de 150/95. No refiere tensiones altas en controles previos. Las tensiones
durante su estancia son normales. Se realiza analítica de orina con resultado de
normalidad. La paciente refiere cefalea sin otra sintomatología asociada. El hemograma,
la coagulación y la bioquímica completa tampoco muestran alteraciones. Ante estos
hallazgos, cómo clasificaría el estado hipertensivo de la paciente y qué tratamiento
propondría:

1. 1. Preeclampsia e ingreso urgente, dada la cefalea de la paciente.


2. 2. Hipertensión crónica y tratamiento antihipertensivo.
3. 3. Hipertensión gestacional y controles ambulatorios de tensión arterial.
4. 4. Preeclampsia y tratamiento antihipertensivo.
Gráfico de respuestas
Comentario

Tema impresciendible para el ENARM. La paciente que nos describen presenta sólo una tensión
alterada siendo los demás controles normales, sin proteinuria, por lo que quedan descartadas las
opciones de preclampsia. No refiere otras tensiones altas previamente, por lo que no parece que
presentara una hipertensión crónica que viniera de antes de la gestación. Por tanto parece
corresponderse a una tensión asociada al embarazo, en un momento puntual, por lo que en
principio no habría que administrar antihipertensivos de entrada, si no que se debería realizar un

 
 
 
 
control de las tensiones de la paciente para valorar tratamiento en el caso de repetirse
continuamente el estado hipertensivo de la misma.(R3)

156. Lactante de 4 meses de edad, alimentado hasta hace una semana con lactancia
materna exclusiva, actualmente han introducido fórmula de inicio y cereales adecuados
para su edad. Comienza con evacuaciones más liquidas con hebras de sangre y algún
vómito aislado. Señale la opción más adecuada sobre el manejo de este paciente:

1. 1. Se debe retirar el gluten de la dieta.


2. 2. Se debe administrar fórmula sin lactosa.
3. 3. Se debe retirar la lactosa de la dieta de la madre.
4. 4. Se debe administrar un hidrolizado de proteínas.
Gráfico de respuestas
Comentario

Estamos ante una intolerancia a las proteínas de la leche de vaca, que se ha manifestado una
semana después de la introducción de una fórmula artificial con clínica de colitis alérgica con
sangre en las heces por lo que debemos cambiar la fórmula por un hidrolizado de proteínas, por lo
que no es necesario realizar pruebas invasivas. A los 4 meses todavía no se ha introducido el
gluten por lo que esa opción es incorrecta.(R4)

157. Una niña de 12 años en un estudio cardiológico previo a realizar deporte de alto
rendimieto es diagnosticada de comunicación interauricular. Indique la respuesta
CORRECTA:

1. 1. Precisa profilaxis de la endocarditis bacteriana en las situaciones de riesgo.


2. 2. Para su diagnóstico es imprescindible, la realización de un cateterismo cardíaco.
La sintomatología clínica tardía consiste en hipertensión pulmonar, arritmias auriculares e
3. 3.
insuficiencia cardíaca.
4. 4. Es más frecuente en el sexo masculino.
Gráfico de respuestas
Comentario

La comunicación interauricular (CIA) consiste en la presencia de un defecto en el tabique que


separa ambas aurículas entre sí. Existen tres grupos de CIA:

•   Ostium secundum, que es el más frecuente.


•   Ostium primum, en la porción más baja del septo, considerada una forma parcial de canal
AV común.
•   Tipo “seno venoso”.

La CIA produce un shunt izquierda-derecha a nivel auricular. Esto implica una sobrecarga del
circuito pulmonar y de las cavidades derechas. Afortunadamente, la mayoría de las CIA son leves y
asintomáticas, descubriéndose incidentalmente. En la auscultación existe un refuerzo del primer
tono, un soplo sistólico de hiperaflujo pulmonar y un desdoblamiento fijo del segundo tono, hallazgo
muy vinculado a esta patología.

Con cierta frecuencia, esta anomalía se corrige espontáneamente antes de los dos años. Por este
motivo, la corrección quirúrgica suele realizarse después de los tres años de vida.

 
 
 
 
El tratamiento quirúrgico no está indicado en todos los casos, sino únicamente en los que tienen un
shunt importante (relación entre el flujo pulmonar y el sistémico superior a 1.5-2, según autores).
Estos casos más graves, si no se tratan, producirían hipertensión pulmonar, insuficiencia cardíaca
derecha y arritmias auriculares, tal como explica la respuesta 3.

Recuerde que, hoy día, existen alternativas a la cirugía, cerrándose el defecto vía percutánea,
mediante dispositivos tipo “Amplatzer”.(R3)

Cierre percutáneo de la CIA

158. Señalar la afirmación FALSA sobre la naltrexona:

1. 1. No desarrolla dependencia física.


2. 2. Es un antagonista opiáceo de síntesis.
3. 3. Se metaboliza en el parénquima hepático.
4. 4. Puede comenzarse el tratamiento unas horas después de la última dosis de opiáceos.
Gráfico de respuestas
Comentario
Es una pegunta más concreta sobre el tratamiento de la deshabituación a opioides, la pregunta
parece difícil pero se puede contestar sin dificultad si leemos atentamente. Aunque no supiésemos
nada de las características de la naltrexona, una cosa la debemos de tener clara para iniciar el
tratamiento con éste fármaco debe estar totalmente limpio y desintoxicado de opioides ( test de la
naloxona ) por lo que la opción falsa es la 5,no puede tener opioides porque precipita una
abstinencia que puede persistir más de 24 horas. Deben de estar "limpios" de opioides de acción
corta durante 5- 7 días y de acción larga durante unos 15 días más o menos. Las demás opciones
son correctas ( 1,2,3,4). Manual CTO 4ª Edición, Psiquiatría, Tema 5, Pág. 38.(R4)

159. En el síndrome de Gardner NO suelen hallarse:

1. 1. Quistes epidermoides.
2. 2. Osteomas.
3. 3. Fibromas.
4. 4. Papilomas vellosos múltiples.
Gráfico de respuestas
Comentario

Concepto poco importante para el ENARM. El síndrome de Gardner cursa con polipos
adenomatosos como la poliposis colonica familiar pero también con manifestaciones
extraintestinales como osteomas en craneo, mandíbula y huesos largos, tumores desmoides,
anormalidades dentales, quistes epidermoides y sebáceos, lipomas, fibromas, tumores de tiroides,
glándulas suprarrenales arbol biliar e hígado pero no cursa con papilomas vellosos.(R4)

160. Una mujer de 35 años, sin antecedentes de interés, acude a la consulta de Neurología
por presentar desde hace 5 días inestabilidad al caminar con tendencia a desviarse hacia

 
 
 
 
la izquierda, visión doble en la mirada conjugada horizontal hacia dicho lado y desviación
de la comisura bucal hacia la derecha. La exploración física revela además piramidalismo
en las 4 extremidades. Se realizó una RM cerebral, mostrando lesiones ovoideas de
pequeño tamaño, hiperintensas en T2 y densidad protónica e isointensas en T1,
localizadas en sustancia blanca profunda de ambos hemisferios, pedúnculo cerebeloso
medio izquierdo y protuberancia de ese lado. Lo más probable es que se trate de:

1. 1. Multiinfarto lacunar.
2. 2. Neurolúes.
3. 3. Enfermedad desmielinizante.
4. 4. Metástasis cerebrales.
Gráfico de respuestas
Comentario

Aunque las manifestaciones clínicas pueden ser compatibles con varios diagnósticos, nos están
hablando de lesiones en múltiples regiones de la sustancia blanca, que además aparecen en una
mujer joven. Estos datos deberían hacernos pensar, como primer diagnóstico, en una esclerosis
múltiple, por lo que la respuesta correcta es la 4.(R3)

161. Un lactante comienza con vómitos biliosos poco después de su primera toma. Se
realiza una Rx-abdomen, visualizándose un signo de doble burbuja. Diagnóstico de
sospecha:

1. 1. Onfalocele.
2. 2. Síndrome de Chilaiditi.
3. 3. Atresia duodenal.
4. 4. Estenosis pilórica.
Gráfico de respuestas
Comentario

Sobre la atresia duodenal, le enumeramos los detalles más importantes para el examen ENARM:

- Vómitos biliosos desde el nacimiento (no confundamos con la hiperplasia pilórica, donde
aparecen hacia los 20 días de vida).

- Imagen radiológica típica en “doble burbuja”.

- Asociación clásica: el síndrome de Down.(R3)

162. Primípara, 40 semanas de amenorrea, con monitorización interna intraparto. Se


aprecian desaceleraciones DIP tipo II con todas las contracciones. Se decide realizar una
microtoma de sangre fetal para analizar el pH. Obtenemos un resultado de 7,14. Su
dilatación es de 6 cm. ¿Cuál sería su actitud?:

1. 1. Esperar 5-10 minutos y repetir la prueba para confirmar un resultado tan extraño.
Esperar a que se ponga en dilatación completa y hacer el parto utilizando fórceps, si es
2. 2.
preciso, para así abreviar el expulsivo.
3. 3. Cesárea urgente.
Utilizar un tocolítico para que el feto deje de sufrir con las contracciones y posteriormente
4. 4.
realizar cesárea.
Gráfico de respuestas

 
 
 
 
Comentario

Una pregunta relativamente sencilla. Tenemos un caso clínico de una gestante a término con
monitorización intraparto (está de parto, > 2 cm), y se registran deceleraciones tardías (mal
pronóstico), con lo que realiza microtoma fetal, obteniendo un valor patológico (<7,20); debemos
finalizar la gestación de forma inmediata. El paso siguiente es ver la vía de finalización, y como no
está en dilatación completa (10 cm), ni en tercer plano de Hodge, no podemos finalizar con un
fórceps, así que deberemos realizar una cesárea urgente; opción correcta la 3.(R3)

163. Con respecto a la histopatología del eccema agudo alérgico, señalar lo FALSO:

1. 1. Es característica la presencia de espongiosis.


2. 2. La hiperqueratosis es el hallazgo más frecuente.
3. 3. En la dermis superficial aparecen infiltrados perivasculares fundamentalmente linfocíticos.
4. 4. La espongiosis puede ser de intensidad suficiente como para originar vesiculación.
Gráfico de respuestas
Comentario
La histopatología de la dermatitis de contacto alérgica es más diagnóstica en su fase aguda ya que
las fases subagudas- crónicas son más confusas. Aún así, el estudio histológico en las dermatitis
de contacto es de un valor limitado ya que la distinción entre la forma alérgica y la forma irritativa
no se puede establecer con certeza. En la fase aguda del eccema de contacto alérgico se aprecia
espongiosis, exocitosis linfocitaria y un infiltrado inflamatorio perivascular en la dermis superficial
constituido por linfocitos, histiocitos y eosinófilos. Cuando la espongiosis es intensa puede dar
lugar a la formación de vesículas intraepidérmicas. En las lesiones más evolucionadas (subagudas,
crónicas) se produce hiperplasia epidérmica, a veces psoriosiforme, con acantosis, hiperqueratosis
y paraqueratosis.(R2)

164. En relación con el desarrollo psicomotor, lo siguiente es FALSO:

1. 1. Sostiene la cabeza al tercer mes.


2. 2. Se sienta sin apoyo al cuarto mes.
3. 3. Transfiere un objeto de una mano a la otra a los 6 meses.
4. 4. Gatea a los 11 meses.
Gráfico de respuestas
Comentario

La respuesta correcta es la 2. Repasemos lo siguiente las veces que sean necesarias, año con año
les encanta preguntar en el nacional.

1.5 mes: inicia la sonrisa social.

3 meses: inicia el sostén cefálico.

4 meses: toma objetos grandes con la mano.

5 meses: prensión alternante de objetos.

6 meses: inicia la sedestación, que se completa a los ocho meses.

 
 
 
 
8- 9 meses: oposición del pulgar.

9- 10 meses: inicia la reptación.

10- 11 meses: comienza la bipedestación.

12-15 meses: da los primeros pasos y emite su primera palabra real.

18-22 meses: realiza combinaciones de dos palabras.

24 meses: sube y baja escaleras, corre, apila cuatro o seis cubos para formar una torre.

5-10 años: el niño comprende qu e la muerte es un fenómeno permanente.

(R2)

165. ¿Cuál de estas posibilidades diagnósticas están acordes con este caso?: Paciente
de 2 años de edad, con anemia secundaria a hemorragia, sin dolor y el examen de
abdomen se reporta sin alteraciones:

1. 1. Invaginación intestinal.
2. 2. Fisura anal.
3. 3. Duplicación quística abdominal.
4. 4. Diverticulosis.
Gráfico de respuestas
Comentario

La única posibilidad es la respuesta número 3, ya que:

Respuesta 1: el paciente tendría dolor, evacuaciones color grosella, además de episodios de dolor
abdominal y a la palpación abdominal se podría detectar la invaginación.

Respuesta 2: en caso de fisuras anales, el dolor es muy intenso, por lo que se descarta.

Respuesta 4: a esa edad o hay diveritculosis, sería extremadamente raro!

(R3)

166. El trasplante pulmonar es en la actualidad terapéutica válida para pacientes en


insuficiencia respiratoria terminal en la que se han agotado todos los tratamientos
médico-quirúrgicos. Entre los siguientes, indique la enfermedad que más frecuentemente
se beneficia de este proceder terapéutico:

1. 1. Carcinoma broncogénico.
2. 2. Fibrosis quística.
3. 3. Distrés respiratorio del adulto.
4. 4. Embolismo pulmonar.
Gráfico de respuestas
Comentario

 
 
 
 
Pregunta sencilla sobre indicaciones de trasplante pulmonar.

Basta con conocer las indicaciones de trasplante pulmonar para responder correctamente a esta
pregunta.

En cualquier caso, al explicar el carcinoma broncogénico, el distrés respiratorio, la embolia o la


hemoptisis masiva, no se explica NUNCA el trasplante como opción terapéutica.(R2)

Criterios fisiopatológicos para la indicacióndel trasplante pulmonar

167. Varón de 52 años, mozo de


almacén de profesión, que consulta por presentar cuadro de cefalea intensa que comenzó
mientras trabaja y que ha ido en aumento. Asocia dos episodios de vómitos. Obedece
órdenes simples, pero no complejas. Apertura ocular espontánea y dirigiendo la mirada.
Fuerza y sensibilidad normales. Se realiza TC craneal, que es normal. La siguiente actitud
sería:

1. 1. Alta con vigilancia domiciliaria.


2. 2. Arteriografía cerebral.
3. 3. Punción lumbar.

 
 
 
 
Valoración por psiquiatría por cuadro de somatización con claro interés ganancial por el
4. 4.
paciente.
Gráfico de respuestas
Comentario
No hay comentario(R3)

168. En el protocolo diagnóstico se ha realizado la siguiente prueba (ver imagen), por lo


cual el diagnóstico más probable es:

1. 1. Aneurisma arteria cerebral posterior.


2. 2. Hematoma intraparenquimatoso.
3. 3. Aneurisma arteria cerebral media.
4. 4. Malformación arteriovenosa.
Gráfico de respuestas
Comentario
No hay comentario(R3)

169. Un varón de 45 años acude a consulta por la aparición reciente de un bulto en el


cuello, que ha crecido en el último mes. En la infancia recibió radioterapia. En la
exploración se aprecia un nódulo de unos 4 cm de diámetro en el lóbulo izquierdo, duro,
adherido a estructuras profundas y con 2 adenopatías palpables. ¿Qué actitud
terapéutica inicial decidiría?:

1. 1. Hemitiroidectomía izquierda con biopsia intraoperatoria.


2. 2. Tiroidectomía total.
3. 3. Gammagrafía de tiroides.
4. 4. PAAF nódulo tiroideo.
Gráfico de respuestas
Comentario
Nos presentan a un paciente con un nódulo tiroideo solitario de reciente instauración y crecimiento
rápido, que además como datos para sospechar malignidad presenta el antecedente de
radioterapia en la infancia, existencia de adenopatías y adherencia a planos profundos. El manejo
inicial de estos pacientes con alto riesgo de presentar un carcinoma tiroideo es el mismo que si no
tuviera ningún criterio de sospecha de malignidad, es decir realizar una PAAF. En caso de que la
citología sea informada como carcinoma papilar se realizará tiroidectomía total. Si por el contrario
la PAAF resulta ser no concluyente o existe proliferación folicular se realizaría un hemitiroidectomía
con biopsia intraoperatoria. La gammagrafía y la ecografía cervical no aportan nada para la
decisión terapéutica.(R4)

170. ¿Cuál de los siguientes hallazgos indica peor pronóstico en la insuficiencia aórtica?:

1. 1. Insuficiencia ventricular izquierda.


2. 2. Angina de pecho.
3. 3. Bloqueo de rama derecha.
4. 4. Cardiomegalia en la radiografía de tórax.
Gráfico de respuestas
Comentario
Esta pregunta es útil para repasar la IAo, su pronóstico y su tratamiento. Debes saber que los
pacientes con IAo severa asintomáticos sin disfunción ni dilatación ventricular izquierda NO deben
ser sometidos a cirugía, dado que pueden permanecer en esta situación muchos años. La
aparición de disfunción ventricular izquierda y/o dilatación ventricular empeora el pronóstico de

 
 
 
 
estos pacientes, en los cuales la cirugía SI está indicada. Así pues, el dato de peor pronóstico es la
aparición de Insuficiencia ventricular izquierda (opción 1). El soplo sistólico, el bloqueo de rama
derecha y la cardiomegalia no son datos de peor pronóstico, y por ello en estos casos NO está
indicada la cirugía. La angina de pecho suele deberse a isquemia coronaria.(R1)

171. Según la clasificación histológica de los tumores de ovarios, el Tumor de Brenner pertenece al
grupo de:

1. 1. Tumores epiteliales.
2. 2. Tumores del estroma de los cordones sexuales.
3. 3. Tumores de las células germinales.
4. 4. Tumores de tejidos blandos inespecíficos del ovario.
Gráfico de respuestas
Comentario

Si ha fallado esta pregunta, repasa la siguiente tabla de cáncer de ovario(R1)

172. Lactante de un mes de edad, que acude a control médico, tiene las siguientes
medidas: talla 52 cm, peso: 4 Kg y perímetro cefálico: 37 cm, correspondiéndoles dichas
medidas al percentil 40. Si el niño recibe lactancia materna exclusiva y es un niño sano,
usted esperará en el siguiente control mensual:

1. 1. Que el perímetro cefálico pase el percentil 80.

 
 
 
 
2. 2. Que solo la talla persista en el percentil 40.
3. 3. Que el peso suba al percentil 90.
4. 4. Que todas sus medidas continúan en el percentil 40.
Gráfico de respuestas
Comentario

Por lógica la única respuesta posible sería la respuesta número 4, ya que estar en el percentil 40
habla de normalidad en sus medidas antropométricas.(R4)

173. Una posible complicación tardía de fractura de epicóndilo en niños es la de cúbito


valgo progresivo con afectación de:

1. 1. Nervio mediano.
2. 2. Nervio cubital.
3. 3. Nervio radial.
4. 4. Nervio musculocutáneo.
Gráfico de respuestas
Comentario

Una pregunta sobre una de las fracturas que conviene dominar a la perfección en el niño como es
la fractura supracondílea. Lo principal que no se le puede escapar de las fracturas supracondíleas
es que son la causa más frecuente de síndrome compartimental en el niño. La lesión nerviosa
aguda más relacionada con estas fracturas es la del nervio interóseo anterior, rama del mediano.
Ahora bien, para contestar esta pregunta podemos sabernos la respuesta o usar la picardía: la
deformidad en cúbito valgo se produce por un crecimiento asimétrico de la extremidad lo cual
puede producir una lesión por elongación del nervio que atraviese el epicóndilo medial: este nervio
es el cubital (respuesta 2 correcta).(R2)

174. Paciente de 56 años a la que se le detecta una tumoración en


el cuadrante infero-externo de la mama derecha. Se realiza una mastografía que se
muestra en la imagen. ¿Qué tratamiento realizaría usted?

1. 1. Biopsia intraoperatoria de la lesión y, si es positiva, tumorectomía y ganglio centinela.


2. 2. Biopsia intraoperatoria de la lesión y si es positiva tumorectomía y radioterapia.
Biopsia intraoperatoria de la lesión y si es positiva mastectomía con reconstrucción
3. 3.
mamaria inmediata.
4. 4. Biopsia intraoperatoria de la lesión y si es positiva mastectomía y radioterapia.
Gráfico de respuestas
Comentario

 
 
 
 
En la mastografía, podemos apreciar una tumoración irregular, con bordes mal definidos, en una
paciente de más de 50 años, con lo que el diagnóstico más probable sería el de cáncer de mama.
Aparentemente, se trata de un tumor de pequeño tamaño, por lo que no habrá que hacer (al menos
de inicio) una mastectomía. Lo que sí debemos realizar es una biopsia intraoperatoria, que nos
confirmará la malignidad histológica, y en caso de ser positiva procederíamos al estudio del ganglio
centinela.

Recuerde que, en el pronóstico del cáncer de mama sin metástasis a distancia, el factor pronóstico
más importante es el número de ganglios afectados a nivel axilar, de ahí la importancia del estudio
del ganglio centinela. En cuanto a la radioterapia, se realiza cuando se trata de un tumor de
pequeño tamaño, como parece ser el caso, pero no durante el propio acto quirúrgico, sino de forma
diferida. De este modo (tumorectomía + radioterapia) se alcanzan resultados similares a las
mastectomías clásicas, en términos de supervivencia, sin realizar intervenciones tan
mutilantes.(R1)

175. En la paciente del caso anterior el


resultado definitivo anatomopatológico informa de carcinoma ductal infiltrante, de 34 mm
de diámetro máximo, indiferenciado, con importante invasión vascular, receptores
hormonales negativos, Ki67 30% y Her-2-neu negativo. Señale cuál sería el tratamiento
complementario más adecuado:

1. 1. Radioterapia.
2. 2. Radioterapia y quimioterapia.
3. 3. Radioterapia, quimioterapia y hormonoterapia.
4. 4. Quimioterapia.
Gráfico de respuestas
Comentario

 
 
 
 
Confirmado que se trata de un tumor pequeño (34 mm de diámetro máximo), debemos administrar
radioterapia. Aunque no nos dicen el estado de los ganglios, sí que nos mencionan varios factores
de mal pronóstico: importante invasión vascular y carácter indiferenciado, por lo que sería
aconsejable utilizar quimioterapia. Desde el punto de vista de los receptores hormonales, al ser
negativos, no precisamos administrar tamoxifeno, inhibidores de la aromatasa ni otras formas de
hormonoterapia. Por ello, la respuesta correcta es la 2 y no la 3.(R2)

176. Señale cuál de los siguientes factores NO se considera de mal pronóstico en el


cáncer de mama de la paciente del caso anterior:

1. 1. Receptores hormonales negativos.


2. 2. Grado de diferenciación G3.
3. 3. Her-2-neu negativo.
4. 4. Invasión vascular importante.
Gráfico de respuestas
Comentario

La presencia de Her-2-neu se considera un factor de mal pronóstico. Recuerde que, para las
pacientes en las que es positivo, existe un tratamiento relativamente reciente, el trastuzumab, un
anticuerpo monoclonal anti Her- 2- neu.(R3)

177. A 55-year-old woman comes to her gynecologist for her annual follow-up visit. She
is a heavy smoker since she was 30. On physical examination, a 1.5-cm-diameter lump is
discovered in the upper outer quadrant of her left breast. The lump is tender, as well as
the whole left breast. No axillary or supraclavicular lymph nodes are palpated.
Mammography shows the existence of a nodule with features of malignancy.
Intraoperative biopsy is reported as infiltrating ductal carcinoma and the resected nodule
has tumor-free margins. Which of the following is the most appropriate next step in
management?

1. 1. Radical mastectomy, because the patient has poor prognostic factors, such as her age.
2. 2. Tumorectomy, lymphadenectomy and radiation therapy given to the surgical bed.
3. 3. Subcutaneous mastectomy, lymphadenectomy and letrozole.
4. 4. Chemotherapy, radiation therapy, postoperative hormone therapy (tamoxifen).

(R2)

 
 
 
 

178. which is the wrong answer about


hypertrophy of the pylorus?

1. 1. This condition most commonly occurs in female newborns.


2. 2. Its usual presentation is non bilious vomiting.
3. 3. Frequently constipation is present.
4. 4. Surgery is an important part of treatment.
Gráfico de respuestas
Comentario

La estenosis hipertrófica del píloro es una entidad relativamente frecuente (3 por cada 1000 recién
nacidos vivos), más frecuente en hombres (respuesta 2 falsa), sobre todo de raza blanca y con
mayor frecuencia si son los primogénitos de la familia. La incidencia parece ser mayor en los
grupos sanguíneos B y O. Entre otras asociaciones, se ha relacionado con la administración
intravenosa de prostaglandinas a la madre, y con el uso de eritromicina en el neonato.(R1)

179. Un paciente varón, de 61 años de edad, acude a nuestra consulta refiriendo un


aumento progresivo del número de veces en que se levanta a orinar por las noches. No
refiere tenesmo, pero sí una disminución subjetiva del calibre y de la fuerza del chorro de
orina. Realizamos un tacto rectal, que nos impresiona de hiperplasia benigna de próstata.
Como el paciente refiere que su padre falleció como consecuencia de un cáncer de
próstata, decidimos hacer una determinación del PSA, hallándose éste en valores de
normalidad. Respecto al tratamiento de la HPB, es FALSO que:

1. 1. Hasta la fecha, la cirugía continúa siendo el tratamiento con mejores resultados.


Los alfa-bloqueantes actúan sobre el componente muscular liso de la próstata y cuello
2. 2.
vesical.
3. 3. El finasteride actúa principalmente por disminución del tamaño glandular.
4. 4. Los fitoterápicos tienen un efecto objetivo probado.
Gráfico de respuestas
Comentario

 
 
 
 
Pregunta sencilla, pero muy didáctica. Todas las opciones son correctas con respecto al
tratamiento de la HBP. Recuerda que la fitoterapia se utiliza cuando la sintomatología es leve. No
se conoce claramente su mecanismo de acción (opción 4 falsa) y su eficacia real no ha sido
demostrada hasta ahora. El resto de las opciones enuncian algunos conceptos fundamentales
sobre el tratamiento de esta enfermedad, que no deberían haberte planteado dificultades.(R4)

180. Una paciente de 65 años acude a nuestra consulta con un informe médico en el que
consta el diagnóstico de bocio multinodular con hipertiroidismo subclínico. ¿Cuál de los
siguientes patrones hormonales le corresponde?

1. 1. T4 total normal, T4 libre aumentada, TSH suprimida.


2. 2. T4 total normal, T4 libre normal, TSH suprimida.
3. 3. T4 total baja, T4 libre baja, TSH aumentada.
4. 4. T4 total normal, T4 libre normal, TSH aumentada.
Gráfico de respuestas
Comentario

Recuerda que en el hipertiroidismo subclínico, el tiroides no necesita laa estimulación fisiológica


por parte de la TSH para mantener unas concentraciones normales de hormonas tiroideas. Por
tanto, la T4 (tanto libre como total) estará normal con unos niveles suprimidos de TSH.(R2)

181. A 43-year-old man comes to your office complaining of dyspeptic symptoms for the
past 3 years. A month ago he underwent a breath test for H. pylori that was positive, and
upper GI tract endoscopy revealed chronic superficial gastritis in the antral region, with
negative staining for H. pylori. The following therapeutic measures would be
recommended for the management of this patient EXCEPT:

1. 1. Ranitidine.
2. 2. Magaldrate.
3. 3. H. pylori eradication.
4. 4. Acexamate Zn.
Gráfico de respuestas
Comentario

Una pregunta de cierta dificultad sobre la infección por H. pylori.

Como sabe, H. pylori ha sido implicado en la patogenia de diversas enfermedades (gastritis agudas
y crónicas, úlcera péptica, adenocarcinoma gástrico, linfomas MALT…). Sin embargo, también es
muy frecuente la infección por esta bacteria sin que produzca ningún tipo de síntoma.

Actualmente, la erradicación universal de H. pylori no está aceptada. Son claras indicaciones la


úlcera gastroduodenal y el linfoma primario tipo MALT. Cuando se trata de gastritis crónicas, es
aconsejable su erradicación en los casos en que asocie displasia. Cuando ésta no existe, su
indicación es controvertida, por lo que en el caso que nos presentan la opción 3 sería, como
mínimo, discutible. En cambio, el resto de las opciones son medidas sintomáticas ante el cuadro
dispéptico por el que consulta.(R3)

 
 
 
 

182. Paciente de 22 años que acude


por primera vez a la consulta para realizar una revisión ginecológica. Menarquia a los 14
años, reglas normales, no dolorosas, nuligesta. Le realiza ecografía vaginal, objetivando
en región anexial izquierda lo que se muestra en la imagen. Señale lo INCORRECTO:

1. 1. Es el tumor germinal ovárico más frecuente.


2. 2. Son benignos.
3. 3. Habitualmente cursan con dismenorrea.
4. 4. El tratamiento fundamentalmente es quirúrgico.
Gráfico de respuestas
Comentario

Teniendo en cuenta que se trata de una revisión rutinaria, en una mujer joven, sin factores de
riesgo para tumoración maligna y, por lo demás, asintomática, aparte de las características de la
imagen, debemos concluir que seguramente se trata de un quiste ovárico totalmente benigno.
Observa que se trata de una tumoración completamente regular, tanto en su contorno (redondeado
y liso) como en su contenido. Estos quistes suelen ser, simplemente, hallazgos incidentales y casi
siempre son asintomáticos, por lo que la respuesta incorrecta es la 4.(R4)

183. Usted decide intervenir a la paciente del caso clínico anterior con fines diagnóstico-
terapéuticos. Señale la opción correcta:

La anexectomía laparoscópica es el tratamiento más idóneo, debido al contenido de la


1. 1.
tumoración.
2. 2. La quistectomía laparoscópica sin intraoperatoria es el tratamiento más oportuno.
3. 3. Se recomienda la biopsia intraoperatoria, debido a la edad joven de la paciente.
4. 4. En los embarazos suelen disminuir de tamaño.
Gráfico de respuestas
Comentario

Difícilmente vamos a poder tratar un quiste (tumoración benigna, pero tumoración) con tratamiento
médico. Por ello, lo que debemos hacer es extirparlo, lo cual hoy día es posible mediante cirugía
laparoscópica. Dada la juventud de la paciente y las características ecográficas del quiste, no

 
 
 
 
precisamos biopsia intraoperatoria, ya que se trata de una neoplasia benigna casi con total
seguridad.

Recuerda que una tumoración que sí puede reducir su tamaño con tratamiento médico son los
miomas, utilizando agonistas de la GnRH. No obstante, el mioma no tendría un contenido líquido
(anecoico, como en este caso), sino sólido. Y, en cualquier caso, lo que se suele conseguir es que
reduzcan su tamaño, pero para que desaparezcan por completo sería necesaria la extirpación
quirúrgica.(R2)

184. Señale el diagnóstico que le parece más probable para un RN hijo de madre
diabética, que, tras un parto sin incidencias, presentó a las 12 horas de vida temblores,
letargia y rechazo de las tomas, así como ligera acrocianosis. A los 2 días de vida
presenta ictericia que requiere fototerapia:

1. 1. Hipoglucemia neonatal.
2. 2. Policitemia.
3. 3. Alteración del sistema nervioso central.
4. 4. Síndrome de abstinencia a opiáceos.
Gráfico de respuestas
Comentario

Pregunta sencilla sobre las complicaciones en RN de madres con diabetes gestacional.

Le cuenta que a las 12 horas comienza con temblores, letargia, rechazo a las tomas, acrocianosis
e ictericia, todo compatible con poliglobulia. Respuesta 2 correcta.

En la siguiente imagen repase las complicaciones de la diabetes gestacional.(R2)

 
 
 
 
185. De las siguientes tubulopatías. ¿Cuál se acompaña de litiasis?:

1. 1. ATR distal.
2. 2. ATR tipo IV.
3. 3. Síndrome de Bartter.
4. 4. Glucosuria renal.
Gráfico de respuestas
Comentario
Recuerda la sociación típica (y el porqué) de la ATR tipo I con la litiasis renal. En la ATR I o distal
se produce una alteración en la eliminación distal de H+ y para la conservación de K+ (acidosis
hiperclorémica, hipopotasemia y alteración en la concentración de la orina). La acidosis crónica
intenta ser compensada con el movimiento cálcico, produciéndose una disminución en la
reabsorción renal de calcio , con hiperclaciuria y un leve hiperparatiroidismo secundario con la
consecuente aparición de raquitismo y osteomalacia. La hipercalciuria favorece la aparición de
cálculos de fosfato cálcico. En la ATR tipo II o proximal la hipercalciuria es moderada, por lo que la
incidencia de litiasis es menor (resp 1 falsa), al igual que en la ATR tipo IV, donde la hiperclaciuria
es poco frecuente (resp 3 falsa). En el síndrome de Bartter y en la glucosuria renal no existe
alteración en la excreción renal de calcio ni aumento en la incidencia de nefrolitiasis (resp 4 y 5
falsas). Bibliografía: Tema 11.8 de Nefrología, Manual 4ª Ed, pág. 32- 33.(R1)

186. Secundigesta
de 28 años, con un parto eutócico previo, gestante de 39 semanas. El embarazo ha
cursado con normalidad. Durante el periodo de dilatación se obtiene el registro que se
muestra en la imagen. Condiciones obstétricas: cefálica, líquido amniótico claro, 5 cm, II
plano Hogde. Conducta a seguir:

1. 1. Cesárea.
2. 2. Colocar en decúbito lateral izquierdo.
3. 3. Realizar microtoma fetal.
4. 4. Retirar goteo oxitocina.
Gráfico de respuestas
Comentario

Una pregunta bastante difícil, puesto que dependemos mucho de nuestra capacidad para
interpretar la imagen.

Para empezar, debe darse cuenta de que se trata de un registro cardiotocográfico fetal. Lo
fundamental es que sepa distinguir entre las DIPS tipo I y tipo II. En las tipo I, la bradicardia fetal

 
 
 
 
ocurre exactamente en el mismo instante que la contracción uterina, sin ningún decalaje temporal
entre ellas. Por el contrario, en el tipo II se produce primero la contracción, y varios segundos
después el descenso de la frecuencia cardíaca.

En este caso, observe que no existe una coincidencia temporal perfecta. Esto se aprecia muy
claramente en el centro de la imagen, un poco a la derecha, donde existe un claro decalaje entre la
contracción uterina (línea negra) y el descenso de la frecuencia cardíaca (línea roja). Estamos, por
tanto, ante un registro con DIPS tipo II, que suelen relacionarse con mayor o menor grado de
sufrimiento fetal. Esto nos obliga a medir el pH mediante una microtoma (respuesta 3 correcta), lo
que nos permitirá definir la actitud siguiente. Recuerde que los valores normales del pH fetal están
entre 7.25 y 7.45. Cuando se encuentran por debajo de 7.20, debemos poner fin a la gestación
inmediatamente, y entre 7.20-7.25 podríamos adoptar medidas conservadoras (oxígeno, decúbito
lateral izquierdo, etc.) y repetir la microtoma en quince minutos.(R3)

187. Mujer de 52 años que acude a urgencias por presentar dolor abdominal y tumoración
en región umbilical de 12 horas de evolución, la cual se trata de reducir y con mucho
esfuerzo se consigue pero protruye nuevamente de inmediato, se trata de una hernia:

1. 1. Indirecta.
2. 2. Inguino-escrotal.
3. 3. Incoercible.
4. 4. Por deslizamiento.
Gráfico de respuestas
Comentario

Concepto sobre hernias que se debe conocer. Una hernia es incoercible o irreductible cuando al
reducirse vuelve a protruir. No se debe confundir este concepto con el de estrangulada, que se
refiere a la hernia con compromiso vascular.(R3)

188. A patient who has been recently diagnosed with upper respiratory tract infection by
his physician comes to ER, complaining of a 2-day history of severe otalgia. He also says
that he felt a bloody discharge coming out from his ears that alleviated his pain. Which of
the following is the most likely diagnosis?

1. 1. Influenza virus.
2. 2. Streptococcus pneumoniae.
3. 3. Mycoplasma pneuomoniae.
4. 4. Varicella-zoster virus.
Gráfico de respuestas
Comentario

La infección por Mycoplasma pneumoniae puede producir un cuadro llamado miringitis ampollosa
hemorrágica caracterizado por otalgia intensa que cede con la rotura de las ampollas de contenido
hemorrágico que se sitúan sobre la membrana timpánica causando otorragia. Sospecharemos este
cuadro aún más si, como ocurre en este caso, nos comentan que ocurre en el contexto de una
infección de vías altas.(R1)

189. La complicación crónica más común en la infección congénita por CMV es a nivel
de:

1. 1. SNC.

 
 
 
 
2. 2. Corazón.
3. 3. Sistema hematológico.
4. 4. Sistema inmune.
Gráfico de respuestas
Comentario

Recuerde la siguiente regla:

CMV: Coriorretinitis, Colecistitis acalculosa; Microcefalia; calcificaciones periVentriculares.

El CMV es la causa más frecuente de infección congénita. La infección adquirida en la primera


mitad del embarazo da laugar a un cuadro sintomático que se caracteriza por coriorretinitis,
calcificaciones periventriculares y microcefalia.

Si la infección se adquiere en la segunda mitad de la gestación, que es lo más habitual, cursa de


modo asintomático, pero puede derivar en la aparición de hipoacusia neurosensorial bilateral.

Respuesta 1 correcta.(R1)

190. Un enfermo EPOC con diagnóstico de miastenia gravis recibe tratamiento


prolongado con inmunosupresores. En la Rx tórax de control post IQ por extirpación del
timo, se aprecia una densidad ovalada de diámetro mayor vertical en lóbulo superior
izquierdo, con pequeño menisco radiotransparente en su interior. Se compara dicha
imagen con Rx anteriores, observándose en estas una lesión de morfología idéntica, de
contornos muy definidos, pero de densidad aire en su totalidad, en la misma localización.
Se le realiza una biopsia por BFC y se analiza en laboratorio, informándose como proceso
inflamatorio crónico con multitud de elementos filiformes tabicados y ramificados en 45º.
¿Qué diagnóstico sugiere?:

1. 1. Ca. broncogénico con calcio cristalizado.


2. 2. Absceso pulmonar por Actinomices israelii.
3. 3. Nocardiosis.
4. 4. Aspergiloma
Gráfico de respuestas
Comentario

Lo que nos describen es una imagen redondeada con una semiluna aérea en la parte superior
(menisco radiotransparente). Esta imagen radiológica es muy característica del aspergiloma. La
aparición de elementos filiformes tabicados y con ramificaciones en 45º también va a favor de
Aspergillus así como la inmunosupresión del paciente.(R4)

191. A 35-year-old woman comes to your office complaining of pallor of the fingers
followed by cyanosis and rubor after exposure to cold. She also complains of numbness
and pain. She does not refer any other symptoms. What is the most likely diagnosis?

1. 1. Systemic sclerosis.
2. 2. Buerger's disease.
3. 3. Raynaud's disease.
4. 4. Livedo reticularis.

 
 
 
 
(R4)

192. ¿Cuál es el hecho más constante en el síndrome de ovario poliquístico?:

1. 1. Amenorrea.
2. 2. Anovulación.
3. 3. Obesidad.
4. 4. Polimenorrea.
Gráfico de respuestas
Comentario

Esta pregunta es muy importante porque se debe conocer perfectamente el síndrome de ovario
poliquístico y sus alteraciones hormonales. Es una afección muy frecuente en que está aumentada
la LH con niveles de FSH bajos o inferiores a los normal, por lo que aumenta la relación LH/FSH.
Hay un aumento leve de andrógenos, aumento de la estrona (los andrógenos circulantes son
convertidos a estrona en la grasa periférica) y descenso del estradiol. La LH aumentada estimula la
teca, produciendo más andrógenos (también hay una sobreproducción suprarrenal) que va a
provocar obesidad, hirsutismo y anovulación. La esterilidad es el síntoma más frecuente (73%) y se
debe precisamente a la falta de ovulación. Existe insulinresistencia. La tendencia actual para el
tratamiento de la esterilidad en el SOP es usar como primera opción clomifeno y metformina. Si
falla usar gonadotropinas como 2º opción y , si nuevamente fracasamos, destrucción parcial del
ovario por vía laparoscópica.(R2)

193. En cuanto a los índices urinarios en el diagnóstico del fracaso renal agudo prerrenal,
indique la afirmación INCORRECTA:

1. 1. La osmolalidad urinaria es superior a 400 mOsm/Kgr.


2. 2. El sodio urinario es inferior a 20 mEq/1.
3. 3. El índice de fallo renal (IFR) es superior a 1.
4. 4. La excreción fraccional de Sodio (EFNa) es inferior al 1 %.
Gráfico de respuestas
Comentario
El índice de fracaso renal agudo es similar a la excrección fraccional de Na: EFNa = Na o/ Na p x
Cr p/Cr o; IFR = Na o x Cr p/Cr o; su valor en el FRA prerrenal es inferior al 1%. Las opciones 1, 2,
4 y 5 son habituales en el fracaso renal agudo prerrenal.(R3)

194. Un hombre de 61 años con fiebre, tos y expectoración mucopurulenta, presenta una
consolidación extensa con broncograma aéreo que afecta a los lóbulos superior e inferior
del pulmón izquierdo. Refiere disnea intensa al acostarse sobre el lado izquierdo. La
PaO2 de una muestra de sangre arterial obtenida en esa posición es de 48 mmHg, minutos
antes, una muestra tomada mientras se encontraba acostado sobre el lado derecho
demostraba una PaO2 de 69 mmHg. Este descenso en la PaO2 al acostarse sobre el lado
izquierdo puede ser explicado con mayor probabilidad por:

1. 1. Aumento de la rigidez de la pared torácica en el pulmón situado en posición declive.


2. 2. Cúmulo de edema intersticial en el pulmón situado en posición declive.
3. 3. Aumento del riego sanguíneo en el pulmón situado en posición declive.
Aumento de las resistencias de las vías respiratorias en el pulmón situado en posición
4. 4.
declive.
Gráfico de respuestas
Comentario

 
 
 
 
Si el paciente se tumba sobre el pulmón izquierdo, por efecto de gravedad pasará por él la mayor
parte de la sangre del árbol pulmonar. Dado que el pulmón con neumonía es precisamente éste,
no podrá oxigenar la sangre de forma adecuada, ya que el espacio aéreo (los alveolos) estarán
ocupados por material exudativo, polimorfonucleares, etc...

Por el contrario, si tumbamos al paciente sobre el pulmón sano, la mayor parte de la sangre pasará
por él, con la consiguiente mejoría gasométrica.(R3)

195. Acude al servicio de urgencias una mujer de 62 años postmenopáusica por haber
presentado una metrorragia de dos días de duración en cantidad menor que una regla.
La paciente tiene buen estado general, es sana, no toma medicaciones y tiene un IMC 21.
Como antecedentes ginecológicos destacan 3 partos eutócicos y una salpinguectomía
por gestación ectópica. Menopausia a los 53 años. Señale la respuesta correcta:

1. 1. El cuadro clínico puede explicarse por alteraciones hormonales.


2. 2. El mejor método diagnóstico es el ultrasonido transvaginal.
3. 3. Al ser autolimitado y no ser obesa, podemos tener actitud expectante.
4. 4. Realizaremos una histeroscopía diagnóstica en consulta con toma de biopsia.
Gráfico de respuestas
Comentario

Pregunta básica en el manejo de las metrorragias postmenopáusicas. La paciente no tiene


actividad hormonal, por lo que el sangrado puede deberse a atrofia endometrial o a neoformación
endometrial entre otros. Por tanto, es recomendable realizar un USG que nos oriente, pero la mejor
prueba diagnóstica es la histología, siendo la histeroscopía el mejor método para obtenerla. Por
tanto, ni estaremos expectantes, ni pautaremos estrógenos porque siempre debemos pensar en
que se puede esconder un origen neoplásico.(R4)

196. Masculino de 60 años, cirrótico por VHC en tratamiento con espironolactona y


furosemida a dosis máximas, sin otros antecedentes de interés. Acude a consulta por
aumento de perímetro abdominal y dolor en ambos flancos y espalda. En la exploración
física se observó abdomen distendido y oleada ascítica. En la analítica destacaba: Hb
11.0 g/dl, VCM 80.5 fl, leucocitos 10.000/mm3 con 50% neutrófilos, plaquetas 78.000/mm3,
glucosa 89 mg/dl, urea 101 mg/dl, creatinina 2.5 mg/dl, proteínas totales 5.6 g/dl, albúmina
3.3 g/dl. GOT 88 U/L, GPT 101 U/L, GGT 178 U/L, bilirrubina total 1.8 mg/dl, fosfatasa
alcalina 108 U/L, LDH 187 U/L, sodio 133, potasio 4.8.¿Cuál sería la actitud para el control
de este paciente?:

1. 1. TIPS
2. 2. Paracentesis evacuadora de repetición.
3. 3. Aumentar la dosis de diuréticos.
4. 4. Tratamiento con Peg-IFN y rivabirina.
Gráfico de respuestas
Comentario

Dudo bastente que reguntes un TIPS, pero si te van a preguntar sobre e pcinte cirrótico con as

citis. Se trata de un paciente que acude con ascitis importante no controlada a pesar de diurético a
dosis máximas. Se trata, por tanto, de una ascitis refractaria. Para ello puede plantearse la
realización de paracentesis evacuadotes o TIPS. En este caso nos decantaríamos por ésta última

 
 
 
 
ya que tiene insuficiencia renal y no se observa que sea un paciente con encefalopatía hepática
previa ni un CHILD muy avanzado.

Lo que si debes recorda para el ENARM es la clasificación de CHILD.(R1)

197. ¿Cuál de las siguientes opciones acerca de la mama y la lactancia materna es


FALSA?

Los alveolos desembocan en los conductos galactóforos, que se unen para formar los 12 a
1. 1.
15 conductos que desembocan en el pezón.
Los estrógenos bloquean la acción de la prolactina en la mama, por eso no se produce
2. 2.
eyección láctea hasta que se expulsa la placenta y descienden sus niveles.
Durante los dos-tres primeros días de la lactancia se secreta una leche amarillenta, llamada
3. 3. calostro, que se caracteriza por tener más grasa y menos proteínas y azúcares que la leche
madura.
Entre los factores que contribuyen a mantener la lactancia, encontramos una correcta
alimentación e ingesta hídrica, una buena succión y descarga del pecho en cada toma y
4. 4.
factores psicológicos y sociales que actúan en la regulación de la secreción de prolactina y
oxitocina.
Gráfico de respuestas
Comentario

Durante el embarazo, los estrógenos y la progesterona “bloquean” la acción de la prolactina sobre


la glándula mamaria. Después del alumbramiento, se produce una brusca disminución de estas
hormonas, lo que permite la acción de la prolactina sobre la producción y secreción láctea. Aparte
de estas hormonas, también juegan un papel algunas otras, como el lactógeno placentario.
Recordemos que, además, para que se instaure la lactancia, es necesario que se complete un
reflejo neurohormonal en el que participa la succión por parte del recién nacido.

La primera secreción láctea recibe el nombre de calostro. Es característica de su composición la


riqueza en proteínas, con una escasa proporción de lactosa y lípidos.(R3)

 
 
 
 

198.
Una joven de 17 años consulta por cefaleas frecuentes y epistaxis. Su estatura es de 140
cm, la coloración cutánea es normal y presenta Pterigium colli. La tensión arterial es
175/100 mmHg en ambos brazos, se ausculta un soplo mesosistólico en la parte alta del
borde esternal izquierdo y los pulsos femorales son débiles y simétricos. En el ECG se
aprecia lo que muestra la imagen 12. ¿Cuál de los siguientes diagnósticos es el más
probable?

1. 1. Estenosis aórtica supravalvular.


2. 2. Miocardiopatía hipertrófica obstructiva.
3. 3. Coartación de aorta.
4. 4. Estenosis valvular pulmonar.
Gráfico de respuestas
Comentario

El aspecto más importante que debes dominar respecto a las cardiopatías congénitas es saber
reconocerlas en un cuadro clínico, pues de esta forma es como se preguntan en el MIR. Para eso,
apréndete un par de datos característicos de cada una de ellas. En esta pregunta hay varios datos
que orientan el diagnóstico de coartación de aorta: HTA en un paciente joven con signos debidos a
ésta (cefalea y epistaxis), ausencia o disminución de los pulsos femorales y soplo mesosistólico en
cara anterior del tórax. Recuerda que en los varones la coartación de aorta es dos veces más
frecuente que en mujeres, aunque las mujeres con síndrome de Turner (baja estatura y Pterigium
colli) padecen frecuentemente coartación de aorta. Otras asociaciones de esta cardiopatía que
debes recordar son la válvula aórtica bicúspide y los aneurismas del polígono de Willis. Recuerda
que en el ECG suelen objetivarse datos sugestivos de hipertrofia ventricular izquierda.(R3)

199. Señale cuál de las siguientes es la causa más frecuente de isquemia mesentérica
aguda:

1. 1. Bajo gasto cardíaco.


2. 2. Embolia arterial aguda.
3. 3. Trombosis aguda de la vena mesentérica superior.
4. 4. Trombosis arterial aguda secundaria a una arteriosclerosis.
Gráfico de respuestas
Comentario

 
 
 
 
Concepto importante para el nacional. La isquemia mesentérica aguda es un proceso obstructivo
brusco de la arteria mesentérica debido con más frecuencia a una embolia en pacientes con
fibrilación auricular u otro proceso embólico. Con menos frecuencia se debe a una trombosis in situ
de una placa de arteriosclerosis o a bajo gasto.(R2)

200. .
Paciente primigesta de 35 semanas de gestación que acude a urgencias por percibir
contracciones durante la noche. Exploración cervical: cérvix formado, permeable 1 dedo.
Se realiza registro cardiotocográfico que se muestra en la imagen. ¿Cuál seria la actitud
CORRECTA en esta paciente?

Reposo e hidratación en domicilio. Acudir si presenta contracciones más intensas y


1. 1.
regulares.
2. 2. Ingreso e iniciar tocólisis y maduración pulmonar.
3. 3. Realizar una prueba de Pose, pues el RCTE tiene RAF negativo.
Finalizar la gestación si el Doppler en arteria umbilical está alterado ya que el RAF es
4. 4.
negativo.
Gráfico de respuestas
Comentario

Ya hemos repasado este concepto en simulacros previos. A partir de las 34 semanas de gestación,
se considera que el feto es maduro desde el punto de vista pulmonar, por lo que se dejará
evolucionar el parto en caso de instaurarse, sin necesidad de tratamiento tocolítico ni
corticoterapia. En cuanto al registro cardiotocográfico que nos presentan, puede considerarse
normal, por lo que se descarta la opción 3.

En casos como éste, es suficiente proceder con medidas conservadoras como la 1, con reposo
domiciliario, sin ninguna medida terapéutica adicional. En el caso de que se tratara de un cérvix de
parto, se esperaría a la evolución espontánea del mismo.(R1)

201. Ante una paciente de 42 años, con cáncer de mama, a la que se le ha realizado
tumorectomía y linfadenectomía, con ganglios positivos para infiltración tumoral y
receptores estrogénicos negativos, ¿qué actitud debemos tomar a continuación?:

1. 1. No pautar tratamiento adyuvante, y control a los 6 meses.


2. 2. Radioterapia sólo.
3. 3. Quimioterapia sólo.
4. 4. Radioterapia y quimioterapia.
Gráfico de respuestas

 
 
 
 
Comentario

La opción correcta es la número 4, ya que al ser una tumorectomía estrictamente se tiene que
realizar radioterapia; y al tener ganglios positivos se tendrá que administrar quimioterapia. Al tener
receptores de estrógenos positivos no es necesario la administración de tamoxifeno.(R4)

202. Las acciones biológicas de los estrógenos son las siguientes, EXCEPTO:

1. 1. Inducen el crecimiento del útero.


2. 2. Bajo su acción el moco cervical aumenta la elasticidad.
3. 3. Estimulan el tono vegetativo simpático.
4. 4. Inducen la expresión de receptores de FSH.
Gráfico de respuestas
Comentario
Los estrógenos tiene una liberación bimodal: crecen hasta el pico preovulatorio (24- 36 h antes de
la ovulación) y tiene otro pico menor en la fase lútea. Son tróficos para todo el aparato genital. A
nivel local inducen receptores de FSH. Niveles bajos y moderados inhiben la secreción de FSH y
niveles altos tiene un efecto gatillo sobre LH. Estimulan el crecimiento y la proliferación de los
órganos sexuales femeninos. Bloquean a la PRL en la excreción de la leche. La disminución de los
niveles circulantes de estrógenos tiene que ver en la causa de la osteoporosis
postmenopáusica.(R3)

203. ¿Cuál es la causa más frecuente de conjuntivitis infecciosa neonatal?:

1. 1. Neisseria meningitidis.
2. 2. Neisseria gonorrhoeae.
3. 3. Chlamydia trachomatis.
4. 4. Staphilococcus aureus.
Gráfico de respuestas
Comentario

Hoy por hoy, la causa más frecuente de conjuntivitis neonatal, en tanto en cuanto la enfermedad
gonocócica se circunscribe a medios socioeconómicos deprimidos y de gran promiscuidad, es la
que aparece secundariamente a vaginitis y cervicitis por clamidia. Recuerde que la conjuntivitis por
clamidia puede derivar en el llamado tracoma, y que aparece tras un tiempo de incubación que
oscila entre 5 y 14 días.(R3)

204. En relación con la pericarditis urémica en un paciente con insuficiencia renal crónica
terminal o en diálisis ¿qué medida terapéutica estaría CONTRAINDICADA?:

1. 1. Es una causa mandatoria para iniciar diálisis.


2. 2. Uso de antiagregantes y/o anticoagulantes.
3. 3. Se debe incrementar la dosis y "calidad" de la diálisis.
4. 4. No se utilizan diuréticos ni antiinflamatorios no esteroideos
Gráfico de respuestas
Comentario
La clínica de la IRC es ubicua, si bien recordar la que afecta específicamente al aparato
cardiovascular no viene mal, pues al fin y al cabo el IAM es la causa más frecuente de muerte en
estos enfermos. También pueden padecer cardiopatía urémica, pericarditis urémica, ACVs, ICC e
incluso EAP (por retención de líquidos)? Recuerda que la HTA es la complicación más frecuente en

 
 
 
 
pacientes con IRC terminal. En este caso la cuestión es: ¿qué no deberíamos hacer ante una
pericarditis urémica? Si sabemos que es de características serofibrinosas pero con tendencia a
hemorragia, entonces está claro que no debemos dar ni antiagregantes, ni anticoagulantes, ni
AINEs ante el riesgo de hacer rápidamente un taponamiento cardíaco. Para evitar el bajo gasto y la
hipotensión, recuerda que no se deben dar diuréticos ni en el taponamiento cardíaco ni en el IAM
de ventrículo derecho. Y recuerda también que la pericarditis urémica es indicación de diálisis,
junto con otras muchas.(R2)

205. De entre las modificaciones gravídicas, cual es INCORRECTA:

El aumento de la volemia no conlleva hipertensión arterial al disminuir las resistencias


1. 1.
vasculares periféricas.
Existe un aumento del sistema renina-angiotensina-aldosterona, aunque su efecto quede
2. 2.
bloqueado.
Los uréteres pueden dilatarse desde la semana 8 hasta tres meses después del parto, siendo
3. 3.
más evidente en el uréter izquierdo.
A nivel bucal puede aumentar las pérdidas dentarias debido a un aumento de la salivación
4. 4.
y a una disminución del pH de dicha saliva.
Gráfico de respuestas
Comentario

La volemia aumenta hasta un 40% en la semana 28 y además existe un aumento de 5 litros de


volumen extravascular, lo que puede explicar los edemas en la gestante. El aumento de la volemia
no conlleva hipertensión arterial al disminuir las resistencias vasculares periféricas, debido al efecto
miorrelajante de la progesterona. No obstante, el aumento de estrógenos provoca un aumento del
sistema renina- angiotensina- aldosterona, que daría lugar a hipertensión, pero no es así por el
efecto de la progesterona, que relaja los vasos y bloquea el efecto de la angiotensina II. Los
uréteres pueden dilatarse desde la semana 8, una vez más por la progesterona, y puede durar
hasta tres meses después del parto, siendo más evidente en el uréter derecho por la
dextrorrotación del útero debido al espacio que ocupa el sigma. A nivel bucal el aumento de la
saliva y que además sea más ácida conlleva que puedan aumentar las pérdidas dentarias.(R3)

206. En la apnea obstructiva del sueño grave, el tratamiento más satisfactorio consiste
en:

1. 1. Disminución de peso.
2. 2. Presión positiva continua nasal en las vías respiratorias (PPCV).
3. 3. Uvulopalatofaringoplastia.
4. 4. Presión espiratoria continua nasal (PEEP).
Gráfico de respuestas
Comentario
Es una pregunta de dificultad baja y para su correcta respuesta deberían bastar los conceptos
adquiridos en primera vuelta. Vamos a recordar unas nociones básicas sobre el tratamiento para
responder la pregunta; el primer escalón del tratamiento lo constituyen diversas medidas higiénico-
dietéticas (evitar alcohol y sedantes, reducir peso, evitar dormir en decúbito supino, dispositivos
intraorales...) que podrían bastar si la apnea fuese leve- moderada. Si la apnea es severa o si lo
anterior no es suficiente, en la actualidad lo más usado (y más eficaz) es la CPAP (opción 2
correcta), que previene el desarrollo de la presión subatmosférica crítica que produce el colapso de
las vías respiratorias. Por último y sólo en los casos refractarios graves, se puede recurrir a la
cirugía (uvulopalatofaringoplastia o traqueostomía).(R2)

 
 
 
 
207. Una mujer de 74 años acude a urgencias por dolor torácico y mareo. En el ECG,
destaca una elevación del ST en II, III y aVF y un bloqueo auriculoventricular de tercer
grado. La presión arterial es 80/50mmHg. La primera medida a llevar a cabo será:

1. 1. Nitroglicerina sublingual.
2. 2. Atropina intravenosa.
3. 3. Dobutamina intravenosa.
4. 4. Líquidos intravenosos.
Gráfico de respuestas
Comentario
En los pacientes con infarto agudo de localización inferior es frecuente que se pongan en marcha
reflejos (reflejo de Bezold- Harisch) que producen una descarga del Vago (colinérgico) sobre el
corazón y la circulación produciendo bradicardia sinusal o incluso bloqueo aurículo ventricular y
vasodilatación arterial con hipotensión arterial. El tratamiento de esta reacción vagal es con
líquidos y atropina intravenosa y suele responder de manera adecuada. Con respecto a las
bradiarritmias que pueden ocurrir en la fase aguda del infarto cabe destacar que los bloqueos
aurículo ventriculares y la disfunción sinusal son más frecuentes en los IAM inferiores que en los
anteriores. Los BAV de los IAM inferiores suelen estar localizados a nivel suprahissiano, sulen
tener un mecanismo reflejo vagal por lo que suelen responder a atropina. Sin embargo los BAV de
los IAM anteriores suele estar localizado a nivel del HIS o más abajo, sulen ser mor necrosis
extensa del sistema de conducción y no responden a atropina, marcando un mal pronóstico.(R2)

208. Niño de 4 años con


fiebre de 6 días de evolución y exantema maculopapular en tronco y cara. A la exploración
se detecta una adenopatía laterocervical izquierda, hiperemia conjuntival bilateral sin
secreción, y los labios que aparecen en la imagen. Los padres refieren que le notan
decaído. ¿Cuál es el diagnóstico?

1. 1. Exantema súbito.
2. 2. Enfermedad de Kawasaki.
3. 3. Sarampión.
4. 4. Fiebre faringoconjuntival.
Gráfico de respuestas
Comentario

 
 
 
 
La efermedad de Kawasaki se caracteriza por un cuadro de fiebre elevada de más de 5 días de
evolución, asociado a una serie de hallazgos clínicos: adenopatía (laterocervical mayor de 1cm),
afectación bucal (enrojecimiento y fisuras en labios y lengua aframbuesada), conjuntivitis bilateral
no supurada, afectación de dedos, palmas y plantas y exantema.

La asociación de fiebre junto con 4 de los otros criterios mencionados previamente y la afectación
del estado general, debe hacernos sospechar en un Kawasaki. Respuesta 2 correcta.(R2)

209. Se decide realizar estudios de laboratorio. ¿Cuál de los siguientes datos no sería
compatible con el cuadro que sospechamos?

1. 1. VSG elevada.
2. 2. Leucocitosis.
3. 3. Piuria estéril.
4. 4. Hemocultivo positivo.
Gráfico de respuestas
Comentario

Los criterios de laboratorio de la enfermedad de Kawasaki son los siguientes:

Elevación de reactantes de fase aguda, leucocitosis, elevación de transaminasas,


hipoalbuminemia.

Hemocultivos y urocultivos negativos.

Estos datos asociados a los datos clínicos confirman el diagnóstico.(R4)

210. Señale cuál de las siguientes opciones acerca del cordón umbilical en el recién
nacido es INCORRECTA:

1. 1. El cordón suele desprenderse durante las cuatro primeras semanas de vida.


El desprendimiento retrasado del cordón (posterior al mes) puede orientarnos a un defecto
2. 2.
de la quimiotaxis de neutrófilos.
Los vasos sanguíneos del cordón están funcionalmente cerrados pero son permeables
3. 3.
durante unos días.
Durante el período de tiempo en que los vasos siguen permeables son una puerta de
4. 4.
entrada potencial para infecciones.
Gráfico de respuestas
Comentario

El desprendimiento del cordón umbilical suele ocurrir en las dos primeras semanas de vida, y no
después; el retraso en la caída se asocia con trastornos de la inmunidad Recuerden también que la
onfalitis puede ser la puerta de entrada para el desarrollo de sepsis, dado el acceso vascular que el
ombligo presenta.(R1)

211. Con respecto al megacolon aganglionar congénito (enfermedad de Hirschprung).


Señala lo FALSO:

1. 1. Puede asociarse con enteropatia perdedora de proteínas.


2. 2. Pueden aparecer vómitos biliosos o fecaloideos.

 
 
 
 
La anomalía funcional consiste básicamente en un tono muscular disminuido del segmento
3. 3.
agangliónico del intestino.
4. 4. Pueden tener periodos de estreñimiento alternados con diarrea.
Gráfico de respuestas
Comentario

El megacolon congénito supone la causa más frecuente de obstrucción intestinal baja en el RN. La
enfermedad de Hirschprung se debe a una inervación anómala del colon, secundaria a una
interrupción en la migración neuroblástica dando lugar a una ausencia del sistema nervioso
parasimpático intramural (plexos de Meissner y Auerbach) y una hiperplasia compensadora del
sistema nervioso parasimpático extramural (fibras colinérgicas).

Pueden presentar vómitos fecaloideos, acompañados de signos de deshidratación. Otra forma de


manifestarse es aquella que alterna periodos de estreñimiento con episodios de diarrea que es
posible que lleguen a provocar una enteropatía pierdeproteínas.

La anomalía funcional consiste en un tono muscular aumentado en el segmento agangliónico, lo


cual resulta de utilidad para el diagnóstico mediante manometría anorrectal: ausencia de relajación
del esfínter anal interno ante un aumento de presión a ese nivel (lo normal es la relajación del
mismo).(R3)

212. ¿Cuál de los siguientes NO pertenece al síndrome de Heerfordt?:

1. 1. Aumento bilateral de parótidas.


2. 2. Uveitis anterior.
3. 3. Adenopatías hiliares bilaterales.
4. 4. Parálisis facial.
Gráfico de respuestas
Comentario
El síndrome de Heerfordt- Waldenstrom o fiebre uveoparotídea es una de las dos formas de
presentación de la sarcoidosis aguda ( junto con el síndrome de Lofgren ). Consiste en fiebre,
aumento del tamano de las parótidas, uveítis anterior y parálisis del nervio facial.(R3)

213. En un paciente con fiebre, malestar general y artralgias de comienzo gradual, que en
la exploración se evidenció un soplo mitral de escasa entidad, se obtuvieron tres
hemocultivos a su ingreso. A las 18 horas se observa enturbiamiento de todos los frascos
y en la tinción de Gram, vemos un coco gram-positivo que forma cadenas. Con estos
datos, podemos casi asegurar que padece una endocarditis por:

1. 1. Streptococcus grupo viridans.


Un estreptococo, pero debemos esperar al crecimiento en medios sólidos para
2. 2.
identificarlo.
3. 3. Staphylococcus aureus.
4. 4. Staphylococcus epidermidis.
Gráfico de respuestas
Comentario

Cuando nos hablan de un coco grampositivo que forma cadenas, está claro que debe tratarse de
un estreptococo. Entre las respuestas 1 y 2, la más lógica sería la 2, ya que por los hallazgos
morfológicos no sería posible identificar la especie. Aunque lo más probable es que sea un

 
 
 
 
estreptococo del grupo viridans, la opción 2 es más amplia y, por lo tanto, con menos posibilidades
de equivocarse.(R2)

214. ¿Cuál de las siguientes afirmaciones sobre la enfermedad pélvica inflamatoria es


INCORRECTA?

1. 1. La mayoría de las veces se origina por vía ascendente.


2. 2. Para el diagnóstico es imprescindible la toma de cultivo por laparoscopía.
3. 3. La promiscuidad sexual incrementa el riesgo de padecer la enfermedad.
4. 4. Puede tener como secuela la esterilidad.
Gráfico de respuestas
Comentario

Tema básico para el ENARM, que debes dominar. La enfermedad inflamatoria pélvica (EIP) es una
infección de útero, trompas y ovarios, de origen bacteriano, que alcanza estas regiones por vía
ascendente. La causa más frecuente es Chlamydia trachomatis, seguida por Neisseria
gonorrhoeae. Obviamente, para padecer esta enfermedad, la promiscuidad es un factor de riesgo
muy importante.

Clínicamente se caracteriza por un dolor abdominal bajo, habitualmente bilateral, con leucorrea
acompañante.Otros síntomas sistémicos, como fiebre, náuseas y/o vómitos, indican inflamación
peritoneal y una afectación clínica más grave.

A largo plazo, puede producir alteraciones tubáricas, con fibrosis y retracción, y secundariamente
esterilidad. Otra complicación, menos grave pero más frecuente, sería el dolor pélvico crónico.

El diagnóstico de EIP es fundamentalmente clínico (dolor abdominal bajo + leucorrea, con o sin
fiebre). La laparoscopía es, sin duda, el método diagnóstico más seguro, pero sólo se hace en
casos muy puntuales (dudas importantes, diagnóstico diferencial con otras patologías), dado que
es un método caro y complejo.

Recuerda que de inicio debes ser lo menos inasivo en la mayoría de los casos.

El tratamiento ideal es prevenir la transmisión de ETS. Una vez instaurado el cuadro, pautaríamos
antibioterapia dirigida a los microorganismos más implicados (ceftriaxona + doxiciclina, por
ejemplo).Es infrecuente recurrir a la cirugía para el tratamiento de esta entidad.(R2)

215. Paciente femenino de 52 años con nódulo mamario no doloroso, de 1 cm, en el


cuadrante inferointerno de la mama derecha. La mamografía refiere "nódulo estrellado
con microcalcificaciones agrupadas, compatible con carcinoma". La ecografía no es
concluyente. Se realiza una punción-aspiración con aguja fina (PAAF) que es informada
como "cambios propios de mastopatía fibroquística". ¿Qué actitud debe tomar?

1. 1. Repetir la PAAF.
2. 2. Biopsia excisional.
3. 3. Tumorectomía y linfadenectomía.
4. 4. Mastectomía simple.
Gráfico de respuestas
Comentario

 
 
 
 
Ante un nódulo estrellado con microcalcificaciones agrupadas, la sospecha inicial debe ser un
cáncer de mama. Por este motivo, está indicada la realización de una PAAF. Si el resultado fuese
positivo para cáncer, no haría falta una biopsia posterior para confirmarlo. En cambio, si es
negativo para malignidad, como en este caso, no excluye que en realidad se trate de un cáncer,
porque esta técnica tiene sus limitaciones (por ejemplo, haber pinchado en una zona libre de
cáncer). Dada esta posibilidad, es necesario realizar una prueba más fiable, por lo que debemos
obtener una muestra tisular más amplia, mediante una biopsia.(R2)

216. Mujer de 30 años ingresa en urgencias por fiebre alta y taquipnea, además de un
cierto grado de deshidratación. En la analítica de urgencia se observan los siguientes
datos: urea de 65 mg/dl, Na+: 150 mEq/dl, Cl-: 84 mEq/dl, K+:6,1 mEq/dl y bicarbonato: 15
mEq/dl. En orina hay oliguria y glucosuria. Es cierto que:

1. 1. Estos datos son compatibles con acidosis tubular renal.


2. 2. Existe aumento del anión gap.
3. 3. La pCO2 será probablemente > de 45 mmHg.
4. 4. Tiene una alcalosis metabólica.
Gráfico de respuestas
Comentario

Debes manejar con soltura el equilibrio ácido- base, ya que es un tema muy preguntado en el
examen.

En la valoración de las acidosis metabólicas, es útil conocer el anión gap. Está formado por ácidos
que fisiológicamente forman parte del contenido del plasma y que resultan del metabolismo normal.
Se calcula mediante la siguiente fórmula:

AG = [Na+] – ( [Cl- ] + [HCO3- ] ) En condiciones normales está entre 10 +/- 2 mEq/l.

Si lo calculamos en este caso concreto AG= 150- (84+15)= 51 mEq/l, por lo que en este caso hay
un aumento del anión gap (RC- 2).

No nos dicen el valor del pH, por lo que no sabemos si las respuestas 1, 4 y 5 son ciertas.

La taquipnea provoca disminución de la pCO2, por lo que es muy poco probable que esta sea
superior a 45 mmHg como dice la respuesta 3.(R2)

217. Señale cuál de las siguientes situaciones es indicación de tratar con isoniazida una
infección tuberculosa latente:

1. 1. Mantoux positivo en sujeto de 14 años.


2. 2. Mantoux positivo en sujeto de 68 años.
3. 3. Mantoux negativo detectado en el estudio inicial de un paciente infectado por virus VIH.
Paciente de 66 años en estudio por fiebre sin foco, con un primer Mantoux negativo y un
4. 4.
segundo (realizado 7 días después) positivo.
Gráfico de respuestas
Comentario
Un Mantoux positivo en sujetos menores de 35 años es indicación de tratamiento con Isonizida
(opción 1ª correcta, la 2ª es incorrecta).Las personas que han sido tratadas correctamente
mantienen la inmunidad y no precisan tratamiento adicional (opción 3ª falsa). En un paciente VIH
positivo hay indicación de tratar la infección tuberculosa latente, es deci deben recibir Isoniazida,

 
 
 
 
aquellos que tengan Mantoux positivo (opción 4ª incorrecta).En la opción 5ª, el Mantoux que pasa
de negativo a positivo en 7 días es un fenómeno Booster, que indica infección de larga evolución y
en una persona de 66 años NO es indicación de tratamiento.(R1)

218. ¿Cuál de los siguientes enunciados sobre la preeclampsia es FALSO?:

1. 1. Lo habitual es que aparezca en la segunda mitad del embarazo.


2. 2. Histológicamente, la lesión renal más característica es la endoteliosis glomerular.
Dado que el diazóxido tiene carácter teratógeno, el control tensional se realizaría con
3. 3.
alfametildopa o con atenolol.
4. 4. En general, se acepta que la vía vaginal es preferible a la cesárea.
Gráfico de respuestas
Comentario

Cuidado porque la alfametildopa está indicada pero el atenolol produce CIR y no se puede usar en
el embarazo. Las opciones son alfametildopa o labetalol, junto con hidralazina para las crisis, y el
nifedipino con la precaución de recordar que además de antihipertensivo, tiene efecto
tocolítico.(R3)

219. A 48-year-old male comes to the emergency room with vomiting and severe flank
pain that radiates to his groin. He has had similar episodes before. Kidney ultrasound
study shows a renal stone. 24-hour urine collection shows a urinary calcium excretion of
400mg (normal is less than 300 in males). Serum calcium and PTH levels are normal.
Further investigations do not reveal the cause of his hypercalcemia. Which of the
following treatments is the most advisable for this patient?

1. 1. Furosemide
2. 2. Hydrochlorothiazide
3. 3. Increased salt in his diet
4. 4. Low dose NSAIDs
Gráfico de respuestas
Comentario
Hydrochlorothiazide. Thiazide diuretics lower calcium renal excretion, lowering the chance of further
calculi.(R2)

220. Dentro de las complicaciones de la enterocolitis necrotizante se incluyen las


siguientes EXCEPTO:

1. 1. Síndrome de intestino corto.


2. 2. Perforación intestinal.
3. 3. Alcalosis hiperclorémica.
4. 4. Estenosis intestinal.
Gráfico de respuestas
Comentario

Se pregunta bastante en el ENARM la enterocolitis necrotizante, se enfocan más al típico cuadro


clínico. Las complicaciones de la enterocolitis necrotizante son las siguientes: sepsis de origen
entérico (por gram negativos y anaerobios) y perforación intestinal (y todo lo que este hecho
conlleva, cirugías, resecciones intestinales, síndrome de intestino corto, estenosis secundarias).
Recuerda que, analíticamente, la enterocolitis puede asociar trombopenia, acidosis e
hiponatremia.(R3)

 
 
 
 
221. Paciente de 18 años que acude a urgencias por dolor en hipogastrio, fiebre y
leucorrea amarilla, en su manejo han indicado lo siguiente, cuál le parece inapropiado:

1. 1. Estudio de secreción del cuello uterino.


2. 2. Test de gestación.
3. 3. Tratamiento antibiótico inmediato.
4. 4. Laparotomía exploratoria inmediata.
Gráfico de respuestas
Comentario

El cuadro clínico suena concordante con una enfermedad pélvica inflamatoria, que suele ocurrir en
la mayor parte en mujeres jóvenes con vida sexual activa, con lo que las tres primeras opciones
parecen indicadas. La laparotomía se reserva sólo para casos complicados (abscesos), o cuando
la terapéutica médica no ha servido (respuesta 4 incorrecta), es diagnóstica y terapéutica.(R4)

222. Hombre de 25 años refiere una historia de dos años de evolución consistente en
diarrea y dolor abdominal intermitentes. ¿Cuál de estos grupos de datos le parece más
consistente con el diagnóstico de colon irritable?

1. 1. Hematocrito 42%, albúmina 1,8 g/dl, sangre en heces (-), sigmoidoscopia normal.
Hematocrito 42%, albúmina 4,3 g/dl, sangre en heces (-), volumen de heces 1.000 ml/día y
2. 2.
sigmoidoscopia normal.
Hematocrito 42%, albúmina 4,3 g/dl, sangre en heces (-), volumen de heces 200 ml/día y
3. 3.
sigmoidoscopia normal.
Hematocrito 42%, albúmina 4,3 g/dl, sangre en heces (-), volumen de heces 200 ml/día y
4. 4.
sigmoidoscopia con áreas de eritema intenso en recto y sigma.
Gráfico de respuestas
Comentario

En el síndrome del intestino irritable, la colonoscopia es normal por definición (opcion 4 falsa) , no
existe hipoalbuminemia (opción 1 falsa), no hay sangre oculta en heces ni existe una diarrea tan
cuantiosa como se dice en la respuesta 2. La correcta, por tanto, es la 3. Recuerda que existe un
mayor número de deposiciones diarias, pero el volumen de las heces no está especialmente
aumentado. De hecho, puede hasta ser normal. No olvides tampoco las crisis de dolor cólico
abdominal que pueden aparecer sobre todo por la mañana, ni el moco en heces, datos muy típicos
de esta patología.(R3)

223. A 31-year-old female comes to your office complaining of breast pain and fever for
the past 2 days. She has a 4-week-old baby, who she breastfeeds. Physical examination
reveals: temperature 38,4ºC, and a hard, red, swollen area on her left breast, with no
fluctuation. What treatment would you recommend?

1. 1. Drainage
2. 2. Antibiotics, analgesics and nursing exclusively with the right breast
3. 3. Antibiotics, analgesics and continue breastfeeding
4. 4. Lactation suppression with bromocriptine, along with antibiotics
Gráfico de respuestas

(R3)

 
 
 
 
224. El carcinoma escamoso de cérvix uterino ocupa el quinto lugar de los cánceres que
afectan a la mujer. Se ha demostrado que su origen está íntimamente ligado a:

1. 1. Infección por virus del papiloma humano (HVP).


2. 2. El consumo de tabaco.
3. 3. Infecciones vaginales repetidas por Candida albicans.
4. 4. Infección por herpes virus tipo II.
Gráfico de respuestas
Comentario

Sin duda, el factor de riesgo más destacado e indispensable del carcinoma invasor de cuello es la
infección genital por el papovavirus. El HPV está muy relacionado con la génesis del cáncer de
cuello uterino y con la aparición de SIL. Los tipos más oncogénicos de HPV son: 16 y 18.

El virus del herpes tipo II, reflejo de promiscuidad sexual, se considera promotor de este cáncer. No
olvides que el tabaco también se considera factor de riesgo para este cáncer, pero el HPV se
considera más importante.(R1)

225. Cuando en la exploración de un niño en que sospechamos una cardiopatía


congénita, observamos cianosis sólo en miembros inferiores, pensaremos en:

1. 1. Tetralogía de Fallot.
2. 2. CIA.
3. 3. Ductus permanente con inversión del cortocircuito.
4. 4. Trasposición de grandes vasos.
Gráfico de respuestas
Comentario
En la mayoría de los casos, el ductus no es muy grande y no produce síntomas. Entonces el
ductus es diagnosticado al auscultar el soplo característico. En los ductus con cortocircuito
izquierda- derecha importante, puede producirse insuficiencia cardiaca en la infancia, a veces
incluso en el recién nacido. En estos casos puede encontrarse a un niño con retraso en el
desarrollo, que muestra taquipnea, retracción intercostal o subcostal y hepatomegalia. Cuando el
ductus se descubre en un niño mayor o en un adulto, suele ser un hallazgo casual en un paciente
asintomático. En los casos evolucionados, el aumento de las resistencias vasculares pulmonares
puede invertir el cortocircuito, apareciendo entonces acropaquias y cianosis en la parte inferior del
cuerpo, lo cual se denomina CIANOSIS DIFERENCIAL.(R3)

226. Lactante de 6 meses que consulta por presentar en el curso de un cuadro catarral
de vías altas, fiebre, irritabilidad y llanto. Exploración física: Tª rectal 39,6 ºC, tímpano
derecho hiperémico y abombado, faringe enrojecida con exudado amarillento y secreción
nasal abundante. Resto de exploración compatible con la normalidad. Indicar el
tratamiento más adecuado a seguir:

1. 1. Azitromicina 10 mg/kg/día, 3 días.


2. 2. Amoxicilina 80-90 mg/kg/día, 10 días.
3. 3. Cefixima 8 mg/kg/día, 10 días.
4. 4. Paracetamol 15 mg/kg/dosis.
Gráfico de respuestas
Comentario

 
 
 
 
Pregunta con respecto a una infección de vías respiratorias altas en un lactante de seis meses. Por
la edad del paciente, lo más probable es que la faringoamigdalitis que el paciente muestra sea de
tipo vírico, ya que además se nos refiere clínica de rinitis. El problema es que el paciente presenta
clínica acompañante de otitis media aguda incipiente. Ante las posibilidades terapéuticas lo más
correcto sería no instaurar tratamiento; no obstante, si hubiera que iniciarlo, la opción sería iniciar
amoxicilina a dosis de 80 mg/kg/día durante ocho o diez días.(R2)

227. Una niña de 10 años con atresia biliar, tratada con la técnica de Kasai cuando era
lactante, presenta ahora torpeza progresiva, disminución de los reflejos tendinosos
profundos y ataxia. El diagnóstico más probable es:

1. 1. Encefalopatía hepática.
2. 2. Encefalitis.
3. 3. Deficiencia de vitamina E.
4. 4. Ataxia de Friederich.
Gráfico de respuestas
Comentario

Un signo de que la técnica de Kasay ya no es funcionante, es la clínica relacionada con el déficit


vitamínico de las siguientes vitaminas ADEK. en particular, la vitamina E produce sintomatología
neurológica, la que se describe en la pregunta.(R3)

228. Los padres de un lactante de 5 meses acuden porque su hijo, que asiste a guardería,
presenta desde hace 3 días, coincidiendo con una infección respiratoria de vías altas,
heces líquidas en número de 4 al día, algún vómito alimentario ocasional y temperatura
axilar de 38.3°C. Es alimentado con lactancia mixta. En la exploración, el lactante tiene
buen estado general, está bien nutrido e hidratado y su respiración es adecuada; su peso
es de 4,730 g y ha descendido 70 g respecto al de la semana anterior. El abdomen está
blando y depresible, sin masas ni megalias, y la fontanela, normotensa. Salvo la
presencia de rinorrea acuosa, el resto de la exploración por órganos y aparatos es
normal. De las siguientes afirmaciones, señale la respuesta que considera CORRECTA:

Se debe realizar una estimación de las pérdidas, recomendar un ayuno de 4 horas y


1. 1.
rehidratar durante este tiempo con solución rehidratante oral.
2. 2. Es aconsejable la introducción de cereales de arroz por su efecto astringente.
3. 3. Se debe recomendar el empleo de fórmula sin lactosa.
Se ha de aconsejar reponer las pérdidas tras cada deposición, con solución rehidratante
4. 4.
oral, y continuar con su alimentación habitual.
Gráfico de respuestas
Comentario

Pregunta sencilla y muy importante acerca de la gastroenteritis infecciosa. Reconócela al tratarse


de un lactante que tiene vómitos, diarrea y fiebre. Al ser la exploración física normal, con un buen
estado de hidratación y sin pérdida significativa de peso (< 3 % del peso previo), estará indicado
continuar con su alimentación habitual, reponiendo las pérdidas tras cada vómito o deposición con
una solución de rehidratación oral (opción 4 correcta). La modificación de la dieta habitual no está
indicada al no modificar el curso de la sintomatología. Tampoco está indicada la realización de un
coprocultivo al tratarse con alta probabilidad de una infección vírica sin ningún signo de gravedad
en un paciente inmunocompetente.(R4)

 
 
 
 
229. Mujer de 68 años que consulta por síndrome constitucional en los meses previos.
En la exploración física se palpan adenopatías generalizadas y esplenomegalia. El valor
de la hemoglobina es de 8 g/dL y se detecta un componente monoclonal en el suero.
¿Cuál es el diagnóstico más probable?:

1. 1. Linfoma linfoplasmocitoide.
2. 2. Mieloma múltiple.
3. 3. Enfermedad de Hodgkin.
4. 4. Amiloidosis AL.
Gráfico de respuestas
Comentario

A pesar de que en presencia de un componente monoclonal en el suero el diagnóstico más


probable entre los citados es el mieloma múltiple, este proceso no cursa con adenopatías ni
esplenomegalia, por lo que el linfoma linfoplasmocitoide, es decir la macroglobulinemia de
Waldenström, es el diagnóstico más probable. La LLC casi nunca cursa con componente
monoclonal y el linfoma de Hodgkin tampoco. La amiloidosis AL lo tiene en orina (cadenas ligeras).
Respuesta 1 correcta.(R1)

230. Hombre de 43 años que recibe un trasplante


renal de donante cadáver. A los 10 días de la intervención comienza con fiebre, disnea
intensa y tos seca. Entre sus antecedentes destacaba que su esposa había presentado
un cuadro "catarral" en los días previos al comienzo de la clínica respiratoria del paciente.
Se realizó una radiografía de tórax y una tomografía computerizada (ver imagen). ¿Cuál
es su interpretación clínico-radiológica del caso?

Se observa un infiltrado en un lóbulo pulmonar de tipo alveolar (puesto que tiene el signo
1. 1.
del "broncograma aéreo") sugestivo de neumonía neumocócica.

 
 
 
 
Se observan nódulos pulmonares múltiples, algunos con cavitación de su zona central
2. 2.
sugestivos de aspergilosis invasiva.
Se observa un patrón de infiltrado pulmonar de tipo intersticial que afecta a ambos
3. 3. pulmones, compatible con neumonía bacteriana atípica, neumonía por Pneumocystis
jiroveci o con neumonía viral.
4. 4. Se observa un hidroneumotórax compatible con neumonía bacteriana complicada.
Gráfico de respuestas
Comentario

El patrón radiológico que se nos muestra en este caso no es, en absoluto, una condensación tipo
alveolar, como sucedería típicamente en una neumonía neumocócica. Lo que vemos es todo lo
contrario: un patrón difuso, bilateral, de carácter intersticial, lo que podría encajar en varios
diagnósticos: neumonía viral, bacteriana atípica o incluso por Pneumocystis, como dice la
respuesta 4. En el caso de este paciente, teniendo en cuenta sus antecedentes (trasplante renal),
es posible que esté inmunodeprimido y podría tratarse de cualquiera de estos tres
diagnósticos.(R3)

231. Se realizó una broncoscopia con toma de lavado broncoalveolar en la que el único
resultado microbiológico positivo fue el aislamiento de virus de la Gripe de tipo A (H3N2).
¿Cuál sería su actitud terapéutica en este momento?

Sólo indicaría tratamiento antibiótico ante la posibilidad de sobreinfección bacteriana por


1. 1.
neumococo, S. pyogenes o S. aureus.
2. 2. El paciente debe recibir lo antes posible tratamiento con oseltamivir.
El aislamiento de virus de la Gripe en un lavado broncoalveolar carece de significación
3. 3.
patológica.
4. 4. Sólo trataría a su esposa, como origen probable de la transmisión de la infección.
Gráfico de respuestas
Comentario

En el contexto de una gripe, se puede desarrollar una neumonía por sobreinfección bacteriana
(neumococo, S. pyogenes, S. aureus) o, menos frecuentemente, como consecuencia del propio
virus. Cuando se trata de una sobreinfección, normalmente veríamos un patrón compatible con
bronconeumonía, o incluso una neumonía lobar, datos que no encontramos en esta radiografía de
tórax. Lo que sí vemos es un patrón intersticial difuso, más sugestivo de neumonía de origen vírico,
que en este caso podríamos atribuir a la propia gripe. Por ello, dado que se trata de una gripe con
una complicación grave, estaría indicado el tratamiento con oseltamivir, un inhibidor de la
neuraminidasa.(R2)

232. El desarrollo motor fino del niño se evalúa con la siguiente actividad:

1. 1. Girar el tronco sobre su eje.


2. 2. Sostener la cabeza.
3. 3. Pararse sin apoyo.
4. 4. Armar torres con cubos.
Gráfico de respuestas
Comentario

Supongamos que no tiene el conocimiento específico para resolverlo, pero si tiene otras
herramientas que puede utilizar para contestarlo. Si se fija hay tres respuestas que tratan de
movimientos gruesos como la 1, 2 y 3, por ende la respuesta correcta es la 4.(R4)

 
 
 
 
233. Respecto a la muerta súbita del lactante, señale cuál de las siguientes opciones NO
es un factor de riesgo:

1. 1. El sexo masculino.
2. 2. El crecimiento intrauterino retardado.
3. 3. El uso de chupón precozmente.
4. 4. El colchón blando.
Gráfico de respuestas
Comentario

El uso del chupón se cree que es un factor protector del SMSL, el resto de las opciones son
factores de riesgo aunque hay que recordar que de los principales factores de riesgo el primero
sería la posición en prono para dormir y el segundo sería el tabaquismo materno.(R3)

234. En el caso de una amenaza de parto prematuro ¿en cuál se las siguientes situaciones
estaría indicado realizar un test de fibronectina para orientar la actitud terapéutica?:

1. 1. Cérvix sin modificar y dinámica uterina inexistente.


2. 2. Cérvix 4cm y dinámica uterina regular.
3. 3. Cervicometría de 12 mm y dinámica uterina regular.
4. 4. Cervicometría de 33 mm y dinámica uterina regular.
Gráfico de respuestas
Comentario
El test de fibronectina tiene un alto valor predictivo negativo por tanto nos pueden indicar que
pacientes tienen un riesgo muy reducido de tener un parto prematuro. Por tanto es de utilidad en
pacientes que no cumplen los 2 criterios de APP: modificación cervical y dinámica uterina regular.
Entre las opciones que nos proponen, el test de fibronectina sólo es útil en la opción 4 en la que no
tenemos un cérvix modificado pero si contracciones uterinas.(R4)

235. Los miomas uterinos submucosos son tumoraciones benignas de tejido muscular,
cuyo signo más frecuente es:

1. 1. Sangrado genital
2. 2. Dolor pélvico
3. 3. Leucorrea
4. 4. Ulcera cervical
Gráfico de respuestas
Comentario

La clínica más frecuente de los miomas submucosos es el sangrado, de hecho es tan importante
que es la causa más frecuente de indicación quirúrurgica debido a la anemia que puede generar en
las pacientes, el resto de las respuestas no tienen sentido.(R1)

236. Una mujer de 75 años acude a urgencias por presentar una tumoración dura y
dolorosa en la región inguinal izquierda a la palpación. Este cuadro probablemente
corresponde a:

1. 1. Hernia inguinal directa estrangulada.


2. 2. Herna inguinal indirecta estrangulada.
3. 3. Hernia femoral estrangulada.

 
 
 
 
4. 4. Aneurisma femoral.
Gráfico de respuestas
Comentario

Es importante que estudie y entienda los distintos tipos de hernias, sus características y sus
complicaciones. En concreto ya se ha preguntado en el ENARM un caso de hernia estrangulada.
La dificultad de esta pregunta es que hace referencia a dos conceptos. En primer lugar debemos
saber que la hernia inguinal más frecuente en la mujer es la hernia inguinal indirecta. Pero de igual
modo debemos tener presente que la hernia con más probabilidad de estrangularse es la hernia
crural o femoral. Como se indica en esta pregunta, la hernia crural es típica de mujeres mayores.
Por supuesto, es básico saber identificar un cuadro de estrangulación herniaria. Se compone de un
cuadro de obstrucción intestinal y aparición de masa dolorosa generalmente en la ingle.(R2)

237. Mujer de 32 años de edad con antecedentes de uso de anticonceptivos orales y


migrañas, que acude al ginecólogo por presentar un cuadro de prurito en región genital,
con secreción vaginal aumentada de 10 días de evolución, al que desde hace dos días,
según refiere la paciente, se ha añadido dolor, tanto en la micción como al mantener
relaciones sexuales. El pH de la secreción vaginal orienta hacia la identificación del
agente etiológico. ¿En qué germen pensaría si el pH es superior a 4.5?:

1. 1. Candida.
2. 2. Trichomonas vaginalis.
3. 3. Chlamydia.
4. 4. Gonococo.
Gráfico de respuestas
Comentario

Los agentes productores de vulvovaginitis son: Candida, Trichomonas, Gardnerella.

La infección por Candida (a la que predisponen: embarazo, anticonceptivos orales, corticoides,


antibióticos de amplio espectro...) produce prurito y un flujo blanquecino, espeso, en grumos. El
cultivo es el método más sensible y específico, y es característico que el pH vaginal no se
modifique.

Por otro lado, la infección por Trichomonas produce una leucorrea abundante, ligeramente
maloliente, amarillogrisácea o verde amarillenta, espumosa y con burbujas. A la exploración, el
cérvix puede presentar aspecto en fresa, y es característico un pH superior a 4.5 (opción 2
correcta).

Por último, Gardnerella produce una leucorrea blancogrisácea, maloliente, con olor a pescado en
descomposición. El pH es también sería mayor de 4.5, con dos características a recordar: olor a
pescado tras la prueba de las aminas y visualización en microscopía de células clave o clue
cells.(R2)

238. En relación a las enfermedades pulmonares del recién nacido señale lo incorrecto:

1. 1. La enfermedad de membrana hialina es por déficit de surfactante pulmonar.


2. 2. La taquipnea transitoria se presenta posterior a cesáreas.
3. 3. La aspiración del meconio se da en recién nacidos a termino.
4. 4. La aspiración del meconio no se asocia a hipertensión pulmonar.

 
 
 
 
Gráfico de respuestas
Comentario

En esta pregunta tiene que leer muy bien las respuestas. Para empezar le están preguntando lo
INCORRECTO. Partiendo de esto al leer la opción 4 (que la aspiración del meconio NO se asocia a
HTP), es obvio que esta es la respuesta que debe elegir, ya que esta asociación sí se
presenta.(R4)

239. En un paciente con hiperaldosteronismo primario, la diferencia entre hiperplasia


nodular y adenoma único debe establecerse por:

1. 1. Pruebas funcionales.
2. 2. Aldosterona y renina plasmáticas.
3. 3. Pruebas de imagen.
4. 4. Niveles de potasio.
Gráfico de respuestas
Comentario
Dentro de las posibles causas de un hiperaldosteronismo primario, la más frecuente es la
existencia de un adenoma unilateral (síndrome de Conn), seguido de la hiperplasia suprarrenal
bilateral. Para distinguir estas 2 patologías, las pruebas que ofracen una mayor rentabilidad son las
técinas de imagen, y dentro de ellas la TAC heliciodal con cortes finos. Otras de utilidad son la
RMN, gammagrafía con yodocolesterol y en raras ocasiones es necesario la realización de un
cateterismo de las venas suprarrenales. Dentro de las pruebas funcionales, en ocasiones, el test
postural puede ser útil para distinguir entre estos 2 distintos orígenes. En esta prueba se determina
la aldosterona y la renina basales y tras 3 horas de deambulación, encontrando una disminución o
no modificación de la aldosterona en los casos de adenoma unilateral. En la hiperplasia existe una
elevación de la aldosterona tras este test. De cualquier forma la rentabilidad es mucho menor que
con las técnicas de imagen. La aldosterona se encuentra elevada y la ARP disminuida en los 2
casos, puede existir hipopotasemia en ambos, y la clínica no ofrece ninguna ayuda.(R3)

240. Niño de 1 año de edad que acude a urgencias por presentar nistagmo horizontal
bilateral. En la exploración destaca una masa abdominal. Respecto a la patología que
sospecha indique la respuesta FALSA:

1. 1. Puede producir demencia progresiva.


2. 2. Puede asociarse a un hematoma lineal en el párpado.
3. 3. Las catecolaminas en orina suelen encontrarse elevadas.
Puede encontrarse un Síndrome de Pepper asociado con afectación hepática difusa sin
4. 4.
hepatomegalia.
Gráfico de respuestas
Comentario

Una pregunta difícil cuya contestación correcta implica un buen conocimiento del tema.

El diagnóstico de sospecha sería un neuroblastoma debido a que es la causa más frecuente de


masa abdominal en la infancia y puede manifestarse con movimientos oculares como síndrome de
Kinsbourne u opsoclono- mioclono que puede progresar a demencia progresiva.

Puede asociarse a hematoma lineal del párpado. Un dato específico de este tumor que aparece en
el 90% de los enfermos es la elevación de catecolaminas en orina de 24 horas.

 
 
 
 
El síndrome de Pepper se ha descrito en lactantes con neuroblastoma por afectación hepática
difusa CON hepatomegalia.(R4)

241. En relación a la Enfermedad Inflamatoria Pélvica, señale la opción INCORRECTA:

Si la paciente es portadora de DIU es necesario retirarlo previamente al inicio de cualquier


1. 1.
tratamiento antibiótico.
La esterilización tubárica (ligadura tubárica) disminuye el riesgo de Enfermedad
2. 2.
Inflamatoria Pélvica.
El tratamiento puede hacerse de forma ambulatoria mediante una combinación antibiótica
3. 3.
que posea amplio espectro de actividad.
Las pacientes con un diagnostico dudoso de Enfermedad Inflamatoria Pélvica deben ser
4. 4.
tratadas, ya que es preferible el tratamiento precoz para evitar secuelas.
Gráfico de respuestas
Comentario

La respuesta incorrecta es la 1: no es imprescindible retirar el DIU si hay enfermedad inflamatoria


pélvica, y si se hace, no tiene por qué ser antes de empezar el tratamiento antibiótico. El
tratamiento antibiótico no se debe retrasar.

-La respuesta 2 es verdadera: clásicamente se dice que la ligadura tubárica protege contra la EIP.

-La respuesta 3 es verdadera: según la gravedad, el tratamiento puede ser ambulatorio u


hospitalario.

- La respuesta 4 es verdadera: ante la sospecha, se debe empezar tratamiento empírico, porque es


mucho lo que nos jugamos si se deja sin tratar.(R1)

242. Señale la causa más probable de hipoxemia en un paciente con la siguiente


gasometría arterial a nivel del mar: pH = 7.32, PaO2 = 58 mm Hg, PaCO2 = 60 mm Hg,
gradiente alvéolo-arterial de O2 = 15 mm Hg:

1. 1. Trastorno de la difusión.
2. 2. Shunt (cortocircuito).
3. 3. Hipoventilación.
4. 4. Disminución de la PO2 en el aire inspirado.
Gráfico de respuestas
Comentario
Se trata de una hipoxemia combinada con hipercapnia, es decir por hipoventilación. La
hipoventilación es pura, de causa extraparenquimatosa puesto que el gradiente alveolo- arterial de
oxigeno es menor de 15 mmHg. En el caso de que fuera disminución de la PO2 inspirada,
encontraríamos hiperventilacion con hipocapnia.(R3)

243. Which of the following statements is FALSE in relation to analytical parameters of


malnutrition?

The degree of hypoalbuminemia does not appear to correlate with clinical manifestations
1. 1.
in cases of kwashiorkor and kwashiorkor-marasm.
IGF-I levels is a very sensitive marker that gets easily affected by acute nutritional
2. 2.
changes so that it may be used to assess response to nutritional treatment.

 
 
 
 
Zinc deficit may occur even in the absence of diarrhea. The best parameter to assess Zinc
3. 3.
body deposits is to determine Zinc levels inside neutrophiles.
The most important factors that determine anemia in these patients are hypoproteinemia
4. 4.
and chronic infections.
Gráfico de respuestas
Comentario

Esta pregunta del tema de malnutrición es muy importante. Recuerde que hay una serie de
proteínas séricas que aportan información sobre el estado nutricional: transferrina, proteína
transportadora de retinol, prealbúmina y albúmina.

La albúmina es la de vida media más larga y se usa para la monitorización del estado nutricional a
largo plazo. Es mejor parámetro de estrés que nutricional ya que sus valores se ven influenciados
por muchos otros factores.

Las otras, de vida media más corta, son útiles para valorar cambios agudos.

El IGF- I (somatostatina) también es un buen marcador de la respuesta al aporte nutricional. En


cualquier caso, el dato de laboratorio más característico del kwashiorkor es la disminución de la
albúmina. En todos los tipos de malnutrición existe déficit de micronutrientes asociado (hierro,
B12...) y es éste déficit el factor más importante, pero no el único, que determina la anemia de
estos pacientes.

Recuerde que una de las manifestaciones más grave y constante es la inmunodeficiencia


secundaria. Son muy frecuentes los déficits de magnesio y potasio.(R4)

244. A 19-month-old child is brought to the emergency department with a history of fever
accompanied by vesicles with central umbilication that are located predominantly on both
cheeks. Thorough physical examination reveals submaxilar adenopathies. He has a
history of atopic dermatitis and began treatment with topical corticosteroids last week.
What is the most likely diagnosis?

1. 1. Eczema herpeticum
2. 2. Varicella infection
3. 3. Rubella
4. 4. Measles
Gráfico de respuestas
Comentario
Eczema herpeticum. Eczema herpeticum, also known as a form of Kaposi varicelliform eruption
caused by viral infection, usually by herpes simplex virus, is an extensive cutaneous vesicular
eruption that arises from pre-existing skin disease, usually atopic dermatitis. Local administration of
corticosteroids is the precipitating factor in this case.(R1)

245. Un paciente consulta por debilidad y en su hemograma se objetivan los siguientes


valores: hematíes 2.800.000/mm3, leucocitos 3.200/mm3, plaquetas 85.000/mm3. En la
exploración física se aprecia esplenomegalia. El diagnóstico MENOS probable sería:

1. 1. Aplasia medular.
2. 2. Cirrosis hepática.
3. 3. Enfermedad de Hodgkin.
4. 4. Enfermedad de Gaucher.

 
 
 
 
Gráfico de respuestas
Comentario
Pregunta de dificultad media. Para sacarla debéis recordar que la disminución de las tres líneas
celulares (pancitopenia); es un signo bastante inespecífico, por lo que podríamos encontrarlo en
cualquiera de las cinco opciones del enunciado. Pero hay un dato muy importante que no hay que
olvidar para el MIR, la aplasia de la medula ósea es una enfermedad sin ninguna evidencia de
infiltración neoplásica ni de síndrome mieloproliferativo, por lo que no tienen esplenomegalia
(opción 1 correcta) ni adenopatías.(R1)

246. La herpangina es característicamente producido por:

1. 1. Corynebacterium hemolyticum.
2. 2. Corynebacterium parvum.
3. 3. Coxsackievirus.
4. 4. Virus herpes simple.
Gráfico de respuestas
Comentario

Tema preguntado constantemente. Dentro de las infecciones que debe recordar producida por
Coxsackievirus son las siguientes:

Grupo A: herpangina, conjuntivitis hemorrágica aguda, enfermedad de mano-pie-boca.

Grupo B: pleurodinia, miocarditis, pericarditis y hepatitis.(R3)

247. Paciente de 25 años con una gestación sin controlar que presenta una amenorrea de
25 semanas. Refiere que esta mañana ha presentado dolor tipo contracciones. A la
exploración se evidencia un cérvix sin modificar. Se realiza un USG transvaginal con una
cervicometría de 37 mm. Feto en presentación cefálica con una biometría acorde a 26
semanas. En el RCTG aparece cierta irritabilidad uterina. ¿Cuál sería el siguiente paso a
realizar?:

1. 1. Alta sin tratamiento.


2. 2. Alta con progesterona vaginal
3. 3. Realizar test de fibronectina cervical.
4. 4. Ingreso y tratamiento con corticoides y tocolíticos.
Gráfico de respuestas
Comentario

Los datos que nos aporta el caso clínico informan de una gestación pretérmino, que presenta
escasa dinámica uterina y un cérvix >30 mm, lo que nos deja relativamente tranquilos ya que la
probabilidad de parto pretérmino es muy baja. No obstante en la batería de pruebas que podemos
realizar en este tipo de situaciones tenemos también disponemos del test de la fibronectina, con un
alto valor predictivo negativo, por lo que si tenemos opción a su realización sería mas correcto el
enfoque diagnóstico del caso. Si el resultado fuera positivo deberíamos entonces tratar a la
paciente como una amenaza de parto prematuro, si en cambio fuera negativo, no sería
necesario.(R3)

248. En un recién nacido con moniliasis oral, la fuente más frecuente de infección es:

 
 
 
 
1. 1. La vagina materna.
2. 2. Los objetos contaminados.
3. 3. La utilización de Ag NO3.
4. 4. El contacto con portadores hospitalarios.
Gráfico de respuestas
Comentario

La fuente más frecuente de infección por Candida en un recién nacido no ingresado en una unidad
de cuidados intensivos neonatal es la infección vaginal de la madre por este hongo. En los
lactantes y neonatos más mayores, lo más frecuente es que aparezca secundariamente a
perpetuidad del reservorio en el seno materno.(R1)

249. Atendemos a una mujer en trabajo de parto y un interno te ha pedido que le expliques
en que consiste el mecanismo de parto de una presentación de vértice. Le explicas que
una vez que se produce el paso del mayor de los diámetros de la cabeza fetal por el
estrecho superior de la pelvis (enclavamiento de la cabeza fetal), generalmente en
posición transversal, al ir descendiendo la cabeza, ésta se suele flexionar como
consecuencia de la presión que sobre ella ejercen los tejidos blandos pélvicos, pasando
entonces el diámetro suboccipitobregmático a ser el mayor en ese momento. Usted debe
saber que el diámetro SOB (suboccipitobregmático) mide aproximadamente:

1. 1. 12 cm.
2. 2. 9.5 cm.
3. 3. 10.5 cm.
4. 4. 8 cm.
Gráfico de respuestas
Comentario

Pregunta +++ importante para el nacional, seguro cuando leiste la pregunta te pareció larguísima
para lo que te cuestionaron finalmente, bueno pues cuando aprezcan este tipo de preguntas
primero debes leer las respuestas y la parte final del caso clínico, para ahorrar tiempo y evitas el
choro del interno etc. El diámetro suboccipito- bregmático es la distancia desde la región
suboccipital hasta el bregma o fontanela menor. El tamaño en los fetos a término es de
aproximadamente 9.5 cm. Es el diámetro anteroposterior presentado cuando la cabeza fetal está
flexionada. Es más pequeño que otros diámetros lo que justifica que la flexión de la cabeza
favorezca la buena evolución del expulsivo fetal. Recuerde que en los casos extremos de deflexión
de la cabeza, como son la frente y la cara mentoposterior, se contraindica la vía vaginal.(R2)

250. Paciente de 31 años, de 36 semanas de gestación ingresa por bolsa rota. A las 4
horas del ingreso comienza con dinámica regular alcanzando los 3 cm de dilatación y
cérvix borrado. 6 horas más tarde alcanza dilatación completa y de pronto comienza con
sangrado escaso y oscuro, abdomen doloroso a la palpación y bradicardia fetal. Decide
finalizar el parto, cuyas condiciones obstétricas son: cefálica, occipitoanterior, cabeza
no encajada. ¿Cuál es la actitud más correcta?:

1. 1. Ventosa obstétrica por la altura de la presentación.


2. 2. Cesárea urgente.
3. 3. Maniobra de Kristeller.
4. 4. Espátulas de Thierry.
Gráfico de respuestas
Comentario

 
 
 
 
En este caso clínico tenemos una gestante a término en trabajo de parto que comienza con un
abruptio cuando alcanza dilatación completa. La clave es que la presentación no alcanza el III
plano (no está encajada) por eso la cesárea es obligada.(R2)

251. Mujer de 92 años, hipertensa, muy


delgada, que acude a urgencias por distensión abdominal y vómitos alimenticios. A la
exploración abdominal no presenta cicatrices, ni hernias palpables, el abdomen está
distendido y timpánico, y no muestra signos de irritación peritoneal. El tacto rectal es
negativo. Sólo refiere un intenso dolor de inicio reciente en la cara interna del muslo,
aunque no cuenta antecedente traumático. En la QS se detecta una elevación discreta de
la creatinina y amilasa séricas acompañado de hipopotasemia. Se le realiza la siguiente
radiografía de abdomen. Indique los hallazgos radiológicos que más se adecuen al
resultado de la misma:

1. 1. Obstrucción de intestino grueso.


2. 2. Edema de asas intestinales.
3. 3. Obstrucción de intestino delgado con cambio de calibre a nivel inguinal.
4. 4. Hidronefrosis.
Gráfico de respuestas
Comentario

Pregunta complicada al tener que interpretar una radiografía de abdomen. Lo que se observa en la
radiografía es una imagen en pilas de moneda, compatible con distensión del intestino delgado
(por una probable obstrucción). Se diferencia de la obstrucción del intestino grueso porque si te
fijas, las válvulas coniventes atraviesan todo el diámetro de la luz GI (cosa que no ocurre en el
intestino grueso dilatado, donde las haustras no atraviesan todo el diámetro intestinal).(R3)

252. ¿Qué medidas terapéuticas iniciaría en esta paciente pese a no tener un diagnóstico
de certeza?

 
 
 
 
1. 1. Sonda nasogástrica (SNG), hidratación por vía periférica.
2. 2. Antibióticos de amplio espectro.
3. 3. Tratamiento antitrombótico.
4. 4. Gastrografín por SNG y repetir la RX de abdomen a las 24 horas.
Gráfico de respuestas
Comentario
Las medidas terapeúticas corresponde al tratamiento que se realiza a cualquier paciente que
acude a Urgencias con un cuadro de probable obstrucción intestinal: colocación de SNG en
aspiración intermitente (para descongestionar el tubo digestivo), rehidratación y reposición
electrolítica (ante cualquier obstrucción intestinal se produce un edema de pared intestinal y
creación de tercer espacio, con riesgo a deshidratación, pérdida de electrolitos y
hemoconcentración) y dieta absoluta. Solo en el caso de sospechar sepsis asociada se
administrará antibióticos (metronidazol y cefotaxima).(R1)

253. Tras hidratar a la paciente, se le realiza un escáner de abdomen donde se confirma


el atrapamiento de un asa en la pelvis. ¿Qué tratamiento indicaría para la patología que
presenta la paciente?

Continuaría con SNG y sueros hasta que la paciente tolerara y comenzara con tránsito
1. 1.
intestinal.
2. 2. Cirugía urgente.
3. 3. Antibioterapia intravenosa.
4. 4. Laparoscopia diagnóstica y actuación según los hallazgos.
Gráfico de respuestas
Comentario
Existen múltiples indicaciones de cirugía urgente en pacientes con obstrucción intestinal: hernias
estranguladas o incarceradas, sospecha de estrangulación intestinal por hernia interna, peritonitis,
neumoperitoneo, obstrucción de un asa cerrada, vólvulo de localización diferente a sigma... Lo que
está claro en este caso es la necesidad de cirugía (tras el diagnostico realizado con el TC
abdominal la única opción terapeútica es cirugía urgente).(R2)

254. La presbicia en el paciente miope de 3 dioptrías y 65 años de edad se corrige:

1. 1. Con productos vasodilatadores.


2. 2. Con ejercicios especiales.
3. 3. Con cristales esféricos convergentes.
4. 4. No es necesaria la corrección.
Gráfico de respuestas
Comentario
La presbicia se caracteriza por una incapacidad para enfocar los objetos cercanos por pérdida de
la capacidad de acomodación del cristalino, perdiéndose convergencia, constituyendo en cierto
modo una forma de ?hipermetropización?. Por todo ello la corrección se debe realizar como si
fuera un hipermétrope, es decir, aportándole al ojo ese poder convergente que ha perdido, por eso
se usan lentes esféricas convergentes o positivas.(R4)

255. Mujer de 75 años de edad que acude a urgencias por mal estado general. Refiere
además diarrea acuosa de 4 días de evolución con dificultad para ingesta oral. En la
exploración física, la paciente presenta signos de depleción de volumen y la tensión
arterial es de 90/55 mmHg. La analítica demuestra Cr 2,5 mg/dl, urea 150 mg/dl, Na 139
mmol/l, K 3,6 mmol/l. En orina, Na 14 mmol/l. ¿Cuál es el diagnóstico más probable?:

 
 
 
 
1. 1. NTA.
2. 2. NTIA.
3. 3. FRA funcional.
4. 4. Vasculitis.
Gráfico de respuestas
Comentario
Caso clínico típico de FRA funcional, en paciente deshidratada, hipotensa y con Na o < 20.(R3)

256. Pre escolar de 3 años con cuadro catarral, fiebre de 39 ºC y exantema maculoso que
se inició en la cara y se ha generalizado, y que afecta a palmas y plantas. En la exploración
se observa, además, un exantema rojo en mucosa bucal, facies abotargada, congestión
nasal mucopurulenta, párpados hinchados y ojos rojos con lagrimeo intenso. ¿Con cuál
de las siguientes complicaciones NO se ha relacionado esta enfermedad infecciosa?:

1. 1. Orquitis.
2. 2. Laringitis estenosante.
3. 3. Mastoiditis.
4. 4. Panencefalitis esclerosante subaguda.
Gráfico de respuestas
Comentario

El cuadro clínico que nos describen corresponde a un sarampión. Son datos especialmente
sugestivos la fiebre alta y el exantema con afectación palmoplantar. También son típicos el
enantema faríngeo, la congestión nasal y la conjuntivitis bilateral. Por ello, la respuesta correcta es
la 1, ya que esa complicación no es típica del sarampión, sino de la parotiditis.(R1)

257. ¿Qué actitud tomaría ante un enfermo ingresado por presentar un infarto de
miocardio que presenta extrasístoles ventriculares esporádicas, sin deterioro
hemodinámico?

1. 1. Lidocaína profiláctica (25 mg).


2. 2. Lidocaína en dosis terapéuticas (50 mg).
3. 3. Gluconato cálcico en dosis bajas.
No se da tratamiento específico, los betabloqueantes son útiles de por sí para suprimir la
4. 4.
actividad ventricular ectópica.
Gráfico de respuestas
Comentario

Las extrasístoles frecuentes son prácticamente la norma en el infarto agudo de miocardio y no


precisan tratamiento antiarrítmico específico, como la lidocaína. El tratamiento es la monitorización
y el uso de betabloqueantes, que el paciente ya debería recibir por presentar un IAM. En caso de
otras arritmias que pueden ocurrir en la fase aguda del infarto, la de mayor repercusión es la
fibrilación ventricular primaria, que es la causa más frecuente de mortalidad antes de llegar al
hospital. La fibrilación ventricular secundaria (que es la que aparece pasadas las primeras 48- 72
horas) es muy grave y, a diferencia de la primaria, afecta gravemente al pronóstico a largo plazo
del paciente. Las taquicardias ventriculares monomórficas sostenidas no suelen ser muy frecuentes
en la fase aguda del IAM. El tratamiento es similar al de las TVMS fuera de la fase aguda del IAM.
Si el paciente está hemodinámicamente estable, se puede hacer un intento de cardioversión
farmacológica, y en ese caso suele emplearse la lidocaína o la amiodarona. Si no cede, se
realizaría cardioversión eléctrica.(R4)

 
 
 
 
258. ¿Cuál es el tratamiento de elección de un paciente con LES que tenía un sedimento
de orina y una creatinina normales y que quince días después presenta creatinina de 3,5
mg/dl, hematuria, proteinuria y cilindros hemáticos en el sedimento de orina?:

1. 1. Biopsia renal y actuar según resultado.


2. 2. Prednisona 1-2 mg/Kg/día en dos o tres dosis.
3. 3. Ciclofosfamida 1g/Kg en bolo i.v.
4. 4. AINEs.
Gráfico de respuestas
Comentario
En un paciente con LES en el que se observa deterioro de la función renal y actividad inflamatoria
en el sedimento urinario debemos iniciar inmediatamente el tratamiento con corticoides en dosis
altas sin esperar al resultado de la biopsia. La espera de varios días al resultado de la misma
puede hacer que la función renal se deteriore aún más, dificultando su control posterior. En esta
pregunta puedes tener duda con la respuesta 3 pero debes descartarla por dos motivos: no
tenemos confirmación de que se trate de una proliferativa difusa para añadir a la prednisona
ciclofosamida y la dosis que te plantean es incorrecta, siendo la que utilizamos mucho menor (750
mg/m2).(R2)

259. La localización más frecuente de la primoinfección del herpes genital es:

1. 1. Vulva.
2. 2. Uretra.
3. 3. Vagina.
4. 4. Cérvix uterino.
Gráfico de respuestas
Comentario

Cuando repasemos esta pregunta conviene - de paso- echarle un vistazo al diagnóstico diferencial
de las ulceraciones genitales, tema muy preguntado en el examen ENARM. Sobre el herpes genital
debemos saber los datos que nos permitan diferenciarlo de otras ulceraciones genitales; causado
por VHS II en un 80%, típicamente se presenta a los 3- 14 días del contacto como vesículas o
úlceras dolorosas agrupadas en racimo sobre una base eritematosa, junto con adenopatías
bilaterales y dolorosas (ojo a esto). En el varón suele presentarse en el surco balanoprepucial o en
el prepucio, mientras que en la mujer el lugar más frecuente de la primuinfección es el cervix
uterino. Las recurrencias son menos dolorosas, siendo la tasa de recurrencias más alta cuando el
herpes genital está causado por VHS II. Llegamos al diagnóstico a través del citodiagnóstico de
Tzanck, y realizamos el tratamiento con aciclovir. Opción 4 correcta.(R4)

260. En un paciente con un síndrome del pronador podremos encontrar los siguientes
signos clínicos, con la EXCEPCION de:

1. 1. Signo de Tinel, a nivel de la muñeca, negativo.


2. 2. Atrofia de la musculatura tenar
3. 3. Dolor a nivel de la muñeca y del codo.
4. 4. Test de Phalen positivo.
Gráfico de respuestas
Comentario
El síndrome del pronador o del escritor se debe a una compresión proximal del nervio mediano.
Proximalmente Se puede comprimir este nervio en múltiples localizaciones (apófisis y ligamento de
Struthers, lacerto fibroso bicipital, inserción humeral y cubital del pronador redondo, arco fibroso del
flexor superficial de los dedos). La clínica es el dolor en la cara anterior del antebrazo y afectación

 
 
 
 
motora similar al síndrome del túnel carpiano: parestesias que afectan a la cara volar de los dedos
primer, segundo, tercero y mitad radial del cuarto, la atrofia y pérdida de función de la musculatura
tenar ocurre de forma tardía. El test de Phalen y el Tinel a nivel de la muñeca son negativos, ya
que el compromiso es más proximal. E tratamiento inicial consiste en limitación de actividades que
desencadenen la clínica AINES, los casos refractarios se tratan de forma quirúrgica.(R4)

261. En una mujer embarazada de 11 semanas que lleva tres días sangrando por
genitales, con muchas náuseas, útero mayor que su amenorrea y unos valores de beta
HCG muy elevados, ¿en cuál de las siguientes patologías debe pensarse?

1. 1. Amenaza de aborto.
2. 2. Aborto diferido.
3. 3. Mola hidatídica.
4. 4. Amenaza de aborto en un útero con miomas.
Gráfico de respuestas
Comentario

La enfermedad trofoblástica engloba una serie de entidades en las que existe una proliferación
anormal relacionada con la gestación. Entre ellas, se encuentra la mola hidatiforme (respuesta
correcta 3), cuyas manifestaciones clínicas son:

•   Útero mayor del que corresponde al tiempo de amenorrea.


•   Metrorragia.
•   Aparecen con más frecuencia que en embarazos normales manifestaciones como la
hiperémesis gravídica, preeclampsia, alteraciones tiroideas…
•   Las determinaciones semanales de hCG revelarán un mayor incremento que el que se
produce en un embarazo normal.

El diagnóstico se realiza por USG, que mostrará una imagen típica en “copos de nieve”, que
corresponde con las vesículas propias de esta entidad. No se vería saco gestacional ni presencia
de feto.(R3)

262. Una mujer de 21 años presenta una amenorrea secundaria. Los niveles plasmáticos
de gonadotropinas (LH y FSH) son inferiores a 10 UI/ml. Los niveles de prolactina y de
hormona tireoestimulante (TSH) son normales. La paciente no menstrúa tras la
administración de progestágeno, pero sí lo hace al administrar un estrógeno junto con
un progestágeno. ¿Cuál de los diagnósticos que a continuación se relacionan es el más
CORRECTO?

1. 1. Síndrome de ovario poliquístico.


2. 2. Fallo ovárico autoinmune.
3. 3. Tumor hipotalámico o hipofisario.
4. 4. Disgenesia gonadal.
Gráfico de respuestas
Comentario

Pregunta sobre el algoritmo diagnóstico de las amenorreas secundarias.

El caso clínico nos aporta todos los datos necesarios para llegar al diagnóstico etiológico de la
amenorrea secundaria que manifiesta la paciente.

 
 
 
 
La opción 1 (SOP) la descartamos al no responder al test de supresión con progestágenos. La
opcion 2 las podemos descartar, dado que, al ser de origen ovárico, tendrían las gonadotropinas
elevadas.

La respuesta 4, al ser un déficit congénito, produciría una amenorrea primaria, no secundaria. Así
pues, la respuesta correcta es la 3.(R3)

Diagnóstico de las amenorreas secundarias

263. A 5-year-old Russian kid is brought to the emergency department for having two
seizures this evening. He also presents with weakness on his right arm and leg. His
parents report that he has a delayed development in comparison to the rest of the children
they know and also that his skin gets a bluish colour when he plays. He has mild fever
but the rest of vital signs are within normal limits. What is the most likely explanation for
this case?

1. 1. Brain abscess
2. 2. Acute meningitis
3. 3. Temporal Glioma
4. 4. Arnold Chiari malformation
Gráfico de respuestas
Comentario
Brain abscess. A brain abscess is a focal intracranial infection affecting the brain parenchyma, with
the formation of a purulent collection. Fever and inflammatory signs help us rule out tumoral causes

 
 
 
 
and malformations. Meningitis is also a CNS infection that can be accompanied by convulsions but
no other focal neurological deficits.(R1)

264. La leche humana es el alimento natural y apropiado durante el primer año de vida.
¿Cuál de las siguientes aseveraciones al respecto NO es cierta?:

La lactancia natural se asocia con una menor incidencia de alergia o intolerancia a la leche
1. 1.
de vaca.
Están bien reconocidas las ventajas psicológicas de la lactancia al pecho tanto para la
2. 2.
madre como para el niño.
3. 3. No se ha documentado la transmisión de infección por VIH por la leche materna.
La leche humana contiene lactoferrina, que tiene un efecto inhibitorio en el crecimiento de
4. 4.
E. coli.
Gráfico de respuestas
Comentario

Concepto preguntado en simulacro previo. La lactancia materna ofrece múltiples ventajas, lo que
supone que su promoción sea un cometido de los profesionales sanitarios. Sin embargo, existen
una serie de situaciones en las que no se recomienda; una de estas contadas contraindicaciones a
la lactancia materna es la infección materna por VIH en paises desarrollados, ya que el virus puede
ser transmitido de la madre al niño a través de la leche. Recuerde que en los países pobres el
riesgo de mortalidad por desnutrición puede ser mucho mayor que el riesgo de contraer el SIDA
por lo que en estos casos la indicación debe ser individualizada.(R3)

265. Mujer obesa, postmenopáusica y virgen que presenta sangrado vaginal, su primera
opción diagnóstica es:

1. 1. Atrofia endometrial.
2. 2. Cáncer de cérvix.
3. 3. Cáncer de endometrio.
4. 4. Cáncer de vagina.
Gráfico de respuestas
Comentario

Las opciones 2, 3 y 4 pueden explicar el sangrado vaginal, dado que la 1 es un cambio fisiológico
tras la menopausia que no debería producir sangrado. Sin embargo, el cáncer del tracto genital
femenino más frecuente en una persona que nunca ha tenido relaciones sexuales es el cáncer de
endometrio, y más si tenemos en cuenta que es obesa, que es un factor de riesgo para padecer
este cáncer. El cáncer de cérvix es poco frecuente en mujeres que no han tenido relaciones
sexuales, dado que su causa principal es la infección por VPH, y el cáncer de vagina es muy
infrecuente en todas las mujeres.(R3)

266. Niño de 6 años que acude al Servicio de Urgencias con clínica compatible de tetania,
Trousseau positivo, calcio plasmático 6,9 mg/dl. En la exploración destaca que el
paciente presenta manchas blanquecinas en la lengua y en las uñas y un desarrollo
anómalo del esmalte dental. Respecto al cuadro que usted sospecha, señale qué NO
esperaría encontrar:

1. 1. Distrofia ungueal.
2. 2. Diabetes mellitus.
3. 3. Calcificación de la membrana timpánica.

 
 
 
 
4. 4. Enfermedad tiroidea autoinmune en el 70% de los casos.
Gráfico de respuestas
Comentario
Sospechamos un síndrome pluriglandular autoinmune tipo 1 (PGA tipo 1), que se diagnostica
generalmente en la primera década de vida, como en el caso de nuestro paciente, el cual presenta
candidiasis mucocutánea, asociada a hipoparatiroidismo, representado por los signos de
hipocalecemia con un calcio plasmático bajo. En este paciente, no se ha manifestado por el
momento, la insuficiencia suprarrenal, presente hasta en el 70% de los casos. El PGA tipo 1se
hereda con carácter autosómico recesivo a través del gen AIRE del cromosoma 21, si bien la
ausencia de este gen no lo excluye. Pueden asociarse otras enfermedades autoinmunes como la
diabetes mellitus o la enfermedad tiroidea autoinmuine, si bien en un porcentaje muy inferior al del
PGA tipo2, el cual asocia ésta última en un 70% de los casos. En cambio el PGA tipo 1 asocia
enfermedad tiroidea autoinmeune en el 10% de los casos (respuesta 5 falsa).

Por otra parte, el PGA tipo 1 puede asociar distrofia ectodérmica (de naturaleza no autoinmune),
caracterizada por hipoplasia del esmalte dental, distrofia ungueal y calcificación de la membrana
timpánica.(R4)

267. Lactante de 3 meses que presenta tos y dificultad respiratoria de 2 días de evolución,
que no cede pese al tratamiento con broncodilatadores en cámara. Durante el último día,
ha comenzado a rechazar las tomas y a presentar vómitos después de los accesos de
tos. Asocia febrícula. Ahora tiene tiraje sub e intercostal, palidez y abundantes ruidos de
secreciones con la respiración, y en la auscultación presenta hipoventilación
generalizada en ambos campos pulmonares. La saturación transcutánea de oxígeno es
del 90%. ¿Cuál será el tratamiento de elección, según su diagnóstico de sospecha?:

1. 1. Añadir a lo anterior un ciclo corto de corticoides vía oral.


Aerosoles de suero fisiológico más el resto de medidas generales, ya que el
2. 2.
broncodilatador no está siendo eficaz.
3. 3. Aerosoles de adrenalina más las medidas generales.
Aerosoles de adrenalina, medidas generales y antibioterapia vía oral por el riesgo tan alto
4. 4.
de sobreinfección de esta patología.
Gráfico de respuestas
Comentario

Esta pregunta acerca del tratamiento de la bronquiolitis es difícil. El tratamiento en las formas leves
se puede realizar en el domicilio: humedad, posición semiincorporada, fisioterapia y aspirado de
secreciones. Se puede probar la respuesta a salbutamol. El paciente de la pregunta cumple
criterios de ingreso (rechazo de tomas, saturación <93%) y debe recibir oxigeno y adrenalina en
aerosol, ya que es el fármaco mas útil en su tratamiento. Recuerde que los corticoides no han
demostrado eficacia y que los antibióticos sólo se emplean si existe sobre infección bacteriana, que
no existe en este caso. El riesgo de sobreinfección no es alto.(R3)

268. Un paciente de 62 años es visto en Urgencias por dolor lumbar de dos meses de
evolución y malestar general. Refiere cefaleas, visión borrosa, sed intensa y molestias
digestivas. A la exploración física está deshidratado, con una TA de 180/105 mmHg, a 110
Ipm. La analítica sanguínea muestra una hemoglobina de 9,5 g/dl,VSG 90, acidosis
metabólica, creatinina 2,5 mg/dl, proteínas totales 10,2 con un pico monoclonal de IgG
tipo l, y una calcemia de 15 mg/dl. El tratamiento urgente de esta condición incluye el uso
de esteroides, difosfonatos, expansión con suero salino y el uso de un diurético. ¿Cuál?

 
 
 
 
1. 1. Acetazolamida.
2. 2. Amilorida.
3. 3. Hidroclorotiazida.
4. 4. Furosemida.
Gráfico de respuestas
Comentario

El tratamiento comienza con la administración de suero salino fisiológico (2-3 l/día) junto con
furosemida tras corregir el déficit de volumen existente. Los bifosfonatos asociados reducen de
forma eficaz y rápida el calcio sérico con apenas efectos secundarios (pamidronato,
etidronato).(R4)

Actitud ante el hiperparatiroidismo primario

269. An ECG of an asymptomatic adolescent shows sinus rhythm and right bundle branch
block. On physical examination, a widely split S2 is heard, and his chest X-ray film shows
a prominent pulmonary arch. What is the most likely diagnosis?

1. 1. Tetralogy of Fallot
2. 2. Atrial septal defect
3. 3. Patent ductus arteriosus
4. 4. Ventricular septal defect
Gráfico de respuestas
Comentario

La semiología es muy importante en cardiología y puede darnos la respuesta de preguntas como


ésta sin conocer con exactitud las patologías. Desdoblamiento amplio y fijo del segundo ruido +
BRD = comunicación interauricular. Vamos a razonar ahora un poco más. El bloqueo de rama
derecha es consecuencia de la sobrecarga del ventrículo derecho al recibir más volumen
procedente de la AD procedente de la AI (es decir se produce un shunt D-I que sobrecarga las
cavidades derechas). Debido a la sobrecarga del corazón derecho, hay un hiperaflujo pulmonar y
como consecuencia el arco pulmonar prominente en la prueba de imagen. El desdoblamiento del
segundo ruido se da por el retraso del cierre de la valvula pulmonar al tener que derivar más
volumen del normal (que tambien se puede auscultar en BRD, EP y CIV) La única de las
patologías propiestas que reune todas estas características es la CIA.(R2)

 
 
 
 
270. Se identificó un pólipo de aspecto benigno de 2.5 cm de diámetro en el colon
sigmoide de una mujer de 55 años mediante el enema de bario. Aunque la escisión
intentada con la sigmoidoscopia flexible fue incompleta, se extrajeron varias porciones
de tejido adenomatoso que en el examen histológico no pareció maligno. El plan
terapéutico más apropiado sería:

1. 1. Colectomía sigmoide.
2. 2. Resección segmentaria del pólipo con escisión de un margen de 3 cm de colon normal.
3. 3. Enemas de bario periódicos para evaluar el crecimiento de los residuos del pólipo.
4. 4. Sigmoidoscopia repetida para completar la escisión del pólipo.
Gráfico de respuestas
Comentario

Existen varios factores que influyen en la probabilidad de malignización de los pólipos


adenomatosos. Uno de ellos es el tamaño, con el que existe una relación casi directa.Cuando un
pólipo supera los 2 cm, el riesgo de malignización supera el 50% y el estirpe histológico velloso.
Por tanto, si persisten restos de pólipo, sería como dejarle en el abdomen una bomba de relojería.
Está claro que habrá que marcar una opción que permita extirparlo. Entre las que nos ofrecen, la
única correcta es la 4. Dado que el estudio histológico no ha demostrado malignidad, habría que
hacer una resección lo más conservadora posible (excisión del pólipo, simplemente). En caso de
que los restos del pólipo fuesen tumorales, posteriormente podría plantearse una ampliación en
función del resultado.(R4)

271. Señale la respuesta FALSA en relación al tratamiento del cáncer de ovario:

Los tumores más sensibles a la radioterapia son el disgerminoma y el tumor de células de


1. 1.
la granulosa.
2. 2. Hay que hacer apendicectomía en todos los tumores.
3. 3. Hay que hacer omentectomía en todos los tumores.
4. 4. Los estadios Ia y Ib, grados G1 y G2 no precisan más tratamiento que la cirugía.
Gráfico de respuestas
Comentario

Esta pregunta del cancer de ovario no es muy importante y la respuesta muy lógica. En relación a
su tratamiento simplemente debe recordar que es: cirugía citorreductora + poliquimioterapia
adyuvante y dos excepciones:

1.- No se da quimioterapia si está diferenciado y limitado al ovario .

2.- Cirugía parcial conservadora si IAG1 y deseos genésicos.

La cirugía de elección es: histerectomía, doble anexectomía, omentectomía, biopsias múltiples de


peritoneales y lavado peritoneal. La apendicectomía se realiza especialmente en los tumores
mucinosos. La utilidad de la radioterapia en el ovario se reduce casi exclusivamente a la recidiva
de el disgerminoma (homólogo del seminoma testicular) y la tendencia actual es a sustituirla por
quimioterapia. Recordemos la asociación cistoadenoma mucinoso, pseudomixoma peritoneal y
mucocele apendicular.(R2)

272. El “ovario en cinta” o “en banda” es característica de:

1. 1. Pseudohermafroditismo femenino.

 
 
 
 
2. 2. Síndrome de Klinefelter.
3. 3. Síndrome de Turner.
4. 4. Síndrome de Triple X.
Gráfico de respuestas
Comentario

Los ovarios de las pacientes con síndrome de Turner se describen como "cintas" y tan sólo están
constituidos por tejido conectivo (tejido de sostén y unión de otros tejidos y partes del cuerpo). Por
ese motivo se le llama disgenesia gonadal.(R3)

273. Cuando una mujer queda embarazada, el organismo sufre una serie de
modificaciones y algunos parámetros biológicos cambian. ¿Cuál de los siguientes
cambios NO sucede?

1. 1. Aumento de la frecuencia respiratoria y ligero aumento del pH.


2. 2. Aumento de las hormonas tiroideas y de la TBG.
3. 3. Disminución del filtrado glomerular.
4. 4. Disminución de las resistencias periféricas.
Gráfico de respuestas
Comentario

Los cambios que se producen en el organismo durante la gestación han sido preguntados en
varias convocatorias. Y, entre ellos, el más preguntado ha sido el filtrado glomerular, que tienes
que saber que está AUMENTADO (respuesta 3 verdadera). Como consecuencia de ello,
disminuyen los niveles plasmáticos de urea y creatinina, ya que aumenta su aclaramiento.

Desde el punto de vista cardiorrespiratorio, destaca la disminución de las resistencias periféricas y


el aumento de la frecuencia respiratoria, lo que justifica una ligera alcalosis metabólica, por lo que
aumenta el pH.

En cuanto a la función tiroidea, recuerda que la beta- hCG tiene acción “TSH- like”, por lo que
aumenta ligeramente el tamaño de la glándula, aumentando la cifra de hormonas tiroideas y de
tiroglobulina.(R3)

 
 
 
 
274. Un varón de 46 años sin antecedentes psiquiátricos es llevado a urgencias por
presentar desde 2 horas antes un cuadro de obnubilación, desorientación, dificultades
de atención y alucinaciones visuales. Es prioritario descartar las situaciones siguientes,
EXCEPTO:

1. 1. Esquizofrenia.
2. 2. Intoxicación medicamentosa.
3. 3. Encefalopatía metabólica.
4. 4. Encefalopatía de origen infeccioso.
Gráfico de respuestas
Comentario
Una cuestión de concepto básico, pues básicamente nos están hablando de un cuadro confusional
agudo o delirium, bastante preguntada en el MIR, en el que hay que destacar que es de inico
agudo, con desorientación y bajo nivel de conciencia, al que se asocia sintomatología psicótica. La
presencia de sintomatología psicótica no es sinónimo de esquizofrenia, de hecho la existencia de
bajo nivel de conciencia nos descarta este cuadro (en la esquizofrenia puede haber una distorsión
de la realidad, pero viviendo en ella conscientes, orientados y atentos). Como el delirium tiene una
causa orgánica, su tratamiento es el etiológico, por eso es indispensable descartar esos cuadros.
Bibliografía: Manual Psiquiatría CTO- Medicina 4ªEd, pág32.(R1)

275. ¿Cuál de las siguientes formas de la artritis crónica juvenil se asocia más
frecuentemente con iritis aguda de repetición?:

1. 1. Forma sistémica.
2. 2. Oligoarticular y HLA B27 positivo.
3. 3. Poliarticular FR positivo.
4. 4. Poliarticular FR negativo.
Gráfico de respuestas
Comentario

La artritis reumatoide se pregunta mucho en el MIR, no así la artritis crónica juvenil. No te


preocupes si has fallado esta pregunta.

La uveítis anterior es una asociación clásica de la artritis crónica juvenil de comienzo oligoarticular
tardío. Esta forma suele comenzar a partir de los 10 años. Es especialmente frecuente cuando
coexiste con ANA+.(R2)

276. Un paciente de 58 años va a comenzar un tratamiento crónico con digoxina por


presentar una miocardiopatía dilatada idiopática con fibrilación auricular crónica. Una de
las siguientes situaciones NO incrementará el riesgo de que desarrolle cardiotoxicidad
secundaria al tratamiento con digoxina:

1. 1. Hipocalcemia.
2. 2. Tratamiento concomitante con verapamilo.
3. 3. Insuficiencia renal.
4. 4. Hipotiroidismo.
Gráfico de respuestas
Comentario

Pregunta de gran dificultad. La intoxicación digitálica es una complicación del tratamiento con
digoxina debido a su estrecho margen terapéutico (0,5- 2ng/ml). Generalmente la causa más

 
 
 
 
frecuente es la hipopotasemia pero existen múltiples causas. Entre ellas están también la edad
avanzada, la insuficiencia renal, la hipercalcemia, el hipotiroidismo y la administración de ciertos
fármacos (amiodarona, quinidina, verapamil, eritromicina...). El cuadro clínico de una intoxicación
digitálica se caracteriza por náuseas, vómitos, diarrea, alteraciones visuales y sobretodo por
arritmias, las más frecuentes son extrasístoles ventriculares. Otras arritmias típicas son la
taquicardia auricular con bloqueo AV variable, así como el bigeminismo y los bloqueos A- V, que
pueden producir ondas a "en cañón", que ocurre cuando la válvula tricúspide se encuentra cerrada
mientras la AD se contrae.(R1)

277. Ante un cuadro de macrohematuria, oliguria e HTA en un niño de 7 años que presentó
un impétigo 2 semanas antes, ¿qué dato analítico nos extrañaría encontrar?

1. 1. Descenso de la fracción C3 del complemento en suero.


2. 2. Excreción fraccional de sodio (EFNa) en orina superior al 1%.
3. 3. Proteinuria de 2 g/día.
4. 4. Anticuerpos antiestreptolisina 0 (ASLO) bajos.
Gráfico de respuestas
Comentario

Nos describen un síndrome nefrítico que aparece después de un impétigo, por lo que deberíamos
pensar en una GN postestreptocócica. Recuerda que, en esta entidad, desciende el filtrado
glomerular, por lo que el riñón trata de reabsorber todo el sodio que puede, por lo que la excreción
fraccional de sodio no estará aumentada, sino disminuida (inferior a 2).(R2)

278. RNPT presenta episodios de cese de la respiración que se acompaña de Sat O2 del
85% y FC de 80 lpm. Señale la respuesta FALSA:

1. 1. La etiología más frecuente de apnea idiopática es la mixta (central y obstructiva).


2. 2. Es típico que comience en las primeras 24 horas de vida.
3. 3. Está indicado el tratamiento con teofilina o cafeína.
4. 4. En ocasiones es preciso el uso de presión positiva continua nasal.
Gráfico de respuestas
Comentario

Las pausas de apnea se definen como cese de la respiración durante 10 segundos. Las pausas
prolongadas (de duración mayor de 20 segundos) pueden acarrear consecuenias hemodinámicas,
tales como bradicardia, y expresión cromática (el niño puede quedar cianótico). Recuerden para el
ENARM que las pausas de apnea idiopáticas típicas del prematuro aparecen entre el día 2 y el 7
de vida. Las que aparecen el día 1 o después del 7 siempre son secundarias a un cuadro
orgánico.(R2)

279. El sangrado por deprivación después de la administración de progesterona en una


mujer amenorreica indica:

1. 1. Producción de gonadotropinas.
2. 2. Producción endógena de estrógenos.
3. 3. Todo lo anterior.
4. 4. Ninguno de lo anterior.
Gráfico de respuestas
Comentario

 
 
 
 
Ante una amenorrea, lo primero que hay que descartar es una gestación. Posteriormente se hará
determinación de TSH y PRL: si están alteradas, se llevará acabo un tratamiento etiológico. Si son
normales, se continuará el estudio. Se da una pequeña cantidad de progesterona: si la paciente
produce con normalidad la 1a fase del ciclo, la proliferativa, pero no llega a ovular y, por tanto, no
llega a producir progesterona, al administrarle esta progesterona que le faltaba, menstruará, y se
concluye que la causa era anovulación. Si, a pesar de la progesterona, no menstrua, se continuará
el estudio con estrógenos y progesterona.(R3)

280. Paciente de 30 años de edad que


acude a la consulta de reproducción asistida tras 3 años de esterilidad. Sin antecedentes
de interés. USG transvaginal normal. Reglas irregulares. FSH y estradiol el día 4 de ciclo
en los parámetros normales para su edad. Progesterona el día 21 de ciclo con niveles
muy bajos. Se realiza una histerosalpingografía cuyo resultado se muestra en la imagen.
Seminograma con REM (recuperación de espermatozoides móviles) de 2 millones. ¿Cual
es el tratamiento más correcto para esta pareja a la vista de los resultados del estudio de
esterilidad?

1. 1. Coito dirigido.
2. 2. Fecundación in vitro.
3. 3. Inseminación artifical con semen del marido.
4. 4. Biopsia testicular.
Gráfico de respuestas
Comentario

La fecundación in vitro consiste en recoger los ovocitos mediante punción folicular (normalmente
por vía vaginal, con control ecográfico), induciendo previamente la evolución. Se ponen en
contacto los ovocitos extraídos con los espermatozoides y, una vez producida la fecundación, se
transfieren hasta tres embriones a la cavidad uterina. Las indicaciones fundamentales de esta
técnica son:

- Patología tubárica bilateral, como la obstrucción de las trompas (no es este caso, donde la
histerosalpingografía demuestra que están permeables).

- Insuficiente número de espermatozoides para realizar inseminación intrauterina.

- Fracasos repetidos con la inseminación intrauterina, en caso de hacerse.

 
 
 
 
Dado que el seminograma con REM nos muestra un número de espermatozoides inferior a 3
millones, nos decidimos por la fecundación in vitro.(R2)

281. De las siguientes técnicas quirúrgicas, indique en la que NO es esperable, de


entrada, la realización de un estoma:

1. 1. Amputación abdominoperineal.
2. 2. Bypass gástrico laparoscópico.
3. 3. Resección anterior baja con escisión mesorrectal total.
4. 4. Intervención de Hartmann.
Gráfico de respuestas
Comentario
Debemos realizar estomas en 3 situaciones fundamentalmente:de forma temporal para proteger
anastomosis de riesgo como sucede en las anastomosis colo o ileoanales (panproctocolectomía
restauradora con reservorio ileoanal) o colorrectales muy bajas (resección anterior baja con
escisión mesorrectal total). Generalmente usamos ileostomías “de descarga” que se cerrarán tras 6
semanas de la Cirugía inicial, si no ha habido complicaciones de la misma. Algunos centros
prefieren no hacerlas en la Cirugía inicial y la reservan para tratar la complicación (la dehiscencia
de la anastomosis) pero es un aspecto controvertido.en pacientes donde no se puede reestablecer
el tránsito, bien porque no tengan una buena continencia o porque exista un tumor que afecte a los
esfínteres: amputación abdominoperinealsituaciones de urgencia que impidan la realización de una
anastomosis, bien porque haya “que correr” y no se pueda emplear tiempo en hacer una
anastomosis (control de daños), o bien porque exista alto riesgo de fracaso de la misma por sepsis
local o difusa, desnutrición, terapias con corticoides… es el caso de la intervención de Hartmann
por perforaciones de colon izquierdo y sepsis que contraindique la anastomosis primaria.Las
intervenciones gástricas e intestinales rara vez requieren el uso de un estoma, al menos de
entrada.(R2)

282. Es traído a urgencias un paciente politraumatizado que presenta taquipnea,


movimiento paradójico del hemitórax derecho con la respiración, TA 95/60, hematuria, y
múltiples fracturas. ¿Cuál de las siguientes lesiones trataría en primer lugar?:

1. 1. Fractura de fémur.
2. 2. Volet costal.
3. 3. Lesión de la uretra.
4. 4. Fractura abierta de tibia.
Gráfico de respuestas
Comentario

Esta pregunta es muy importante ya que el manejo del politraumatizado es un tema muy
preguntado. Lo prioritario en el manejo de un politraumatizado, y utilizando el acrónimo ABCDE
para recordarlo, es mantener la vía aérea permeable (Airway) seguido de control de la ventilación
(Breathing). En este caso, presenta un volet costal que le provoca un distrés respirátorio, por lo que
será lo primero que tratemos, intentando mediante métodos externos, que la parrilla costal funcione
adecuadamente.(R2)

283. La infección por citomegalovirus durante la gestación puede causar todo lo


siguiente, EXCEPTO:

1. 1. Bajo peso al nacimiento.


2. 2. Hepatomegalia.

 
 
 
 
3. 3. Neumonía.
4. 4. Hidrocefalia.
Gráfico de respuestas
Comentario

Concepto básico para el ENARM. La hidrocefalia en el contexto de una infección connatal se ve,
no en la infección por CMV, sino en la infección por Toxoplasma. No se confunda, pues otras
manifestaciones son comunes a estas dos infecciones (en ambas hay antecedente de CIR,
hepatoesplenomegalia, coriorretinitis y calcificaciones intracraneales - dispersas en la
toxoplasmosis y periventriculares si el agente es CMV- ).(R4)

284. Una mujer de 43 años, con cirrosis hepática VHC +, presenta ascitis no a tensión,
siendo tratada con dieta hiposódica. A las dos semanas de tratamiento, no se objetiva
disminución de peso, y en orina la concentración de sodio es de 50 mEq en 24 h. ¿Cuál
sería su actitud?:

1. 1. Iniciar tratamiento con espironolactona 100 mg/día.


2. 2. Educación sanitaria respecto a la dieta.
3. 3. Iniciar tratamiento con espironolactona 100 mg/día y furosemida 40 mg/día.
4. 4. Restricción hídrica.
Gráfico de respuestas
Comentario

Uno de los temas importantes de la endocrinología, y más preguntados en el MIR es el síndrome


de Cushing, sobre todo el diagnóstico diferencial. Aquí se presenta un caso clínico en el que el
diagnóstico sindrómico ya está confirmado. Sólo queda interpretar el resto de las pruebas, para
llegar al diagnóstico etiológico.

Lo primero que debe observarse son las cifras de ACTH, para saber si es un Cushing ACTH
dependiente o independiente. En caso de que sus cifras estén elevadas o inapropiadamente
normales (dado que los pacientes presentan cortisol elevado y por feed- back negativo deberían
suprimir a la ACTH), se tratará de un Cushing ACTH dependiente.

La adecuada supresión con el test de 8 mg de DXT aparece en los microadenomas hipofisarios


productores de ACTH y en algunos casos de tumores carcinoides (Cushing ectópico). La respuesta
de la ACTH al CRH es normal, luego indica que el eje hipotálamo- hipofisario- suprarrenal está
intacto (típico de los tumores hipofisarios).

La RMN es normal, pero esto no descarta la existencia de un microadenoma, dado que por su
pequeño tamaño pueden no ser visualizados. Ya que su frecuencia es mucho mayor que la de los
carcinoides y en las pruebas diagnósticas presenta las respuestas típicas de un microadenoma, se
debe descartar su existencia con la realización de un cateterismo de los senos petrosos inferiores
(seguiría siendo la causa más frecuente, pese a la negatividad de la RMN). En caso de no existir
diferencias en el nivel de ACTH entre ambos senos, estaría indicada la realización de una TAC
toracoabdominal.(R2)

285. Cardiogenic shock apprears as a result of a failure of the pump action of the heart.
Consequently, acute hypoperfusion and hypoxia of the tissues and organs occur, in spite
of an adequate intravascular volume. Which of the following options has not been shown
to improve survival in patients with cardiogenic shock?

 
 
 
 
1. 1. Administration of oxygen and artificial respiration.
Anti-inflammatory drugs in order to treat systemic inflammatory response syndrome
2. 2.
(SIRS).
When the underlying etiology is cardiac ischemia, perform emergent percutaneous
3. 3.
angioplasty, thrombolytic therapy or revascularization surgical procedure.
Surgical repair when the underlying etiology is mitral insufficiency, interventricular
4. 4.
septum rupture or ventricular free wall rupture.
Gráfico de respuestas
Comentario

En realidad, el nivel de evidencia científica con el que se cuenta para la mayoría de estas
circunstancias, no es demasiado elevado. No obstante el sentido común hace muy razonable que
si existe una situación de compromiso vital como el shock cardiogénico, las medidas que vayan
encaminadas a la corrección de la causa subyacente (revascularización si se trata de un infarto,
cirugía si es una complicación mecánica...), puedan afectar positivamente al pronóstico del
paciente. Las medidas de soporte vital avanzado como la ventilación mecánica y oxigenoterapia o
la contrapulsación aórtica, también por lógica, deben mejorar el devenir del paciente.

Sin embargo, la respuesta inflamatoria sistémica que acontece en la situación de shock,


caracterizada por la liberación de múltiples citoquinas y otras sustancias, si bien puede contribuir a
la disfunción sistólica progresiva no ha de ser combatida con fármacos específicos antiinflamatorios
pues, en ningún caso, se ha demostrado su beneficio y sí incluso se ha detectado un incremento
de ciertos riesgos asociados a su empleo (por ejemplo, los esteroides sistémicos pueden
relacionarse con un mayor riesgo de rotura cardíaca en el infarto agudo de miocardio, los
antiinfamatorios no esteroideos también, ambos facilitan el deterioro renal y la retención
hidrosalina, fomentan la actividad protrombótica plaquetaria que dificulta la normalización de la
función endotelial en la microcirculación, etc.).(R2)

286. A 6-day-old preterm newborn at 28 weeks gestation is brought to the emergency


department presenting with vomiting and breast refusal. The mother has noted blood in
his stools. Bowel sounds are diminished. Abdominal X ray shows gas diffusely
distributed on the intestinal wall. What is the most likely diagnosis?

1. 1. Meconial ileum
2. 2. Pyloric atresia
3. 3. Necrotizing enterocolitis
4. 4. Biliary atresia
Gráfico de respuestas
Comentario
Necrotizing Enterocolitis. Necrotizing enterocolitis (NET) is a life-threatening condition in preterm
children. It must be suspected in children with marked abdominal distension, loss of appetite and
bloody stools. The radiological finding described is called “pneumatosis intestinalis” and it
represents intramural gas within the wall of the bowel.(R3)

287. El tumor ovárico de células germinales más frecuente es el:

1. 1. Disgerminoma.
2. 2. Carcinoma embrionario.
3. 3. Teratoma maduro quístico.
4. 4. Teratoma inmaduro.
Gráfico de respuestas
Comentario
 
 
 
 
Una pregunta relativamente sencilla, ya que se contesta directamente a partir del Manual CTO. Los
tumores ováricos germinales representan el 25% del total. Más de la mitad de ellos se producen en
mujeres jóvenes. Por lo tanto, son los tumores ováricos más frecuentes en este grupo de edad. El
síntoma más frecuente es el dolor y la distensión abdominal.

El tumor ovárico más frecuente, dentro de los de estirpe germinal, es el teratoma quístico maduro
(quiste dermoide), con gran diferencia: 90% del total. El 10% restante son malignos. Está
constituido por tejidos bien diferenciados y sólo es maligno de forma excepcional. En su
arquitectura existe un predominio de tejido ectodérmico: glándulas sebáceas, sudoríparas, pelo…
En algunas ocasiones, este tumor es productor de alfafetoproteína, al igual que muchos tumores
testiculares (excepto el seminoma).(R3)

288. Paciente de 42 años de edad. Refiere que ahora le cuesta más leer el periódico (tiene
que alejarlo). Como único antecedente oftalmológico cita que de niño era hipermétrope y
tuvo que utilizar gafas durante los primeros años de su vida. Respecto a la situación de
este paciente, señale la correcta:

1. 1. Tiene presbicia y deberá corregirse con cristales positivos.


Una miopización tan clara sugiere una catarata nuclear incipiente. Dilatar la pupila y si
2. 2.
presenta catarata, programar para facoemulsificación y LIO.
Lo más importante es medir la PIO pues tiene 40 años y una clínica muy sugestiva de
3. 3.
glaucoma crónico.
4. 4. Se trata de una degeneración macular asociada a la edad.
Gráfico de respuestas
Comentario
Este caso resume muy bien la presentación clínica de la presbicia o vista cansada. Este problema
se presenta habitualmente alrededor de los 40- 50 años, si bien el momento de aparición puede
variar en función de la edad del paciente (algo antes si el paciente es hipermétrope y un poco
después si es miope). La patogenia no se conoce bien, aunque está claro que se debe a la pérdida
de competencia del mecanismo de la acomodación. Al no poder generar la acomodación
necesaria, el sujeto se ve obligado a alejar los objetos para poderlos enfocar. Aunque hoy en día
se han diseñado lentes intraoculares multifocales, que hacen compatible una buena visión cercana
y lejana, la presbicia suele tratarse con cristales convergentes (positivos). Por todo ello la
respuesta correcta es la 1.(R1)

289. El síndrome de Heerfordt-Waldenström se caracteriza por todo lo siguiente,


EXCEPTO:

1. 1. Ser una forma aguda de presentación de la sarcoidosis.


2. 2. Cursar con fiebre.
3. 3. Aparición de uveítis anterior.
4. 4. Presentar lesiones nodulares, dolorosas, en cara anterior de miembros inferiores.
Gráfico de respuestas
Comentario
El síndrome de Heerfordt- Waldenstrom o fiebre uveoparotídea es una de las dos formas de
presentación de la sarcoidosis aguda que se manifiesta con fiebre, aumento del tamano de las
parótidas, uveítis anterior y parálisis del nervio facial.(R4)

290. Hombre de 56 años con cirrosis por VHC que acude a su consulta por aumento de
perímetro abdominal y edemas de miembros inferiores. Los laboratorios reportan: Hb
11.2 g/dl, VCM 80.5 fl, leucocitos 10,000/mm3 con 50% neutrófilos, plaquetas 68,000/mm3,
glucosa 87 mg/dl, urea 25 mg/dl, creatinina 0.3 mg/dl, albúmina 3.0 g/dl, AST 276 U/L, ALT

 
 
 
 
100 U/L, GGT 128 U/L, bilirrubina total 1.8 mg/dl, fosfatasa alcalina 98 U/L, LDH 287 U/L,
sodio 136, potasio 4.8. Se realiza un ultrasonido abdominal en la que se observa un
hígado heterogéneo e irregular con una lesión nodular de 8.5 cm y líquido libre peritoneal.
Se realiza un TC donde se observan los mismos hallazgos siendo la lesión de 9 cm con
realce en fase arterial y lavado precoz en fase venosa. ¿Cuál sería la actitud más
adecuada en este paciente?

1. 1. Iniciar tratamiento antiviral por Peg-IFN y rivabirina.


2. 2. Trasplante hepático.
3. 3. Quimioembolización.
4. 4. Sorafenib.
Gráfico de respuestas
Comentario
El paciente siendo cirrótico y teniendo un nódulo que en el TC tiene ese comportamiento, es
diagnosticado de hepatocarcinoma. Se encuentra en un estadio B en el que tanto el trasplante
como la cirugía resectiva no están indicadas. Lo más indicado sería el intentar disminuir la masa
tumoral con quimioembolización. El sorafenib se reserva para pacientes con enfermedad
extrahepática o cuando existe invasión portal ya que aumenta mínimamente la supervivencia.(R3)

291. Un varón de 35 años acude a su consulta para "hacerse un chequeo". No tiene


antecedentes de interés. Usted le mide la PA, y resulta de 135/100 mm. de Hg. Señale la
respuesta correcta:

Se le debe diagnosticar de HTA sistólica aislada, y se instaurará tratamiento con


1. 1.
propranolol.
No se puede diagnosticar de HTA con una medida aislada, pues son necesarias al menos
2. 2.
15 medidas a lo largo de un año según la OMS.
3. 3. Puede tratarse de una "pseudohipertensión" por tener una arteria braquial rígida.
Es una HTA maligna, pues la P.A. diastólica supera en más de 10 mmHg el límite superior
4. 4.
de la normalidad.
Gráfico de respuestas
Comentario
Es importante que no falles esta pregunta sobre HTA porque es sencilla y sobre todo porque ya ha
caído una pregunta parecida en más de una ocasión en el MIR. Recuerda que para daignosticar a
un paciente de HTA, éste debe mostrar repetidamente cifras de presión arterial superiores a las
correspondientes a su edad y sexo, tomadas en varias ocasiones durante DOS O TRES
SEMANAS (opción 3 falsa). Además, en el diagnóstico de HTA debe descartarse la posibilidad de
que se trate de una ?pseudohipertensión?, que consiste en una PA elevada como consecuencia de
una arteria braquial rígida, y que se sospecha por el signo de Osler (palpación de la arteria radial
después de la desaparición del pulso). En relación con la opción 5, recuerda que la HTA maligna
se define por la presencia de edema de papila y no por cifras tensionales altas.(R3)

292. En el pseudohermafroditismo femenino, el cariotipo es:

1. 1. 45,X
2. 2. 47,XYY
3. 3. 46,XX
4. 4. 47,XXY
Gráfico de respuestas
Comentario

 
 
 
 
Tema poco importante para el ENARM. Con entender el concepto de pseudohermafroditismo es
suficiente para contestar bien a la pregunta.(R3)

293. El quiste que se ubica en las paredes de la vagina, se llama:

1. 1. Quistes de Gartner.
2. 2. Quiste endometriósico.
3. 3. Quiste de Naboth.
4. 4. Quiste seroso.
Gráfico de respuestas
Comentario

Los quistes de inclusión vaginal son los más comunes y se pueden formar como resultado de una
lesión a las paredes vaginales durante el proceso del parto o después de una cirugía.

Los quistes del conducto de Gartner se desarrollan en las paredes laterales de la vagina. Este
conducto está presente en la etapa fetal, pero casi siempre desaparece después del parto. Si
quedan partes del conducto, pueden acumular líquido y convertirse en un quiste de la pared
vaginal más tarde en la vida.

El absceso o quiste de Bartolino es la acumulación de líquido o pus que forma una protuberancia
en una de las glándulas que se encuentran a cada lado de la abertura vaginal.

El quiste de Naboth es un tumor lleno de moco en la superficie del cuello uterino.(R1)

294. ¿Cuál de los siguientes aspectos NO corresponde al síndrome de Bruton?:

1. 1. Agammaglobulinemia ligada al cromosoma X.


2. 2. Ausencia de células plasmáticas en la mucosa intestinal.
3. 3. Afecta ambos sexos.
4. 4. Inmunidad celular normal.
Gráfico de respuestas
Comentario
Pregunta sencilla que no debes fallar, ya que esta enfermedad es de las inmunodeficiencias que
más ha aparecido en el MIR, y este tema guarda mucho peso dentro del bloque de inmunología. La
hipogammaglobulinemia congénita ligada al cromosoma X (síndrome de Bruton) es una deficiencia
de anticuerpos o de células B. Suele debutar entre los 6 meses y el año. Los niños afectados
muestran niveles menores de 100 mg/dL de IgG y no se detectan IgM o IgA: no hay linfopenia y los
linfocitos sólo están discretamente disminuidos en los órganos linfoides. Lo que no hay en ninguna
parte son células plasmáticas ni linfocitos B, lo que constituye el dato característico de esta
inmunodeficiencia congénita. La inmunidad celular está respetada. Las infecciones repetidas se
deben a gérmenes grampositivos, además estos niños tienen mayor frecuencia de artritis
reumatoide y de tumores malignos del tejido linfoide. Como hemos dicho posee una herencia
ligada al cromosoma X, lo que explica que esta enfermedad tenga lugar sobre todo en hombres,
siendo extremadamente rara en mujeres (en el MIR SIEMPRE será un niño, nunca una niña, que
no posea linfocitos B).(R3)

295. La nefrosis lipoídica se caracteriza por todos los siguientes hechos, con una
excepción, señálela:

1. 1. Edema.

 
 
 
 
2. 2. Proteinuria.
3. 3. Hipertensión arterial severa.
4. 4. Hipoalbunimenia.
Gráfico de respuestas
Comentario

La nefrosis lipoídica o enfermedad de cambios mínimos se presenta en el 80% de los casos como
síndrome nefrótico con sedimento urinario benigno. El resto presenta proteinuria no nefrótica. Es la
causa más frecuente de síndrome nefrótico durante la infancia. La proteinuria suele ser altamente
selectiva (predominio de albumina con la correspondiente hipoalbuminemia), mientras que en el
adulto la selectividad es menos predecible. No suele asociar insuficiencia renal y en el 20-30% de
los casos presenta microhematuria en el sedimento urinario. Como parte del síndrome nefrótico
asocia dislpemia y mayor supceptibilidad a infecciones. El complemento es normal. Respuesta 3
incorrecta.(R3)

296. A 30-year-old woman from Cleveland city (Ohio) presents with malaise, asthenia and
skin lesions on her left forearm and hand. She denies any chills, nausea or vomiting. Vital
signs are within normal limits, except for her body temperature, which is 38,2ºC. Her skin
lesions are mostly papulopustular, purplish and well demarcated. Some microabscesses
are seen surrounding the purple rash. Which of the following is the best initial test in this
patient?

1. 1. Wet preparation of purulent material from skin lesions.


2. 2. Skin biopsy.
3. 3. Bacterial and viral cultures of purulent material from skin lesions.
4. 4. Bacterial and fungal cultures of purulent material from skin lesions.
Gráfico de respuestas
Comentario
Wet preparation of purulent material from skin lesions. This patient is presenting with signs and
symptoms suggestive of Blastomycosis (the fact that he is from Cleveland is a very valuable hint
too... remember that disease is endemic in the south-central and north-central US). The yeast form
of the causative agent (Blastomyces spp) can be seen in wet preparations of purulent material
obtained from the skin lesions. TREATMENT: Itraconazole, fluconazole, or amphotericin B.(R1)

 
 
 
 

297. Señale cuál de los siguientes


hallazgos no está presente en la biopsia intestinal (ver la imagen adjunta a la pregunta):

1. 1. Infiltrado inflamatorio de neutrófilos.


2. 2. Hiperplasia de criptas.
3. 3. Atrofia vellositaria.
4. 4. Daño de enterocitos.
Gráfico de respuestas
Comentario

El infiltrado inflamatorio presente en la biopsia de la enfermedad celíaca es de linfocitos T.

Por otro lado, el orden de alteraciones encontradas en la celiaquia son: infiltrado linfocitario,
hiperplasia de criptas y atrofia vellositaria que descriven los grados de MARSH.(R1)

298. Indique cuál de los siguientes cereales está exento de gluten:

1. 1. Triticale.
2. 2. Trigo.
3. 3. Maiz.
4. 4. Cebada.
Gráfico de respuestas
Comentario

Concepto importante sobre la dieta de los pacientes con enfermedad celiaca. El mijo o maíz no
contiene gluten. El trigo, la cebada, el centeno y el triticale sí.

Para recordar los cereales que tiene gluten y los que no hay una regla nemotécnica:

Cereales con gluten: TACC : trigo, avena, cenceno y cebada.

Cereales sin gluten: Tio SAMM : tapioca, soya, arroz, maiz, mijo.(R3)

 
 
 
 
299. ¿A los cuantos meses de edad se recomienda colocar la vacuna contra la hepatitis
A?

1. 1. 12 meses.
2. 2. 6 meses.
3. 3. 4 meses.
4. 4. 2 meses.
Gráfico de respuestas
Comentario

Pregunta fácil sobre los esquemas de vacunación. La aplicación de la vacuna contra hepatitis A se
realizará a partir de los 12 meses y un refuerzo a los 6 meses después. Respuesta 1 correcta. De
todos modos no es una vacuna de obligada aplicación.(R1)

300. La manifestación ocular más frecuente en la artritis reumatoide es:

1. 1. Queratoconjuntivitis seca.
2. 2. Uveitis anterior recidivante.
3. 3. Escleromalacia perforante.
4. 4. Vasculitis retiniana.
Gráfico de respuestas
Comentario
Recuerda que la manifestación ocular más frecuente en la AR es la queratoconjuntivitis seca (la
presentan el 20% de los pacientes con AR que tienen un síndrome de Sjogren secundario). La
escleritis es sin embargo la manifestación más característica. Puede afectar a la parte más
superficial de la esclera (epiescleritis) o bien las capas más profundas, pudiendo en este caso
perforar la esclera e incluso permitir la protrusión a su traves de la coroides (escleromalacia
perforante). La uveitis no es más frecuente que en la población general por lo que no se considera
una manifestación propia de la enfermedad.(R1)

301. Señale lo FALSO en cuanto a la fisiología cardíaca:

En la fase 0 del potencial de acción tiene lugar una entrada rápida de sodio en la célula
1. 1.
miocárdica.
2. 2. La postcarga equivale a la tensión parietal del ventrículo al comienzo de su contracción.
3. 3. La válvula pulmonar se cierra antes que la aórtica.
4. 4. En situaciones de acidosis, la contractilidad miocárdica se encuentra disminuida.
Gráfico de respuestas
Comentario

El tema de fisiología cardíaca es muy importante de cara al examen.

Tanto en el 1R como en el 2R, las válvulas izquierdas se cierran antes que las derechas, es decir,
la mitral se cierra antes que la tricúspide y la aórtica se cierra antes que la pulmonar (respuesta
falsa- 3).

El resto de opciones son correctas.(R3)

302. Señale lo CORRECTO con relación al diagnostico de talla corta:

 
 
 
 
1. 1. En la talla corta familiar las proporciones corporales son normales.
2. 2. La velocidad de crecimiento en el retraso constitucional es paralelo a la edad ósea.
La edad ósea muy retrasada y las proporciones armónicas corresponden a las displasias
3. 3.
esqueléticas.
4. 4. Ninguna de las anteriores.
Gráfico de respuestas
Comentario

Esta es una pregunta fácil, pese a lo complejas que parecen las opciones. En la talla corta familiar,
el crecimiento suele ser armónico.(R1)

303. Paciente de 29 años que presenta dolor lumbar con rigidez matutina desde hace 6
meses. En la exploración presenta un test de Schöber de 6 cm y la radiología muestra
sacroileítis bilateral grado II. El tratamiento adecuado comprende:

1. 1. Fisioterapia más corticoides.


2. 2. Fisioterapia más AINEs.
3. 3. AINEs y sales de oro.
4. 4. AINEs y D-penicilamina.
Gráfico de respuestas
Comentario
Para el diagnóstico de espondilitis anquilosante se utilizan los criterios modificados de Nueva York.
Que comprende hallazgos radiológicos y clínicos. CRITERIO RADIOLOGICO. - Presencia de
sacroileitis radiológica (unilateral de grado III ó IV, o bilateral de grado II) CRITERIOS CLINICOS: -
Dolor lumbar de características inflamatorias (insidioso, empeora con el reposo, rigidez). -
Limitación de la movilidad de la columna lumbar en el plano sagital y frontal (test de Schöber). -
Disminución de la expansión de la expansión torácica en relación con los valores normales para el
sexo y la edad del paciente. El diagnóstico se establece cuando al criterio radiológico (ineludible)
se une al menos uno de los criterios clínicos y en este caso el diagnóstico se puede establecer. La
única medida farmacológica eficaz en el tratamiento de la enfermedad es la administración de
AINEs. Ningún tratamiento de fondo es capaz de evitar o retrasar la evolución hacia la anquilosis.
La fisioterapia, especialmente la natación son muy útiles para mantener la funcionalidad del
paciente.(R2)

304. Hombre de 30 años, tratado con poliquimioterapia por enfermedad de Hodgkin hace
cinco años. Consulta por cansancio y debilidad en los últimos meses. En la BH presenta
leucocitos 4.5 x 109/L, hemoglobina 7 g/dL, VCM 105 fL, plaquetas 80 x 109/L. El estudio
de médula ósea presenta hipercelularidad con 12% de blastos. ¿Cuál es el diagnóstico
del paciente?:

1. 1. Síndrome mielodisplásico primario.


2. 2. Leucemia aguda mieloblástica.
3. 3. Leucemización de la enfermedad de Hodgkin.
4. 4. Anemia refractaria con exceso de blastos secundaria.
Gráfico de respuestas
Comentario
El paciente presenta pancitopenia con macrocitosis eritrocitaria, datos compatibles con
mielodisplasia y leucemia aguda. El porcentaje de blastos medular (de 5% a 19%) es propio de
mielodisplasia, mientras 20% o superior es criterio de leucemia aguda. Dado el antecedente de QT
y el tiempo pasado desde dicho tratamiento, y también la edad del paciente, es más lógica una
mielodisplasia secundaria que primaria. No existe la leucemización de la enfermedad de
Hodgkin.(R4)

 
 
 
 
305. Un paciente es diagnosticado de miocardiopatía restrictiva. Tras el estudio
diagnóstico se llega a averiguar la causa de la misma. ¿Cuál de las siguientes tiene mayor
probabilidad de ser la causa de la miocardiopatía de este paciente?:

1. 1. Amiloidosis.
2. 2. Hemocromatosis.
3. 3. Fibrosis endomiocárdica.
4. 4. Enfermedad endomiocárdica de Loeffler.
Gráfico de respuestas
Comentario
La miocardiopatía restrictiva es un tema poco preguntado en el MIR. La amiloidosis, la
hemocromatosis, la fibrosis endomiocárdica y la enfermedad endomiocárdica de Loeffler son
posibles causas de la Miocardiopatía restrictiva. La sarcoidosis, sin embargo, cuando causa una
miocardiopatía suele ser la dilatada. De éstas, debes recordar sobre todo la amiloidosis, ya que es
la causa más frecuente de miocardiopatía restrictiva en nuestro medio (opción 2 correcta), y la
hemocromatosis, ya que este dato entró en el MIR 2004- 2005.(R1)

306. ¿Cuál es la localización más frecuente de los aneurismas arterioscleróticos?:

1. 1. Aorta torácica.
2. 2. Aorta abdominal distal.
3. 3. Arteria femoral.
4. 4. Arteria poplítea.
Gráfico de respuestas
Comentario
En relación con la etiología de los aneurismas, es importante que tengas claro que la localización
más frecuente de los aneurismas ateroscleróticos es la aorta abdominal infrarrenal, seguida de la
aorta torácica descendente y la arteria poplítea. Así, debes tener siempre presente que todos los
aneurismas de la aorta abdominal deben considerarse arterioescleróticos mientras no se
demuestre lo contrario. Respecto a los aneurismas de aorta torácica, la arterioesclerosis también
es la etiología más frecuente en los localizados en arco aórtico y aorta torácica descendente pero
en los de aorta torácica ascendente deben considerarse otras etiologías como la degeneración
quística de la media y la sífilis.(R2)

307. ¿Cuál de las siguientes hormonas o derivados de las mismas puede recibir una
gestante?:

1. 1. Estrógenos.
2. 2. Progestágenos.
3. 3. Progesterona natural micronizada.
4. 4. Danazol.
Gráfico de respuestas
Comentario

Esta es una pregunta bastante difícil que no debe preocupar si se ha fallado.

Lo más importante que debemos recordar de la progesterona durante el embarazo es que el


principal precursor de la progesterona es el colesterol materno que se utiliza para la
esteroidogénesis fetal así que aunque niveles adecuados de progesterona son necesarios para el

 
 
 
 
bienestar fetal, no es un buen marcador del mismo ya que ante estados como la anencefalia o la
muerte fetal los niveles pueden permanecer altos durante un tiempo.

La progesterona natural actúa en el organismo desarrollando el endometrio, manteniendo el


embarazo e inhibiendo la motilidad uterina. Suprime la acción de las gonadotropinas y también
tiene acción antiestrogénica. Está indicada como sustitución de la progesterona en mujeres
ooforectomizadas, como suplemento durante la fase lútea en la fertilización en ciclo espontáneos e
inducidos y para la amenaza de aborto o en abortadoras habituales por insuficiencia lútea.(R3)

308. Recibe usted en Urgencias a un lactante


de 8 meses cuyos padres refieren que hace 6 horas, mientras estaba jugando, han visto
cómo ingería una pequeña pieza metálica redondeada. El paciente presenta un excelente
estado general, sin vómitos ni dificultad respiratoria. Se realiza una radiografía de tórax
y abdomen que muestra la siguiente imagen. ¿Qué actitud adoptaría usted?

1. 1. Realizaría una laparotomía y extraería el cuerpo extraño.


Administraría una solución evacuante y vigilaría su expulsión con las heces en un
2. 2.
ambiente hospitalario.
3. 3. Realizaría una esofagogastroduodenoscopia de urgencia.
Al haber progresado más allá del esófago le daría el alta, recomendando a los padres
4. 4.
vigilar su evacuación en las heces en los próximos días.
Gráfico de respuestas
Comentario

El manejo de la ingestión de cuerpos extraños en el tracto gastrointestinal en la infancia varía en


función de la sintomatología presentada, así como de las características del objeto en cuestión
(radioopaco o radiolúcido, cortante o no).

Independientemente de las características del objeto, en todos aquellos casos que presenten
sintomatología asociada se debe realizar extracción endoscópica del mismo. Asimismo si se trata

 
 
 
 
de un objeto radiolúcido se realizará su extracción endoscópica independientemente de la
localización que ocupe, al no poderse realizar un seguimiento radiológico correcto.

Finalmente en aquellos casos que se trate de un cuerpo radioopaco, la conducta variará:

- Esófago: extraer. Hay que tener en cuenta que los casos en que el objeto está alojado en el
esófago existe riesgo de aspiración, perforación y formación de fístulas.

- Estómago: extraer si es punzante, es una pila o mide > 2.5x5 cm.

- Intestino: observar. Si después de 1-2 semanas no es expulsado a través de las heces, se


realizará radiografía de abdomen y si después de 4 semanas no es expulsado se realizará
extracción.(R4)

309. El pH de la secreción vaginal orienta hacia la identificación del agente etiológico. En


qué gérmen pensaría si el pH es superior a 4,5:

1. 1. Trichomona vaginalis.
2. 2. Chlamydia.
3. 3. VHS.
4. 4. Gonococo.
Gráfico de respuestas
Comentario

Lo más importante de las infecciones ginecológicas de vagina y vulva es que sepas realizar el
diagnóstico diferencial entre la candidiasis, la tricomoniasis y la vaginosis bacteriana.

La Trichomona es un protozoo de transmisión sexual.

La leucorrea puede ser el único síntoma. Es de baja visosidad, abundante, ligeramente maloliente,
homogénea, color verde- amarillenta, espumosa y con burbujas de aire.

En la exploración, el cervix puede presentar hemorragias puntiformes con “aspecto de fresa”. Esto
hace que el cervix sangre fácilmente.

El ph vaginal es superior a 4,5 (RC- 2).

El tratamiento de elección es metronidazol por vía oral, que también seviría para la vaginosis
bacteriana (Gardnerella).(R1)

310. A 42-year-old woman with chronic C hepatitis virus infection complains of asthenia,
arthralgias, low-grade fever, abdominal pain, dyspnea at rest and paresthesias in her
upper limbs. Which of the following is the most likely diagnosis?

1. 1. Type 2 cryoglobulinemia.
2. 2. Waldeström's macroglobulinemia
3. 3. Systemic lupus erythematosus
4. 4. Chronic lymphocytic leukemia
Gráfico de respuestas
Comentario
 
 
 
 
La crioglobulinemia suele presentar la sintomatología típica de púrpura cutánea, polineuropatía y
afectación renal, sin embargo no necesariamente se tiene que dar en todos los casos. También
podemos ver afectación clínica a otros niveles, como pulmonar, cardíaca o bien abdominal como
en este caso.(R1)

311. A un paciente de 34 años se le indica cirugía de reflujo gastroesofágico por presentar


esofagitis grado II. Con respecto a la funduplicatura tipo Nissen que se le va a realizar,
una de las siguientes afirmaciones NO es correcta:

1. 1. El fundus gástrico rodea el esófago 180º.


2. 2. Si queda muy apretada, puede producir disfagia.
3. 3. Si queda muy laxa, la sintomatología puede recidivar.
4. 4. Si el paciente presentara un esófago corto, asociaríamos una gastroplastia de Collis.
Gráfico de respuestas
Comentario
La funduplicatura de Nissen rodea con el fundus toda la circunferencia del esófago (360º). Las
parciales como el Toupet (180º posterior) sólo se emplean en la actualidad, en la cirugía de la
acalasia, para mantener abierta la miotomía. Es preciso una correcta calibración mediante un tutor,
para que la funduplicatura no quede ni muy apretada, que produciría disfagia y Síndrome de “gas
bloat” (no pueden eructar ni vomitar), ni muy laxa, en la que volvería aparecer el RGE. El abordaje
laparoscópico es el de elección en la actualidad, salvo que existan contraindicaciones.(R1)

312. En relación a la circulación pulmonar, una de las siguientes afirmaciones es FALSA:

1. 1. La presión media normal de la arteria pulmonar es de 15 mmHg.


2. 2. En bipedestación, la perfusión es menor en las bases.
3. 3. La hipoxia alveolar es un importante estímulo para la vasoconstricción pulmonar.
4. 4. La resistencia vascular pulmonar aumenta si existen trombos intraluminales.
Gráfico de respuestas
Comentario

La perfusión pulmonar es mayor en las bases que en los vértices, ya que la sangre tiende a
acumularse en las zonas más declives por simple efecto gravitatorio. Por ello, la relación
ventilación- perfusión es mayor en las bases y menor en los vértices.(R2)

313. ¿Cuál de las siguientes patologías NO es causa de obstrucción de las vías


urinarias?:

1. 1. Trombosis de la vena renal.


2. 2. Carcinoma de cérvix.
3. 3. Fimosis.
4. 4. Fibrosis retroperitoneal.
Gráfico de respuestas
Comentario
El uréter retrocavo es considerado una malformación congénita en la que el trayecto lumbar del
uréter derecho en algún momento se sitúa en una posición posterior a la vena cava, siendo esto
objeto de uropatía obstructiva. El tratamiento es quirúrgico en aquellos casos en los que exista
dolor o deterioro de la función de esa unidad renal, y consiste en la exéresis del segmento
retrocavo con posterior reconstrucción mediante ureteroureterostomia (Respuesta 2 correcta). El
carcinoma de cérvix puede ocasionar atrapamiento ureteral y por tanto sería otra causa de uropatía
obstructiva (Respuesta 3 correcta). Un anillo fimótico que condicione una total obliteración del

 
 
 
 
orificio prepucial puede condicionar uropatía obstructiva infravesical y por tanto bilateral
(Respuesta 4 correcta). La fibrosis retroperitoneal puede ser de etiología no filiada (idiopática) o
bien y más común, secundaria a procesos intercurrentes como radioterapia, quimioterapia, cirugías
sobre aorta abdominal.... Esta puede condicional una compresión extrínseca de los uréteres y
uropatía obstructiva (Respuesta 5 correcta). La trombosis de la arteria renal condiciona una clínica
de hematuria y dolor en fosa renal secundaria a aumento del tamaño renal por congestión venosa.
Hay que sospechar esta posibilidad en un adulto con síndrome nefrótico o en un recién nacido con
algún factor de deshidratación. No produce uropatía obstructiva encontrándose la vía urinaria
expedita (Respuesta 1 incorrecta).(R1)

314. El tratamiento de elección del delirium tremens es:

1. 1. Vitamina B.
2. 2. Clordiazepóxido.
3. 3. Difenilhidantoína.
4. 4. Corticosteroides.
Gráfico de respuestas
Comentario

Es una pregunta directa que se contesta rápidamente, importante cara al ENARM porque es muy
preguntado. Podemos descartar de entrada las dos últimas opciones. Los corticoides (opción 4) no
se usan en el delirium tremens. Si tuviese convulsiones como complicación del delirium se usaría
diacepam no difenilhidantoína. (opción 3). La duda está entre la vitamina B y el clordiazepóxido. La
vitamina B se usa para evitar las secuelas neurológicas (opción 1) y no disminuir la ansiedad e
inquietud del delirium que para ello usaríamos el clordiazepóxido (opción 2 correcta) que es el
tratamiento de fondo.(R2)

315. La sintomatología de inicio más frecuente en un paciente con neurinoma del acústico
es:

1. 1. Acúfenos, hipoacusia y dolor facial.


2. 2. Acúfenos únicamente.
3. 3. Vértigo típico.
4. 4. Acúfenos, hipoacusia y alteraciones del equilibrio.
Gráfico de respuestas
Comentario

La clínica habitual de presentación del neurinoma del acústico es la hipoacusia retrococlear con
acúfeno unilateral de larga evolución. Puede presentar inestabilidad pero es muy rara la presencia
de vértigo típico intenso. Un 10 % de los neurinomas debutan como sordera brusca. Cuando el
tumor alcanza un tamaño considerable puede presentar dolor facial por afectación del V par o
ataxia por compresión cerebelosa.(R4)

316. Con respecto a la depresión endógena, ¿cuál de estos datos NO es típico?:

1. 1. Depresión más intensa por las tardes.


2. 2. Pérdida de peso.
3. 3. Respuesta a antidepresivos y terapia electroconvulsiva.
4. 4. Fatiga y disminución de la energía.
Gráfico de respuestas
Comentario

 
 
 
 
La duda razonable está entre la 2 y la 5. La fatiga intensa con sensación de pesadez de los
miembros de predominio vespertino es propia de la depresión atípica, pero en la 5 no especifican
tanto, y se limitan a hablar de anergia y astenia, que se dan en todas las depresiones, aunque más
en las endógenas, pues en las neurótico- reactivas domina la clínica ansiosa sobre la anergia. Así
que la claramente errónea es la respuesta 2: en la endógena se está peor por la mañana.(R1)

317. Si al segundo día de vida un recién nacido pretérmino tiene en suero sanguíneo 30
mg por 100 de bilirrubina no conjugada y 9 por 100 de hemoglobina se tratará con:

1. 1. Antibióticos.
2. 2. Transfusión sanguínea.
3. 3. Exanguinotransfusión.
4. 4. Se puede esperar al día siguiente sin hacer nada.
Gráfico de respuestas
Comentario

El manejo de la ictericia depende de unas tablas que establecen, para cada hora de vida, la
conducta que hemos de seguir. Lógicamente, nadie puede recordar esta gráfica de memoria. Pero,
ante una hiperbilirrubiemia tan importante como la propuesta en el enunciado (especialmente si
aparece en un contexto de anemia), debemos pensar en una isoinmunización anti-D grave, con
riesgo de kernícterus, e iniciar exanguinotransfusión.(R3)

318. Recién nacido de término recibe al nacer asistencia respiratoria con bolsa y máscara
con una presión máxima superior a 20 cmH2O por un período de 3 minutos. Al llegar a
hospitalización, presenta taquipnea, ronquidos y cianosis con desviación a la derecha
del choque de la punta. El tórax es asimétrico y a la auscultación se detecta
hipoventilación en el hemitórax izquierdo. El diagnóstico más probable es:

1. 1. Síndrome de corazón izquierdo hipoplásico.


2. 2. Neumomediastino.
3. 3. Neumotórax.
4. 4. Fístula traqueoesofágica
Gráfico de respuestas
Comentario

Niño a término con antecedente de presión positiva, que posteriormente presenta desviación a la
derecha del choque de la punta, cianosis, tórax asimétrico y en la auscultación hipoventilación
izquierda: el diagnóstico es neumotórax, respuesta 3.(R3)

319. Which of the following is the most common cause of colonic obstruction?

1. 1. Sigmoid colon cancer.


2. 2. Diverticulitis of the sigmoid colon.
3. 3. Cecal volvulus.
4. 4. Postoperative adhesions.
Gráfico de respuestas
Comentario

La causa más frecuente de obstrucción del colon es el cáncer colorrectal (más frecuente en recto y
sigma). Recordemos que la etiología de la obstrucción del intestino delgado es diferente ya que la

 
 
 
 
causa más frecuente son las adherencias y la segunda las hernias (que son la etiología más
frecuente en pacientes sin cirugía previa).(R1)

320. A 17-year-old caucasian male,


presentes to the ER complaining of shortness of breath for the last 30 minutes
accompanied by non-productive cough and wheezes. He also refers oppressive chest
pain. His medical history is unremarkable. Which of the following findings is present in
the associated image?

1. 1. Normal CXR study


2. 2. Lung hyperinflation
3. 3. Pneumothorax
4. 4. Bronchial wall thickening
Gráfico de respuestas
Comentario

Lo que nos están describiendo es una crisis asmática. Aunque es posible encontrarnos signos
como aplanamiento diafragmático y otras manifestaciones de hiperinsuflación pulmonar, esto sólo
sucede en los casos más graves. La manifestación radiológica más frecuente en una crisis
asmática es una radiografía de tórax normal, como la que nos muestran en esta imagen.(R1)

321. Mujer de 32 años con clínica de una semana de evolución consistente en febrícula,
eritema nodoso, inflamación periarticular de tobillos y uveítis anterior. En la radiografía
de tórax presenta adenopatías hiliares bilaterales. ¿Cuál es el diagnóstico?:

1. 1. Tuberculosis ganglionar mediastínica.


2. 2. Sarcoidosis tipo síndrome de Löfgren.
3. 3. Lupus eritematoso sistémico.
4. 4. Poliarteritis nodosa.
Gráfico de respuestas
Comentario

 
 
 
 
El síndrome de Löfgren es una forma aguda de sarcoidodis que puede asociar fiebre, artritis,
eritema nodoso y adenopatías hiliares. La uveítis anterior, aunque puede verse, es más típica del
síndrome de Heerfordt-Waldenström, que es otra forma aguda de la sarcoidosis que asocia fiebre,
uveítis, parotiditis y parálisis facial.(R2)

322. Uno de los siguientes NO es un factor predictor de buena respuesta al interferón en


la hepatitis crónica B:

1. 1. Infecciones producidas por la cepa mutante precore.


2. 2. Mujeres.
3. 3. Corta duración de la enfermedad.
4. 4. Niveles elevados de GPT.
Gráfico de respuestas
Comentario

Los factores predictores de buena respuesta son: el sexo femenino, edad joven, corta historia de
enfermedad, niveles bajos de DNA- VHB, niveles elevados de transaminasas, la escasa fibrosis en
la biopsia hepática y un estado inmunológico adecuado (inmunocompetentes). Sin embargo, la
cepa mutante precore suele ser resistente al tratamiento con interferon. Por ello, el tratamiento que
suele emplearse es la lamivudina. Recuerde que, en caso de no responder a lamivudina, el
siguiente recurso terapéutico sería el adefovir, entecavir o tenofovir.(R1)

323. Si le llega una niña de 10 años con metrorragias y síntomas de pubertad precoz de
rápido desarrollo, junto con sintomatología de hipertiroidismo (bocio, taquicardia,
temblor, etc.), debe sospechar:

1. 1. Estruma ovárico.
2. 2. Tumor ovárico de células germinales productor de HCG.
3. 3. Tumor hipofisario secretor de GH.
4. 4. Pseudopubertad precoz.
Gráfico de respuestas
Comentario

Esta pregunta de tumores de ovario no es muy importante pero puede ser útil para recordar los
tumores ováricos que secretan hormonas.

El estruma ovarii es una rara entidad de mujeres jóvenes que se engloba dentro de los teratomas y
produce hormonas tiroideas con el consiguiente desarrollo de hipertiroidismo pero no justificaría la
pubertad precoz. (el hipotiroidismo grave si puede desencadenar una pubertad precoz ).

El androblastoma ovárico es un tumor que reproduce elementos testiculares y produce andrógenos


y constituye la primera causa de virilización de origen ovárico.

El GHoma produciría gigantismo en esta caso. El tumor ovárico de células germínales productor de
HCG es el coriocarcinoma, un tumor muy infrecuente. La HCG tiene actividad TSH-like al tener la
subunidad alfa común. Recuerda que la enfermedad trofoblástica puede dar lugar a hipertiroidismo
también.(R2)

324. Niño de 8 años con síndrome de Down. Desde siempre han observado distensión
abdominal y apetito caprichoso. Hace un mes, en un examen rutinario se detectó una

 
 
 
 
anemia microcítica e hipocroma. ¿Cuál de las siguientes actitudes sería la adecuada para
diagnosticar la causa más probable de este cuadro clínico?:

1. 1. Determinar Ac antiendomisio IgA y antigliadina IgG.


Tratar con hierro oral durante 3 meses, y al finalizar, hacer una biometría hemática,
2. 2.
ferritina y estudio de hemoglobinas.
3. 3. Determinar ferritina e investigar sangre oculta en heces.
4. 4. Determinar ferritina y hacer un ultrasonido abdominal.
Gráfico de respuestas
Comentario

Lo que debemos sospechar en este caso es una enfermedad celíaca. Tenga en cuenta que esta
enfermedad no siempre cursa con un cuadro florido… En muchas ocasiones, se manifiesta con
una clínica digestiva poco específica y con el déficit de algún nutriente (en este caso, la anemia
que padece es compatible con ferropenia). Por otra parte, la enfermedad celíaca es más frecuente
en pacientes con síndrome de Down, detalle que debemos valorar. Dado que nos piden la actitud
orientada a la causa más probable del cuadro, habría que dirigir el estudio hacia esta enfermedad
(respuesta 1 correcta).(R1)

325. Neonato de 24 horas de vida que presenta esplenomegalia de 7 cm con discreta


hepatomegalia. En la inspección encontramos grandes ampollas fláccidas en manos y
pies, que se descaman dejando áreas denudadas, y rinorrea con obstrucción nasal que
le impide la toma adecuada de alimento. En la palpación se descubren linfadenopatías
diseminadas y llanto con la palpación de los codos y rodillas. En la radiografía de las
extremidades encontramos zonas de refuerzo en las epífisis que forman dichas
articulaciones. ¿Qué tipo de patología sospecha en este niño?:

1. 1. Sífilis congénita tardía.


2. 2. Sífilis congénita precoz.
3. 3. Infección por estafilococo productor de toxina exfoliativa.
4. 4. Infección connatal por VIH.
Gráfico de respuestas
Comentario

Se trata de un cuadro de sífilis precoz, en el que nos encontramos algunas de las características
más sobresalientes del mismo, como las ampollas fláccidas palmoplantares (pénfigo sifilítico), la
rinitis y la osteítis. Recuerde que, debido a la afectación ósea, el niño sufre importantes dolores con
el movimiento osteomuscular, por lo que tiende a estar inmóvil o a moverse poco (pseudoparálisis
de Parrot).(R2)

326. Con respecto a la isquemia mesentérica aguda, indique la opción que considere
INCORRECTA:

1. 1. La causa más frecuente es la embolia de la arteria mesentérica superior.


2. 2. Todas las embolias arteriales deben ser tratadas quirúrgicamente.
La fluoresceína puede ayudar a identificar los segmentos isquémicos que precisan
3. 3.
resección.
4. 4. Los vasodilatadores son el tratamiento de elección de las trombosis mesentéricas.
Gráfico de respuestas
Comentario

 
 
 
 
El 50% de las isquemias mesentéricas son producidas por un émbolo en la arteria mesentérica,
generalmente en pacientes con patología embolígena (fibrilación auricular, valvulopatías…). El
tratamiento en estos casos es quirúrgico, mediante la resección de segmentos no viables y
restablecimiento de la permeabilidad del vaso (embolectomía).

Existen varias técnicas para delimitar el segmento de intestino no viable y que debemos resecar,
como el eco- doppler para ver el flujo arterial o la inyección de sustancias fluorescentes como la
fluoresceína, que se fija en lugares donde existe flujo sanguíneo. Gran parte de las trombosis
mesentéricas agudas no tienen afectación de toda la pared de intestino y pueden recuperarse con
tratamiento médico que consiste en la anticoagulación y que ha de mantenerse de por vida. Los
vasodilatadores como la papaverina se reservan para las isquemias no oclusivas ya que en su
patogenia puede incluirse la vasoconstricción o el vasoespasmo.(R2)

327. ¿En cuál de las circunstancias que se enumeran a continuación NO esperaría


encontrar elevado el cociente entre la urea y la creatinina plasmática?:

1. 1. Rabdomiólisis.
2. 2. Dieta rica en proteínas.
3. 3. Hemorragia digestiva.
4. 4. Deshidratación.
Gráfico de respuestas
Comentario
Nos están preguntando por el cociente BUN / Creatinina plasmática, que quizás no es
excesivamente importante para el MIR, aunque en clínica se use de vez en cuando. Normalmente
la urea o BUN (nitrógeno ureico) y la creatinina aumentan paralelamente según disminuye el
filtrado glomerular,Es elemental que sepas en cuáles de estas circunstancias disminuye el filtrado
glomerular: En la rabdomiólisis al destruirse el músculo salen moléculas de mioglobina que si son
filtradas taponan los túbulos renales (exactamente ocasionan necrosis tubular tóxica) produciendo
insuficiencia renal, así aumenta tanto el BUN como la creatinina plasmática quedando el cociente
igual. En el resto de circunstancias concurre un aumento de BUN por el metabolismo que sufren
las proteínas de la sangre, pero esto normalmente no va acompañado de una disminución del
filtrado glomerular, así la relación BUN/creatinina aumenta.(R1)

328. Atiende usted a una


paciente de 44 años que fue diagnosticada a los 21 años de lupus eritematoso sistémico
habiendo presentado en este tiempo cuadro de artritis con afectación y deformidad de

 
 
 
 
las manos (ver imagen), nefritis proliferativa difusa, pericarditis, lesión cutánea facial en
"alas de mariposa" para la cual ha recibido tratamientos con AINE, corticoides a dosis
altas y, actualmente recibe tratamiento con 15 mg de prednisona. Acude a la consulta
refiriendo, desde hace 5 días, la presencia de un dolor acusado sobre la ingle derecha
que aparece fundamentalmente con el apoyo y mejora con el reposo. En la exploración
física es anodina, aunque tiene algo de dolor en las rotaciones de la cadera. Se evidencia
una lesión en el dorso de la nariz eritematosa, sobreelevada, con un centro algo atrófico
y, en las manos, una tendencia a la deformidad en "cuello de cisne" de los dedos sin
artritis activa de ninguna localización. El resto de la exploración es normal. Se solicitó
una radiografía de pelvis y una ecografía de la cadera en la que no se observa derrame.
La analítica mostraba una leve anemia con una bioquímica normal, ANA de 1/640 y anti
DNA de 38 , VSG de 18 y PCR: 0,7 (N menor de 0,5) y el resto normal. ¿Cuál sería su
diagnóstico de sospecha, la prueba que solicitaría y el tratamiento con respecto al
problema que puede haber desarrollado esta paciente?

Cree que la paciente puede haber desarrollado una osteonecrosis de la cadera por lo que
1. 1. indicaría tratamiento con reposo y descarga de la articulación y solicitaría una resonancia
magnética.
Cree que lo que la paciente tiene se debe a un brote de su lupus con artritis de la cadera
2. 2.
por lo que precisará aumentar la dosis de corticoides y añadir un antiinflamatorio.
Cree que la paciente podría haber sufrido una pseudofractura debido a una osteomalacia
3. 3. secundaria a su enfermedad renal. Indicaría tratamiento analgésico y suplementos de
vitamina D.
Cree que el dolor es debido a la extensión de la artropatía de Jaccoud que presenta la
4. 4.
paciente por lo que solicita una gammagrafía ósea e indica tratamiento con metotrexato.
Gráfico de respuestas
Comentario

Ante el desarrollo de un dolor de ritmo mecánico en rodillas o caderas en un paciente que toma
corticoides de forma crónica (sobre todo en una paciente con lupus), deberíamos siempre
descartar como primera opción la presencia de una ostenoecrosis aséptica . Dado que en esta
complicación la radiografía simple puede no mostrar hallazgos durante las primeras semanas, se
debería solicitar una resonancia para descartar esta posibilidad.

No parece tratarse de un cuadro inflamatorio por su enfermedad, ya que la ecografía no muestra


derrame, lo que, asociado a la ausencia de fiebre también nos haría descartar razonablemente la
posbilidad de que se trate de una artritis séptica, y tampoco se trata de una artropatía de Jaccoud,
que es una deformidad típica de esta enfermedad que afecta a las manos. La osteomalacia
aparece en pacientes ancianos o con algún tipo de déficit o fármacos que interfieren con la
absorción de la vitamina D y también en situaciones de insuficiencia renal. Aunque esta paciente
ha tenido una nefritis lúpica, no parece que tenga afectación de la función renal.(R1)

329. Varón de 43 años, con síntomas dispépticos de 3 años de evolución, que consulta
porque hace un mes se le realizó un test del aliento para H. pylori, y ha sido positivo. Se
realizó posteriormente una endoscopia en la que se le diagnosticó de gastritis crónica
superficial en región antral, con tinciones negativas para H. pylori. Las siguientes
medidas terapéuticas serían recomendables para el manejo de este paciente, EXCEPTO:

1. 1. Ranitidina.
2. 2. Magaldrato.
3. 3. Erradicación de H. pylori.
4. 4. Acexamato de Zn.

 
 
 
 
Gráfico de respuestas
Comentario

Una pregunta de cierta dificultad sobre la infección por H. pylori.

Como sabes, H. pylori ha sido implicado en la patogenia de diversas enfermedades (gastritis


agudas y crónicas, ulcus péptico, adenocarcinoma gástrico, linfomas MALT…). Sin embargo,
también es muy frecuente la infección por esta bacteria sin que produzca ningún tipo de síntoma.

Actualmente, la erradicación universal de H. pylori no está aceptada. Son claras indicaciones la


úlcera gastroduodenal y el linfoma primario tipo MALT. Cuando se trata de gastritis crónicas, es
aconsejable su erradicación en los casos en que asocie displasia. Cuando ésta no existe, su
indicación es controvertida, por lo que en el caso que nos presentan la opción 4 sería, como
mínimo, discutible. En cambio, el resto de las opciones son medidas sintomáticas ante el cuadro
dispéptico por el que consulta.(R3)

340. Un niño de 10 años ha sufrido una herida en la pierna derecha con un instrumento
oxidado 24 horas antes. La madre refiere que el niño recibió todas las vacunas del primer
año de vida. Qué indicaría usted para prevenir tétanos?:

1. 1. Inmunoglobulina antitetánica.
2. 2. Vacuna DT.
3. 3. Suero antitetánico.
4. 4. Ninguna de las anteriores.
Gráfico de respuestas
Comentario

Ya que el paciente recibió todas las vacunas durante su infancia y tiene 10 años, lo indicado es
revacunar con toxoide tetánico, respuesta 2 correcta.(R2)

341. Las convulsiones febriles se presentan con mayor frecuencia en:

1. 1. En el recién nacido.
2. 2. De 1 a 2 años de edad.
3. 3. A los 6 años de edad.
4. 4. A los 8 años de edad.
Gráfico de respuestas
Comentario

Las crisis febriles son un proceso típico de la edad infantil (entre los 5 meses y los 3 años). Se
relacionan con un aumento de la temperatura, lo que da lugar a una crisis el primer dia del proceso
febril, independientemente del origen del mismo. Las crisis febriles simples son generalizadas,
duran menos de 15 minutos, presentan buena recuperación posterior y los hallazgos en el periodo
intercritico son normales o negativos. Con frecuencia existen antecedenes familiares de crisis
febriles. Hay que diferenciarlas de las crisis febriles complejas que si que tienen patologia
subyacente, deben desaparecer a los 5 años de edad.(R2)

342. Niña de 18 meses que es traída a Urgencias por presentar crisis de tos,
desencadenadas por la ingesta de agua y alimento, y que suelen terminar con una pausa
de apnea o con vómitos de características mucosas. Como antecedentes de interés, sólo

 
 
 
 
destaca un cuadro catarral que apareció hace una semana. En la exploración física
destaca una hemorragia subconjuntival y una discreta hernia umbilical. Respecto a la
enfermedad que usted sospecha, señale cuál de las siguientes afirmaciones es cierta:

1. 1. El agente causal en un Paramyxovirus.


2. 2. La enfermedad suele conferir inmunidad duradera, de por vida.
3. 3. El tratamiento de elección es la penicilina benzatina i.m.
En el laboraotorio es típica la existencia de una importante leucocitosis con linfocitosis
4. 4.
absoluta.
Gráfico de respuestas
Comentario

El cuadro que nos describen corresponde a una tos ferina. Esta enfermedad es una infección
producida principalmente por Bordetella pertussis. Afecta preferentemente a menores de un año.
Clínicamente, produce un pródromo inespecífico, de tipo catarral, y seguidamente aparece la fase
que caracteriza a la enfermedad, que es la de tos paroxística. Produce accesos repentinos de tos,
con series repetitivas de múltiples toses enérgicas, acompañándose después de un ruido
inspiratorio que recibe el nombre de gallo, al pasar aire a través de una glotis cerrada. Las
complicaciones de la tos ferina son:

- Neumonía: La más frecuente. Suele ser por sobreinfección bacteriana secundaria.

- Convulsiones, cuya causa no está bien establecida.

- Otras: Prolapso rectal, hemorragia subconjuntiva, hernia umbilical, epistaxis… Estas


complicaciones se deben a la intensa presión positiva intratorácica que se produce en relación con
la tos.

A pesar de tratarse de una infección bacteriana, es característica la presencia de linfocitosis en el


hemograma. El tratamiento antibiótico de la tos ferina es la eritromicina durante dos semanas.(R4)

343. ¿Cuál de los siguientes datos NO es propio de la denominada leucemia de células


peludas?:

1. 1. Pancitopenia.
2. 2. Esplenomegalía.
3. 3. Adenopatías prominentes.
4. 4. Infecciones recidivantes.
Gráfico de respuestas
Comentario
El cuadro típico de la enfermedad es pancitopenia por mielofibrosis e hiperesplenismo,
esplenomegalia sin adenopatías e infecciones por inmunodeficiencia fundamentalmente
celular.(R3)

344. Respecto a los factores favorecedores de la EPI, señale la FALSA:

1. 1. Los anticonceptivos hormonales protegen frente a la EIP.


2. 2. Los DIU son los más frecuentemente asociados a EIP.
3. 3. Se da con menor frecuencia en nulíparas.
4. 4. Más frecuente en clases bajas.
Gráfico de respuestas

 
 
 
 
Comentario

La EPI es un tema que debe dominar al 100% para el nacional. La respuesta 3 es falsa; se da con
mayor frecuencia en nulíparas. Se puede responder aunque sea por eliminación: el resto de
opciones son ciertas.(R3)

345. Paciente de 30 años nuligesta, con última regla hace 20 días, que acude a la consulta
por presentar esterilidad y dis¬menorrea importante. En las exploraciones que se realizan
destaca tumoración anexial en el ultrasonido y elevación moderada de Ca 125 sérico.
Cuál es el diagnostico de presunción:

1. 1. Embarazo ectópico
2. 2. Cáncer de ovario
3. 3. Mioma Uterino
4. 4. Endometriosis
Gráfico de respuestas
Comentario

La endometriosis consiste en la presencia de tejido endometrial fuera de la cavidad uterina. Sus


manifestaciones principales son:

- dolor: dismenorrea y dispareunia.

- alteraciones menstruales.

- infertilidad.

- distensión abdominal.

Como característica, cabe destacar que puede elevar el Ca125, por lo que se debe hacer el
diagnóstico diferencial con el carcinoma de ovario.(R4)

346. Un paciente previamente sano es ingresado por una sepsis de origen abdominal. A
las 24 horas del ingreso comienza con disnea y taquipnea de creciente intensidad. Una
radiografía de tórax muestra infiltrados alveolares bilaterales, de predominio en bases,
que pasadas unas horas se extienden por los dos pulmones. La PaO2 es de 52 mmHg, a
pesar de la oxigenoterapia en altas concentraciones. ¿Cuál es su diagnóstico?:

1. 1. Absceso en hemiabdomen superior.


2. 2. Síndrome de distrés respiratorio del adulto.
3. 3. Hemorragia alveolar.
4. 4. Derrame por pancreatitis aguda necrohemorrágica.
Gráfico de respuestas
Comentario

El SDRA se define por un infiltrado alveolar bilateral sin causa cardiogénica, junto a una PaO2 < 55
mmHg con una FiO2 > 50%. La causa más frecuente es la sepsis bacteriana, como sucede en este
caso.(R2)

 
 
 
 
347. RN de 40 h de vida que presenta valores de bilirrubinas en riesgo intermedio bajo,
que actitud tomaría usted:

1. 1. Tomaría control de bilirrubinas en 4 a 6 horas.


2. 2. Colocaría en fototerapia preventiva hasta control en 12h.
3. 3. Tomaría control de bilirrubinas en 24-48h.
4. 4. Realizaría exanguineotransfusión.
Gráfico de respuestas
Comentario

Pregunta dificil sobre el Nomograma de Bhutani. Revise el siguiente esquema.(R3)

348. Un paciente de 38 años, diagnosticado de enfermedad de Graves, presenta, tras 2


semanas de tratamiento con metimazol, fiebre de 40 ºC y odinofagia. Ante esta situación,
se debe descartar inmediatamente:

1. 1. Faringitis estreptocócica.
2. 2. Tiroiditis postmetimazol.
3. 3. Esofagitis por Candida.
4. 4. Agranulocitosis.
Gráfico de respuestas
Comentario

La agranulocitosis por antitiroideos es un efecto adverso muy poco frecuente. Sin embargo, debido
a su gravedad, conviene tenerlo en cuenta y avisar a todos los pacientes que reciben tratamiento
con este tipo de fármacos. Es un efecto idiosincrático, aunque suele darse en los primeros meses
del tratamiento, coincidiendo con las dosis más altas del fármaco. A diferencia de otros efectos
adversos que producen los antitiroideos, éste sí aparece con los otros fármacos del mismo grupo.
Por tanto, ante un agranulocitosis no pueden darse otros antitiroideos y los pacientes precisan
tratamiento ablativo definitivo. Una vez se ha diagnosticado la agranulocitosis por antitiroideos, el
fármaco debe suspenderse, el paciente ingresar en el hospital con aislamiento de contacto al
menos y recibir tratamiento antibiótico que depende de la severidad del caso. El uso de
estimuladores de colonias hematopoyéticas (GM- CSF) está hoy en día más discutido y en todo
caso su uso debe reservarse para los pacientes más graves.(R4)
 

 
 
 
 
349 ¿Que microorganismos cubre la vacuna pentavalente?

1. 1. Difteria, tosferina, tétanos, hepatitis B, H. influenzae B.


2. 2. Difteria, tosferina, tétanos, H. influenzae B, polio.
3. 3. Difteria, tosferina, hepatitis B, polio, tétanos
4. 4. BCG, DPT, polio, sarampión y paperas.
Gráfico de respuestas
Comentario

La secuencia correcta de vacunación es la que sale reflejada en la respuesta 2.(R2)

350. Paciente diagnosticado de pancreatitis crónica y pseudoquistes que acude por dolor
abdominal epigástrico, vómitos alimenticios y pérdida de peso. La complicación que
probablemente presenta este paciente es:

1. 1. Obstrucción duodenal.
2. 2. Obstrucción del colédoco.
3. 3. Fístula pancreática a colon transverso.
4. 4. Fístula pancreática a estómago.
Gráfico de respuestas
Comentario

Cuando se produce un pseudoquiste, lo habitual es la resolución espontánea, por lo que suele


bastar un seguimiento ecográfico periódico. No obstante, los pseudoquistes ocupan espacio y se
encuentran en el triángulo duodenopancreático… Por ello, cuando el tamaño es considerable,
pueden producir compresión duodenal (respuesta 1 correcta), produciendo un cuadro como el que
nos describen.(R1)

351. En un hombre de 41 años que acude a consulta por un nódulo en el lóbulo tiroideo
derecho se ha realizado una punción-aspiración con aguja fina del nódulo. El análisis
citológico de la muestra obtenida tras la punción podría permitirnos:

1. 1. Distinguir entre un adenoma folicular y un carcinoma folicular mínimamente invasivo.


Identificar las áreas de invasión capsular y/o vascular características de un carcinoma
2. 2.
folicular.
3. 3. Distinguir entre un tumor de células de Hürthle y un adenoma folicular con oxifilia.
4. 4. Identificar las características nucleares típicas de un carcinoma papilar.
Gráfico de respuestas
Comentario

Esta pregunta no viene textualmente en el Manual, pero se puede intentar resolver con un poco de
sentido común, a partir de la información que sí tenemos sobre la enfermedad que nos preguntan.
Las respuestas 1 y 2 son claramente falsas, puesto que para distinguir entre un adenoma folicular
y un carcinoma folicular es necesario demostrar la presencia de invasión capsular y vascular, y
esto sólo es posible mediante la biopsia.

La respuesta 4 podría descartarse por la misma razón: aunque la PAAF permita identificar células
de Hürtle, no será suficiente para diferenciar un adenoma de un carcinoma. Nos quedarían, por
tanto, las respuestas 3 y 5. Aunque no tengamos claras cuáles son las características nucleares
típicas del carcinoma papilar de tiroides, sabemos que un análisis citológico sería suficiente para

 
 
 
 
detectarlas. Por otro lado, la hiperplasia de células C del carcinoma medular podría pasar
desapercibida en la PAAF en caso de existir. Por tanto, la respuesta correcta es la 5.(R4)

352. La alimentación complementaria cumple los siguientes objtivos. Marque la


afirmación verdadera con (V); la falsa con (F) A. Los alimentos son estimulos y vehículos
de nutrientes. B. El éxito depende del cumplimiento del esquema de alimentación. C.
Refuerza la relación madre – niño, y estimula el desarrollo de funciones psicomotoras. D.
Busca cubrir los requerimientos nutricionales, adecuado al crecimiento y actividad del
niño. E. El inicio puede ser antes del cuarto mes, ya que no existe contraindicaciones.

1. 1. V F V F V.
2. 2. F V F F V.
3. 3. V F V V F.
4. 4. V V F F V.
Gráfico de respuestas
Comentario

Los alimentos son estimulos y vehículos de nutrientes: verdadero

El éxito depende del cumplimiento del esquema de alimentación. Falso.

Es verdadero que refuerza la relación madre-hijo.

Es verdadero que busca cubrir los requerimientos nutricionales.

Es falso que el inicio es antes del cuarto mes.(R3)

353. Paciente de 52 años,


menopausia hace 3 años, que presenta sangrado vaginal escaso desde hace una semana.
En la exploración ginecológica se confirma sangrado procedente de cavidad uterina, sin
visualizarse lesiones a nivel cervical. No se delimitan masas pélvicas a la palpación
bimanual. Usted le realiza un ultrasonido vaginal y encuentra los hallazgos que se
muestran en la imagen. ¿Que patología sospecharía y como realizaría el diagnostico de
confirmación?:

 
 
 
 
1. 1. Quiste de ovario. Realizo punción dirigida por ecografía y citología del líquido.
2. 2. Mioma submucoso. Realizo legrado fraccionado.
3. 3. Pólipo endometrial. Realizo histeroscopia diagnóstica y toma de biopsias.
4. 4. Cáncer de endometrio. Realizo biopsia con cánula de Pipelle.
Gráfico de respuestas
Comentario
El diagnostico de un pólipo endometrial se realizara idealmente mediante una histeroscópia
diagnostica, que nos permite ver la cavidad endometrial y tomar biopsias dirigidas. La cánula de
Pipelle y la de Corner, así como el legrado fraccionado, nos permiten hacer biopsias a ciegas, no
asegurando así que hemos tomado muestra de la zona donde esta la patología.(R3)

354. En qué casos haría una aspiración traqueal

1. 1. RN con apgar 9 – 10
2. 2. Prematuro con Distress Respiratorio leve o moderado
3. 3. Macrosomico con llanto vigoroso
4. 4. Post – maduro deprimido con líquido amniótico meconial denso
Gráfico de respuestas
Comentario

La respuesta correcta es la 4, puesto que se debe aspirar el líquido meconial de la orofaringe del
recien nacido antes de que empiece a llorar y lo aspire hacia los pulmones produciendo una
neumonitis química.

RN con apgar 9-10 no necesita este soporte, en el prematuro, seguramente el distrés será
producido por un SMH y el macrosoma, aunque tenga meconio, si ya ha llorado no tiene indicación
la aspiración.(R4)

355. Respecto a la prevención de la transmisión vertical madre-hijo de la infección por


VIH es INCORRECTO decir:

1. 1. La lactancia materna se contraindica en nuestro medio.


El riesgo de transmisión perinatal se incrementa a partir de 12 horas de rotura de
2. 2.
membranas.
El recién nacido ha de recibir tratamiento antirretroviral que se debe iniciar antes de las 6
3. 3.
horas de vida y mantener durante al menos 4 semanas.
Las técnicas serológicas no son fiables en el diagnóstico de infección VIH en el niño
4. 4.
durante los 18 primeros meses.
Gráfico de respuestas
Comentario

La mayoría de las infecciones congénitas por VIH se adquieren durante el parto. La tasa de
transmisión del VIH ha descendido mucho en los últimos años gracias al conocimiento de los
mecanismos de transmisión y las medidas profilácticas. Entre éstas se incluyen el cribado universal
a todas las embarazadas, la terapia antirretroviral de gran actividad durante la gestación, la
cesárea electiva (en casos muy concretos se puede optar por el parto vaginal), la contraindicación
de la lactancia materna y el tratamiento del RN de forma precoz y prolongada.

Uno de los factores de riesgo más importantes de transmisión perinatal es la rotura de membranas,
siendo desaconsejable permitir que se prolongue más allá de 4 horas. El diagnóstico de la
infección connatal por VIH es fundamentalmente virológico, mediante técnicas moleculares, siendo

 
 
 
 
inútil la serología hasta los 18 meses por paso de anticuerpos maternos a través de la
placenta.(R2)

356. A 48-year-old male comes to your office complaining of abdominal cramps, that
usually start 30 minutes after meals, along with weakness and sweating. His medical
history is positive for hypertension, that is being treated with amlodipine; and obesity, for
which he underwent a partial gastrectomy 3 weeks ago. Physical examination reveals: BP
135/85 mm Hg, pulse 70/min and temperature 36,5ºC. What is the most likely diagnosis?

1. 1. Peptic ulcer disease


2. 2. Achalasia
3. 3. Late dumping syndrome
4. 4. Early dumping syndrome
Gráfico de respuestas
Comentario
Early dumping syndrome. Postgastrectomy syndromes include small capacity, dumping syndrome,
bile gastritis, afferent loop syndrome, efferent loop syndrome, anemia, and metabolic bone disease.
Clinically significant dumping syndrome occurs in approximately 10% of patients after any type of
gastric surgery, and it has characteristic alimentary and systemic manifestations. The clinical
presentation of dumping syndrome can be divided into gastrointestinal (GI) symptoms and
vasomotor symptoms. GI symptoms include early satiety, crampy abdominal pain, nausea,
vomiting, and explosive diarrhea. Vasomotor symptoms include diaphoresis, flushing, dizziness,
palpitations, and an intense desire to lie down. Most patients with early dumping have both GI and
vasomotor symptoms, while patients with late dumping have mostly vasomotor symptoms.(R4)

357. La aparición de braquialgia y síndrome de Horner en el mismo lado debe sugerir:

1. 1. Un ganglioneuroma.
2. 2. Un aneurisma aórtico.
3. 3. Una costilla cervical.
4. 4. Un tumor de Pancoast.
Gráfico de respuestas
Comentario

Pregunta sencilla sobre un tema que debes dominar, el cáncer de pulmón. El tumor de Pancoast
cursa con un síndrome de Horner (por afectación del simpático cervical), dolor en la cara cubital del
brazo y antebrazo (por afectación del plexo braquial) y erosión de las primeras costillas. Se debe al
crecimiento de un tumor en el vértice pulmonar que penetra en el canal neural y destruye C8, T1 y
T2, siendo su causa más frecuente el cáncer de pulmón, sobre todo el epidermoide. Luego la
respuesta correcta es la 4.(R4)

358. Paciente femenino de 14 años que acude a consulta por presentar menstruaciones
muy irregulares. La exploración física es normal, presentando un desarrollo mamario y
de vello axilar-pubiano adecuado para su edad. En el estudio de laboratorio, las
hormonas sexuales presentan niveles de FSH aumentados respecto a los de LH durante
todo el ciclo y falta el pico preovulatorio de LH. Con estos datos ¿cuál cree que puede
ser su diagnóstico?:

1. 1. Disfunción del sistema hipotálamo-hipofisario.


2. 2. Síndrome del ovario poliquístico.
3. 3. Anorexia nerviosa.

 
 
 
 
4. 4. Mioma uterino.
Gráfico de respuestas
Comentario

Es muy importante que conozca el diagnóstico diferencial de la amenorrea. Aunque en este caso
no se nos describa lo primero que se debe hacer es una prueba de embarazo, y una determinación
de TSH y PRL. En caso de que todo esto sea normal se debe hacer una prueba con administración
de progesterona durante 5 días y en caso de que se produzca la regla será debida a anovulación.
Pero en este caso nos describen el hecho de que no existe pico preovulatorio de LH, por lo que
estará anulada la secrección pulsatil de GnRH que es la responsable de este pico, por lo tanto esto
va a ser la causa de la amenorrea, la disfunción del sistema hipotálamo-hipofisario.(R1)

359.
Which of the following options should not be included in the management of the patient
with pyloric stenosis?

1. 1. Rehydration and correction of hydroelectrolytic disorders, should these be encountered.


2. 2. Ramstedt pyloromyotomy is the gold standard for the surgical treatment of this condition.
If the patient undergoes surgical treatment, post-operative fasting during 5 days is
3. 3.
recommended, until the swelling resolves.
Persistent vomiting after surgical treatment suggests incomplete pyloromyotomy, gastritis,
4. 4.
GERD or some associated cause of obstruction.
Gráfico de respuestas
Comentario
En primer lugar es necesaria la corrección de líquidos y electrolitos del pacientes, tal y como dice la
respuesta 1. Posteriormente, el tratamiento es quirúrgico, mediante la piloromiotomía de Fredet-
Ramstedt. En esta técnica, implementada en 1911, se realiza el corte de la serosa y luego de la
capa muscular circular del píloro, sin cortar la mucosa; luego se deja sin suturar la muscular y se
cierra la serosa. La intervención generalmente se realiza por laparotomía, aunque también se
puede a través de técnicas laparoscópicas. En el posoperatorio se inicia la alimentación habitual
del niño, siendo la tendencia actual iniciar a las 6 h, una vez que el paciente se ha recuperado de
la anestesia, con leche materna o fórmula sin diluir al volumen completo. Pueden aparecer vómitos
posoperatorios, que ceden entre la segunda y quinta toma. Un estudio relacionado con el inicio de
la alimentación del lactante en el postoperatorio, indica que la alimentación ad libitum después de
la recuperación anestésica, es segura y puede acortar la estancia hospitalaria. No obstante, si los
vómitos son persistentes tras la cirugía debe sospecharse una cirugía incompleta u otras causas
de obstrucción, tal y como se comenta en la respuesta 4.(R3)

 
 
 
 

360.
A 20-day-old boy is brought to the emergency department by his parents because he has
been continuously vomiting after breastfeeding for the past 48 hours. He usually "spits
up" a bit after eating. He has no fever and physical examination is unremarkable. An
abdominal echography is performed and the most relevant finding is shown in the image.
Which of the following options is false?

1. 1. The diagnostic suspicion is hypertrophic pyloric stenosis.


2. 2. The vomits are typically non-bilious.
The ultrasound diagnostic criteria are: Pyloric muscle thickness greater than 4 mm or
3. 3.
pyloric canal length greater than 14 mm.
Conservative management should be initially proposed, leaving surgical treatment as the
4. 4.
last resort when all other treatments have failed.
Gráfico de respuestas
Comentario

Caso clínico típico de estenosis hipertrófica del píloro, diagnóstico al que debería haber llegado sin
ayuda de la imagen adjunta. El protagonista suele ser, como en este caso, un niño de unos 20 días
de vida, con vómitos inicialmente ocasionales, pero que después aparecen en relación con cada
toma. Por supuesto, los vómitos no son biliosos, ya que la región estenosada es proximal a la
desembocadura de las vías biliares. Desde el punto de vista de la técnica de examen, al pedirnos
la falsa, la consecuencia lógica es que existan tres opciones ciertas. Como en varias de ellas se
habla de “estenosis pilórica” (1), “engrosamiento del músculo pilórico” (2), “masa pilórica” (3), el
diagnóstico es evidente, con foto o sin ella. En la imagen adjunta podemos ver un engrosamiento
del píloro en la parte izquierda de la imagen, y una imagen redondeada, en “dona”, en la derecha,
una imagen transversal del píloro hipertrofiado. Sin embargo, insistimos una vez más en que las
fotografías, en muchos casos, ejercen un papel fundamentalmente decorativo.(R4)

361. Llevan a su consultorio a un paciente de 3 años, con el antecedente que 10 días


previos a la consulta le diagnostican otitis media aguda. Recibió tratamiento con
amoxicilina por 7 días. Actualmente no refiere dolor ni fiebre. Al examen presenta
secreción liquida en oído afectado, la membrana timpánica no es eritematosa. La
conducta mas adecuada sería:

1. 1. Continuar con la amoxicilina por 3 días más.


2. 2. Dar un curso de antibióticos con amoxicilin-clavulanato.
3. 3. Dar corticoides.
4. 4. Seguimiento al paciente.
Gráfico de respuestas
Comentario

 
 
 
 
La respuesta correcta es la 4, ya que la pauta antimicrobiana fue adecuada, amoxicilina por 7 días.
El paciente ya no presenta fiebre, ni dolor, la membrana timpánica no es eritematosa, solo tiene
secreción líquida, no purulenta.(R4)

362. La psoriasis es una enfermedad común de la piel, de afectación individual muy


variable. En cuanto a su presentación en la población general, señale lo que le parece
FALSO:

1. 1. Es una enfermedad más frecuente en la raza blanca que en el resto.


La edad de aparición presenta una frecuencia bimodal, con un primer pico en la segunda
2. 2.
década de la vida, y otro en torno a los 55-60 años.
3. 3. Aunque es más frecuente en mujeres, la edad de aparición es más temprana en los varones.
Los individuos con HLA-B13 y HLA-B17 tienen más riesgo de psoriasis que la población
4. 4.
general.
Gráfico de respuestas
Comentario

Una pregunta de cierta dificultad, ya que exige conocer algunos detalles no muy preguntados sobre
la psoriasis. Aunque es algo más frecuente en hombres, tiene tendencia a aparecer antes en
mujeres, por lo que la respuesta correcta es la 3. No olvides que es más frecuente en la raza
blanca, tiende a debutar entre los 20- 30 años y existe un segundo pico hacia los 50- 60 años.(R3)

363. Paciente de 55 años, fumador e hipertenso que acude a urgencias con dolor
centrotorácico de características isquémicas. En el ECG presenta elevación del ST en II,
III, aVF. De las opciones siguientes señala la FALSA:

1. 1. La arteria del nodo sinusal depende de la coronaria izquierda en el 60% de los casos.
2. 2. La mayor parte de los IAM posteroinferiores se deben a patología de la coronaria derecha.
3. 3. Los IAM inferiores se suelen unir a manifestaciones gastrointestinales.
4. 4. La arteria del nodo AV depende de la coronaria derecha.
Gráfico de respuestas
Comentario
La respuesta 1 es falsa porque la arteria del nodo sinusal depende en un 55% de la coronaria
derecha y un 45% de la arteria circunfleja (rama de la izquierda). Es una pregunta difícil, pues si no
conocemos este dato nos podemos liar. Es muy curiosa la respuesta 3, pero es cierta, pues los
infartos inferiores suelen cursar con activación del parasimpático (vagales) y puden producirse
náuseas y vómitos. También hay que recordar la respuesta 5, ya que por esa razón son más
frecuentes los bloqueos AV en los infartos inferiores que en los anteriores.(R1)

364. A 15-year-old female comes to the doctor's office presenting with hirsutism and
irregular menses. Physical examination shows adequate height and mild overweight, a
deep voice, periareolar hair with a normal breast development and clitoris hypertrophy.
Hormonal serum levels are: LH 25 IU/L, FSH 10 IU/L, 17 hydroxyprogesterone 500 ng/mL,
Serum testosterone normal, DHEA normal. What is the most likely diagnosis?

1. 1. Polycystic ovary syndrome


2. 2. Congenital adrenal hyperplasia
3. 3. Cushing's syndrome
4. 4. Conn's syndrome
Gráfico de respuestas
Comentario

 
 
 
 
Congenital adrenal hyperplasia. The girl in this case has signs of hyperandrogenism. Polycystic
ovarian syndrome is one of the most common causes but increased levels of 17-
hydroxyprogesterone and normal testosterone levels suggest a different diagnosis. Congenital
adrenal hyperplasia must be considered.(R2)

365. Señale cuál de los siguientes trastornos NO es una indicación de estudio y


erradicación de H. pylori:

1. 1. Dispepsia no ulcerosa refractaria.


2. 2. Linfoma MALT de bajo grado.
3. 3. Úlcera duodenal.
4. 4. Familiares en 2 grado de pacientes afectos de adenocarcinoma gástrico.
Gráfico de respuestas
Comentario
Son los familiares de primer grado de pacientes afectos de cáncer gástrico los que deben
estudiarse y erradicarse en caso de positividad, dado que tienen más resigo que la población
general de desarrollar un adenocarcinoma gástrico. El resto, son indicaciones reconocidas en el
consenso de Maastrict III sobre estudio y tratamiento de H. Pylori.(R4)

366. ¿Cuál de las siguientes formas clínicas de psoriasis puede mostrar una evolución
hacia la curación espontánea en corto tiempo?:

1. 1. Psoriasis minor (forma de tipo eczema seborreico de cuero cabelludo y cara).


2. 2. Psoriasis en gotas de aparición en jóvenes.
3. 3. Psoriasis ungueal (onicodistrofia psoriásica).
4. 4. Psoriasis eritrodérmico.
Gráfico de respuestas
Comentario
Si conocemos las formas clínicas del psoriasis veremos que se trata de una pregunta muy sencilla.
La respuesta correcta es el psoriasis en gotas, una forma clínica de buen pronóstico que cursa con
brotes de lesiones de pequeño tamaño en tronco y que es más frecuente en niños pequeños tras
infecciones faríngeas por estreptococo. Página 19. Manual CTO 4ª Edición.(R2)

 
 
 
 

367. A 72-year-old female comes to the ER


complaining of back pain for the last three weeks. Her past medical history is significant
for giant-cell arteritis treated with high-dose steroid therapy. She denies any trauma but
refers over effort few days ago. She has no fever or other symptoms. Palpation of the
paravertebral muscles is painful. There are no sensory or motor deficits. The lateral X ray
of the lumbar spine is shown below. Which of the following statements is false?

1. 1. The patient has at least three fractures in vertebrae


2. 2. In 75% of the cases, in this type of fractures, surgical fixation is required
3. 3. Steroid use and advanced age are risk factors for these fractures
4. 4. Conservative treatment is the preferred initial choice in most of these cases.
Gráfico de respuestas
Comentario

Reparemos en el hecho de que la paciente tiene factores de riesgo de osteoporosis, como el ser
mujer y la toma de corticoides. Debemos saber que el manejo de las fracturas vertebrales es
fundamentalmente sintomático, con analgesia y reposo cuando duelen.

El tratamiento quirúrgico sólo estaría indicado cuando el tratamiento conservador no resulte eficaz,
siempre que persista edema óseo en la RMN (respuesta 2 correcta).(R2)

368. La causa más común de secreción sanguínea por el pezón en mujeres pre-
menopáusicas es:

1. 1. Enfermedad de Paget.
2. 2. Papiloma.

 
 
 
 
3. 3. Carcinoma de mama.
4. 4. Fibroadenoma de mama.
Gráfico de respuestas
Comentario

La secreción patológica a través del pezón puede ser unilateral o localizada a un único conducto.
La causa más frecuente de secreción patológica es el papiloma 52-57%. La secreción asociada al
papiloma puede ser clara o hemorrágica. Los papilomas solitarios pueden presentar áreas de atipia
o carcinoma ductal in situ. El resto de casos se debe a ectasia ductal y otros cambios benignos.

En cuanto a malignidad se pueden presentar en 5-15 % de pacientes con descarga anómala. El


más frecuente es el carcinoma ductal in situ.(R2)

369. Respecto al laringocele señale lo INCORRECTO:

1. 1. Es una dilatación del sáculo o apéndice del ventrículo de Morgagni.


2. 2. Puede presentar una etiología tumoral en un 2-10% de los casos (tumor de ventrículo)
3. 3. Pueden rellenarse modo (laringomucocele) o pus (laringopiocele)
La mayoría son bilaterales por una laxitud natural de la membrana tirohioidea, más
4. 4.
frecuentemente en hombres.
Gráfico de respuestas
Comentario
La mayoría son unilaterales, asintomáticos y más frecuentemente en hombres.(R4)

370. A 50-year-old man presents to the emergency room with sudden chest pain and
dyspnea. His past medical history is significant for diabetes mellitus and hypertension.
He has smoked two packs of cigarettes per day since he was 20 years old and he drinks
two cans of beer a day. The patient has had recent surgery of a right femur fracture. On
arrival to the emergency room a right bundle branch block not previously described is
appreciated. Echocardiography shows right ventricular dilation with preserved LVEF.
Which of the following is the most likely diagnosis?

1. 1. Right ventricle infarction


2. 2. Cardiac tamponade
3. 3. Left anterior descending artery occlusion
4. 4. Pulmonary embolism
Gráfico de respuestas
Comentario
Pulmonary embolism. Pulmonary embolism is a trending topic in USMLE exams. The risk factors for
PE are: previous embolic events, prolonged inmobilization (bed, long airplane trips), surgery,
adenocarcinmoa, obesity, tobacco, oral contraception…70% of patients have previous deep vein
thrombosis (swollen and painful leg). Most common symptoms are: sudden shortness of breath and
pleuritic chest pain, tachicardia and tachipnea. ECG shows sinus tachicardia and may show
SIQIIIT3 complex, CXR is normal. An angioCT of pulmonary arteries is the diagnostic test of choice.
Normal D-Dimer levels have great negative predictive value. Anticoagulation should be started as
soon as possible and fibrinolitic treatment is indicated if the patient presents right ventricular
dysfunction or shock.(R4)

371. Señale la afirmación FALSA respecto al sarcoma de Kaposi epidémico:

1. 1. Aparece en personas más jóvenes que el sarcoma de Kaposi clásico.

 
 
 
 
2. 2. La afectación mucosa es muy frecuente, especialmente la oral.
3. 3. El 5-10% no tienen afectación cutánea.
4. 4. La afectación visceral más frecuente es la ganglionar.
Gráfico de respuestas
Comentario
Pregunta sobre conceptos básicos de la asignatura. El sarcoma de Kaposi clásico afecta a
pacientes ancianos y deteriorados, mientras que el epidémico afecta frecuentemente a varones
homosexuales VIH positivos, resultando muy raro el dasarrollo en los demás grupos de riesgo de
SIDA. La presencia de lesiones en la mucosa oral es un dato muy característico del sarcoma de
Kaposi asociado a SIDA. La afectación viscera se produce en el 75 % de los pacientes, siendo los
organos más afectados los ganglios linfáticos en un 50%, el traco gastrointestinal y el pulmón. La
respuesta incorrecta es la 3.(R3)

372. Usted ve en consulta por primera vez a un paciente de 55 años, diagnosticado de


enfermedad pulmonar obstructiva crónica (EPOC). Desde los 15 años, había fumado un
promedio de 20 cigarrillos al día (40 paquetes-año). El síntoma fundamental es la tos, con
expectoración abundante, especialmente por las mañanas. Esto lleva ocurriéndole varios
años y, según dice, raro es el día que no expectora al despertarse. En la exploración física
destacan abundantes roncus en la auscultación y edemas hasta los tobillos. ¿Cuál de las
siguientes actuaciones considera FALSA en este paciente?:

1. 1. Es probable que exista cor pulmonale crónico.


2. 2. En este paciente no son aconsejables los diuréticos.
Si el paciente tuviera enfisema en vez de bronquitis crónica, la auscultación sería
3. 3.
diferente.
4. 4. En el hemograma no sería sorprendente encontrar un aumento del valor HCTO.
Gráfico de respuestas
Comentario

Aunque el uso de diuréticos no cambiará el pronóstico del paciente a largo plazo, sí que tendrían
su utilidad para el tratamiento de los edemas, por lo que la respuesta 3 es falsa. Como sabes, este
grupo de fármacos se utiliza mucho en la insuficiencia cardíaca, aliviando los edemas cuando es
derecha, y la disnea si es izquierda.(R2)

373. El factor predisponente más frecuente de infección urinaria recurrente en niñas es.

1. 1. Nefrolitiasis.
2. 2. Diverticulos vesicales.
3. 3. Reflujo vesicoureteral.
4. 4. Hidronefrosis.
Gráfico de respuestas
Comentario

Pregunta fácil sobre epidemilogía de infección de vías urinarias. En caso de niñas, la causa más
frecuente de infecciones recurrentes de vías urinarias son debido a reflujo vesicoureteral.
Respuesta 3 correcta. La opción 4 la descarta ya que la presencia de hidronefrosis es
consecuencia del reflujo. La opción 1 la descarts inmediatamente ya que, ¿consideras que es muy
frecuente la aparición de nefrolitiasis en niños?.(R3)

 
 
 
 
374. En relación a la reanimación en neonatos, seleccione la respuesta CORRECTA.

Aproximandamente el 1% de los recién nacidos requiere de algún tipo de asistencia para


1. 1.
iniciar la respiración al nacer.
La administración de medicamentos es raramente necesaria en la reanimación de los
2. 2.
recién nacidos.
El “ABC” de la reanimación neonatal es igual a la que se realiza en todo paciente
3. 3.
pediátrico.
4. 4. El masaje cardiaco es siempre necesario en la reanimación de los recién nacidos.
Gráfico de respuestas
Comentario

Pregunta sencilla, que puede llegar a la respuesta correcta por descarte. 1% de asistencia en los
recién nacidos es un número muy bajo, mientras que el ABC, ha cambiado, actualmente es CAB y
el masaje cardiaco no siempre es necesario, por lo que la respuesta correcta es la 2.(R2)

375. Respecto a la traqueotomía es cierto:

1. 1. Una saturación de oxigeno superior al 90% la contraindica.


2. 2. Siempre es permanente.
Se debe realizar en pacientes ingresados en UVI que llevan intubados más de diez días y
3. 3.
se prevee una intubación prolongada por patología de base.
4. 4. Actualmente todas se hacen percutáneas.
Gráfico de respuestas
Comentario
Las intubaciones prolongadas pueden originar granulomas laríngeos y estenosis laringo-
traqueales, por lo que deben ser evitadas. La mayoría de las traqueotomías son de carácter
temporal y sólo se pueden hacer percútaneas en pacientes seleccionados. En niños la
traqueotomía se debe evitar, porque la decanulación es casi imposible. Se puede tener una disnea
severa alta con saturaciones buenas, porque el paciente compensa la al paso del aire con tiraje
muscular, pero cuando éste falla, se desatura en escasos minutos.(R3)

376. Respecto a las amenorreas primarias, es FALSO que:

1. 1. La causa más frecuente son las disgenesias gonadales.


El síndrome de Swyer es una disgenesia gonadal en la que es frecuente la aparición de
2. 2.
talla baja y malformaciones asociadas.
3. 3. El síndrome de Morris tiene cariotipo 46,XY y puede desarrollar disgerminoma.
En la hiperplasia suprarrenal congénita aparecen genitales externos masculinizados,
4. 4.
debido a un trastorno enzimático de la síntesis del cortisol.
Gráfico de respuestas
Comentario

En esta pregunta se nos pregunta aspectos simples de las amenorreas primarias.

La causa más frecuente de las amenorreas primarias (paciente 16 años o más que aún no ha
tenido la menstruación) es la disgenesia gonadal, de ellas la más frecuente es el síndrome de
Turner (asocia fenotipo Turner, talla baja, infantilismo sexual, pterigium colli, cúbito valgo).

La disgenesia gonadal pura no asocia ningún fenotipo especial (talla baja, malformaciones
características), y el tipo 46,XY se denomina síndrome de Swyer (respuesta falsa es la 2).

 
 
 
 
El síndrome de Rokitansky asocia malformaciones renoureterales al derivar su origen embriológico
de los conductos de Müller.

El síndrome de Morris es la feminización testicular y sus testículos intraabdominales pueden


degenerar en un disgerminoma.

La hiperplasia suprarrenal congénita se debe a un déficit enzimático en la cadena de síntesis del


colesterol, siendo los más frecuentes los déficit de 21, 17 y 11 hidroxilasa.(R2)

377. Un hombre de 30 años con una historia de 10 años de evolución de colitis ulcerosa
se somete a una colectomía total con anastomosis ileoanal. ¿Cuál de las siguientes
manifestaciones extracolónicas es de esperar que NO regrese tras la intervención?:

1. 1. Colangitis esclerosante.
2. 2. Gonartritis.
3. 3. Iritis, episcleritis.
4. 4. Sacroileítis.
Gráfico de respuestas
Comentario

Concepto poco preguntado hasta ahora en el ENARM. En la colitis ulcerosa hay asociadas
manifestaciones extraintestinales. Algunas de ellas tienen relación en su curso evolutivo con el
estado de actividad de la enfermedad inflamatoria intestinal, pero otras, en cambio, son
independientes de la actividad intestinal. Así, en las que sí dependen de la actividad, si
conseguimos controlar el brote de colitis ulcerosa, también mejorará la manifestación
extraintestinal. En general, aquellas entidades que cursen con cierto grado de fibrosis o esclerosis
(como la colangitis esclerosante primaria), son independientes de la actividad de la enfermedad
intestinal y, por tanto, no regresarán tras la colectomía total.(R1)

378. Mujer de 46 años, de profesión secretaria. Como antecedentes destaca una ulcera
duodenal correctamente tratada hace 3 años y una depresión controlada con
antidepresivos tricíclicos. Desde hace 7 días presenta lesiones papulosas, foliculares y
queratósicas que asientan sobre una base eritematosa. Además existe una
queratodermia palmoplantar importante. El eritema tiene un tono amarillo anaranjado. El
cuero cabelludo también se encuentra afectado. Señale la FALSA en relación a esta
enfermedad:

1. 1. A menudo existen zonas de piel sana entre las placas afectas.


2. 2. El etretinato mejora la enfermedad, pero no acorta el curso clínico.
3. 3. La etiología es desconocida.
4. 4. La forma circunscrita afecta al 25% de los casos y es típica de los adultos.
Gráfico de respuestas
Comentario

La pitiriasis rubra pilaris obedece a un trastorno de la queratinización, que se engloba dentro de


las enfermedades eritematodescamativas. Puede ser adquirida o hereditaria autosómica
dominante (en este caso, aparece en la infancia). Las características clínicas más importantes son:

• Placas descamativas anaranjadas, con islotes de piel normal dentro de ellas.

 
 
 
 
• Pápulas queratósicas foliculares en zonas de extensión.

Puede confundirse con la psoriasis, que es su principal diagnóstico diferencial. Para su tratamiento,
se emplean queratolíticos y el acitretino. En alguna ocasión, responde a tratamiento con
PUVA.(R4)

379. Un paciente de 40 años, sin antecedentes de interés, consulta por disfagia. Ésta no
se produce al iniciar la deglución, sino unos segundos después. El cuadro ha
evolucionado progresivamente durante los últimos tres años, y actualmente es
constante. Ocasionalmente, padece episodios de dolor retroesternal, de carácter
opresivo. Ha perdido peso, aunque no de forma significativa, desde el comienzo del
cuadro (4 kg en 3 años). Refiere la reciente aparición de regurgitaciones no ácidas desde
hace pocos meses. Señale el diagnóstico más probable:

1. 1. Cáncer de esófago.
2. 2. Espasmo esofágico difuso.
3. 3. Cáncer de esófago.
4. 4. Acalasia esofágica.
Gráfico de respuestas
Comentario

Una disfagia progresiva asociada a pérdida de peso y regurgitaciones no ácidas, debe hacerte
pensar en acalasia.

Los ca de esófago también provocarían disfagia progresiva y pérdida de peso, pero nos hablan de
4 kg en 3 años, un cáncer de esófago hubiera provocado una mayor afectación.(R4)

380.   Mujer de 46 años que vive en un lugar de la


Mancha, acude a Urgencias en el mes de agosto. Refiere que tres días antes, de manera
brusca, comienza con fiebre de 40 °C acompañada de cefalea intensa y hoy nota la

 
 
 
 
aparición de manera diseminada de las lesiones que se exponen en la porción inferior
de la fotografía, que también afectan palmas y plantas. A la exploración se observa,
además de estas lesiones, en cara posterior de muslo derecho otra lesión mostrada en
la porción superior de la fotografía. El examen físico no reveló otras anomalías. Los
análisis mostraron una hemoglobina de 14,1 g/dl, leucocitos 4.300 mm 3 , con discreta
linfopenia, GOT 68 U/L, GPT 47 U/L. Las radiografías de tórax fueron normales. ¿Cuál es
el agente etiológico que verosímilmente condiciona el cuadro?

1. 1. Treponema pallidum.
2. 2. Borrelia burgdorferi.
3. 3. Bartonella henselae.
4. 4. Rickettsia conorii.
Gráfico de respuestas
Comentario

Respuesta vinculada a la imagen n.º 14 del examen MIR. La enfermedad que se describe es una
Fiebre Botonosa Mediterránea. Es coherente con los factores de riesgo: ocurre en verano y en
medio rural. Es coherente con el cuadro clínico: fiebre alta, cefalea intensa, alteración leve de
enzimas hepáticos y exantema cutáneo que característicamente afecta a las palmas y a las
plantas. También es coherente con la fotografía que se facilita, en la cual se ve una lesión
compatible con mancha negra: úlcera necrótica de pequeño tamaño rodeada de un halo
eritematoso. Por tanto, la respuesta correcta es que esta enfermedad está producida por Rickettsia
conorii y que el tratamiento de primera elección es doxiciclina.

Treponemapallidum es el agente causal de la sífilis. También produce fiebre y exantema que


afecta a palmas y plantas en la sífilis secundaria pero no se describe factor de riesgo para
enfermedad de transmisión sexual y no tendría que tener mancha negra. Borrellia burdogferi es el
agente causal de la enfermedad de Lyme. Se transmite por picadura de garrapata, al igual que la
Fiebre Botonosa Mediterránea, pero no tiene mancha negra y la afectación cutánea inicial no es en
forma de exantema generalizado. La afectación cutánea de la primera fase es una lesión “en
diana” denominada eritema crónicomigrans. Bartonella henselae produce la enfermedad por
arañazo de gato y lesiones cutáneas de tipo “vascular” en pacientes infectados por VIH (la
denominada angiomatosis bacilar). No es compatible con el cuadro clínico y fotográfico que se
describe. Coxiella burnetii produce la denominada Fiebre Q: no tiene lesiones cutáneas. El cuadro
agudo suele ser una neumonía atípica acompañada de hepatitis y el cuadro crónico una
endocarditis sobre válvula aórtica con hemocultivos negativos.(R4)

381. En estas circunstancias el tratamiento más adecuado para esta enferma es:

1. 1. Penicilina G sódica i.v.


2. 2. Doxiciclina.
3. 3. Cotrimoxazol.
4. 4. Gentamicina.
Gráfico de respuestas
Comentario

Respuesta vinculada a la imagen n.º 14 del examen MIR. La enfermedad que se describe es una
Fiebre Botonosa Mediterránea. Es coherente con los factores de riesgo: ocurre en verano y en
medio rural. Es coherente con el cuadro clínico: fiebre alta, cefalea intensa, alteración leve de
enzimas hepáticos y exantema cutáneo que característicamente afecta a las palmas y a las

 
 
 
 
plantas. También es coherente con la fotografía que se facilita, en la cual se ve una lesión
compatible con mancha negra: úlcera necrótica de pequeño tamaño rodeada de un halo
eritematoso. Por tanto, la respuesta correcta es que esta enfermedad está producida por Rickettsia
conorii y que el tratamiento de primera elección es doxiciclina.

Treponemapallidum es el agente causal de la sífilis. También produce fiebre y exantema que


afecta a palmas y plantas en la sífilis secundaria pero no se describe factor de riesgo para
enfermedad de transmisión sexual y no tendría que tener mancha negra. Borrellia burdogferi es el
agente causal de la enfermedad de Lyme. Se transmite por picadura de garrapata, al igual que la
Fiebre Botonosa Mediterránea, pero no tiene mancha negra y la afectación cutánea inicial no es en
forma de exantema generalizado. La afectación cutánea de la primera fase es una lesión “en
diana” denominada eritema crónicomigrans. Bartonella henselae produce la enfermedad por
arañazo de gato y lesiones cutáneas de tipo “vascular” en pacientes infectados por VIH (la
denominada angiomatosis bacilar). No es compatible con el cuadro clínico y fotográfico que se
describe. Coxiella burnetii produce la denominada Fiebre Q: no tiene lesiones cutáneas. El cuadro
agudo suele ser una neumonía atípica acompañada de hepatitis y el cuadro crónico una
endocarditis sobre válvula aórtica con hemocultivos negativos.(R2)

382. Respecto a la profilaxis de la eritroblastosis fetal por isoinmunización Rh de la madre con


gammaglobulina anti-Rh, ¿cuál de las siguientes afirmaciones es la verdadera?

Sin profilaxis, la probabilidad de que una mujer Rh negativa con hijo Rh positivo se
1. 1.
inmunice tras el primer parto es de 1 entre 1.000.
Hoy se recomienda administrar, alrededor de las 28 semanas de edad gestacional, una
2. 2. dosis de 250 microgramos de gammaglobulina anti-Rh a las embarazadas Rh negativas no
inmunizadas, cuando el Rh del padre es positivo o desconocido.
La profilaxis postparto se debe administrar antes de que transcurran 7 días de la fecha del
3. 3.
parto.
En caso de realizar una amniocentesis precoz a las mujeres Rh negativas no inmunizadas,
4. 4. antes de las 20 semanas de edad gestacional, no es necesario hacer profilaxis porque a esa
edad todavía no se han manifestado los antígenos de los hematíes fetales.
Gráfico de respuestas
Comentario

La profilaxis con gammaglobulina se debe hacer en todas aquellas embarazadas Rh (-), siempre
que el test de Coombs indirecto sea negativo (es decir, no esté ya inmunizada) y que el marido NO
sea Rh(-) conocido (es decir, siempre que sea Rh (+) o desconocido) (respuesta correcta la 2) y
posteriormente, si el feto es Rh (+), una nueva dosis dentro de las 72 primeras horas posparto.

La respuesta 1 es incorrecta porque la probabilidad de que una mujer Rh- con hijo Rh+ se
inmunice tras el primer parto es mayor de 1/1.000.

En situaciones donde se registran antecedentes obstétricos de riesgo, con compromiso fetal


predecible, y en edad gestacional precoz para realizar transfusión intrauterina (TIU), la terapia de
elección es administrar altas dosis de gammaglobulina endovenosa (400 mg/kg/día durante cinco
días o 2 g/kg/día durante dos días) en ciclos cada 21 días.

 
 
 
 
La respuesta 4 es incorrecta. Se debe administrar gammaglobulina anti-D tras aborto, embarazo
ectópico, amniocentesis, funiculocentesis o versión cefálica externa.(R2)

Profilaxis de la isoinmunización Rh

383. Señale la respuesta INCORRECTA en relación a la CIA tipo ostium secundum:

1. 1. El defecto suele localizarse en la región de la fosa oval.


2. 2. Es más frecuente en varones.
3. 3. El primer tono es fuerte y a menudo se ausculta un soplo diastólico precoz.
4. 4. El segundo tono es siempre único.
Gráfico de respuestas
Comentario
Lo más importante de la exploración física de la CIA tipo ostium secundum es la auscultación, en la
que se objetiva un primer tono fuerte con soplo sistólico eyectivo de hiperaflujo pulmonar y, lo más
característico, un desdoblamiento amplio y fijo del segundo tono, que se produce como
consecuencia del retraso de la sístole del ventrículo derecho, puesto que en la CIA el ventrículo
derecho recibe sangre de la aurícula derecha y aurícula izquierda. Ten en cuenta que el
desdoblamiento del 2º ruido puede hacerse menos evidente o incluso desaparecer, si aparece
hipertensión pulmonar.(R4)

384. Una muchacha de 14 años, de tipo asténico, que juega de pívot en un equipo de
baloncesto y calza un número 8, viene sufriendo esguinces repetitivos en sus muñecas
y tobillos. Hace algunos días sus familiares han descubierto que tiene la espalda "algo
torcida" hacia un lado, y la escápula derecha algo más elevada que la izquierda.
Últimamente nota que se fatiga mucho y palpitaciones que se incrementan
excesivamente a los pocos minutos de estar jugando. También ha apreciado que está
perdiendo agudeza visual. No refiere ningún otro antecedente. ¿Cuál de los siguientes es
el diagnóstico más probable?

 
 
 
 
1. 1. Aracnodactilia.
2. 2. Acondroplasia.
3. 3. Osteogénesis imperfecta.
4. 4. Osteopatía de Paget.
Gráfico de respuestas
Comentario

Es un caso clínico muy representativo del síndrome de Marfan. Por ello, la respuesta correcta es la
1, ya que la aracnodactilia es especialmente frencuente en esta enfermedad. La pregunta no está,
estrictamente, bien construida, ya que la aracnodactilia no es una enfermedad como tal… Pero el
resto de las opciones no encajarían, por lo que es la mejor solución.

Respecto a la paciente de la pregunta, merece la pena darse cuenta de los siguientes detalles:

- Talla elevada (juega de pívot) y tipo asténico. Ambos elementos son característicos del fenotipo
del síndrome de Marfan.

- Calza un 8 (no está mal para una niña de 14 años…).

- Esguinces de repetición, lo que traduce hiperlaxitud.

- Presencia de escoliosis.

- Sensación disneica y palpitaciones. Recuerda que el síndrome de Marfan se asocia al prolapso


de la válvula mitral.

- Pérdida de agudeza visual. Esto puede ocurrir en el Marfan, ya que se asocia a enfermedades del
cristalino (colobomas, subluxación…).

En la acromegalia, también podría haber talla elevada, pies grandes y pérdida de agudeza por
compresión hipofisaria en relación con el adenoma… Pero no explicaría la hiperlaxitud, las
palpitaciones ni la escoliosis.(R1)

385. Paciente masculino de 73 años, con antecedentes de insuficiencia cardíaca


congestiva en tratamiento con digoxina, diuréticos e IECA, que acude a urgencias por
dolor abdominal agudo y diarrea sanguinolenta. En la exploración, el paciente presenta
TA 90/60, arrítmico, abdomen doloroso sin signos de irritación peritoneal y ausencia de
ruidos intestinales. En el enema opaco se observan imágenes en huella digital y en la
colonoscopia colitis en colon izquierdo, respetando el recto. Ud. sospecha que el
paciente presenta:

1. 1. Colitis isquémica.
2. 2. Angiodisplasia intestinal.
3. 3. Angina intestinal.
4. 4. Isquemia mesentérica aguda.
Gráfico de respuestas
Comentario

La colitis isquémica se debe a una insuficiencia circulatoria a nivel del colon. Lo más frecuente es
que obedezca a causas no oclusivas (por ejemplo, hipoperfusión por bajo gasto en relación con

 
 
 
 
una insuficiencia cardíaca descompensada). La región más frecuentemente afectada es el ángulo
esplénico del colon (punto de Griffith), porque es una de las zonas peor irrigadas del colon. En
cambio, no se suele afectar el recto, puesto que su irrigación no sólo depende del sistema de las
mesentéricas (hemorroidal superior), sino que también recibe irrigación de otro origen (hemorroidal
media, rama de la hipogástrica, y hemorroidal inferior, rama de la pudenda).

La clínica característica es un dolor moderado tipo cólico, de inicio súbito, y que puede
acompañarse de rectorragia o diarrea sanguinolenta. Es poco frecuente el vómito. Puede
manifestarse de forma fulminante, pero lo más frecuente es que se trate de un cuadro subagudo.
Cuando progresa hacia la gangrena, el dolor se vuelve constante y aparecen síntomas de
abdomen agudo. En casos crónicos, pueden producirse estenosis segmentarias de colon, con el
consiguiente riesgo de obstrucción.

El diagnóstico definitivo se realiza por colonoscopia. Rara vez se recurre al enema opaco, el cual
revela el dato clásico de las “impresiones digitales” o estenosis en el segmento cólico afectado. Por
supuesto, el enema opaco está contraindicado en pacientes con sospecha de gangrena. La
arteriografía no es útil y no demuestra anomalías vasculares, lo cual sugiere que se trata de una
enfermedad de la microvascularización.(R1)

386. Lactante de 5 meses que ha presentado episodio de tos, sofocación y náuseas tras
estar jugando con un juguete pequeño. Si se hubiera producido una aspiración de cuerpo
extraño y hubiera quedado impactado en el bronquio derecho, ejerciendo obstrucción
completa bidireccional en inspiración y espiración, ¿qué esperaría encontrar en la
radiografía de tórax?:

Atelectasia obstructiva en el lado derecho con desplazamiento del mediastino y corazón


1. 1.
hacia el lado derecho en inspiración y espiración.
Enfisema obstructivo en el lado derecho con desplazamiento del mediastino en espiración
2. 2.
hacia el lado izquierdo.
Atelectasia del lado derecho con desplazamiento del mediastino y corazón hacia el lado
3. 3.
izquierdo.
Enfisema obstructivo del lado derecho con desplazamiento del mediastino hacia el lado
4. 4.
derecho.
Gráfico de respuestas
Comentario
Esta pregunta acerca de los hallazgos radiológicos en el caso de aspiración es importante ya que
han sido preguntados en alguna ocasión. Recuerda que se debe hacer una radiografía en
inspiración y expiración ante la sospecha de cuerpo extraño en vía aérea (también en caso de
neumotórax). Puede mostrar el CE si se trata de un cuerpo radiopaco. Pueden aparecer signos de
enfisema obstructivo ( hiperinsuflación con aumento de los espacios intercostales, horizontalización
del diafragma y desplazamiento mediastínico al hemitorax contralateral), o atelectasia (en caso de
obstrucción completa de un bronquio) con elevación del hemidiafragma del lado afecto y
desplazamiento del mediastino hacia ese mismo hemitórax.(R1)

387. En nuestro medio, la etiología más frecuente de vértigo periférico es:

1. 1. VPPB.
2. 2. Neuronitis vestibular.
3. 3. Migraña basilar.
4. 4. Fractura del peñasco por accidente de tráfico.
Gráfico de respuestas
Comentario

 
 
 
 
El 40-50% de los mareos son vértigos periféricos. De estos, más de la mitad son VPPB. La
mayoría son idiopáticos, y de los de causa conocida los más frecuentes se relacionan con catarros
y tras traumatismos craneoencefálicos.(R1)

388. Una paciente diagnosticada de esclerosis sistémica presenta a lo largo de su


enfermedad los siguientes síntomas clínicos. En relación a cada uno de ellos, se describe
a continuación un tratamiento adecuado específico. Señale de los descritos la asociación
que juzgue INCORRECTA:

El tratamiento con prednisona a dosis bajas de 10 mg/día o menos está indicado en el


1. 1. tratamiento de artritis refractaria a AINE y en casos de afectación cutánea precoz para
reducir el edema en fases iniciales.
2. 2. Los antagonistas del calcio pueden ser útiles en el fenómeno de Raynaud.
3. 3. La colchicina es muy efectiva en las manifestaciones articulares de ésta enfermedad.
Los glucocorticoides están indicados en la miositis inflamatoria y pleuropericarditis. No
4. 4. deben usarse sin embargo en la forma primaria de afectación insidiosa muscular de la
enfermedad.
Gráfico de respuestas
Comentario
El tratamiento de la esclerodermia ha sido menos preguntado ultimamente, pero debes conocerlo.
Los fármacos más novedosos son los IECAs para las crisis renales, que han salvado vidas a base
de disminuir la insuficiencia renal. La D- penicilamina se utiliza como tratamiento de los cambios
cutáneos, y consigue modificar en cierto modo la evolución de la enfermedad visceral. La
colchicina se usa con el mismo fin, pero no modifica el curso de la enfermedad. No sirve para
brotes de artritis, se requieren AINEs o corticoides para éstos. El ac. paraminobenzoico y la
vitamina E tienen las mismas aplicaciones que la colchicina. Los vasodilatadores como
antagonistas del calcio son tratamiento del fenómeno de Raynaud, para evitar la vasoconstricción
funcional. Para manifestaciones severas viscerales, como la fibrosis pulmonar incipiente o la
miositis (aunque la mayoría de las veces la miopatía es por limitación cutánea), los corticoides son
de elección, asociados o no a inmunosupresores.(R3)

389. Señale la causa más frecuente de degeneración macular:

1. 1. Degeneración macular senil.


2. 2. Coriorretinopatía serosa central.
3. 3. Estrías angioides.
4. 4. Edema macular cistoide.
Gráfico de respuestas
Comentario

La causa más frecuente de degeneración macular es la degeneración macular asociada a la edad


(DMAE). Es la primera causa de ceguera no corregible en personas mayores en el mundo
occidental. Afecta a un 12% de la población entre 65 y 75 años, y hasta a un 30% entre los 75 y los
85 años.(R1)

390. La causa más frecuente de convulsiones en el periodo neonatal es:

1. 1. Procesos metabólicos.
2. 2. Encefalopatía hipóxico-isquémica.
3. 3. Traumatismos durante el parto.
4. 4. Enfermedades maternas.
Gráfico de respuestas

 
 
 
 
Comentario

La etiología de las convulsiones se ha preguntado con cierta frecuencia en el MIR. Debes conocer
al menos la causa más frecuente para cada grupo de edad, que te señalamos en la siguiente
tabla.(R2)

391. A 78-year-old caucasian male presents to the Emergency Department with


continuous abdominal pain for the last 5 hours. He reports an episode of diarrhea without
blood or pus at the onset of the pain. No nausea or vomiting. His medical history includes
diabetes mellitus type 2 and hypertension. Physical examination shows diffuse
abdominal tenderness and pain without rebound or guarding. The EKG shows a
previously unknown atrial fibrillation. Abdominal x-ray shows no signs of bowel
obstruction. Laboratory work-up shows: Hb 13.5 g/dL RWC 9.500 /mm3 Serum sodium
144 mEq/L Serum potassium 5.8 mEq/L Chloride 104 mEq/L BUN 18 mg/dL Serum
creatinine 1.2 mg/dL Serum lactate 4.2 mmol/L Which of the following diagnostic tests is
most appropriate?

1. 1. Abdominal Ultrasound
2. 2. Abdominal CT without iodinated contrast
3. 3. Exploratory laparotomy
4. 4. Computed Tomography Angiogram
Gráfico de respuestas
Comentario
Computed Tomography Angiogram. We suspect acute mesenteric ischemia. In an eldery patient
with a cardiovascular disorder who presents with abdominal pain with no apparent etiology and an
unspecific physical exam we should immediately suspect ischemia and perform a CT-
angiography.(R4)

392. Sobre el "síndrome medular central", ¿qué NO le parece correcto?:

1. 1. El pronóstico es bueno.
2. 2. La espondilosis cervical predispone a este cuadro.
3. 3. La sensibilidad en miembros inferiores no está conservada.
4. 4. Son variables las alteraciones de la función vesical.
Gráfico de respuestas
Comentario
Es una pregunta difícil ya que requiere un conocimiento avanzado de este síndrome para contestar
bien. No te preocupes si has fallado. El síndrome medular central consiste en la afectación para la
sensibilidad térmica y dolorosa en el nivel compresivo medular mientras que la sensibilidad táctil se
encuentra conservada. Por esto decimos que produce un déficit sensitivo suspendido y disociado.
Si se afecta el asta anterior medular se produce amiotrofia, paresia y arreflexia segmentaria. Puede
haber parálisis espástica por debajo del nivel lesional si alcanza al haz piramidal cruzado. El
síndrome medular central aparece en la siringomielia, hidromielia y tumores centromedulares.(R3)

 
 
 
 

393. Recién nacido a término, CIR


simétrico, perímetro cefálico 30 cm. No se puede realizar ultrasonido cerebral
transfontanelar, por lo que realiza una TAC que puede ver en la imagen adjunta. ¿Cuál es
su diagnóstico de presunción?

1. 1. Metabolopatía.
2. 2. Infección congénita por herpes humano tipo 5.
3. 3. Hemorragia intracraneal.
4. 4. Cromosomopatía.
Gráfico de respuestas
Comentario

La infección congénita por citomegalovirus (herpes humano tipo 5) produce microcefalia y


calcificaciones periventriculares.

Recuerde CMV: Coriorretinitis, Microcefalia y calcificaciones periVentriculares.(R2)

394. ¿Qué otras lesiones presentará su paciente?

1. 1. Acidosis láctica congénita.


2. 2. Exoftalmos.
3. 3. Labio leporino.
4. 4. Coriorretinitis.
Gráfico de respuestas
Comentario

La infección congénita por citomegalovirus produce, además, coriorretinitis.

 
 
 
 
Recuerde CMV: Coriorretinitis, Microcefalia y calcificaciones periVentriculares.(R4)

395. ¿En presencia de cuál de los siguientes procesos deberemos buscar la existencia
de un tumor maligno del tracto respiratorio o digestivo alto?:

1. 1. Enfermedad de Paget mamaria.


2. 2. Dermatitis seborreica.
3. 3. Síndrome de Reiter.
4. 4. Acroqueratosis paraneoplásica.
Gráfico de respuestas
Comentario
Las dermatosis paraneoplásicas apenas se preguntan en el MIR. No te preocupes por fallar esta
pregunta, que es por otra parte bastante difícil. De estas dermatosis, debes limitarte a saber a qué
tumor se asocian y qué aspecto tienen las lesiones. La acroqueratosis paraneoplásica, o síndrome
de Bazex, se asocia a tumores con tal frecuencia que es absolutamente extraordinario que
aparezca sin relación con neoplasias. Casi el 100% de estos pacientes padecen algún cáncer,
siendo el más frecuente el carcinoma epidermoide de vías aerodigestivas altas. Clínicamente,
cursa con importantes alteraciones ungueales y una erupción de placas eritematosas en zonas
acras (pulpejos de dedos, nariz y orejas), cubiertas por escamas muy adherentes que producen
sangrado al ser desprendidas.(R4)

396. ¿Cuál es la causa de la mayor parte de los casos de crisis aplásicas transitorias que
aparecen de forma brusca en los pacientes con enfermedad hemolítica crónica?

1. 1. Enfermedades autoinmunes.
2. 2. Parvovirus B19.
3. 3. Toxicidad farmacológica.
4. 4. Papilomavirus.
Gráfico de respuestas
Comentario

Recuerda que el parvovirus B19, agente del eritema infeccioso, es responsable de crisis aplásicas
en pacientes con enfermedades hemolíticas crónicas (como la anemia falciforme o la esferocitosis
hereditaria).(R2)

397. Un paciente alcohólico presenta en su domicilio, durante la convalecencia de una


neumonía, un cuadro de confusión y ataxia severa. La exploración muestra un nistagmus
horizontal en ojo derecho, en la mirada hacia la derecha. La familia asegura que no ha
bebido alcohol. ¿Cuál es su opinión?

1. 1. El paciente ha bebido, aunque la familia lo niegue.


2. 2. El cuadro es sugerente de una enfermedad de Marchiafava-Bignami.
3. 3. Debe recibir tiamina con urgencia.
4. 4. El primer diagnóstico a descartar es el de hematoma subdural.
Gráfico de respuestas
Comentario

La aparición de un cuadro de confusión, ataxia y nistagmo horizontal en un paciente alcohólico,


debe hacernos pensar en una encefalopatía de Wernicke, por lo que administraremos como
tratamiento tiamina o vitamina B1.(R3)

 
 
 
 
398. Una mujer de 72 años, con bronquitis crónica, obesa y con tres hijos, consulta por
episodios frecuentes de escapes involuntarios de orina, aunque poco abundantes.
Suelen desencadenarse en coincidencia con episodios de tos, con la risa o con la
maniobra de Valsalva. El residuo postmiccional es normal. Sistemático de orina y
urocultivo normales. Teniendo en cuenta el diagnóstico más probable, una de las
siguientes medidas NO estaría recomendada:

1. 1. Oxibutinina.
2. 2. Ejercicios del suelo pélvico.
3. 3. Duloxetina.
4. 4. Tratamiento quirúrgico si fracasan las medidas conservadoras.
Gráfico de respuestas
Comentario

Se trata de una paciente que presenta incontinencia urinaria de esfuerzo, patología que se produce
por una pérdida de competencia del esfínter vesical, generalmente por pérdida de parte de la
competencia del soporte del piso pélvico. Todos los tratamientos descritos están indicados en
distintas fases de esta patología excepto la oxibutinina, antiespasmódico vesical utilizado en los
casos de incontinencia de urgencia.(R1)

399. El síntoma que con mayor frecuencia provoca la necesidad de asistencia médica en
los casos de obstrucción de las vías urinarias es:

1. 1. Oligoanuria.
2. 2. Malestar general.
3. 3. Dolor.
4. 4. Náuseas y vómitos.
Gráfico de respuestas
Comentario

La litiasis ureteral se manifiesta clínicamente como cólico nefrítico. El dolor es de características


cólicas, irradiado hacia genitales (siguiendo la trayectoria del uréter) y generalmente se extiende
hacia esta zona desde la fosa renal. Es debido a la hiperpresión en la vía urinaria, proximalmente a
la obstrucción, y el motivo de consulta es casi siempre el dolor que produce. Puede acompañarse
de otros síntomas, como hematuria o cortejo vegetativo (palidez, sudoración, sensación nauseosa),
pero no suelen ser la razón por la que el paciente consulta.(R3)

400. Mujer de 55 años acude por obesidad a nuestra consulta. En la exploración destaca
aumento discreto de tiroides de consistencia dura. Las hormonas tiroideas: TSH 21
mUI/ml (0,5-5), T4L 0,8 mg/dl (1,2-2,2), la gammagrafía tiroidea captación disminuida e
irregular del trazador. Los anticuerpos antiperoxidasa se encuentran elevados. Señale la
etiología más probable:

1. 1. Tiroiditis de Quervain.
2. 2. Tiroiditis crónica con tirotoxicosis transitoria (silente).
3. 3. Tiroiditis crónica linfocitaria.
4. 4. Tiroiditis de Riedel.
Gráfico de respuestas
Comentario
Pregunta en relación con el diagnóstico diferencial del hipotiroidismo. Los datos más relevantes de
este caso clínico son la existencia de bocio, una captación irregular del trazador, y sobre todo la

 
 
 
 
elevación de los anticuerpos antitiroideos. El diagnóstico más probable de esta paciente será la
tiroiditis crónica autoinmune, que es la causa más frecuente de hipotiroidismo primario. No
debemos sospechar ni la tiroiditis subaguda ni la silente, dado que no existen antecedentes de una
fase de hipertiroidismo inicial y dado que los anticuerpos están muy elevados. La tiroiditis de Riedel
no suele presentar hipotiroidismo, y lo más llamativo de esta entidad es el bocio grande y pétreo
que obliga a descartar la existencia de un carcinoma anaplásico.(R3)

401. Una mujer de 65 años, previamente sana, consulta por metrorragia escasa de tres
semanas de evolución. El examen ginecológico y el USG son normales. ¿Qué actitud
tomaría?:

1. 1. Realizar estudio de coagulación.


2. 2. Solicitar exploración pélvica con TC.
3. 3. Histerectomía.
4. 4. Biopsia endometrial.
Gráfico de respuestas
Comentario

Ante una mujer postmenopaúsica con metrorragia aunqu eel USG sea normal debemos descartar
una patología endometrial (benigna o maligna).

Para ello recurrimos a la biopsia guiada por histeroscopía, que actualmente constituye el método
diagnóstico de elección en estos casos.

Recordad que la metrorragia es el principal síntoma, y habitualmente el más precoz de la mayoría


de tumores genitales.(R4)

402. Paciente mujer de 46 años de edad, antecedentes de obesidad y uso de


anticonceptivos orales durante 15 años, antecedentes familiares de hermana con cáncer
de mama, acude a la revisión con el ginecólogo, y al realizar la mastografía se aprecia un
nódulo denso y espiculado en el cuadrante superoexterno de la mama derecha, sugestivo
de malignidad. Se realiza a la paciente una PAAF, que confirma la existencia de la lesión,
siendo etiquetada como carcinoma intraductal. Se realiza estudio de extensión, en el que
no se aprecian lesiones sugerentes de proceso metastático. Con estos datos se
programa a la paciente para realizar tratamiento quirúrgico de la lesión. Al informar a la
paciente, esta solicita información acerca de los factores pronóstico de la enfermedad
que padece. ¿Cuál es el factor pronóstico más importante del cáncer de mama?

1. 1. Grado histológico de diferenciación tumoral.


2. 2. Tipo histológico.
3. 3. Receptores hormonales.
4. 4. Infiltración ganglionar.
Gráfico de respuestas
Comentario

De los factores pronósticos del cáncer de mama es fundamental recordar que el más importante
es el número de ganglios afectados.(R4)

 
 
 
 
403. La principal utilidad de los fijadores externos en Traumatología es el tratamiento de:

1. 1. Fracturas vertebrales inestables.


2. 2. Fracturas cerradas de fémur.
3. 3. Fracturas de marco obturador de pelvis.
4. 4. Fracturas abiertas de tibia.
Gráfico de respuestas
Comentario
La fijación externa es un dispositivo que permite inmovilizar fracturas sin abordar el foco de
fractura, lo cual minimiza la posibilidad de infección en fracturas de alto riesgo como las fracturas
abiertas o infectadas. Así mismo es de utilidad cuando queremos obtener una síntesis estable de
una fractura de forma rápida y con poca morbilidad asociada, como puede ser el caso de una
fractura inestable de pelvis, en la que lo prioritario es la inestabilidad hemodinámica por el
sangrado de vasos retroperitoneales. Las fracturas del marco obturador son fracturas de ramas
típicas del anciano y son estables.(R4)

404. Un hombre de 54 años acude a su consulta por presentar desde hace tres meses
dolor en epigastrio irradiado a ambos hipocondrios y pérdida de 5 kg de peso en dicho
periodo. Las pruebas de laboratorio evidenciaron FA 789, GGT 564, resto normal. Una
ecografia no mostró alteraciones, presentando la vía biliar un calibre normal. ¿Cuál cree
que es la prueba que le aportaría más información sobre la etiología?

1. 1. Biopsia hepática.
2. 2. Colangiopancreatografía retrógrada endoscópica.
3. 3. TC.
4. 4. RMN.
Gráfico de respuestas
Comentario

Tienes que leer muy bien lo que te están preguntando, porque en eso consiste la dificultad de esta
pregunta. Efectivamente, la clínica orienta a cáncer de páncreas, aunque también podría tratarse
de un colangiocarcinoma, y otros tumores digestivos tampoco son imposibles.

Ahora vas a ver por qué la has fallado, porque el diagnóstico sí lo habrás orientado bien. Lo que te
preguntan no es la mejor prueba de imagen (entonces sería el TAC), sino la prueba que MÁS
INFORMACIÓN aporte sobre la etiología. Date cuenta de que la CPRE te permite detectar un
cáncer de páncreas, no sólo el TAC (cuando hay cáncer, se distorsiona la arquitectura de los
conductos intrapancreáticos, estando íntegros en caso contrario). Pero además te permite TOMAR
BIOPSIAS, lo que con el TAC o la RMN es imposible, y un mejor estudio de las vías biliares.(R2)

405. Paciente de 77 años incorporado a un programa de diálisis periódica desde hace 5


años por nefropatía secundaria a litiasis coraliforme bilateral. En los últimos 6 meses
destaca: síndrome constitucional clínico, dolor sordo no incapacitante en fosa renal
derecha, no fiebre y no focalidad infecciosa clara. No otros datos clínicos de interés. En
las analíticas mensuales se objetivan parámetros de desnutrición en incremento que
incluyen: disminución de albúmina, de colesterol y de potasio. Así mismo incremento
llamativo de la anemia de características normocíticas y normocrómicas, habiendo
pasado de precisar 3000 U. de eritropoyetina subcutáneas semanales a 21000 U./semana.
No tenía pautado Fe en el último año ni había recibido transfusiones con anterioridad.
Ante: Hgb 8 g/dl, Hto 25%, VCM 93, Fe sérico 22 mg/dl, saturación de transferrina 15% y

 
 
 
 
ferritina sérica de 1000 ng/dl. ¿Cuál de las siguientes respuestas NO sería cierta en este
contexto?:

1. 1. Resistencia clara a la eritropoyetina de 6 meses de evolución.


2. 2. Proceso inflamatorio crónico oculto.
Administración de Fe intravenoso a grandes dosis como coadyuvante del tratamiento con
3. 3.
eritropoyetina.
Tras resultado diagnóstico se practicó nefrectomía derecha con resultado
4. 4.
anatomopatológico de pionefrosis crónica.
Gráfico de respuestas
Comentario
El manejo de la anemia en la insuficiencia renal crónica fue preguntado en la convocatoria 2005-
2006, no te extrañe que lo vuelvan a preguntar en la próxima. Definimos la resistencia a la EPO
como el fracaso en conseguir la concentración diana de hemoglobina o la necesidad de recibir más
de 20.000 U/semana. La causa más común es el déficit absoluto o funcional de hierro. Así
debemos valorar pérdidas sanguíneas crónicas, infección/inflamación, hiperparatiroidismo,
toxicidad por aluminio, fármacos, diálisis inadecuada? En hematología habrás estudiado la anemia
por trastorno crónico, en ella el Fe está disminuído en plasma porque está bloqueada su salida a
plasma, la saturación de transferrina es normal o baja pero lo más importante como es este caso la
ferritina está aumentada. En el tratamiento de este tipo de anemia no se administra Fe (es
contraproducente) porque hay de sobra en los depósitos, pero no puede salir, lo que se necesita es
un tratamiento etiológico de la infección/inflamación crónica en el riñón derecho (la clínica lo
indicaba: síndrome constitucional, dolor crónico en fosa derecha?). En el diagnóstico estaba
acertada la solicitud de un TAC abdominal.(R3)

406. ¿Cuál de las siguientes medidas le parece prioritaria ante un paciente en shock
hemodinámico tras accidente de coche con un traumatismo tóraco-abdomino-pélvico
con herida abierta en hemitórax izquierdo?:

1. 1. Aspiración de los vómitos.


2. 2. Aporte de volumen.
3. 3. Laparotomía exploratoria urgente.
4. 4. Rx tórax.
Gráfico de respuestas
Comentario

Preguntamuy importante de un tema que hemos repasado cada simulacro Debe dominar el
ABCDE en el paciente politraumatizado.

1. Airway (vía aérea) más protección de la columna cervical: lo 1º que hay que hacer es asegurar
la permeabilidad de la vía aérea, con extracción de cuerpos extraños, aspiración de vómitos
(RC- 1) y elevación de la barbilla con tracción anterior de la mandíbula. Se usará tanto la cánula
de Guedel como la intubación en caso de ser necesario.

2. Breathing (respiración y ventilación).

3. Circulación (valoración y tratamiento del estado de shock) con control de los puntos sangrantes
activos.

4. Disability (lesiones neurológicas).

 
 
 
 
5. Exposición completa del paciente para detectar posibles lesiones ocultas.
(R1)

407. ¿Cuál de los siguientes procedimientos es el más seguro para diagnosticar el


sufrimiento fetal intraparto?

1. 1. La auscultación fetal.
2. 2. El ultrasonido doppler color.
3. 3. El perfil biofísico fetal.
4. 4. La microtoma fetal.
Gráfico de respuestas
Comentario

Pregunta muy fácil sobre métodos de valoración de bienestar fetal intraparto que hemos repasado
previamente.

Las opciones 2 y 3 son métodos de valoración anteparto. El resto de las opciones son
posibilidades para utilizar durante el período de dilatación y expulsivo, pero la opción 4 es el único
parámetro bioquímico, así pues es la mejor forma de diagnosticar un sufrimiento fetal, que es una
situación de acidosis.(R4)

408. Niño de 8 años de edad, llega a urgencias con un un cuadro de 2 días de evolución,
caracterizado por orina oscura y “estar hinchado”. Tiene como antecedente una infección
de la piel hace dos semanas. Al examen físico: costras de color miel, edema de miembros
inferiores, hepatomegalia, palidez e hipertensión arterial. El diagnostico probable es:

1. 1. Glomerulonefritis post-estreptocócica.
2. 2. Síndrome nefrótico.
3. 3. Nefritis lúpica.
4. 4. Hematuria benigna.
Gráfico de respuestas
Comentario

Pregunta sencilla. Se tiene que identificar los datos importantes:

1.- Antecedente de infección cutánea.

2.- Cambio en la coloración de la orina, edemas e hipertensión.

Todo esto orienta a un síndrome nefrítico, por razones obvias post-estreptocócica.(R1)

409. ¿Cuál de los siguientes cuadros tiene peor pronóstico?:

1. 1. Crisis de angustia.
2. 2. Síndrome ansioso-depresivo.
3. 3. Ansiedad reactiva.
4. 4. Fobia simple.
Gráfico de respuestas
Comentario

 
 
 
 
Es una pregunta con un poco de truco, pues hay que ser hábil al distinguir pronóstico con
gravedad. Seguramente si nos pidiesen gravedad pensaríamos en la depresión endógena, por su
riesgo de recaídas, y su intensidad, sin embargo suelen responder bien al tratamiento con
antidepresivos (ADT, IMAO, ISRS...). Lo mismo ocurre con las crisis de angustia, que su
tratamiento es muy efectivo, tanto con antidepresivos como con terapia cognitivo- conductual. La
ansiedad reactiva, es decir, ante un suceso, es bastante frecuente, y cede tras el desencadenante,
con buen pronóstico. Al igual ocurre con la fobia simple, que suele ser limitada al objeto, y con
buena respuesta a técnica de exposición. Sin embargo, el síndrome ansioso- depresivo es un
complejo heterogéneo, generalmente asociado a problemáticas diversas, cuyo abordaje es
dificultoso y poco satisfactorio. Bibliografía: Manual Psiquiatría CTO- Medicina 4ªEd, pág 3.(R2)

410. Con respecto al cáncer de mama y embarazo, señale la respuesta FALSA:

1. 1. Es la neoplasia maligna más detectada durante el embarazo.


2. 2. Tiene peor pronóstico porque es más agresivo por el ambiente hormonal.
3. 3. El tipo histológico más frecuente es el carcinoma ductal infiltrante.
Si se opta por la cirugía conservadora, la radioterapia debe demorarse hasta después del
4. 4.
parto, pero en las 6 semanas siguientes a la cirugía.
Gráfico de respuestas
Comentario

Son todas verdaderas menos la opción 2. Es verdad que tiene peor pronóstico, pero no porque sea
más agresivo, sino por el retraso en el diagnóstico del cáncer de mama en la mujer embarazada.
Por eso se recomienda la exploración sistemática de las mamas en la gestación y el puerperio, así
como la realización de ultrasonido, mamografía y PAAF ante cualquier lesión sospechosa.(R2)

411. Niño con nauseas, vómitos, dolor en hipogastrio y diarrea con moco y sangre
además de tenesmo. En su vivienda crían cerdos. ¿Cuál es la etiología más probable del
cuadro diarreico?

1. 1. Giardia.
2. 2. Balantidiasis.
3. 3. Estrongiloidosis.
4. 4. Cisticercosis.
Gráfico de respuestas
Comentario

Siempre en preguntas tipo test te darán datos importante que debe tomar en cuenta. En este caso
le estan diciendo que CRIAN CERDOS!!!.

Con este dato inmediatamente debe pensar en una infección por Balantidium coli.

Balantidium coli es el más grande y única protozoo ciliado que infecta a los seres humanos.

Se presenta en regiones tropicales y subtropicales y en países en vías de desarrollo. Puede


transmitirlos cerdos y monos. La transmisión es a través de la ingestión de quistes

MANIFESTACIONES CLÍNICAS - pueden ocurrir tres formas de infección por B. coli: la excreción
de quistes asintomáticos, colitis aguda y la infección crónica. Los síntomas de la colitis aguda

 
 
 
 
incluyen náuseas, vómitos, pérdida de peso y dolor abdominal. Las heces pueden ser líquidos o
con disentería. La infección crónica es relativamente poco común, que consiste en episodios
intermitentes de dolor abdominal y diarrea con sangre.

DIAGNÓSTICO - detección de trofozoitos o quistes en los exámenes de heces o raspados de la


mucosa obtenidos en la colonoscopia o sigmoidoscopia.

TRATAMIENTO - El tratamiento en niños consiste en metronidazol.(R2)

412. ¿En cuál de los siguientes supuestos estaría MENOS indicada, de entrada, la
angioplastia coronaria transluminal percutánea (ACTP)?

1. 1. Lesión severa del tronco de la coronaria izquierda.


2. 2. Estenosis proximal de la arteria descendente anterior.
3. 3. Estenosis de by-pass aortocoronario con vena safena.
4. 4. Estenosis de injerto mamario-coronario.
Gráfico de respuestas
Comentario

Para acertar esta pregunta, bastaría conocer las indicaciones de la revascularización quirúrgica,
que son:

- Estenosis mayor al 50% del tronco de la coronaria izquierda.

- Enfermedad de dos vasos, siendo uno de ellos la descendente anterior proximal.

- Enfermedad de tres vasos.

Recuerde que el nivel crítico de estenosis (“estenosis severa”), para todos los vasos salvo el tronco
coronario izquierdo, es del 70%. Teniendo en cuenta estos datos, la respuesta correcta es la 1, que
se beneficia más de la cirugía que de la ACTP.(R1)

413. Adulto mayor con antecedente de apendicetomía. Acude por cuadro de oclusión
intestinal, por lo que se coloca SNG. Después de 24 h, el enfermo no evoluciona
favorablemente; ahora la actitud será:

1. 1. Descompresión con enemas.


2. 2. Colectomía + ileostomía.
3. 3. Continuar observación 48 h.
4. 4. Intervención de Hartmann.
Gráfico de respuestas
Comentario

En cuadros de obstrucción intestinal que no mejoren con medidas no invasivas, el siguiente paso
será realizar una colectomía con ileostomía. Las respuestas dadas podrían ser impugnables, pero
en este tipo de examenes como el ENARM habrá que elegir la respuesta que se acerque a la
realidad.(R2)

 
 
 
 
414. ¿Cuál es el signo radiológico más frecuente de la osteomielitis crónica en el niño?

1. 1. Fístula.
2. 2. Periostitis.
3. 3. Secuestros.
4. 4. Luxación patológica.
Gráfico de respuestas
Comentario

El signo radiológico más frecuente de la osteomielitis crónica en el niño es la periostitis.(R2)

415. La presencia de proteínas en la orina puede ser un marcador importante de


enfermedad renal. Señale la respuesta INCORRECTA:

La excreción urinaria de proteínas, superior a 3 g/24 horas supone, en la práctica, que


1. 1.
exista afectación glomerular.
La rara presencia de proteinuria selectiva (IgG/albúmina) implica un mal pronóstico en la
2. 2.
nefritis de cambios mínimos.
3. 3. La microalbuminuria es factor pronóstico de la nefropatía diabética.
La proteína de Tam-Horsfall se compone de microproteínas secretadas por las células
4. 4.
tubulares.
Gráfico de respuestas
Comentario

El funcionamiento normal del glomérulo impide la filtración de proteínas, de manera que una
excreción elevada de proteínas, superior a 3 g/24 horas, supone en la práctica una clara afectación
glomerular (de hecho, se considera criterio de síndrome nefrótico, al ser una proteinuria masiva).

La nefritis de cambios mínimos es la causa más frecuente de síndrome nefrótico en niño,


produciéndose una proteinuria que suele ser altamente selectiva (predominio de albúmina), lo cual
no indica peor pronóstico, de hecho el pronóstico de esta patología es excelente a largo plazo al
tratarla con corticoides, con una supervivencia a los 15 años superior al 90%. Un dato revelador
acerca de la nefropatía diabética es la microalbuminuria, que es el mejor marcador precoz de
neuropatía en diabéticos y también parece predecir la mortalidad cardiovascular.(R2)

416. Which of the following diseases does not affect sebaceous-gland-rich areas of the
skin?

1. 1. Pityriasis rosea.
2. 2. Seborrheic dermatitis.
3. 3. Darier's disease.
4. 4. Pemphigus foliaceus.
Gráfico de respuestas
Comentario

Pregunta compleja, difícil de contestar de un tema que seguramente no preguntaran. En general, la


enfermedades eritematoescamosas tienen una distribución en áreas seborreicas típica, como en la
Enfermedad de Darier y la Dermatitis Seborreica.

El Pénfigo Foliáceo es un tipo específico que se caracteriza por la aparición de las lesiones
ampollosas en áreas seborreicas (respuesta 4 falsa).

 
 
 
 
La enfermedad que NO se presenta en áreas seborreicas, sino que, característicamente, sus
lesiones aparecen sobre las líneas de tensión de la piel es la Pitiriasis Rosada de Gibert (respuesta
1 correcta).(R1)

417. Indique la respuesta correcta: en el esófago de Barrett, la realización de una técnica


antirreflujo, con demostración manométrica y pHmétrica, de ser eficaz:

1. 1. Hace retroceder siempre las alteraciones de la mucosa esofágica.


2. 2. No hace retroceder la enfermedad, pero impide la evolución a carcinoma.
3. 3. No desaparece el potencial para que se desarrolle un adenocarcinoma.
Tras dos estudios consecutivos de pHmetría esofágica normal, separados entre sí por al
4. 4.
menos 12 meses, puede prescindirse de nuevas valoraciones del paciente.
Gráfico de respuestas
Comentario

El esófago de Barrett consiste en la presencia de un epitelio columnar de tipo intestinal, que


aparece en algunos pacientes con esofagitis por reflujo gastroesofágico. Este cambio del tipo de
epitelio obedece a fenómenos de metaplasia, e implica un riesgo de malignización. Por ello, habrá
que vigilarlo endoscópicamente de forma periódica (respuesta 4 y 5 falsas), ya que existe potencial
de malignización (respuesta 2 falsa). La respuesta 1 es también incorrecta (de hecho, la sola
presencia del término “SIEMPRE” debería levantar sospechas). La única correcta es la 3. Lo que
hace la técnica antirreflujo es reducir el riesgo de progresión a displasia y, si la hubiera, desciende
el riesgo de malignización, pero no se elimina.(R3)

418. ¿Cuál de las siguientes opciones relacionadas con el consumo de sustancias es


INCORRECTA?

La intoxicación producida por sustancias se define como un síndrome reversible que se


1. 1. caracteriza fundamentalmente por la aparición de cambios psicológicos o
comportamentales desadaptativos y fisiológicos.
La abstinencia se define como un síndrome específico de una sustancia debido al cese o
2. 2. reducción de su consumo, que causa un malestar clínicamente significativo y/o un
deterioro de la actividad laboral y social.
El concepto de tolerancia a una sustancia hace referencia al hecho de que con el consumo
3. 3.
continuado se precisan cada vez dosis menores para producir el mismo efecto.
El síndrome de abstinencia producido por la falta de la dosis de la sustancia consumida
4. 4. tiene características específicas para cada sustancia y se alivia con un nuevo consumo de
la misma.
Gráfico de respuestas
Comentario

Muy larga en las opciones, pero si las lees con cuidado resulta fácil de contestar (¡además, el tema
es importante!).

La tolerancia se define como:

•   Necesidad de AUMENTAR las dosis para conseguir el efecto deseado y/o


•   DISMINUCIÓN del efecto cuando se mantiene la misma dosis.

El resto de las definiciones (intoxicación, abstinencia, abuso o consumo perjudicial) son las
descritas en las otras respuestas.(R3)

 
 
 
 

419. Paciente de 30 años que acude a su


consulta por una citología positiva para HSIL. Usted realiza una colposcopía y objetiva
los hallazgos que se muestran en la imagen. Ante dicha imagen, debemos:

Mantener una actitud expectante, puesto que se trata de una imagen en mosaico, y realizar
1. 1.
nuevo control citológico en 6 meses.
La lesión es altamente sospechosa para carcinoma invasor, por lo que hay que iniciar
2. 2.
estudio de extensión.
Se trata de un hallazgo colposcópico normal, por lo que la paciente realizará nuevo
3. 3.
control citológico en un año.
4. 4. Se trata de una zona acetoblanca, por lo que realizaremos una biopsia de la zona.
Gráfico de respuestas
Comentario

Ante una citología cervical alterada, el siguiente paso es la realización de una colposcopía, como
han hecho el caso que nos describen. Recuerde que los hallazgos colposcópicos anormales son:

- Epitelio acetoblanco.

- Epitelio yodonegativo.

- Leucoplasia debida a necrosis y queratinización.

- Neoformación vascular.

- Ulceración del epitelio.

Se debe realizar una biopsia de la zona sospechosa y actuar en función del resultado AP que
obtengamos.(R4)

420. Señale cuál de los siguientes NO parece aumentar el riesgo de sufrir cáncer de
cérvix:

1. 1. Obesidad.

 
 
 
 
2. 2. Promiscuidad.
3. 3. Tabaquismo.
4. 4. Inicio precoz de relaciones sexuales.
Gráfico de respuestas
Comentario

Tenga cuidado doctor, porque, con mucha frecuencia, tratarán que confunda los diferentes factores
de riesgo de los cánceres ginecológicos. La obesidad es un factor de riesgo para cáncer de mama
y, sobre todo, endometrio. Sin embargo, la infección por VPH y la promiscuidad sexual se asocian
al cáncer de cérvix.

Tenga cuidado también con el tabaco, que lo mencionan en una opción. Recuerde que fumar
aumenta el riesgo de cáncer de cérvix, pero no de mama.(R1)

421. Respecto a las manifestaciones clínicas de la poliarteritis nudosa, es FALSO que:

La afectación articular se expresa por poliartritis deformante en las grandes articulaciones


1. 1.
de las extremidades.
La forma más característica de la neuropatía periférica es la mononeuritis múltiple,
2. 2.
aunque puede manifestarse en forma de polineuropatía sensitivomotora.
3. 3. El dolor abdominal es el síntoma más frecuente de la afectación gastrointestinal.
4. 4. La afectación genital se puede traducir en forma de orquiepididimitis o prostatitis.
Gráfico de respuestas
Comentario
LA poliarteritis nodosa o PAN clásica es una vasculitis necrotizante sistémica que puede afectar a
múltiples órganos. Lo más frecuente es la afectación renal que aparece en forma de hipertensión
renal y e insuficiencia renal, y la del sistema musculoesquelético que puede manifestarse como
artralgias, mialgias o artritis no deformante. El siguiente órgano afectado en frecuencia es el
sistema nervioso periférico ( aprox en la mitad de los pacientes) que se afecta en forma de
mononeuritis múltiple típicamente, pudiendo presentar también polineuropatía. El tubo digestivo se
afecta en el 44% de los pacientes y cursa en forma de dolor abdominal generalmente, pudiendo
provocar hemorragias, infartos y perforaciones. LA piel se afecta también de forma frecuente
(aparece púrpura, úlceras, nódulos, lívedo, Raynaud...): El corazón ( insuficiencia cardiaca, IAM,
pericarditis) se afecta en el 35%; el sistema genitourinario ( dolor testicular, ovárico o de epidídimo)
en un 25% y, por último, la afectación del sistema nervioso central ( ACV, alteración del estado
mental, convulsiones#etc) es la menos frecuente ( en un 23%).(R1)

422. El síndrome de Meigs, se acompaña típicamente de:

1. 1. Adenoma ovárico.
2. 2. Fibroma ovárico.
3. 3. Sarcoma ovárico.
4. 4. Tumor de Krukenberg.
Gráfico de respuestas
Comentario

El síndrome de Meigs se define como la presencia de ascitis e hidrotórax asociados con un tumor
benigno de ovario.

El síndrome se asocia, generalmente, con un fibroma, un tecoma o un tumor de Brenner.(R2)

 
 
 
 
423. Mujer de 73 años que empieza con cuadro de escalofríos, fiebre de 39 ºC, ictericia y
dolor en hipocondrio derecho. Ecografía: vía biliar dilatada y cálculo en colédoco. Se
hacen hemocultivos y se inicia tratamiento antibiótico. ¿Qué germen es el menos
necesario que quede cubierto por los antibióticos en espera del resultado del cultivo?

1. 1. Estaphylococus aureus.
2. 2. Escherichia coli.
3. 3. Bacteroides.
4. 4. Klebsiella.
Gráfico de respuestas
Comentario

La clínica que presenta esta paciente es compatible con un cuadro de colangitis. Esto viene
apoyado por la ecografía, que muestra una vía biliar dilatada y un cálculo en el colédoco, aparte de
la clásica tríada de Charcot (ictericia, fiebre, dolor). Dado que se trata de una infección biliar, habría
que pensar en anaerobios y gramnegativos, por lo que la opción incorrecta es S. aureus, que
habitualmente no coloniza esta región anatómica.(R1)

424. Un paciente con diagnóstico de depresión mayor es traído al servicio de Urgencias.


Se sospecha un intento autolítico con dosis altas de paracetamol. Indique la afirmación
con la que NO está de acuerdo:

1. 1. Si han pasado más de 72 horas de la ingesta, el peligro ya ha pasado.


El tratamiento inmediato consiste en lavado gástrico, medidas de sostén y carbón activado
2. 2.
oral, si han pasado menos de 30 minutos desde la ingesta.
El aporte de grupos sulfhidrilo liga los metabolitos tóxicos y estimula la síntesis y
3. 3.
almacenamiento de glutation.
4. 4. Si el paciente se recupera, probablemente no presente secuelas hepáticas.
Gráfico de respuestas
Comentario

La intoxicación por paracetamol produce inicialmente una clínica poco específica, con síntomas
digestivos (náuseas, vómitos, dispepsia…). Más adelante, cuando han pasado 48- 72 horas, se
produce lo verdaderamente típico: una hepatitis fulminante de origen tóxico. Por ello, que hayan
pasado 72 horas desde la ingesta no significa que esté exento de riesgo.

La hepatitis por paracetamol puede ser mortal en el momento agudo, pero si se recupera es difícil
que produzca secuelas. El antídoto de esta intoxicación es la N- Acetil- Cisteína, cuya molécula
contiene un grupo sulfhidrilo, y esto contribuye a reponer el glutatión deficitario.(R1)

425. ¿Qué tipo de hernia se estrangula con mayor frecuencia?:

1. 1. Inguinal directa.
2. 2. Inguinal indirecta.
3. 3. Hernia de Spiegel.
4. 4. Hernia femoral.
Gráfico de respuestas
Comentario

Es habitual encontrar preguntas sobre hernias. Es importante que conozca todos los tipos posibles
y además añada algunos datos epidemiológicos que seguro serán de mucha utilidad.

 
 
 
 
La probabilidad de estrangulación de una hernia depende de la amplitud del orificio herniario. Así
es muy poco probable que se estrangule una hernia directa ya que es debida a una pérdida de
fuerza difusa de la fascia transversalis. En el otro extremo encontramos la hernia femoral o crural;
en este caso debido al cuello estrecho de la hernia, existe gran posibilidad de estrangulación.(R4)

426. Un anciano de 87 años es traído al servicio de Urgencias de su hospital por presentar


un cuadro compatible con una obstrucción intestinal. Señale cuál de las siguientes es la
entidad con mayores probabilidades de ser la responsable de esta situación:

1. 1. Cáncer de colon.
2. 2. Íleo biliar.
3. 3. Invaginación intestinal.
4. 4. Vólvulo de sigma.
Gráfico de respuestas
Comentario

Recuerda las causas más frecuentes de obstrucción intestinal:

- En población general: síndrome adherencial post- cirugía

- En pacientes sin antecedentes de cirugía abdominal: hernias

- En intestino grueso: cáncer de colon

Como las dos primeras causas no se dan como opciones, nos decantaremos por el cáncer de
colon.(R1)

427. Una amenorrea primaria acompañada de dolor abdominal, en la fase de Tanner 5, y


con una masa en la línea media abdominal, lo más probable es que se deba a:

1. 1. Hipertiroidismo.
2. 2. Ovarios poliquísticos.
3. 3. Himen imperforado.
4. 4. Enfermedad inflamatoria crónica del intestino.
Gráfico de respuestas
Comentario

Tema que hemOs repasado previamente. Lo más importante de las amenorreas es conocer su
diagnóstico diferencial, además debe conocer unos datos básicos de las causas más frecuentes o
más características.

El diagnóstico del himen imperforado se basa en la exploración genital que se debe realizar a toda
niña prepúber en la primera visita al pediatra. El acúmulo menstrual retenido puede producir dolor
abdominal. Cura con la incisión y evacuación del contenido vaginal.(R2)

428. Refiriéndose al hiperparatiroidismo primario. ¿Qué afirmación es FALSA?:

1. 1. La litiasis renal es su forma más frecuente de presentación.


2. 2. Puede producir nefrocalcinosis.

 
 
 
 
3. 3. Es causa de osteopenia difusa y fracturas de cuerpos vertebrales.
Es causa de alteración de la función tubular renal dando lugar a una incapacidad para
4. 4.
concentrar la orina y para excretar ácido.
Gráfico de respuestas
Comentario

El hiperparatiroidismo primario es la causa más frecuente de hipercalcemia. El exceso de PTH se


manifiesta clínicamente a dos niveles: sobre el riñón, por la hipercalcemia con hipercalciuria
mantenida, produce litiasis recidivante y nefrocalcinosis, que provoca disfunción tubular con
poliuria (por incapacidad de reabsorción de Na) y acidosis (por incapacidad para excretar ácido).
Sobre el hueso la PTH actúa aumentando la resorción. La forma más grave es la clásica osteítis
fibrosa quística y se caracteriza por quistes óseos en los huesos largos y tumores pardos,
formados por acúmulos de osteoclastos gigantes multinucleares. Es más frecuente encontrar
resorción subperióstica en las falanges, osteopenia localizada (cráneo en sal y pimienta), o pérdida
selectiva del hueso cortical (osteopenia difusa), que puede llegar a producir fracturas patológicas.
Sin embargo, la mayoría de los casos de hiperparatiroidismo que se diagnostican hoy día son
asintomáticos y se detectan por una hipercalcemia fortuita.(R1)

429. Si usted evalua la puntuación de Apgar al minuto y solamente se encuentra 30 latidos


por minutos, el puntaje que le corresponde es:

1. 1. 1.
2. 2. 3.
3. 3. 5.
4. 4. 7.
Gráfico de respuestas
Comentario

0 - 1 - 2 son los parámetros que se pueden utilizar en el APGAR.

Color de la piel: todo azul - extremidades azules - normal.

ritmo cardíaco: no pulso - <100 - >100.

irritabilidad: no respuesta - mueca, llanto, debil tos- movimiento.

actitud: ninguno - alguna flexión - movimiento activo.

respiración: ausente - débil. irregular - fuerte.

(R1)

 
 
 
 

430. El cáncer de mama es un tumor dependiente de estrógenos, lo que permite el uso de


múltiples tratamientos hormonales, aparte de los quimioterápicos, que también pueden
estar indicados. Señale, de entre los siguientes, qué fármaco es un antiestrógeno puro:

1. 1. Raloxifeno.
2. 2. Tamoxifeno.
3. 3. Fulvestrant.
4. 4. Letrozol.
Gráfico de respuestas
Comentario

El tamoxifeno y raloxifeno son moduladores selectivos de los receptores estrogénicos, pero no


antiestrógenos puros. El letrozol es un inhibidor reversible de la aromatasa más selectivo, por lo
que no afecta a hormonas tiroideas, adrenales... El único antagonista puro que existe entre estas
cuatro opciones es el fulvestrant.(R3)

431. Varón de 50 años, sin antecedentes de interés, que desde hace 4 meses presenta
cefalea, pérdida de memoria, y en la última semana se ha añadido disminución del nivel
de conciencia y varios accidentes isquémicos transitorios en territorios de ambas
arterias cerebrales medias. En la analítica destacan solo una VSG elevada, siendo las
serologías para sífilis, VIH, CMV, herpes simple, varicela y Borrelia negativas, así como
los ANA y ANCA. Radiografía de tórax, ecocardiograma y estudio Doppler de troncos
supraaórticos sin alteraciones. La TC y la RMN objetivan múltiples infartos en corteza y
sustancia blanca. La angiografía no mostró lesiones. En la biopsia cerebral se evidenció
afectación de arteriolas con lesiones vasculíticas granulomatosas. El diagnóstico que
Ud. sospecha en este paciente es:

1. 1. Enfermedad de Buerger.
2. 2. Enfermedad de Eales.
3. 3. Vasculitis primaria del SNC.
4. 4. Enfermedad de Churg-Strauss.
Gráfico de respuestas
Comentario
La vasculitis aislada del SNC es un proceso grave e infrecuente con afectación principal de
arteriolas. La clínica suele ser bastante inespecífica, aunque es típico la aparición de cefalea,
deterioro progresivo de las funciones superiores y aparición de múltiples infartos distribuido por
ambos hemisferios cerebrales.

 
 
 
 
Dado el cuadro clínico podemos descartar las otra cuatro opciones: Wegener (infiltrados bilaterales
cavitados no migratorios, afectación de vía aérea superior y frecuente afectación renal); Churg-
Strauss (asma resistente a tratamiento, eosinofilia e infiltrados pulmonares no cavitados
migratorios); Buerger (isquemia importante de extremidades en varones jóvenes fumadores) e
Eales (vasculitis retiniana).(R3)

432. Durante los primeros días de un RN, la madre produce pequeños volúmenes de
calostro, es recomendable:

1. 1. Suplementar con suero y leche.


2. 2. Suplementar con leche
3. 3. No dar ninguna suplementación.
4. 4. Ninguna de las anteriores.
Gráfico de respuestas
Comentario

Durante los primero días cuando la madre produce calostro, este es suficiente para la alimentación
del RN, por lo que no hay que dar ningún tipo de suplementación. Respuesta 3 correcta.(R3)

433. Se realizó una


punción de la colección paravertebral que se envió para estudio histológico y
microbiológico. En la imagen se muestra el resultado de la tinción con la técnica de Zielh-
Neelsen. Señale la respuesta correcta:

Se observan cocobacilos Gram negativos, compatible por tanto con el diagnóstico de


1. 1.
infección por Brucella melitensis.
Se observan cocos Gram positivos en racimos, compatible con el diagnóstico de infección
2. 2.
por Staphylococcus aureus, probablemente secundaria a endocarditis.
Se observan bacilos Gram negativos, compatible con el diagnóstico de infección por
3. 3.
Escherichia coli, probablemente secundaria a infección urinaria complicada del anciano.
Se observan bacilos ácido-alcohol resistentes, compatible con el diagnóstico de infección
4. 4. por Mycobacterium tuberculosis. Debe iniciar tratamiento antituberculoso con cuatro
fármacos.
Gráfico de respuestas
Comentario

Cuando la tinción de Ziehl- Neelsen es positiva, se aprecian los bacilos tal como se muestra en la
sección derecha de la imagen: rojizos sobre fondo azul. Podría haber confusión con Nocardia
asteroides es también un microorganismo ácido alcohol resistente, pero no viene dentro de las

 
 
 
 
opciones y además el contexto clínico sería el de un paciente con abscesos pulmonares y
cerebrales al mismo tiempo. Sin embargo, el caso clínico y la imagen microbiológica sí encajan
bien en una posible tuberculosis, que actualmente se debe tratar con cuatro fármacos Respuesta 4
correcta.(R4)

434. ¿Cuál de las siguientes afirmaciones es correcta en relación al divertículo


faringoesofágico o divertículo de Zenker?

1. 1. Suele diagnosticarse en pacientes jóvenes.


2. 2. El tratamiento incluye la miotomía del músculo cricofaríngeo.
3. 3. La pirosis es su síntoma principal.
4. 4. Se localiza siempre en la cara anterior de la hipofaringe.
Gráfico de respuestas
Comentario

El divertículo de Zenker se localiza en la parte posterior de la hipofaringe, por encima del músculo
cricofaríngeo y debajo del músculo constrictor inferior de la faringe. Se originan por pulsión, debido
a una mala coordinación de la musculatura faríngea. Puede causar halitosis, regurgitación, disfagia
orofaríngea e incluso una obstrucción completa por compresión.

Como complicaciones, puede producir episodios de broncoaspiración, formación de fístulas entre


el divertículo y la tráquea, hemorragia intradiverticular (sobre todo con la aspirina) y, más
raramente, la aparición de un carcinoma epidermoide dentro del divertículo. La colocación de una
sonda nasogástrica o la realización de una endoscopia en estos pacientes tiene riesgo de
perforación del divertículo, por lo que deben evitarse. El tratamiento es quirúrgico, realizando una
miotomía cricofaríngea y extirpando el divertículo. Si es pequeño, la miotomía aislada puede ser
suficiente.(R2)

Divertículo de Zenker

 
 
 
 
435. Un paciente diabético de 60 años consulta por primera vez respecto al tratamiento
de su enfermedad, ¿qué objetivo a alcanzar de los siguientes le recomendaría en primer
lugar?

1. 1. Abandono del hábito tabáquico.


2. 2. Mantener un índice de masa corporal (IMC) menor de 21.
3. 3. Realizarse glucemia capilar basal a diario.
4. 4. Evitar las grasas animales en la dieta.
Gráfico de respuestas
Comentario

Se trata de una pregunta sobre recomendaciones en la diabetes tipo 2, dado que la edad del
paciente de la pregunta nos sugiere este tipo de diabetes con mayor probabilidad. Dado que los
pacientes diabéticos presentan un riesgo aumentado de enfermedad cardiovascular, la
deshabituación del hábito tabáquico es una recomendación inicial prioritaria y fundamental en este
grupo. Por ello, la respuesta número 2 es correcta. El control lipídico y tensional también es
importante. Entre los objetivos tensionales se recomienda que las cifras de presión arterial no
sobrepasen los 130 mmHg de sistólica y 80 mmHg de diastólica, por ello, la respuesta número 1 no
es correcta.

Respecto al control de peso, objetivo es un Índice de Masa Corporal (IMC) por debajo de 25
(respuesta 3 incorrecta) sin llegar a la desnutrición. Se recomienda una dieta hipocalórica y
ejercicio físico en aquellos que presentan sobrepeso u obesidad; hipercalórica si el paciente está
desnutrido; y normocalórica si presenta normopeso. En cuanto a las características de la dieta, la
distribución de los macronutrientes (hidratos de carbono, proteínas y lípidos) no tiene por qué
diferir de la población general. En este sentido, el aporte de grasas saturadas (grasas animales,
entre otras) debe ser menor del 7%, por lo que la opción número 5 “evitar las grasas animales” nos
puede inducir a marcarla como respuesta correcta. Y es verdad que se debe hacer un consumo
moderado de este tipo de grasas, pero desde luego es mucho más relevante para su riesgo
cardiovascular, el abandono del hábito tabáquico. Finalmente, los beneficios del autocontrol
domiciliario en paciente con DM tipo 2 son controvertidos. Como norma general, los autocontroles
no estarían indicados en pacientes con DM tipo 2 en tratamiento dietético o con antidiabéticos
orales que no produzcan hipoglucemia, y sí lo estarían en aquellos con insulina o antidiabéticos
orales que pueden producir hipoglucemia. Por ello, la respuesta 4 es falsa.(R1)
Objetivos control metabólico ADA 2010

PARÁMETROS OBJETIVO

HbA1c < 7%

Glucemia prepandrial 70-130 mg/dl

Glucemia pospandrial < 180 mg/dl

 
 
 
 

Tensión arterial < 130/80 mmHg

Colesterol LDL < 100 mg/dlSe recomienda < 70 mg/dl en pacientes con
enfermedad cardiovascular manifiesta

Colesterol HDL > 40 mg/dl en varón y > 50 mg/dl en mujer

Triglicéridos < 150 mg/dl

Suspensión hábito
tabáquico

Paciente
hospitalizado

Paciente crítico < 180 mg/dl (rango 140-180 mg/dl)

Paciente no crítico Preprandial < 140 mg/dlGlucemia en cualquier momento <


180 mg/dl(No clara evidencia para la recomendación)

436. Recién nacido hombre, con


antecedente de oligoamnios, presenta distrés respiratorio grave que requiere intubación
y ventilación mecánica. En las primeras 24 horas se complica, apareciendo una masa

 
 
 
 
suprapúbica, elevación de las cifras de creatinina con oligoanuria y empeoramiento
respiratorio con la siguiente radiografía. ¿Qué sospecharía usted?

1. 1. Enfermedad de membrana hialina.


2. 2. Síndrome de aspiración meconial.
3. 3. Encefalopatía hipóxico-isquémica con vejiga neurógena.
4. 4. Válvulas de uretra posterior.
Gráfico de respuestas
Comentario

El líquido amniótico se produce a partir de la orina fetal. Por este motivo, muchas alteraciones
urológicas de carácter obstructivo pueden cursar con oligoamnios. De las opciones que nos
ofrecen, la única que justifica este dato es la respuesta 4, las válvulas de uretra posterior.

Los recién nacidos con oligoamnios presentan una mayor incidencia de problemas respiratorios en
este momento de la vida, pero la clave para resolver esta pregunta no sería ésta (sólo es un dato
acompañante), sino la elevación de la creatinina y la oligoanuria.(R4)

437. En el caso anterior, ¿cuál sería el tratamiento etiológico?

1. 1. Ureterostomía bilateral.
2. 2. Ampliación vesical.
3. 3. Resección transuretral.
4. 4. Punción vesical.
Gráfico de respuestas
Comentario

El tratamiento de las válvulas de uretra posterior consisten, sencillamente, en eliminarlas in situ, es


decir, mediante resección transuretral. Entre las opciones que nos ofrecen, no encontramos
demasiadas dudas, ya que el resto de las respuestas nos hablan de intervenir vejiga o uréteres,
cuando el problema primario está en la uretra. Respuesta 3 correcta.(R3)

438. Un paciente de 32 años acude a su consulta por incremento de tamaño testicular en


los últimos dos meses. A la exploración se palpa nódulo testicular derecho. El
ultrasonido muestra imagen intraparenquimatosa testicular derecha con calcificaciones
en su interior. Señale la respuesta CORRECTA:

1. 1. Se debe realizar una orquiectomía inguinal previa a criopreservación seminal.


2. 2. Se debe realizar una biopsia transescrotal.
3. 3. La presencia de calcificaciones indica buen pronóstico.
4. 4. Se debe realizar un TAC toraco-abdominal.
Gráfico de respuestas
Comentario
Se debe completar el estudio de extensión con una TAC, si bien esto no debe retrasar la
orquiectomía. La criopreservación seminal se debe realizar previa al tratamiento con
quimioterapia.(R4)

439. La causa más frecuente del Síndrome de Cushing en la práctica diaria es:

1. 1. Un adenoma hipofisario productor de ACTH.


2. 2. Un tumor bienigno o maligno de las suprerrenales.

 
 
 
 
3. 3. La hiperplasia suprerrenal.
4. 4. La toma prolongada de corticoides.
Gráfico de respuestas
Comentario
Pregunta directa y muy sencilla en relación con la etiología del Cushing. La causa más frecuente
de síndrome de Cushing es la administración exógena prolongada de glucocorticoides (exógeno).
Dentro del Cushing de origen endógeno, la más frecuente es la existencia de un microadenoma o
macroadenoma hipofisario productor de ACTH. Otras causas menos frecuentes son el de origen
adrenal y el producido por secreción ectópica de ACTH.(R4)

440. A 46-year-old woman comes to your office due to depressed mood over the past 6
months. She has lost 6 kg, and she no longer goes to her reading club, which she used
to enjoy very much. She denies having any suicidal plans. She is diagnosed with major
depressive disorder and started on sertraline. Two months later, she comes for a follow-
up visit, and refers no improvement at all. You decide to change her medication. What
would be the most appropriate treatment in this case?

1. 1. Tranylcypromine
2. 2. Citalopram
3. 3. Amitriptyline
4. 4. Valproic acid
Gráfico de respuestas
Comentario
Citalopram.
(R2)

441. En un paciente varón de mediana edad que presenta dolor en la columna


dorsolumbar y sacroileítis de larga evolución con limitación de la movilidad, la aparición
de forma brusca de dolor ocular, fotofobia y aumento de lagrimeo puede sugerirnos el
diagnóstico de:

1. 1. AR.
2. 2. Síndrome de Reiter.
3. 3. Espondilitis anquilosante.
4. 4. Artropatía psoriásica.
Gráfico de respuestas
Comentario
La afectación del esqueleto axial en este paciente a nivel de la columna dorsal y lumbar hace muy
improbable los diagnósticos de LES y AR. Aunque en el Síndrome de Reiter (artritis reactiva) y en
la artropatia psoriasica esta afectación es más habitual la ausencia de afectación de articulaciones
periféricas pone muy en duda esos diagnósticos. El cuadro clínico descrito es compatible con
espondilitis anquilosante. Además la complicación ocular que mencionan, sugestiva de uveitis
anterior es la más frecuente manifestación extrarticular de la Espondilitis anquilosante.(R3)

442.Respecto a las mastitis puerperales, ¿cuál de las siguientes afirmaciones es FALSA?

1. 1. El germen causal más frecuente es el Streptococcus agalactiae.


2. 2. En caso de abscesos, se debe aplicar calor local y proceder al drenaje quirúrgico.
3. 3. El tratamiento de elección son los betalactámicos.
4. 4. Son más frecuentes en primíparas.
Gráfico de respuestas

 
 
 
 
Comentario

Las mastitis son infecciones muy frecuentes durante la lactancia. La producen gérmenes que
pasan de la piel a la glándula mamaria a través de grietas y heridas que se forman durante este
período. Ante una mastitis, debes pensar en microorganismos grampositivos, siendo el más
frecuente S. aureus (respuesta 1 falsa).

El tratamiento de la mastitis se realiza con antibióticos y analgésicos, evacuación periódica del


pecho y aplicación de calor local. En el caso de que se forme un absceso, será necesario
completar el tratamiento anterior con el drenaje quirúrgico. No es necesario inhibir la lactancia,
salvo que sea una mastitis bilateral.(R1)

443. Recibe en consulta a los padres de un niño sano de tres años que inició el ciclo
escolar en el jardín de niños. Les preocupa que el niño llore excesivamente cada vez que
la madre lo deja en la escuela. ¿Cuál de las siguientes es la MEJOR respuesta para los
padres, en relación a esta conducta?:

1. 1. Es adecuada para la edad, desde el punto de vista madurativo.


2. 2. Indica que el niño no tiene la edad ideal para concurrir a Jardín.
3. 3. Justifica derivación a Psicopatología.
4. 4. Ninguna de las anteriores.
Gráfico de respuestas
Comentario

El niño tiene 3 años de edad, por lo que la respuesta correcta es la 1. Es adecuada para la edad
que el niño tenga angustia de separación.(R1)

444. En relación con el examen físico del RN, marque la CORRECTA:

1. 1. El test de Silverman considera el quejido inspiratorio.


2. 2. El soplo grado II es suave, fácilmente audible y con fremito.
3. 3. Dos vasos umbilicales puede verse en trisomia 18.
4. 4. La fosita sacrococcigea es resto de la extremidad caudal del tubo neural.
Gráfico de respuestas
Comentario

El síndrome de Edwards, también conocido como trisomía 18, es un tipo de aneuploidía humana
que se caracteriza usualmente por la presencia de un cromosoma adicional completo en el par 18.
También se puede presentar por la presencia parcial del cromosoma 18 (translocación
desequilibrada) o por mosaicismo en las células fetales.

Dada la alta tasa de mortalidad postnatal de esta enfermedad genética, no existe a día de hoy un
tratamiento eficaz. El trabajo clínico se restringe al diagnóstico prematuro para poder supervisar el
embarazo de forma adecuada (o dar consejo genético a la familia, para que tenga en cuenta lo que
supone un Edwards, de modo que decidan si interrumpir el embarazo o no) y a la preparación
psicológica de los padres para una eventual muerte perinatal inminente o el retraso mental y las
incapacidades físicas en los escasos sobrevivientes.

El diagnóstico se realiza entre la semana 12 y 20 del embarazo mediante técnicas


ultrasonográficas. Ante la presencia de cualquier malformación que presuma una aberración
cromosómica, se

 
 
 
 
confirma el diagnóstico mediante amniocentesis, cordocentesis o biopsias de tejido placentario. En
el caso de la amniocentesis, las muestras son cultivadas mediante protocolos citogenéticos
convencionales para el aislamiento y proliferación de células fetales (como los amniocitos).
Posteriormente, con técnicas de hibridación (FISH) se tratan las células con sondas fluorescentes
para confirmar la presencia de alguna aberración cromosómica, en este caso, la presencia de un
cromosoma 18 o un fragmento del mismo de más.(R3)

445. En las anastomosis de esófago, con respecto a la fuga anastomótica, señale la


CORRECTA:

1. 1. Es frecuente porque el esófago no tiene muscular.


2. 2. Es lo mismo que una dehiscencia de la anastomosis.
3. 3. Junto con las resecciones anteriores de recto son las que más riesgo presentan.
4. 4. El tratamiento suele ser, de entrada, quirúrgico.
Gráfico de respuestas
Comentario

Es importante diferenciar entre una fuga, en la que se filtra el contenido del tubo digestivo, sin que
exista solución de continuidad franca, de una dehiscencia, en la que parte de la sutura se ha ido y
existe un “agujero” de mayor o menor medida.

La mayoría de las fugas producen una colección circundante, que si está bien drenada, suele
responder bien al tratamiento conservador (antibióticos y reposo digestivo). Sin embargo si la
dehiscencia es grande suele precisar de cirugía porque el paciente empeora rápidamente. Las
anastomosis que más riesgo de complicaciones son las de esófago y recto (las más distales y
proximales) porque carecen de serosa y las sutura es menos firme.(R3)

446. Los cristales de pirofosfato cálcico en el microscopio de luz polarizada tienen:

1. 1. Birrefringencia débilmente positiva y forma rectangular.


2. 2. Birrefringencia fuertemente negativa y aspecto en aguja.
3. 3. Birrefringencia débilmente negativa y aspecto en aguja.
4. 4. Ausencia de birrefringencia.
Gráfico de respuestas
Comentario
El aspecto del cristal en el microscopio de luz polarizada es la técnica específica para poder
establecer el diagnóstico de las artritis por microcristales. Los cristales de oxalato cálcico suelen
tener forma piramidal o bipiramidal con birrefringencia fuertemente positiva (respuesta 1). Los
cristales de pirofosfato cálcico tienen birrefringencia débilmente positiva y forma rectangular
(respuesta 2 correcta). Los de ácido úrico son microcristales de forma en aguja y birrefringencia
negativa (respuesta 3). Finalmente, los cristales de hidroxiapatita son muy pequeños, y no se
pueden identificar en el microscopio óptico, precisándose el microscopio electrónico o la difracción
de rayos X para su correcta identificación. No tiene, por lo tanto, birrefringencia (respuesta 5).(R1)

447. A 30-year-old woman is brought to the emergency department with a 5-day history
of cough, fever and watery sputum. Her previous medical history is insignificant. She
denies tobacco use. She has had a dog for the past three years. Ausculation reveals
decreased breath sounds in the lower fields of the right hemithorax. A thin-walled cavitary
lesion in the right lower lobe is seen on her chest X-ray film. Which of the following is the
most likely diagnosis?

 
 
 
 
1. 1. Pulmonary hydatid cyst.
2. 2. Cavitated mass.
3. 3. Lung abscess.
4. 4. Pulmonary tuberculosis.
Gráfico de respuestas
Comentario
Pregunta en forma de caso clínico relativamente fácil si conocemos algunos datos típicos acerca
de la enfermedad en cuestión (Hidatidosis). Fíjate bien que en esta pregunta el antecedente del
perro pesa mucho. Solamente con este dato podemos empezar a descartar todas las opciones
menos la 1. Recuerda que la hidatidosis es una enfermedad producida por una tenia que parasita a
los perros. Los humanos suponen un huésped accidental dentro de su ciclo vital (se adquiere
ingiriendo los huevos liberados por los perros en las deposiciones). Las larvas no pueden madurar
en los humanos y en cambio lo que pasa es que se generan quistes rellenos de líquido (frecuentes
en hígado y pulmón, también en cerebro). Recuerda que es típica la vómica, al romperse un quiste
pulmonar hacia la vía aérea, como en este caso; y que también es típico en la radiografía de
abdomen una masa hepática calcificada ( en la TAC también veremos una masa hepática, con
frecuencia polilobulada, más frecuentes en lóbulo derecho). Podemos asumir que en este caso el
quiste se ha roto y está infectado. Recuerda la utilidad de la punción guiada por eco más
aspiración y posterior instilación de alcohol o suero hipertónico, como opción de tratamiento
eficiente y segura frente a la cirugía abierta (bastante más riesgosa).(R1)

448. En el diagnóstico de la coartación de aorta, NO se presenta uno de los datos


siguientes. Señale cuál:

1. 1. Signo de Roesler.
2. 2. Signo de la "E" en el esofagograma con bario.
3. 3. Signo del "doble contorno".
Palpación de pulsos disminuidos en miembros inferiores, con pulsos normales en
4. 4.
miembros superiores.
Gráfico de respuestas
Comentario

Lo primero que debes recordar es la asimetría que existe entre los pulsos de los MMSS (normales
o aumentados, porque los vasos subclavios salen antes de la coartación) y los de los MMII
(disminuidos, porque obviamente las arterias ilíacas y femorales son distales a la coartación).
Debido a la disminución de flujo sanguíneo a las zonas distales a la obstrucción, se desarrolla
circulación colateral para suplir el déficit y de ahí surgen el resto de signos clínicos y RX:

- Muescas costales en la RX tórax, por gran desarrollo de las arterias subcostales (signo de
Roesler)

- Signo del “3” en la Rx tórax. Se debe a la dilatación pre y postestenótica que ocurre a nivel de la
propia coartación.

- Signo de la “E” en el esofagograma con bario. Su explicación es semejante al signo del 3. Se


observa en estos pacientes si se realiza un esofagograma con bario.

Lo que no es típico en absoluto es el signo del doble contorno. Éste signo traduce crecimiento
auricular izquierdo y se ve en la proyección posteroanterior en las RX tórax. Lo que se ve es un
doble contorno en la silueta de la aurícula derecha. Por supuesto que una coartación de aorta
puede tener este signo, por crecimiento auricular izquierdo, para compensar un VI hipertrófico,
pero esto no nos vale para hacer el diagnóstico de coartación.(R3)

 
 
 
 
449. El tumor orbitario más frecuente en el adulto es:

1. 1. Hemangioma.
2. 2. Quiste dermoide.
3. 3. Rabdomiosarcoma.
4. 4. Meningioma.
Gráfico de respuestas
Comentario
Del tema de los tumores de la órbita no es necesario que entres en mucho detalle, pero sí que te
quedes con los datos clave. El hemangioma cavernoso es el tumor orbitario más frecuente en los
adultos, es de carácter benigno y es la causa más frecuente de exoftalmía tumoral en el adulto. Es
de predominio femenino y suele localizarse en el espacio graso posterior al globo ocular. Las
opciones 2, 3 y 4 son tumores orbitarios típicos de la infancia. De ellos puedes recordar que el
glioma del nervio óptico se asocia a la neurofibromatosis tipo I.(R1)

450. ¿Cuál de las siguientes afirmaciones sobre las características clínicas de la artritis
reumatoide es INCORRECTA?

La afectación de las articulaciones interfalángicas proximales es más frecuente que las


1. 1.
distales.
2. 2. La manifestación pulmonar más frecuente es la pleuritis, aunque suele ser asintomática.
La evidencia radiológica de aumento de partes blandas periarticular forma parte de sus
3. 3.
criterios diagnósticos.
La artritis reumatoide del adulto se asocia con un aumento de la frecuencia de ojo seco,
4. 4.
episcleritis e incluso escleritis.
Gráfico de respuestas
Comentario
Los hallazgos radiológicos que forman parte de los criterios diagnósticos son las erosiones o
descalcificaciones óseas inequívocas en las zonas adyacentes a las articulaciones afectadas, pero
no el aumento de partes blandas periarticular.(R3)

451. A 5-year-old female patient is brought to the emergency room presenting with fever
and pain in her right leg. Physical examination shows a swollen leg, especially on the
anterior tibial area, that is tender to palpation. Blood tests and cultures are pending.
Which of the following is the most likely diagnosis?

1. 1. Osteomyelitis of the tibia


2. 2. Septic Arthritis
3. 3. Sickle cells anemia
4. 4. Acute lymphoblastic leukemia
Gráfico de respuestas
Comentario
Osteomyelitis. Pain, fever and inflammatory signs are suggestive of a systemic infection. The
primary source can be easily be located in the tibial bone area. The most common pathogen
isolated is Staphylococcus aureus. Antibiotics must be started promptly.(R1)

 
 
 
 

¿Cuál es causa de muerte más frecuente en


estos pacientes? (Imagen 6)

1. 1. Shock séptico.
2. 2. Disfunción progresiva del ventrículo derecho.
3. 3. Hemoptisis masiva.
4. 4. Infarto agudo de miocardio.
Gráfico de respuestas
Comentario

En la fisiopatología del TEP, además de las alteraciones en el intercambio gaseoso, el aumento de


la resistencia al flujo aéreo y la disminución de la distensibilidad pulmonar, hay un aumento de la
resistencia vascular pulmonar (por obstrucción vascular o liberación de agentes neurohumorales
como la serotonina por las plaquetas), aumentando la tensión sobre el ventrículo derecho y
pudiendo desencadenar un cuadro de insuficiencia cardíaca derecha, que suele ser la causa
inmediata de fallecimiento más común en el tromboembolismo pulmonar.(R2)

Hombre de 63 años,
fumador importante e intervenido hace un mes por neoplasia de sigma, acude al Servicio
de Urgencias por presentar disnea de 48 horas de evolución que comenzó bruscamente
y se ha acompañado de un esputo hemoptoico y dolor torácico derecho de características
pleuríticas. En la exploración el paciente está ansioso, sudoroso, cianótico y se constata
una temperatura de 37,8 °C;una frecuencia respiratoria de 34 rpm; y una frecuencia
cardíaca de 120 lpm. La presión arterial es 100/55 mmHg. En la auscultación
cardiopulmonar se comprueba la existencia de taquicardia, un segundo ruido pulmonar
fuerte y un roce pleural derecho. La gasometría arterial respirando aire ambiente muestra
los siguientes datos: pH 7,32; PCO2 28 mmHg; PO2 50 mmHg. En el ECG se observa una
onda T invertida de V1 a V4. Tras la administración de oxígeno, la saturación de O2,
medida por pulsioximetría es de 90%. Se le realiza radiografía de tórax y TC espiral de
tórax con contraste (imagen 6). De las siguientes opciones en relación con el manejo de
este paciente, ¿cuál le parece la más acertada en este caso?

1. 1. Cefotaxima i.v. 2 g/8 horas más azitromicina i.v. 500 mg/día y tubo de drenaje pleural.

 
 
 
 
2. 2. Morfina i.v. 5 mg en dos minutos e iniciar infusión de dopamina 2-5 µg/kg/min.
3. 3. Activador tisular del plasminógeno 100 mg i.v. en dos horas seguida de anticoagulación.
4. 4. Aspirina 325 mg por vía oral y nitroglicerina sublingual.
Gráfico de respuestas
Comentario

A pesar de la extensión del caso clínico, es una pregunta sencilla sobre un tema preguntado
muchas veces : el tromboembolismo pulmonar (TEP). Este paciente tiene factores de riesgo
(tabaquismo y, sobre todo, una intervención quirúrgica hace menos de tres meses) y además nos
dan un dato muy típico: la disnea de comienzo brusco (con sólo este dato ya se debería pensar en
el TEP). El resto de datos que nos dan apoyan este diagnóstico: hemoptisis y dolor pleurítico
(orientan a TEP periférico), taquipnea, taquicardia… La gasometría también es compatible, revela
hipoxemia e hipocapnia (aunque la PCO2 puede ser normal o incluso elevarse en algunos casos).
El ECG es inespecífico, aunque es frecuente que, como en este caso, muestre anomalías
inespecíficas del ST-T en derivaciones anteriores.

Como este paciente tiene una alta probabilidad clínica de TEP, se realiza una TC, que muestra la
imagen, donde se confirma el diagnóstico (TEP masivo derecho), siendo además muy llamativo el
derrame pleural derecho. Para su tratamiento, en pacientes hemodinámicamente estables, la
anticoagulación aislada ofrece buenos resultados, pero en aquellos hemodinámicamente
inestables (TEP masivo), como en el presente caso, intentaremos eliminar el coágulo, ya sea
mediante trombólisis o embolectomía, por lo que la opción correcta sólo puede ser la 3, trombólisis
cuanto antes, seguida de anticoagulación (ésta tendrá una duración variable según si es el primer
episodio, los factores de riesgo son reversibles, etc.).(R3)

Tratamiento del TEP según la gravedadde presentación

 
 
 
 

Cuando una mujer queda embarazada, el organismo sufre una serie de modificaciones y
algunos parámetros biológicos cambian. ¿Cuál de los siguientes cambios NO sucede?

1. 1. Aumento de la frecuencia respiratoria y ligero aumento del pH.


2. 2. Aumento de las hormonas tiroideas y de la TBG.
3. 3. Disminución del filtrado glomerular.
4. 4. Disminución de las resistencias periféricas.
Gráfico de respuestas
Comentario

Los cambios que se producen en el organismo durante la gestación han sido preguntados en
varias convocatorias. Y, entre ellos, el más preguntado ha sido el filtrado glomerular, que tiene que
saber que está AUMENTADO (respuesta 3 verdadera). Como consecuencia de ello, disminuyen
los niveles plasmáticos de urea y creatinina, ya que aumenta su aclaramiento.

Desde el punto de vista cardiorrespiratorio, destaca la disminución de las resistencias periféricas y


el aumento de la frecuencia respiratoria, lo que justifica una ligera alcalosis metabólica, por lo que
aumenta el pH.

En cuanto a la función tiroidea, recuerda que la beta-hCG tiene acción “TSH- like”, por lo que
aumenta ligeramente el tamaño de la glándula, aumentando la cifra de hormonas tiroideas y de
tiroglobulina.(R3)

¿Cuál de los siguientes cánceres invasivos es el más frecuente en México?

1. 1. El cáncer de mama.
2. 2. El cáncer de ovario.
3. 3. El cáncer de vagina.
4. 4. El cáncer de cérvix.
Gráfico de respuestas
Comentario

Pregunta relativamente sencilla. El cáncer más frecuente en México es el de mama, seguido por el
cérvico-uterino. Respuesta 1 correcta.(R1)

El sitio más frecuente de metástasis a distancia del cáncer de mama es:

1. 1. Pulmón.
2. 2. Hueso.
3. 3. Mama contra lateral.
4. 4. Hígado.
Gráfico de respuestas
Comentario

El sitio de metástasis más frecuente es el pulmón. Aunque también podría afectar estructuras
óseas, hígado y mama contralateral.(R1)

 
 
 
 
La depresión del segmento ST en el electrocardiograma indica:

1. 1. Isquemia subendocárdica.
2. 2. Isquemia subepicárdica.
3. 3. Lesión subendocárdica.
4. 4. Lesión subepicárdica.
Gráfico de respuestas
Comentario
Es una pregunta de electrocardiografía, que no queda más remedio que aprenderse de memoria.
Recordemos. - Isquemia: cambios en la onda T. - Isquemia subendocárdica: T alta y picuda. -
Isquemia subepicárdica: T negativa simétrica. - Lesión: cambios en el segemnto ST. - Lesión
subendocárdica: descenso del ST. - Lesión subepicárdica: ascenso del ST. - Necrosis: onda Q
significativa.(R3)

La serotonina se ha implicado en la etiopatogenia de todos los trastornos siguientes


EXCEPTO uno:

1. 1. Trastorno obsesivo-compulsivo.
2. 2. Síndrome de Gilles de la Tourette.
3. 3. Trastorno por angustia.
4. 4. Trastornos por control de los impulsos.
Gráfico de respuestas
Comentario
Los estados hiposerotoninérgicos se han relacionado con la depresión, con los trastornos de
ansiedad (ansiedad generalizada, trastorno de angustia, fobia social, estrés postraumático,
trastorno adaptativo), los trastornos de la impulsividad y el TOC. El síndrome de Gilles de la
Tourette, aunque tiene impulsividad, se atribuye a una disfunción hiperdopaminérgica que
responde específicamente a neurolépticos incisivos: haloperidol o pimocide.(R2)

¿Cuál de los siguientes factores es el más importante como elemento pronóstico en el


cáncer de mama?

1. 1. Edad del paciente.


2. 2. Existencia de metástasis ganglionar regional.
3. 3. Invasión tumoral de vasos linfáticos / venosos.
4. 4. Si amamanto en algún momento.
Gráfico de respuestas
Comentario

El factor pronóstico más importante en el cáncer de mama es la existencia de afección ganglionar.


Respuesta 2 correcta.

Entre otros se encuentran:

Tamaño tumoral mayor de 2 cm, edad menor de 35 años, grado histológico G2 o G3,
multicentricidad, invasión vascular o linfática, actividad aumentada de angiogénesis, componente
intraductal extenso, márgenes quirúrgicos afectados o escasos. Receptores estrogénicos
negativos, alteración del oncogén C-erB2 y del gen supreso tumoral p-53, gestación, cáncer
inflamatorio, comedocarcinoma, cáncer fijo a pectoral y/o costilla e infiltración cutánea.(R2)

 
 
 
 
Hombre de 70 años de edad que acude a urgencias con un cuadro de dolor abdominal
difuso, náuseas y vómitos. En la anamnesis nos cuenta que desde hace medio año nota
un cambio de ritmo intestinal. A la exploración se observa un abdomen distendido, sin
visceromegalias y sin signos de rebote. En la radiografía se aprecia dilatación del colon.
¿En qué cuadro clínico deberíamos pensar?:

1. 1. Diverticulosis de sigma.
2. 2. Colitis isquémica.
3. 3. Cáncer de sigmoides.
4. 4. Vólvulus de ciego.
Gráfico de respuestas
Comentario

La presencia de distensión abdominal junto con cambios en el hábito defecatorio sugieren


fuertemente un problema obstructivo. Los hallazgos radiológicos también respaldan esta
posibilidad. Por lo tanto, de las opciones que nos ofrecen, debemos elegir entre las que produzcan
algún tipo de obstrucción, que son la 3 y 4. Entre ellas, es fácil salir de dudas. Si se tratara de un
cáncer de intestino delgado o de un vólvulo de ciego, no habría distensión del colon. La
obstrucción debería ser distal al mismo para que pudiera existir esta distensión, por lo que la
respuesta correcta es la 3.(R3)

Una paciente de 22 años de edad consulta por dolor en la rodilla derecha que en los
últimos tiempos ha aumentado. Nota chasquidos y le incapacita para las actividades
deportivas, pero no para su trabajo diario. El dolor mejora con el reposo y cada vez menos
con los analgésicos y antiinflamatorios. En la radiografía se observa un fragmento
redondeado en el polo superoexterno de la rótula. ¿Cuál sería el primer diagnóstico a
tener en cuenta?

1. 1. Rótula bipartita sintomática.


2. 2. Fractura del polo superior de la rótula.
3. 3. Condromalacia rotuliana.
4. 4. Osteocondritis disecante.
Gráfico de respuestas
Comentario

La imagen que nos describen en este caso es diagnóstica, si no hay antecedente traumático que
pudiera explicar la aparición de una zona independiente de la rótula. La rótula bipartita es una
anomalía relativamente frecuente, se debe a la falta de fusión de los centros de osificación
accesorios y se ha descrito en un 2 al 5% de las rodillas, siendo la más frecuente la que se localiza
en el polo supero- externo, como sucede en nuestro paciente.

En la mayoría de los casos, constituye un hallazgo casual y no precisa de tratamiento, excepto en


los casos en que se hace dolorosa, que puede ser agudo tras un traumatismo o progresivo.
Primero intentaremos un tratamiento conservador mediante analgésicos y AINEs habituales, y en
los casos refractarios estará indicada la extirpación del fragmento.(R1)

 
 
 
 

 
 
 
 

 
 
 
 

 
 
 
 

 
 
 
 

 
 
 
 

 
 
 
 

 
 
 
 

 
 
 
 

 
 
 
 

 
 
 
 

 
 
 
 

 
 
 
 

 
 

S-ar putea să vă placă și